Você está na página 1de 332

lsat-1-1

1. Mr. West: Well, Ms. Smith, by how much do you plan to increase your donation to the
cultural society this
year? You know how many worth while projects we do.
Ms. Smith: I’m not so sure of that. I was very upset about the statue you purchased last
month. I think I’ll give no more money to your cause.
Mr. West: That’s all right: we’ll just put you down for the same amount that you gave last
year.
Which one of the following words or phrases has been misinterpreted in the
conversation?
(A) “increase”
(B) “you know”
(C) “worthwhile”
(D) “no more”
(E) “same amount”

偶不明白为什么是 D 选项。而且 “ we’ll just put you down for the same amount that you
gave last year.”是什么意思?

的理解是 no more 意为不再多


put you down 应该有降低的意思
所以第二个人对第一个人的意思进行了误解

第一题的捐献者是说我明年不会比今年捐的更多了
而被捐献的人则认为那就是和去年一样多,殊不知捐献者可能今年就不捐了,呵呵,犯傻了。逻辑错误

no more, 是不再给了, 不捐钱了. 也许误解成了 not more, 不再多了.

put down 就是写下来, write down.

5. Emperor: The enemy empire across the sea has harassed us for centuries. I want to
conquer it and stop it once and for all. What advice can you give me?

Admiral: If you cross the sea, a mighty empire will fall.

Emperor: In that case, prepare the troops. We set sail tonight.

Of the following, the strongest criticism of the Emperor’s decision to invade would be that
it.

1
(A) Is certain to lead to the emperor’s defeat

(B) Is based on opinion rather than objective facts about troop strength

(C) Contradicts the Admiral’s statement

(D) Fails to consider fully the possible meanings of the Admiral’s advice

(E) Is a futile strategy for solving the problem at hand

答案:D,我觉得 B 也行啊,为什么不对呢?

请教做对话题有什么好办法吗?

不要脱离原文的逻辑. B is out of scope. 比如说原文讲根据一个 survey 做了一个结论, 首选答案应该是和


survey 有关, 例如 survey 不够 representative. 这道题 D 和原文的逻辑有直接的关系, 首选.
ya. empire can be wither enemy's or his own

6. No senator spoke at the convention unless he or she was a Democrat. No Democrat


both spoke at the convention and was a senator

Which one of the following conclusions can be correctly drawn from the statements
above?

(A) N one but senators spoke at the convention.

(B) No Democrat spoke at the convention.

(C) Only Democrats spoke at the convention.

(D) No senator spoke at the convention.

(E) Some Democrat senators spoke at the convention.

答案:D

我完全没有思路这种题该怎么做呢?

第一句说明会议上如果有参议员发言的话,他只能是民主党的参议员,第二句讲发言的人要么是民主党员 ,
要么是参议员。根据第一句,否定掉参议员,只剩民主党了。

2
M:Just wrote a lot and got deleted. So I will be short this time.

There are two things you need to do in CR: understanding and figuring out line of reasoning. The first is about
knowing what each sentence or word means. The second is about knowing what connections are among those
sentences or words.

I think you can easily understand the words and sentences. So the key here is the reasoning. And the key of that is
the key word.

A unless B ==> A --> neg B. I will go unless you go ==> I will go --> you do not go (or you go --> I will not go).

Not A unless B ==> A --> B (or neg B --> neg A). Note that it is a bit different from above.

No ... both ==> either ... or... Democrat either spoke or was a senator.

Now you should put them together and figure out the connection among all the elements. The answer is pretty
clear.

i am mindfree nn' fans


Pls allow me to show something interesting :)

L1 No senator spoke at the convention unless he or she was Democrat.


-(Senator + spoke) =-Democrat
----->If a senator spoke at the convention, he or she must be a Democrat.
------>Senator + spoke = Democrat
L2 No Democrat both spoke at the convention and was a senator.
-(Democrat + spoke) =Senator
----->ALL Democrat spoke at the convention was not a senator.
----->Democrat + spoke= -Senator

推导
Senator + spoke =Democrat
Democrat + spoke =-Senator
-------------------------------------------------------
Senator +spoke +spoke = -Sentor
2Senator=-2spoke
spoke=-Senator
senator=not + spoke

A.Spoke=Senator
B.Spoke=-Democrat
(迷惑选项)C.Only Democrats spoke at the convention.
从仅有的以知条件中推不出 Democrats=spoke 来.

3
D.Spoke=-Senator(答案)
E.Democratsenator=Spoke(无关)

7. If Sarah were a concert pianist for a major orchestra, she would be famous. She is not
a concert pianist since she is not famous.

The conclusion above is unsound because the author does not consider that

(A) Sarah could be a famous actress.

(B) Sarah could be a harpist for a major orchestra.

(C) Sarah could be a pianist with a rock group.

(D) Sarah could be a concert pianist with a minor orchestra.

(E) Sarah could be famous for another reason.

答案:D

我认为是范围差异。题干说了因为 a concert pianist for a major orchestra---famous, 所以不 famous---not a


concert pianist。这是推不出的。就像答案所说,可以是 minor orchestra 的。

9. Free public education is the best form of education there is. Therefore, we must fight to
ensure its continued existence; that is, we must be ready to defend the principle of
equality of educational opportunity. Because this principle is we worth defending, it is
clear that free public education is better than any other form of education.

Which one of the following illustrates the same weak reasoning as found in the passage?

(A) I love music, and that’s why I listen to it constantly. I have my stereo or radio on every
waking minute. Since I play music all the time, I must really love it.

(B) Books are my most valuable possessions. My books are like my friends—each
pleases me in different ways. Just as I would give up everything to save my friends, so
too with my books.

(C) I would much rather be poor and respected than be rich and despised. To have the
respect of others is far more valuable than to have millions of dollars.

(D) I have never been betrayed by any of my friends. They have been true to me through
good times and bad. Therefore I will never betray any of my friends.

4
(E) Because every plant I have ever seen has green leaves, I have concluded that all
plants must have green leaves. This looks like a plant but it does not have green leaves,
so it cannot be a plant.
循环论证到底是个什么东东啊!!其他的选项还有什么论证错误吗??我怎么一点都看不
出来呢?
答案:B

是循环论证,就是需要用结论来证明论据和论点.换言之,前提和结论互为前提。

10. Some people say that the scarcity of food is a function of the finite limits of the earth’s
resources, coupled with a relentless rate of population growth. This analysis fails to
recognize, however, that much of t he world’ s agricultural resources are used to feed
livestock instead of people. In the United States, for example, almost one-half of the
agricultural acreage is devoted to crops fed to livestock. A steer reduces twenty-one
pounds of inexpensive grain to one pound of expensive meat. Thus, the scarcity of food
is not merely a function of limited resources and population growth.
Which one of the following is an assumption that would allow the conclusion in the
argument to be properly drawn ?
(A) People prefer eating meat to eating grain.
(B) Meat is twenty-one times more expensive than grain.
(C) The limits of the earth’s agricultural resources are not finite.
(D) More than one-half of the agricultural acreage in the United States is devoted to
crops fed to humans.
(E) Growing crops for human consumption of the acreage currently devoted to crops for
livestock will yield more food for more people.

答案:E

因为粮食还可以用于家畜的喂养,
结论:粮食的缺少不仅仅代表资源和人类成长的减少

我认为其中的隐含条件是如果将应用于人的同等粮食用于家畜所产生的效益,对人类的成长更有益,那么
粮食的减少就不仅仅代表资源和人类成长的减少
(感觉就是在翻译题目)

如果按照 CXD 的说法,这应该算假设中的在 A 与 B 之间建立本质联系

可能做逻辑题的方法之一就是找 A 与 B 之间的关系与差异,似乎有一部分题是这样出的
还有如何在烦杂的选项中,比较挑选出你的答案,非常考验专心与耐心

以上解释如有错漏,望大家指出,小弟也只刚入门

5
为什么说食物的稀 QUE 不仅仅是由于有限的资源和人口的增长,是因为现在有部分的土地被用来种植牲
畜的食物了。如果把这部分土地拿来用人吃的粮食,那么土地资源就不是那么地稀 QUE 了,也就是还涉及
到所有土地在人用粮食种植与牲畜用粮食种植的分配问题。本题的关键解题句在:……are used to feed
livestock instead of people.

没有那么复杂,只是理解它的意思就可以了,以不变应万变,考试的时候哪有时间去给你这样想啊
反正是给牲畜吃的粮食如果给人吃的话可以比吃肉养活更多的人,所以养动物是浪费。想简单一些吧,简
简单单才是真啊。呵呵,陋见不要见笑

make it symple
题干:有很多的土地种植的粮食现在用于喂猪 =〉不用担心粮食缺乏
读 完 题 干 ,脑 中 有 了 隐 含 的 假 设 : 那些土地种的东 西 能给人 吃(我觉得这是题干 跳过的一个 mid-
conclusion)
看选项,got it. E. 用 not E 验证一下,没错。

if food is not merely the function of population and resource, say, comsumption of crop by liverstock also taken as
a factor, food = f(resource, population,liverstock).
the definition of food with 3 factors need to be defended. and we defend is with choice E, claiming that "Growing
crops for human consumption on the acreage currently devoted to crops for livestock will yield more food for
more people".

for you amusement.


but also what I am thinking when encountering this question.

请教 LSAT-1-1-11,12
11. Hanifah: A recent survey shows that there are fewer people who drive only on
weekends than there are people who drive to work each weekday. As a result, weekend-
only drives are involved in fewer accidents. Therefore, insurance rates should be
adjusted so that rates would be significantly higher for the regular commuters.

Katsu: I can’t agree with your conclusion. The same study also showed that, although
weekend-only drives are involved in fewer accidents, when considered on the basis of
accidents-per-mile-driven their records are worse than those of regular commuters.
Therefore, insurance rates should be adjusted to increase the rates of weekend-only
drivers over those of regular commuters.

In the conversation above, Katus does which one of the following?

(A) Katsu disagrees with each of the premises of the argument that Hanifah offers.

(B) Katsu cites additional evidence stating that weekend-only drivers are actually

6
involved in a greater number of accidents than regular commuters.

(C) Katsu accuses Hanifah of using inaccurate statistical information.

(D) Kasu proves that Hanifah didn’t read the entire report that was cited.

(E) Katsu disagrees with Hanifah over how accident records are to be evaluated for
insurance rates.

答案:E,又是一个对话题,请问 A 为什么不对呢?

12. If Country X does not intervene militarily in Country Y, then the whole region will
definitely fall under enemy influence.

It most logically follows from the statement above that, if Country X does intervene
militarily in Country Y, then the whole region.

(A) Will definitely fall under enemy influence

(B) Will probably fall under enemy influence

(C) Will probably not fall under enemy influence

(D) Will definitely not fall under enemy influence

(E) May or may not fall under enemy influence

答案:E,请教做题的思路。

请问 C、E 有什么区别吗?

probably 表示很有可能的意思. 程度上超过 possible. 你可以这么理解, 题目告诉你一件事可能发生,也可能不


发生, 你不能认为一种可能要大于另一种可能.

我记得有一道题与此题类似. 是说某些人中一部分如何如何. 一个选项是 most of ...就是错的. 因为原文并没


有讲程度. 而此题的 probably 就相当于 most.

Probably is stronger than possible or likely. If something is probably, there should be around 80% chance that it
will happen. So C is not supported by the reasoning. Basically, we have no idea on the likelihood of falling under
or not falling under. Either is possible but we do not know which is more likely.

7
13. Top college graduates are having more difficulty demonstrating their superiority to
prospective employers than did the top students of twenty years ago when an honors
degree was distinction enough. Today’s employers are less impressed with the honors
degree. Twenty years ago no more than 10 percent of a given class graduated with
honors. Today, however, because of grade inflation, the honors degree goes to more than
50 percent of a graduating class, Therefore, to restore confidence in the degrees they
award, colleges must take steps to control grade inflation.

Which one of the following is an assumption that, if true, would support the conclusion in
the passage?

(A) Today’s students are not higher achievers than the students of twenty years ago.

(B) Awarding too many honors degrees causes colleges to inflate grades.

(C) Today’s employers rely on honors ranking in making their hiring decisions.

(D) It is not easy for students with low grades to obtain jobs.

(E) Colleges must make employers aware of the criteria used to determine who receives
an honors degree.

现在的“牛人”比以前的“牛人”更难以向顾主证明其卓越,因为过去的“牛人”凭“牛人”的等
级就可以了,而现在的顾主对“牛人”的等级不太感冒了。原因是现在的学生中得到“牛人”
等级的很多,所以学校一定要把得到“牛人”等级的人数控制一下。如果,现在的学生果然优
秀,那么更多的人得到“牛人”就是应该的,而文中认为不应该给这么多“牛人”,那假设一
定是,本来不应该有这么多牛人的,所以 A 是假设。

14. Either Perry’s faction or Tucker’s faction, but not both, will win control of the
government. If Perry’s faction wins, the nation will suffer economically. If Tucker’s faction
wins, the nation will suffer militarily.

Given the statements in the passage, which one of the following statements must be
true?

(A) It is possible, but not certain, that the nation will neither suffer economically nor suffer
militarily.

(B) If the nation suffers economically, it is certain that Perry’s faction has won control of
the government.

(C) It is certain that the nation will suffer either economically or militarily, and also certain
that it will not suffer both.

8
(D) If the nation suffers militarily, it is possible, but not certain, that Tucker’s faction has
won control of the government.

(E) If the nation suffers both economically and militarily, it is certain that neither Perry’s
faction nor Tucker’s has won control of the government.

由原文,可得 1。Perry’s faction wins——》the nation will suffer economically


2。Tucker’s faction wins——》the nation will suffer militarily
3 。 Either Perry’s faction or Tucker’s faction, but not both, will win control of the
government 可推出 A 错
B 的逻辑关系为 the nation will suffer economically——》Perry’s faction wins,无法从原文
的出,所以错
同理。可推出其他。
D 强调了 it is possible, but not certain。所以正确。

原命题为:A-->B
逆命题为:B-->A
否命题为:非A-->非B
逆否命题为:非B-->非A

原命题和逆否命题为等价命题.如果原命题成立,逆否命题成立.vice versa.
逆命题和否命题为等价命题,如果逆命题成立,否命题成立.vice versa.

举例:
原命题:吃多了-->肚子涨
逆命题:肚子涨-->吃多了
否命题:没吃多--> 肚子不涨
逆否命题:肚子不涨-->没吃多

原命题是正确的.所以逆否成立.
但是逆命题和否命题不一定成立.肚子涨也许是水喝多了.

16. Investigators concluded that human failure was not responsible for the fatal airplane
crash last August, and since that time new and more stringent rules for identifying and
reporting mechanical problems have been in effect. That accounts for the fact that
reports of airplane mechanical problems have increased in frequency by 50 percent since
last August.

Which one of the following is an assumption underlying the argument in the passage?

9
(A) Airplane travel is still relatively safe, despite the increase in reported mechanical
problems.

(B) Mechanical problems in airplanes have increased dramatically since last August.

(C) Mechanical problems in airplanes have not increased by 50 percent since last
August.

(D) Airlines are less reluctant to report mechanical problems than they previously were.

(E) Mechanical problems in airplanes have become easier to detect since last August.

answer:C not+weaken

典 型的 排除他因. 原文 认为 是严 格的 新规章使得 mechanical problem 被更多的检测出来. 言下之意是


mechanical problem 本身并没有同幅增长. 而 C 就是这个意思, 既不是因为 mechanical problem 本身增加了
50%造成的. 这类题目的题干为:A-->B, assumption: 不是 C-->B

1-1-17
The peculiar evil of silencing the expression of an opinion is that it robs the human race.
It takes from posterity, as well as the existing generation, and from those who dissent
from the opinion even more than from those who hold it. If the opinion is right, they are
deprived of the opportunity of exchanging error for truth; if it is wrong, they lose what is
almost as great a benefit: the clearer perception and livelier impression of truth, produced
by its collision with error.
Which one of the following best expresses the conclusion presented in the argument?
A. Silencing the expression of an opinion is robbing the human race.
B. Silencing the expression of an opinion harms those who dissent more than those who
agree.
C. Anyone who agrees with an opinion would not want to silence its expression.
D. Gaining a clearer perception and livelier impression of truth is a great benefit.
E. The greatest benefit is the opportunity of exchanging truth for error.
take from
减去; 减损; 降低
The answer is A.
A 实际上就是题干第一句话,为什么是 conclusion 呢?这个题目的解题思路是什么呢?
原文开头第一句话,就是结论,原文其他的话是围绕这一结论展开的。A 是这句话的
rephrase. 这种类型的题目,就是通过一堆废话来把 examinee 来绕晕,忘了原来的结论是
什么了。D 错的原因: D 存在的前提是 IF IT IS WRONG,但 D 中没说.IF IT IS RIGHT,D 选项
便错了

19. Some good cooks are gourmet cooks who pride themselves on always using

10
extravagantly rich ingredients in elaborate recipes. Some good cooks can be
characterized as fast-food cooks. They may use rich ingredients as long as the recipes
are easy to follow and take little time. Other good cooks are health food enthusiasts, who
are concerned primarily with the nutritional value of food. But even though not all good
cooks are big eaters, they all enjoy preparing and serving food.

If the information in the passage is true, which one of the following CANNOT be true?

(A) Most good cooks do not use extravagantly rich ingredients.

(B) Everyone who enjoys preparing and serving food is a good cook.

(C) More good cooks who use extravagantly rich ingredients are big eaters than are good
cooks who do not use such ingredients.

(D) There are fewer good cooks who enjoy serving and preparing food than there are
good cooks who are big eaters.

(E) Gourmet cooks, fast-food cooks, and cooks who are health food enthusiasts are all
big eaters.
答案是 D
我的问题:我认为 D 和 E 都对啊!
D 的原因是不是:the number of good cooks who enjoy serving and preparing food >=the
number of good cooks who are big eaters。对不?
但是 E 也不对啊!文中不是说:not all good cooks are big eaters.E 中不是说 are all big
eaters 吗!不正好相反吗?那为什么不选 E 呢??(抓耳挠腮的搞不懂,呵呵)

一些 Some good cooks 是 gourmet ,一些 good cooks 是 fast-food cooks,最后一些是


health food

enthusiasts。
But even though not all good cooks 是 BIG EATER,但所有的 they all enjoy preparing
and serving food.

A B C E 皆为可能对也可能不对,D 必错:因为 good cooks 并不 100%是 BIG EATER,


但 100%是 enjoy

preparing and serving food.换句话说 good cook 中 enjoy preparing and serving food 一定
大于等于(不小

于)BIG EATER。
请 注 意 E 的 表 述 : 文 章 讲 的 是 good cook 是 gourment,fast,health , E 是 在 讲
gourment,fast,health 而不是

11
good cook 与文章完全没有关系,所以也是可能对。

LSAT-1-1-21
If the city council institutes new parking regulations, city revenues will surely increase,
since studies have conclusively shown that, if such parking regulations are put into effect,
there is an increase in parking violations, and an increase in parking violations will result
in a greater number of parking fines collected.

21. Which one of the following is closest, in terms of its logical features, to the reasoning
used in the argument above?

(A) Last year’s increase in revenues can be easily explained. That was the year the city
council instituted new parking regulations. No doubt the new law brought with it an
increase in the number of parking violations.

(B) If taxes were increased, this act would naturally result in increased revenues for the
city, and increased revenues would make some desirable social programs possible. So, if
taxes were increased, some desirable social programs would become possible.

(C) Henry says that, if the city council goes into closed session, an important matter of
personnel policy is being discussed. However, no personnel matters were discussed at
the council meeting, so if Henry is right, the council did not go into closed session.

(D) All cars parked on the north side of the street were ticketed last night, and the same
cars were towed away this morning. So beware a car ticketed in this city also gets towed
away.

(E) Allen says that, if the city council institutes new parking regulations, it is unlikely that
revenues for the city will increase. If Allen is right, then the parking regulation plan should
not be instituted.

请问本题的 logic feature 是什么? 这样的题怎么入手做? 我找不到门道阿!

本题的 logic feature 是: 在"前提"和"结论"间通过因果关系来"搭桥".

22. If the statements in the passage are true, which one of the following must also be
true?

(A) Unless there is an increase in the number of parking violations in the city, city
revenues will not increase.

12
(B) If the city council institutes new parking regulations, the council will fall from favor with
the citizens.

(C) The city council will institute new parking regulations only if an increase in city
revenues can be expected to result.

(D) If the city council’s new regulations cause more parking violators to be ticketed, the
city revenues will increase.

(E) Unless the city institutes a complex system of parking regulations, the city cannot
expect traffic violations to increase.
答案是:D
我的问题:为什么 C 不对啊!
文中的关系不应该是这个样子的吗:regulations put into effect-->increase in violations---
>greater number of parking fines--->revenues increase.(应该是这个样子吧!)
而 C 不就是 regulations put into effect-->revenues increase 吗?为什么不对!哪里错了啊!

22 题的结构是“如果新的停车规则实施,则会导致财政收入增加。理由是,新规则会导致违
规的增多,违规的增多会导致罚款增多,(进而增加财政收入)。”
22 题的问实际上是问必要条件,C 的意思是只有会增加财政收入才会实施新的泊车规则,
这个说法已经超出了文中所给出的逻辑问题的范畴,文中只是解释了新泊车规则和财政收
入的正向关系,并没有讲新规则的实施是以增加财政收入为前提的。而 D 正好可以衔接罚
款增多和财政收入增多的关系。

23. The function of government is to satisfy the genuine wants of the masses, and
government cannot satisfy those wants unless it is informed about what those wants are.
Freedom of speech ensures that such information will reach the ears of government
officials. Therefore, freedom of speech is indispensable for a healthy state.

Which one of the following, if true, would NOT undermine the conclusion of the
argument?

(A) People most often do not know what they genuinely want.

(B) Freedom of speech tends ultimately to undermine social order, and social order is a
prerequisite for satisfying the wants of the masses.

(C) The proper function of government is not to satisfy wants, but to provide equality of
opportunity.

(D) Freedom of speech is not sufficient for satisfying the wants of the masses: social
order is necessary as well.

13
(E) Rulers already know what the people want.
答案是:D
我的问题是:削弱的题用不用搞清楚文中每句话之间的逻辑关系!我现在对这道题的关系只
能看到这个程度:government satisfy wants--->know about what those wants are.(这是我
能 看 出 来 的 ) Freedom of speech ensures that such information will reach the ears of
government officials.(看到这逻辑关系就乱了!不知道这个 freedom of speech 该放在哪里?
是 个 什 么 条 件 啊 ) 结 论 : freedom of speech is indispensable for a healthy state. 这 个
freedom of speech 应该是 a healthy state 的必要条件???(总之一句话逻辑关系太乱了,
以至于看选项的时候没有个方向,完全凭感觉!)

为什么 D 对,如果搞不清其逻辑关系也没关系啊。咱还有秘密武器:排除法。让我们看一下:

A 说人们不知道他们想要什么,既然连要什么都不知道,那么让他们去 speech,他们也说
不出什么东东(需要),政府就更不知道了。所以 weaken。

B 说 social order 是必须的,freedom of speech 会损害到 social order,那么绝对不能给


freedom of speech,所以 B 也是 weaken。

C 说政府的职能是保证机会均等(不是满足人们的需要),那么也就不要人们发表什么高
见(speech),所以也是 weaken.

E 说当权者已经知道了人们想要什么,那么也就不需要人们说出他们想要的了,所以也是
weaken.

呵呵,只剩 D 了,D 说 freedom of speech 对于潢足人们的需要是必需的,但光有它还不


够(还得有 social order,其实这里有没有或者是不是 social order 都不会影响结果)。

文中说 FREEDOM OF SPEECH 是不可少的,只是必要条件,所以补充另一个必要条件不


会有 UNDERMINE 的效果

24. An unbiased observer of everyday encounters in Western societies would surely not
find many instances of unkindness by people under 65 toward people over 65. There are
undoubtedly incidents of unkindness based on age, and these warrant reproof. However,
the very fact that such reproof occurs and is generally accepted implies that our Western
societies basically respect the elderly. The same conclusion can be drawn from a recent
survey finding: 71 percent of the under 65 population agreed with the statement that
“people over 65 receive too little respect from society”, while only 44percent of the over-
65 population, the target of the alleged irreverence agreed with it.

The author concludes that Western societies basically respect the elderly partly because.

14
(A) people under 65 are just as kind to people over 65 as they are to people of their own
age group

(B) survey data suggest that fewer people over 65 than under 65 get too little respect

(C) disrespect for the elderly does not go so far as to result in actual harm

(D) survey data suggest that people over 65 are more aware of incidents involving
disrespect to the elderly than are people under 65

(E) incidents of unkindness to the elderly are neither common nor generally accepted in
Western societies

答案:E

B 在原文并没有讨论. survey 谈到的是 perception, 而且并没有涉及对 65 岁以下人的尊敬的问题. 你对 survey


的内容在好好理解一下:71%的 65 岁以下的人同意"社会对 65 岁以上的人缺乏尊敬", 而只有 44%的 65 岁以
上的人同意这个说法.

15
LSAT-1-4

7. A recent survey showed that many workers in a certain company are dissatisfied with
their jobs. The survey also showed that most of the dissatisfied workers believe that they
have little control over their job assignments. Therefore, to increase workers job
satisfaction the company’s management need only concentrate on changing workers’
beliefs regarding the degree of control they have over their job assignments.

Which one of the following, if also shown by the survey, would most seriously call into
question the conclusion made by the author of the passage?

(A) The dissatisfied workers feel that their wages are too low and working conditions are
unsatisfactory.

(B) The number of workers in the company who are satisfied with their jobs is greater
than the number who are dissatisfied.

(C) The workers in the company are more dissatisfied than workers in other companies

(D) Most people in company management believe that the workers already have too
much control over their work.

(E) The workers in the company who are satisfied with their jobs believe that they have a
lot of control over their job assignments.

我觉得 A 与此题目不相关啊,D 为什么不对呢

这道题问:如果 survey 中出现何结果,可以 weaken 此题的结论?

16
首先要清楚题干的结论是 Therefore 之后的话,是如何提高工人的 job satisfaction?结论
是:only focus on changing workers' belief regarding the degree of control。

所以要 weaken 就有两个方向:一个直接说 increase degree of control 不行;另一个间接


说除了 degree of control 的问题之外还有其他问题,言下之意说"only focus on degree of
control"其实也不能提高 job satisfaction。正确选项 A 说的就是后者:就是工资和工作环境
也都不行,意即 weaken 了结论。

D 选项脱离了结论,而且“经理层认为有 control”不代表真的有,没有 weaken 结论的作用。


一般选项里含有“believe/think"等主观词的选项几乎不可能为正确选项,因为不代表客观。

8. Dr. Sheila Porter plans to run an experiment using nursing students. Each student will
be shown either a pleasant nature film or a disturbing horror film. Each student will be
observed by someone who—looking only at the student’s facial expressions—must
ascertain which film is being shown. Students shown the horror movie are told to hide
their feelings in order to convince the observer that they are watching a pleasant film. Dr.
Porter hypothesizes that all the students in the experiment who are convincing will be
among the best at working with patients. The hypothesis will be tested by comparing the
convincing students and unconvincing students in terms of their performance with
patents.

Which one of the following incidents best illustrates Dr. Porter’s hypothesis?

(A) Niles, the most convincing student in the experiment, later went on to become a
physician.

(B) After graduating, Yoshiro, a nursing student who was convincing in the experiment,
helped care for Bram, a patient at a hospital. Bram recovered from his operation.

(C) After graduating, Kim, a nursing student in the experiment who watched the nature
film, was removed from the staff of a hospital for unacceptable performance in patient
care.

(D) Daria, a nursing student who was convincing in the experiment, later received “A’s” in
those classes in which working with patients in a teaching hospital was the sole basis of
her grades.

(E) Marite, a nursing student who was not convincing in the experiment, later quit nursing
school.

17
答案: D,不知道该题的出题点,请指教。

首先第一题要在阅读时注意提炼出要点:本题提干的假定是: convincing--- 〉 will be


among “the best'' at" working with patients" .只有选项 D 同时体现了这两者。
1, 8 题
本题的原文:Dr.P 设计一个实验来检测护士生。让这些护士生看 pleasure 电影 or 恐怖电影。
并且让那些看了恐怖电影的护士生在别人面前努力隐藏自己的感受,以便让别人相信他看
的不是恐怖电影而是 pleasure 电影。Dr.P 说那些能够很好隐藏自己的感受的护士生也能够
among the best at working with patient.
问:下面那个例子最能够说明 Dr.P 的假设?
B:Y 学生在实验中表现出色,毕业后 Y helped care patient Bram, Bram 后来在他的护理
下康复了。这里只讲了 Y 在毕业后护理病人,并没有讲他是否 working with patient best.
D: D 学生在实验中表现出色,后来 D 在那些仅将 working with patents 作为评分标准的课
程中得了“A”。也就是说 D 在实验中表现出色,后来他也能够 working with patient best.

9. Those who think with a hierarchical mentality strive for situations in which their side is
dominant and the other side is submissive. In contrast, communal thinkers strive for
parity among all sides. Therefore, achieving parity of nuclear weaponry between the East
and the West is not enough for Western military generals.

Which one of the following assumptions would provide the most support for the
conclusion above?

(A) Western military generals do not have the same mentality as do Eastern military
generals.

(B) Parity in nuclear weaponry requires that military generals from both the East and the
West think in communal terms.

(C) Western military generals want parity with respect to strength in nuclear weaponry
between the East and the West.

(D) Western military generals’ thinking about relative strength in nuclear weaponry is
hierarchical.

(E) The thinking of military generals with respect to relative strength in nuclear weaponry
is either hierarchical or communal.

这个题目偶也看不明白是啥意思啊,大牛们帮俺翻译一下

18
1-4-10
It has always been difficult to understand the basis of politics in the People’s Republic of
China. Because the system is effectively closed, it is impossible to know with any degree
of confidence who is allied with whom and for what reasons. Yet Chinese politics does
exhibit many of the external characteristics of factional political systems, as found in
more open societies. It is legitimate to conclude, therefore, that China has a factional
political system.
A. All open political systems are factional political systems.
B. All factional political systems are closed political systems.
C. All closed political systems are factional political systems.
D. China’s political system is more open than many existing factional political systems.
E. China’s political system is more closed than all existing factional political systems.
The answer is C. Why?

应该是一个支持题或假设题,也只有 C 能推出中国政治体系是一个派系的政治体系

11. Since no one returns from death, we can never be certain about what passes through
the mind of the dying person. For the unconscious, the confused, and the heavily
sedated, these final moments are probably meaningless. However, for the mentally alert,
it is quite possible that death presents itself as an unbelievably glorious experience, a
flight into an entirely new universe of sensation. Why should we think so? Some people
who have been reprieved from “certain” death at the last moment have experienced what
goes through the consciousness of those who are not so fortunate. For example,
parachutists who have survived falls report experiences that resemble psychedelic “trips.”

The primary point of the argument in the passage is

(A) no one returns from death

(B) dying can be a glorious experience

(C) we can never know what passes through the mind of a dying person

(D) some people are reprieved from death at the last moment

(E) some people “die”, yet live to report their, experiences.

这个题目我在 B/E 之间犹豫了很久,结果还是选错了.答案是 B,我选的是 E.我觉得原文的论


证中对 E 的描述是比较正确的.对 B 而言很片面.请指点??

问题问的是原文的 primary point。
这段文字的 primary point 是 however 后的内容,B 正确。而 E 只讲了原文中提到的一个事

19
实。

12. Aristotle wrote that a tyrant would be well advised to put on the appearance of
uncommon devotion to religion. Subjects are more tolerant of unjust treatment from a
ruler whom they consider god-rearing and pious. Moreover as most subjects believer that
even the gods are on the side of the ruler, the subjects are less apt to move against him.

Which one of the following is an assumption on which Aristotle’s argument depends?

(A) The subjects of tyrannical rulers typically believe that there is a power other than the
mortal.

(B) A tyrant cannot rule unless he has divine power on his side.

(C) The subjects of tyrannical rulers can rarely be fooled by appearances.

(D) Tyrants who are devoted to religion will not treat their subjects unjustly

(E) For a tyrant, the appearance of uncommon devotion to religion is a more effective
means of ruling than unjust treatment

答案:A, A 有点是 GAP 的假设,请问 B 为什么不行?


当问假设题时,有两个选项拿不定,该怎么办?取非好象不是太好用。比如 B 取非:
暴君可以统治,除非有神的保佐,怪怪的。我经常用这种办法拿不定。

B 还是充分必要条件的问题. 原文指出 tyrant 可以用 XXX 方法统治,说明这种方法是充分条


件. B 把这中方法变成了必要条件,必须用这种方法统治才行. 你看到这种答案要在原文中找
逻辑关系, 有没有提到非 XXX 不可. 我以前举的例子:某制度保证福利,所以必须有某制度才
能保证福利. 这里就有 A-->B,所以 B-->A 的逻辑关系. 在此文中不存在这种关系, 所以 B 错.

14. All those who keep a journal will be heard by the next generation. Some of these
journal writers are true artists, others humorous observers of the commonplace, and still
others insufferable egotists who feel compelled to record their every thought.

If the statements above are true, which one of the following must be true?

(A) Not all of those who are humorous observers of the commonplace will be heard by
the next generation.

(B) Everyone who will be heard by the next generation keeps a journal.

(C) The next generation will hear both insufferable egotists and true artists.

20
(D) Some of those who keep journals are not true artists, humorous observers of the
commonplace, or insufferable egotists.

(E) The next generation will bear some of those who are true artists but not all of them.

答案:C,这种  If the statements above are true, which one of the following must be
true?题,我有做有错,有什么好办法吗?

要想:如果 all humorous observers of the commonplace 都记日志, A 就对.所以 A 可对可错.

B.你要想记日志 -->will be heard, 记日志只是一个充分条件 , 其它事也可以导致 will be


heard, 所以可对可错.
C.对.两种人都有记日志的, 而记日志一定导致 will be heard, 可以正推出来.
D.和原文相反.
E.如果 true artists 都记日志,那么 E 就错.

只要你所想的反例不违背原文的意思, 无论多 absurd,都可以用.

17. If the artificial is not better than the natural, to what end are all the arts of life? To dig,
to plow, to build, to wear clothes—all are direct violations of the injunction to follow
nature.

Which one of the following is an assumption made by the author of the passage?

(A) The arts of life have no useful end.

(B) The artificial is not better than the natural.

(C) Digging, plowing, building, and wearing clothes are better than nature.

(D) The injunction to follow nature should not be violated.

(E) The arts of life are indirect means of following nature

答案:C,我不明白这题的结论是什么?

1. 原文的论述中有个 gap,也可以看成是概念的跳跃. 原文的结论是以问题的形式提出来的,


既作者认为 arts of life 证明 artificial can be better than natural. 逻辑关系是:digging...都是
artificial, 所以 arts of life 中, artificial can be better than natural. 这里的 gap 是:digging 等
一定要 better than natural, 才能做为论据. 我再举个例子说明这类题型:都说一班的学生比
二班的聪明,可是张学友怎么就比一班的王胖子聪明呢?assumption 就是张学友是二班的.这
个 gap 不填上就没有结论了.

21
18. If the author’s argument were challenged on the grounds that the construction of
buildings has adverse effects on the natural environment, which of the following replies
might the author use to respond to the challenge logically?
(A) There are human activities, such as making music, that are environmentally
harmless.
(B) Harming the environment is not an end, of purpose, of the arts of life.
(C) The construction could involve the use of natural, not artificial, materials.
(D) Constructing buildings is not an “art of life.”
(E) Even if the natural environment is disturbed by the construction of buildings, it is
improved for human

19. There are at least three people in the room. At most two people in the room recognize
each other. At least one person in the room recognizes everybody else in the room.
Which one of the following is NOT consistent with the above?
(A) Four people are in the room
(B) No two people in the room recognize each other.
(C) At most one person in the room recognizes everybody else in the room.
(D) Anyone in the room who recognizes any other person in the room is also recognized
by that person..
(E) Two people in the room recognize every one else in the room.
answer:D my choice:C

First of all, this is more of a logic game time of question in LSAT. You really do not need
to pay attention to it as GMAT does not use this type of question.

D is not consistent. If there are at least three people in the room, say three: x,y,z. x
recognizes y and z. With choice D, y and z should recognize x too. Then it violate "At
most two people in the room recognize each other".

20. Abolish taxes, and real taxpayers would find that their disposable incomes have
increased. Abolish taxes, and public employees would find that their incomes have
disappeared.
Which one of the following is a logical conclusion that depends on information in both of
the statements above?
(A) Public offices should be abolished so that disposable incomes will rise.
(B) The only real taxpayers are those who would have more to spend if they did not pay
taxes.
(C) Public employees are not real taxpayers.
(D) Public employees’ incomes should not be taxed since they come from taxes.

22
(E) If there were no taxes, then public employees could not be paid.
Answer: C 我知道对。。可是我觉得 E 好象也对。

因 为 , Abolish taxes, and public employees would find that their incomes have
disappeared.
abolish taxes= no taxes, disppeared= could not be paid
谢谢!

21. A low-pressure weather system is approaching Plainville; rainfall results from about
70 percent of such systems in the Plainville area. Moreover, the current season, spring, is
the time of year in which thundershowers, which sometimes result from low-pressure
systems, are most likely to occur in Plainville.
Knowing which one of the following, in addition to the information above, would be most
useful for determining the probability that Plainville will have a thundershower soon?
(A) the percentage of thundershowers in Plainville that occur in the spring
(B) the percentage of spring rainfalls in Plainville that are thundershowers
(C) the percentage of thundershowers in Plainville that result from low-pressure systems
(D) whether low-pressure systems in other areas are associated with rainfall
(E) whether Plainville has more or fewer thundershowers each spring than do near by
towns

Why is the correct B (where there is no mentioning of 'low-pressure system' thinng)


instead of C?

You did not truly understand the question. Very often that ETS tests your understanding
by giving a look-like choice.

Do the math here. We know that 70% chance the low pressure will cause rain fall.

B: We know the chance that rain fall will be thundershower. Let's assume it is 20%. Then
the chance that the low pressure will cause thudershower is 70%*20%=14%

C: is irrelevant. You picked C because you either did not think too hard or you did not
understand what C means. If we know that 50% of thundershowers are caused by low
pressure, we still do not know the chance that low pressure will cause thundershowers.

1-4-22

23
It is illogical to infer a second and different effect from a cause which is known only by
one particular effect. This is incorrect because the inferred effect must necessarily be
produced by some different characteristic of the cause than is the observed effect, which
already serves entirely to describe the cause.
Which one of the following arguments makes the same logical error as the one described
by the author in the passage?
(A) An anonymous donor gave a thousand dollars to our historical society. I would guess
that that individual also volunteers at the children’s hospital.
(B) The radioactive material caused a genetic mutation, which, in turn, caused the birth
defect. Therefore, the radioactive material caused the birth defect.
(C) The tiny, unseen atom is the source of immense power. It must be its highly complex
structure that produces this power.
(D) The city orchestra received more funds from the local government this year than ever
before. Clearly this administration is more civic-minded than previous ones.
(E) If I heat water, which is a liquid, it evaporates. If I heat hundreds of other liquids like
water, they evaporate. Therefore, if I heat any liquid like water, it will evaporate
答案是 A,
我对题干的理解是充分条件与必要条件搞混了。充分条件可以推出好几个结论,不一定是只
有一个。题干中讲一种原因会推出一种特别的结论,而这种结论可以完全解释这个原因,所
以从这种原因中再推出别的结论就不对了。其实,结果对于原因只是必要性而已,而原因对
结果是充分性的,所以错误在于忽略了一个原因可以导致若干个结果,每个结果只是必要
而已,即把必要性当做充分性了。我觉得 5 个选项都不对。答案 A 如何解释?
P27 T22.
这道题的题干首先说明,我们通过了解一个特定的现象(或叫影响)所认识一个事物,对
这个事物推断出第二个并且是不同的效果是不合逻辑的。
然后进一步阐述了这为什么是错误的。是因为,我们要想推出来另一个不同现象,必须是从
这个事物的一些不同的特点出发得到的,不能是已经观察到的现象。而且我们对这事物的认
识完全是基于通过这个以观察到的现象进行的描述。

简而言之,就是,我们所了解的这个事物的全部就是一个特定的现象,没法在这个基础上
得到第二个不同的现象
In A, the second effect is "volunteers at the children’s hospital". The particualr effect and
pbserved effect is "gave a thousand dollars to our historical society". The error described
in the passage is that it is wrong to conclude that another different effect is derived from
the same cause of an observed effect, which is the only effect of that cause.

In A, it is wrong to conclude that volunteer is the effect of the same cause that make the
person donate money. It should be from another cause. Here the cause is not specified.

23. Just as a bicycle chain may be too tight, so may one’s carefulness and
conscientiousness be so tense as to hinder the running of one’s mind.

24
Which one of the following most closely parallels the reasoning used in the argument
above?

(A) Just as a clock may be wound too tightly, so may one’s time be spent fruitlessly in the
pursuit of perfection

(B) Just as a carousel may spin too quickly, so may one’s rapid concentration on several
problems prevent a resolution of difficulties.

(C) Just as a machine may be oiled too much, so may one’s heavy drinking of alcoholic
beverages lead to complete dissipation

(D) Just as a raging river may be frozen into stillness during the winter, so may one’s
career falter at certain times of the year

(E) Just as a boxer may become too tense before a big fight, so may one’s personal
concerns stand in the way of professional success

答案:b 题中 too tight—be so tense,b 答案中 too quickly—rapid

25. All of the best comedians have had unhappy childhoods. Yet, many people who have
had happy childhoods are good comedians, and some good comedians who have had
miserably unhappy childhoods are happy adults.

If the statements in the passage are true, which one of the following CANNOT be true?

(A) The proportion of good comedians who had unhappy childhoods is greater than the
proportion of the best comedians who did.

(B) Some good comedians have had unhappy childhoods and are unhappy adults.

(C) Most of the best comedians are happy adults.

(D) More good comedians have had unhappy childhoods than have had happy
childhoods.

(E) The proportion of comedians who are happy adults is higher than the proportion who
are unhappy adults.

这个题目答案是 A.我理解的不清楚.选项 C/D/E,为什么是正确的呢?

原文推理如下:

25
(1) best comedians → unhappy childhoods
(2) some happy childhoods → good comedians
(3) some good comedians + unhappy childhoods → happy adults
问:哪一个不可能从原文推出?(找一个与原文推理矛盾的项,并不是要找一个可以从原
文推出的项)

A : good comedians + unhappy childhoods 的 比 例   > best comedians + unhappy


childhoods 的比例。
原文推理 (1)得出 best comedians 百分百 unhappy childhoods;
原 文 推 理   (2)+(3) 得 出 一些 good comedians 有 happy childhoods , 一 些 good
comedians 有 unhappy childhoods. 所以 good comedians 当然不能百分百地 unhappy
childhoods.
所以 A 项与原文明确矛盾。

B: 原文推理(3) some good comedians + unhappy childhoods → happy adults


我 们 将 “ some good comedians + unhappy childhoods” 理 解 成 “ good comedians +
unhappy childhoods”这个全集的一个子集 W,W 可以是 happy 的。
我们可以将 B 项“some good comedians + unhappy childhoods”理解成全集的另一个子集
X,X 当然可以是 unhappy 的。
所以 B 项与原文不矛盾。

C:
原文(1)+(3)
(1) best comedians → unhappy childhoods
(3) some good comedians + unhappy childhoods → happy adults
best comedians → good comedians(common sense), 而一些 good comedians + unhappy
childhoods → happy adults ,那当然有可能 most best comedians → happy adults
C 项与原文不矛盾

D:good comedians + unhappy childhoods > good comedians + happy childhoods


找找看,与原文三个推理的哪一个矛盾?没有

E:comedians + happy adults 占 comedians 的比例>comedians + unhappy adults 占


comedians 的比例。
原文只讲了 best comedians 和 good comedians,至于 comedians 总体情况如何,原文没有
讲。所以 E 项的结论自然也是有可能的。

不晓得我想得对不对,不过要讲清楚确实有点费劲。这种题只要推不出与原文明确相反的推
理就是 can be true 的。

26
Lsat-2-2

1. Some people believe that witnessing violence in movies will discharge aggressive
energy. Does watching someone else eat fill one’s own stomach?

In which one of the following does the reasoning most closely parallel that employed in
the passage?

(A) Some people think appropriating supplies at work for their won personal use is
morally wrong. Isn’t shoplifting morally wrong?

(B) Some people think nationalism is defensible. Hasn’t nationalism been the excuse for
committing abominable crimes?

(C) Some people that boxing is fixed just because wrestling usually is. Are the two sports
managed by the same sort of people?

(D) Some people think that economists can control inflation. Can meteorologists make
the sun shine?

27
(E) Some people think workaholics are compensating for a lack of interpersonal skills.
However, aren’t most doctors workaholics?

这个题目原文读懂了,但是提取什么信息导致原文选 D 呢?

类比是两个逻辑上类似的东西的比较,不是自己和自己比较

其它的好象花梨鼠与 mindfree 两位讲过

3. More than a year ago, the city announced that police would crack down on illegally
parked cars and that resources would be diverted from writing speeding tickets to
ticketing illegally parked cars. But no crackdown has taken place. The police chief claims
that resources have had to be diverted from writing speeding tickets to combating the
city’s staggering drug problem. Yet the police are still writing as many speeding tickets as
ever. Therefore, the excuse about resources being tied up in fighting drug-related crime
simply is not true.

The conclusion in the passage depends on the assumption that

(A) every member of the police force is qualified to work on combating the city’s drug
problem
(B) drug-related crime is not as serious a problem for the city as the police chief claims it
is
(C) writing speeding tickets should be as important a priority for the city as combating
drug-related crime
(D) the police could be cracking down on illegally parked cars and combating the drug
problem without having to reduce writing speeding tickets
(E) the police cannot continue writing as many speeding tickets as ever while diverting
resources to combating drug-related crime
E 是答案
能够看出 E 是 not+削弱,D 是削弱
但对原题中的逻辑思路和步骤还分析不清楚
希望大家帮助...

Therefore, the excuse about resources being tied up in fighting drug-related crime simply is not true.z 这是结论。
作者推出结论,光靠 Yet the police are still writing as many speeding tickets as ever.这一 premise 不行,隐含的
条件是做超速记录和以前一样多,就无法完成打击毒品。(一心不能二用的意思)
More than a year ago, the city announced that police would crack down on illegally parked cars and that resources

28
would be diverted from writing speeding tickets to ticketing illegally parked cars. But no crackdown has taken
place.这是废话没用。

writing speeding tickets 何以前一样多--->resources 没用在 fighting drug-related crime 上


assumption 就应该是 resouces 不能被同时用在两个上面

4. Dried grass clippings mixed into garden soil gradually decompose, providing nutrients
for beneficial soil bacteria. This results in better-than-average plant growth. Yet mixing
fresh grass clippings into garden soil usually causes poorer-than-average plant growth.
Which one of the following, if true, most helps to explain the difference in plant growth
described above?
(A) The number of beneficial soil bacteria increases whenever any kind of plant material
is mixed into garden soil.
(B) Nutrients released by dried grass clippings are immediately available to beneficial soil
bacteria.
(C) Some dried grass clippings retain nutrients originally derived from commercial lawn
fertilizers, and thus provide additional enrichment to the soil.
(D) Fresh grass clippings mixed into soil decompose rapidly, generating high levels of
heat that kill beneficial soil bacteria.
(E) When a mix of fresh and dried grass clippings is mixed into garden soil, plant growth
often decreases.
答案是 D。那么 B 呢? 如果说干草释放的营养能更快被有益土壤细菌吸收也有助与解释上
述不同啊。
还是谈一下我自己的感受吧!

前一阶段,我也有过这样,尤其是 weaken 和 support 的题目,就是看懂了,可是不知道选什么?

后来自己反思总结一下。

就像 mindfree 牛牛说的:其实题目中可能有好几层的逻辑关系,你应该明白题目要求你的是对哪一层的逻
辑关系。就是一条线。
就像这道题目,题目要求你解释的就是最后一句话,为什么新鲜的草放进去就有反作用。
只要能把这个矛盾给解决了,回过头来看 B,那就是无关。
题目并没有告诉你 immediately 和 gradually 的区别。
而且 B 更没有提到 fresh grass 的作用。

后来在看 LSAT 演绎题的时候,就感觉能沿着一条线走。


比如,weaken,要知道是 weaken 什么东东,然后往上想,究竟有哪几点 support 这个东东,既然只有这几
点(通常就一两点),那么答案也就差不多。
LSAT 尤其题目长的,有很多背景知识都是废话,纯粹就是占用你大脑的容量。

这是我的一点感受而已,还请希望多批评和指点。

29
5. A gas tax of one cent per gallon would raise one billion dollars per year at current
consumption rates. Since a tax of fifty cents per gallon would therefore raise fifty billion
dollars per year, it seems a perfect way to deal with the federal budget deficit. This tax
would have the additional advantage that the resulting drop in the demand for gasoline
would be ecologically sound and would keep our country from being too dependent on
foreign oil producers.

Which one of the following most clearly identifies an error in the author’s reasoning?

A) The author cites irrelevant data.

(B) The author relies on incorrect current consumption figures.

(C) The author makes incompatible assumptions.

(D) The author mistakes an effect for a cause.

(E) The author appeals to conscience rather than reason.

答案:C,我看了好几遍,都不知为什么是这个错误,能否讲讲?

这道题,我是这样想的。
前面一部分,有一内涵的假设:当消费量提高时才会提高税收收入,从而弥补财政赤子。
后面一部分,又有一内涵的假设:但消费量减少时才会有利于生态保护。
这两个假设是矛盾的。
所有 C 正确。

7. There is no reason why the work of scientists has to be officially confirmed before
being published. There is a system in place for the confirmation or disconfirmation of
scientific finding, namely, the replication of results by other scientists. Poor scientific work
on the part of any one scientist, which can include anything from careless reporting
practices to fraud, is not harmful. It will be exposed and rendered harmless when other
scientists conduct the experiments and obtain disconfirmatory results.

Which one of the following, if true, would weaken the argument?

(A) Scientific experiments can go unchallenged for many years before they are
replicated.

30
(B) Most scientists work in universities, where their work is submitted to peer review
before publication.

(C) Most scientists are under pressure to make their work accessible to the scrutiny of
replication.

(D) In scientific experiments, careless reporting is more common than fraud.

(E) Most scientists work as part of a team rather than alone.

答案:A,我是用排除法得的 A,可是,若 A 是:) Scientific experiments can go unchallenged for


many years after they are replicated. 就削弱了结论:
It will be exposed and rendered harmless when other scientists conduct the experiments
and obtain disconfirmatory results.
可是 A 说的是 before,能削弱吗?

做题不能只看一个字啊, 要理解整个句子的意思. 原文认为 works of scientists 发表后其他


的 scientists 会 重 复 实 验 进 而 验 证 ,从而使得 poor works 被 rendered harmless. A 的意思
削弱了原文的论证, 认为很多实验在发表后很多年都不会有人去重复验证 , 否定了原文的
论 据 , 说 明 可 能 cause harm. 你 要 想 用 after, 就 改 成 Scientific experiments can go
unchallenged for many years after they are published. 你改后的 A 不对. replication 本身
就是 challenge 的一步.

9. Governments have only response to public criticism of socially necessary services:


regulation of the activity of providing those services. But governments inevitably make
the activity more expensive by regulating it, and that is particularly troublesome in these
times of strained financial resources. However, since public criticism of child-care
services has undermined all confidence in such services, and since such services are
socially necessary, the government is certain response.
Which one of the following statements can be inferred from the passage?
A).The quality of child care will improve.
B).The cost of providing child care services will increase.
C).The government will use the funding to foster the advances in child care.
D).If public criticism of policy is strongly voiced, the government is certain to respond
E). If child care services are not regulated, the cost of providing child care will not
increase.
The answer is B. About this question, I have no idea, can any master help analyze.
Thanks in advance.

文章的的推理是这样的:

31
1.政府只对有必要的服务进行回应,并作出规定.
2.政府进行规定的服务成本都会增加.
3. child care service 是政府必须作出回应的服务.

因此, child care service 的成本会提高.


用无关词排除法可排除 A、C、D(因为归纳题只要是文中没有提供的信息均是无关选项)。

E 是原文的否命题,不正确。 只有逆否命题才是等价的。

11. “If the forest continues to disappear at its present pace, the koala will approach
extinction,” said the biologist.

“So all that is needed to save the koala is to stop deforestation,” said the politician.

Which one of the following statements is consistent with the biologist’s claim but not with
the politician’s claim?

(A) Deforestation continues and the koala becomes extinct.

(B) Deforestation is stopped and the koala becomes extinct.

(C) Reforestation begins and the koala survives.

(D) Deforestation is slowed and the koala survives.

(E) Deforestation is slowed and the koala approaches extinction.

这个题目答案是 B,让我的确有些不明白.请解释一下 B 为什么正确?

没有问题啊. 从逻辑关系的角度分析:
Biologist: forest disappear (A)--> extinction (B)
Politician: stop deforestation (非 A)--> no extinction (非 B)
两个论题是不同的, 相当与互为否命题.
B 对. stop deforestation, extinction. 按 politician 的逻辑是不可能发生的, 因为非 A-->非 B.
但 biologist 的逻辑依然可以成立. 当 A-->B, 非 A might or might not lead to 非 B

这位大牛的解释我看了,但是我还是没有明白,请再次解释!

下:
先分析原文逻辑关系

32
Biologist: forest disappear (A)--> extinction (B)
Politician: stop deforestation (非 A)--> no extinction (非 B)
题目是问:哪个与 Politician 不同, 与 biologist 相同.
我是挨个试了一下.到 b 就找到了.b 是说 stop deforestation -->extinction
明显与 politician 相反.
同时由 biologist, A 只是 B 的一种原因,A 是 B 的充分条件, 不能得出就是必要条件,
因此其他条件也同样能推出 B,
而选项 b 中正是选了一个条件,stop deforestation
所以选项 b 可以符合 biologist 的逻辑推理, 因此为答案

LSAT2-1

13. If retail stores experience a decrease in revenues during this holiday season, then
either attitudes toward extravagant gift-giving have changed or prices have risen beyond
the level most people can afford. If attitudes have changed, then we all have something
to celebrate this season. If prices have risen beyond the level most people can afford,
then it must be that salaries have not kept pace with rising prices during the past year.
Assume the premises above to be true. If salaries have kept pace with rising prices
during the past year, which one of the following must be true?
(A) Attitudes toward extravagant gift-giving have changed.
(B) Retail stores will not experience a decrease in retail sales during this holiday season.
(C) Prices in retail stores have not risen beyond the level that most people can afford
during this holiday season.
(D) Attitudes toward extravagant gift-giving have not changed, and stores will not
experience a decrease in revenues during this holiday season.
(E) Either attitudes toward extravagant gift-giving have changed or prices have risen
beyond the level that most people can afford during this holiday season.

这里面有两个原因一个结果:

结果:retail stores experience a decrease in revenues during this holiday season


原因 1:attitudes toward extravagant gift-giving have changed;
原因 2:prices have risen beyond the level most people can afford;而原因 2 产生于:

原因 21:salaries have not kept pace with rising prices during the past year.

通过逆否,只能是原因 21 不成立,所以原因 2 不成立。而我们不知道原因 1 成立与否,所


以只能推论到这里。MUST TRUE 的推论题必须保证推论成立不需要任何额外的假设条件。

题干给的前提:

1. 收入下降->(态度改变)or (价格离谱)

33
【逆否命题:(态度没改变)and (价格不离谱)->收入没下降】

2.价格离谱->工资不配套 【逆否命题:工资配套->价格不离谱】

现在如果已知:工资配套

可以得到 C:价格不离谱。

但要推出 B(收入没下降)必须还要有条件(态度没改变)。

14. The “suicide wave” that followed the United States stock market crash of October
1929 is more legend than fact. Careful examination of the monthly figures on the causes
of death in 1929 shows that the number of suicides in October and in November was
comparatively low. In only three other months were the monthly figures lower. During the
summer months, when the stock market was flourishing, the number of suicides was
substantially higher.

Which one of the following, if true, would best challenge the conclusion of the passage?

(A) The suicide rate is influenced by many psychological, interpersonal, and societal
factors during any given historical period.

(B) October and November have almost always had relatively high suicide rates, even
during the 1920s and 1930s.

(C) The suicide rate in October and November of 1929 was considerably higher than the
average for those months during several preceding and following years.

(D) During the years surrounding the stock market crash, suicide rates were typically
lower at the beginning of any calendar year than toward the end of that year.

(E) Because of seasonal differences, the number of suicides in October and November of
1929 would not be expected to be the same as those for other months.

(1)这个题目的答案选 C,我觉得 C 就是无关的.uring several preceding and following years


怎么可以对 1929 年的高或产生影响呢?

(2)原文的"In only three other months were the monthly figures lower."是什么意思?

首先搞清楚这题的结论:1929 年 10 月的自杀风潮是被人瞎传的,不是事实。
那么要削弱就是要找可以支持这次自杀风潮确实是事实的 evidence.原文的提供的 evidence 是(1)number
of suicides in October and in November was comparatively low. ( 2 ) In only three other months were the

34
monthly figures lower.(3 ) During the summer months, when the stock market was flourishing, the number of
suicides was substantially higher. 从而作者认为 10 月的自杀率并不是同年的 12 个月中最高的,相反有好几
个月比它高,所以人们是把 stock market crash 和自杀率牵强附会了。
我们要寻求的证据是支持自杀风潮的。c 表达了这样一个证据,即它和前后几年的相同时期比较都要高,
所以 1929 年 10 月的 stock market crash 至少和 suicide wave 有一定的 corelation. 所以就起到了削弱原结论的
作用。

15. A well-known sports figure found that combining publicity tours with playing tours led
to problems, so she stopped combining the two. She no longer allows bookstore
appearances and playing in competition to occur in the same city within the same trip.
This week she is traveling to London to play in a major competition, so during her stay in
London she will not be making any publicity appearances at any bookstore in London.

Which one of the following most closely parallels the reasoning used in the passage?

(A) Wherever there is an Acme Bugkiller, many wasps are killed. The Z family garden has
an Acme Bugkiller, so any wasps remaining in the garden will soon be killed.

(B) The only times that the hospital’s emergency room staff attends to relatively less
serious emergencies are times when there is no critical emergency to attend to. On
Monday night the emergency room staff attended to a series of fairly minor emergencies,
so there must not have been any critical emergencies to take care of at the time.

(C) Tomato plants require hot summers to thrive. Farms in the cool summers of country Y
probably do not have thriving tomato plants.

(D) Higher grades lead to better job opportunities, and studying leads to higher grades.
Therefore, studying will lead to better job opportunities.

(E) Butter knives are not sharp. Q was not murdered with a sharp blade, so suspect X’s
butter knife may have been the murder weapon.

这个题目题目的确读懂了,但是我提取不了什么信息作题?
答案选 B

1The advanced technology of ski boots and bindings has brought a dramatic drop in the
incidence of injuries that occur on the slopes of ski resorts: from 9 injuries per 1,000
skiers in 1950 to 3 in 1980. As a result, the remainder of ski-related injuries, which
includes all injuries occurring on the premises of a ski resort but not on the slopes, rose
from 10 percent of all ski-related injuries in 1950 to 25 percent in 1980. The incidence of
these injuries, including accidents such as falling down steps, increases with the amount

35
of alcohol consumed per skier.

17. Which one of the following conflicts with information in the passage?

(A) The number of ski injuries that occurred on the slopes was greater in 1980 than in
1950.

(B) A skier was less likely to be injured on the slopes in 1950 than in 1980.

(C) The reporting of ski injuries became more accurate between 1950 and 1980.

(D) The total number of skiers dropped between 1950 and 1980.

(E) Some ski-related injuries occurred in 1980 to people who were not skiing.

答案:B

A 不对,原文说的是百分比,有可能人数会变,那么两个一乘就不好说了。

同意 1stzhang 的意见. 在牵涉到数字和比较的题目中, 要注意的一个问题是:比较的是绝对的数字(absolute


value)还是比率(rate, percentage).

18. Learning how to build a nest plays an important part in the breeding success of birds.
For example, Dr. Snow has recorded the success of a number of blackbirds in several
successive years. He finds that birds nesting for the first time are less successful in
breeding than are older birds, and also less successful than they themselves are a year
later. This cannot be a mere matter of size and strength, since blackbirds, like the great
majority of birds, are fully grown when they leave the nest. it is difficult to avoid the
conclusion that they benefit by their nesting experience.

Which one of the following, if true, would most weaken the argument?

(A) Blackbirds build better nests than other birds.

(B) The capacity of blackbirds to lay viable eggs increases with each successive trial
during the first few years of reproduction.

(C) The breeding success of birds nesting for the second time is greater than that of birds
nesting for the first time.

36
(D) Smaller and weaker blackbirds breed just as successfully as bigger and stronger
blackbirds.

(E) Up to 25 percent of all birds are killed by predators before they start to nest.

答案:B,B 为什么是削弱呢?

B 削弱在于它指出是其它原因造成了文中的现象. 这道题也是很典型的:A 和 B 同时发生,于


是结论为 A-->B. 这类题一般问 assumption, 答案可能是:1. 不是 B-->A 2. 不是其它原因造
成 B. 这里的削弱就是第二种情况, 否定了 assumption:是其它原因造成的, 原结论就不成立
了.

XX 通过观察认为孵蛋的成功率和筑巢有关(A,B 同时发生==>A-->B). 答案 B 指出蛋的质量


是原因.

我以前回复过此题, CD 不支持全文搜索实在是...唉...
B 削弱在于它指出是其它原因造成了文中的现象. 这道题也是很典型的:A 和 B 同时发生,于是结论为 A-->B.
这类题一般问 assumption, 答案可能是:1. 不是 B-->A 2. 不是其它原因造成 B. 这里的削弱就是第二种情况,
否定了 assumption:是其它原因造成的, 原结论就不成立了.

XX 通过观察认为孵蛋的成功率和筑巢有关(A,B 同时发生==>A-->B). 答案 B 指出蛋的质量是原因.

20. All savings accounts are interest-bearing accounts. The interest from some interest-
bearing accounts is tax-free., so there must be some savings accounts that have tax-free
interest.

Which one of the following arguments is flawed in a way most similar to the way in which
the passage is flawed?

(A) All artists are intellectuals. Some great photographers are artists. Therefore, some
great photographers must be intellectuals.

(B) All great photographers are artists. All artists are intellectuals. Therefore, some great
photographers must be intellectuals.

(C) All great photographers are artists. Some artists are intellectuals. therefore, some
great photographers are intellectuals.

(D) All great photographers are artists. Some great photographers are intellectuals.
Therefore, some artists must be intellectuals.

37
(E) All great photographers are artists. No artists are intellectuals. Therefore, some great
photographers must not be intellectuals.

这个题目从原文归纳了什么逻辑错误去作题目呢?答案选 C

主要的问题就是混淆了必要条件和充分条件,或者说是充分性不足的情况下的出结论

interest-bearing account:组合词,负担利息的,带利息的帐户。其实在逻辑题中象这种词不一定非要理解才
能做题,记住其是元素某某就可以了。
原文推理:
因为:savings accounts (A)--> interest-bearing account(B);
   SOME interest-bearing account(SOME B) --> tax free(C)
所以:SOME A--> C

选项 C:
因为:great photographers(A) -->artists(B);
   SOME artists(SOME B) -->intellectuals(C)
所以:SOME A -->C

23
Despite improvements in treatment for asthma, the death rate form this disease has
doubled during the past decade from its previous rate. Two possible explanations for this
increase have been offered. First, the recording of deaths due to asthma has become
more widespread and accurate in the past decade than it had been previously. Second,
there has been an increase in urban pollution. However, since the rate of deaths due to
asthma has increased dramatically even in cities with long-standing, comprehensive
medical records and with little or no urban pollution, one must instead conclude that the
cause of increased deaths is the use of bronchial inhalers by asthma sufferers to relieve
their symptoms.

23. Which one of the following is an assumption on which the argument depends?

(A) Urban pollution has not doubled in the past decade.

(B) Doctors and patients generally ignore the role of allergies in asthma.

(C) Bronchial inhalers are unsafe, even when used according to the recommended
instructions.

(D) The use of bronchial inhalers aggravates other diseases that frequently occur among
asthma sufferers and that often lead to fatal outcomes even when the asthma itself does
not.

38
(E) Increased urban pollution, improved recording of asthma deaths, and the use of
bronchial inhalers are the only possible explanations of the increased death rate due to
asthma.

这个题目只有选 E!但是这个 argument 的结论好象不明显.

没有读到 E 之前,我认为 D 是相关的,但 E 确实合理的解释出文章可以直接得出这个结论的原因,即 12


不成,那一定是 3 了,可见 3 一定原本就是原因之一

24. There is little point in looking to artists for insights into political issues. Most of them
hold political views that are less insightful than those of any reasonably well-educated
person who is not an artist. Indeed, when taken as a whole, the statements made by
artists, including those considered to be great, indicate that artistic talent and political
insight are rarely found together.

Which one of the following can be inferred from the passage?

(A) There are no artists who have insights into political issues.

(B) A thorough education in art makers a person reasonably well educated.

(C) Every reasonably well-educated person who s not an artist has more insight into
political issues than any artist.

(D) Politicians rarely have any artistic talent.

(E) Some artists are no less politically insightful than some reasonably well-educated
persons who are not artists.

答案:E,其它选项可排除。
是 LSAT 中常见的题型. Most of people at CD are now studying GMAT ==> Some of those
people at CD are not studying. 所以说逻辑题目中的每个字都很重要,尤其是这种表程度,范
围,态度的词.

39
Section 2-4

1. A major art theft from a museum was remarkable in that the pieces stolen clearly had
been carefully selected. The criterion for selection, however, clearly had not been
greatest estimated market value. It follows that the theft was specifically carried out to
suit the taste of some individual collector for whose private collection the pieces were
destined.

The argument tacitly appeals to which one of the following principles?


(A) Any art theft can, on the evidence of the selection of pieces stolen, be categorized as
committed either at the direction of a single known individual or at the direction of a group
of known individuals.
(B) Any art theft committed at the direction of a single individual results in a pattern of
works taken and works left alone that defies rational analysis.

40
(C) The pattern of works taken and works left alone can sometimes distinguish one type
of art theft from another.
(D) Art thefts committed with no preexisting plan for the disposition of the stolen works do
not always involve theft of the most valuable pieces only.
(E) The pattern of works taken and works left alone in an art theft can be particularly
damaging to the integrity of the remaining collection.

偶选 D,答案为 C。why D?

No preexisting plan 错。

3. The United States has never been a great international trader. It found most of its raw
materials and customers for finished products within its own borders. The terrible
consequences of this situation have become apparent, as this country now owes the
largest foreign debt in the world and is a playground for wealthy foreign investors. The
moral is clear: a country can no more live without foreign trade than a dog can live by
eating its own tail.

In order to advance her point of view, the author does each of the following EXCEPT

(A) draw on an analogy

(B) appeal to historical fact

(C) identify a cause and an effect

(D) suggest a cause of the current economic situation

(E) question the ethical basis of an economic situation

答案:E,可是原文中不是有 The moral is clear: a country can no more live without foreign
trade than a dog can live by eating its own tail. ?
1. "It found most of its raw materials and customers for finished products within its own
borders." You can telll this is historical fact as he says in the next sentence "...now owes
the largest foreign debt in the world and is...."
刚查了字典, moral 在这里应该是教训的意思, draw a moral.

5. A government agency publishes ratings of airlines, ranking highest the airlines that

41
have the smallest proportion of late flights. The agency’s purpose is to establish an
objective measure of the relative efficiency of different airlines’ personnel in meeting
published flight schedules.

Which one of the following, if true, would tend to invalidate use of the ratings for the
agency’s purpose?

(A) Travelers sometimes have no choice of airlines for a given trip at a given time.

(B) Flights are often made late by bad weather conditions that affect some airlines more
that others.

(C) The flight schedules of all airlines allow extra time for flights that go into or out of very
busy airports.

(D) Airline personnel are aware that the government agency is monitoring all airline flights
for lateness.

(E) Flights are defined as “late” only if they arrive more that fifteen minutes past their
scheduled arrival time, and a record is made of how much alter than fifteen minutes they
are.

这个题目我在 A/B 之间犹豫了很久.答案是 B,为什么 A 不对?原文的最后一句话应该如何


理解呢?

原文:government agency 制定 rating, highest rating 表示 airline 有 smallest proporation


of 飞机晚点。the translation of last sentence, I try to translate it as concise and correct as I
can. agency 的目的是建立一个相对有效的客观标准来度量不同机组在执行公布飞行时间
表方面的准确性。infrim an assumption that is the peronnel that make the flights late, and
B suggest an another excuse, which is the bad weather that affects more on some
airlines than on other airlines.
6. Although this bottle is labeled “vinegar,” no fizzing occurred when some of the liquid in
it was added to powder from this box labeled “baking soda.” But when an acidic liquid
such as vinegar is added to baking soda the resulting mixture fizzes, so this bottle clearly
has been mislabeled.
A flaw in the reasoning in the argument above is that this argument
(A) ignores the possibility that the bottle contained an acidic liquid other than vinegar
(B) fails to exclude an alternative explanation for the observed effect
(C) depends on the use of the imprecise term “fizz”
(D) does not take into account the fact that scientific principles can be definitively tested
only under controlled laboratory conditions
(E) assumes that the fact of a labeling error is proof of an intention to deceive
这种题目的解题思路是怎么样的?
答案是 B,可我怎么觉得 B 中的 exclude 应该换成 include?

42
Exclude 和 Include 是相对而言的:

如果问题要求找到 Flaw in the conclusion, 则固定住推理,结论还应该包含 "baking


soda" mislabel 的情形。这时(B)中应该用 Include 替换 Exclude 才能成为答案;
如果问题要求找到 Flaw in the reasoning(如本题), 则固定住结论,推理中需要排除
"baking soda" mislabel 的情况才能得到这个结论。这是应该用 Exclude。

7. Marine biologists have long thought that variation in the shell color of aquatic snails
evolved as a protective camouflage against birds and other predators. Brown shells
seem to be more frequent when the underlying seafloor is dark-colored and white shells
more frequent when the underlying seafloor is light-colored. A new theory has been
advanced, however, that claims that shell color is related to physiological stress
associated with heat absorption. According to this theory, brown shells will be more
prevalent in areas where the wave action of the sea is great and thus heat absorption
from the Sun is minimized, whereas white shells will be more numerous in calmer waters
where the snails will absorb more heat from the Sun’s rays.
Evidence that would strongly favor the new theory over the traditional theory would be
the discovery of a large majority of
(A) dark-shelled snails in a calm inlet with a dark, rocky bottom and many predators
(B) dark-shelled snails in a calm inlet with a white, sandy bottom
(C) light-shelled snails in an inlet with much wave action and a dark, rocky bottom
(D) light-shelled snails in a calm inlet with a dark, rocky bottom and many predators
(E) light-shelled snails in a calm inlet with a white, sandy bottom and many predators

answer D. how can it be judged that the snails have dard and rocky ...? thanks

If light shell in sandy area, it is for the old theory. New theory basically states that the color of shell is determined
by the wave movement and heat, not color of the area. So D is for the new theory: light shell and dark bottom.

憧憬 MM, 这题涉及新旧两种理论。旧理论说海底贝壳类东东的颜色是它的保护色,如果海底深色,则它
深色,如果海底浅色,则它也浅色,(这样可以躲避 predator). 新理论则说,贝壳的颜色和吸热有关。如果
大海波浪多,则吸热少,那么贝壳深色的多。如果大海平静,则吸热多,贝壳浅色多。答案 D 说浅色贝壳在
一个平静,黑暗的海底,并且有很多 predator. 如果按照旧理论,一个黑暗海底,又有很多 predator, 那么采
用保护色则贝壳类应为深色。但是这里是浅色,所以与旧理论矛盾。那么按照新理论,平静大海,贝壳就为
浅色,正是 D 所说。其他答案均有和新理论矛盾之处。

8. Measurements of the extent of amino-acid decomposition in fragments of eggshell


found at archaeological sites in such places as southern Africa can be used to obtain

43
accurate dates for sites up to 200,000 years old. Because the decomposition is slower in
cool climates, the technique can be used to obtain accurate dates for sites almost a
million years old in cooler regions.

The information above provides the most support for which one of the following
conclusions?

(A) The oldest archaeological sites are not in southern Africa, but rather in cooler regions
of the world.

(B) The amino-acid decomposition that enables eggshells to be used in dating does not
take place in other organic matter found at ancient archaeological sites.

(C) If the site being dated had been subject to large unsuspected climatic fluctuations
during the time the eggshell has been at the site, application of the technique is less
likely to yield accurate results.

(D) After 200,000 ears in a cool climate, less than one-fifth of the amino acids in a
fragment of eggshell that would provide material for dating with the technique will have
decomposed and will thus no longer be suitable for examination by the technique.

(E) Fragments of eggshell are more likely to be found at ancient archaeological sites in
warm regions of the world than at such sites in cooler regions.

答案:C,

请问 conclusion 与一般的归纳题有什么区别吗?归纳题中如果出现原文没有的概念就排除,
如 C 中出现了 climatic fluctuation?是不是方法上有什么不同?

这里的 fluctuation 并不是新概念, 它是原文中一个概念的非 . 原文讲在不同的气候带中


amino-acid 的分解速度不同, 可以用于计算. 这里的隐含条件是气候带的温度比较恒定.
fluctuation 就是否定这个隐含条件.

这类题目也教常见, 所以在阅读是不要停留在表面上, 不要只是看字眼, 要理解题意.

9. Advertisement: Clark brand-name parts are made for cars manufactured in this
country. They satisfy all of our government automotive test-the toughest such tests in the
world. With foreign-made parts, you never know which might be reliable and which are
cheap look-alikes that are poorly constructed and liable to cost you hundreds of dollars in
repairs. Therefore, be smart and insist on brand-name parts by Clark for your car.

The argument requires the assumption that

44
(A) Clark parts are available only in this country

(B) foreign-made parts are not suitable for cars manufactured in this country

(C) no foreign-made parts satisfy our government standards

(D) parts that satisfy our government standards are not as poorly constructed as cheap
foreign-made parts

(E) if parts are made for cars manufactured in our country, they are not poorly
constructed

答案:D,我觉得这个是 bridge 型的假设题,但是 C 为什么不行呢?也是个 bridge 啊?

你没有把题目完全理解. 记住原文的内容要全面理解.
原文说顾客在选择 foreign parts 时,分不清质优和质差(poorly constructed), 而 Clarks 的
parts 都通过了政府检测, 所以要选就选 Clark parts. 这里的 gap 是分清质优质差和通过政
府检测. D 是标准答案.

C 是标准错误答案, 以后见到类似选项要警惕. C 不能填补原文逻辑中的 gap, C 对此题是无


关选项. 取非后:Not all ...有的通过,有的没通过, 原文的逻辑照样成立.

10. Even if a crime that has been committed by computer is discovered and reported, the
odds of being both arrested and convicted greatly favor the criminal.
Each of the following, if true, supports the claim above EXCEPT:
(A) The preparation of computer-fraud cases takes much more time than is required for
average fraud cases, and the productivity of prosecutors is evaluated by the number of
good cases made.
(B) In most police departments, officers are rotated through different assignments every
two or three years, a shorter time than it takes to become proficient as a computer-crime
investigator.
(C) The priorities of local police departments, under whose jurisdiction most computer
crime falls, are weighted toward visible street crime that communities perceive as
threatening.
(D) Computer criminals have rarely been sentenced to serve time in prison, because
prisons are overcrowded with violent criminals and drug offenders.
(E) The many police officers who are untrained in computers often inadvertently destroy
the physical evidence of computer crime.

10. 即使一个通过电脑作案的罪犯被发现和告发了, 被捕和被定罪的几率大大有利于这个


罪犯。

45
A. 准备电脑相关的犯罪诉讼花的时间比平均的时间多很多,而公诉人的工作效率是由处理
完成的案件数量来评价的。
B.在大多数警察部门,警官们每两三年在不同的岗位上轮岗,这么长的时间不足以使他们
成为专业的计算机犯罪调查人员。
C。在地方警察部门的审判下,大多数计算机犯罪......,他们优先考虑可见的被社区视作具有
威胁性的街头犯罪。
D.计算机罪犯很少被判以需入狱服刑,因为监狱里已经关满了暴力罪犯和毒贩子。
E.许多没有受过计算机方面训练的警官常常将电脑犯罪的实物证据不可挽回的毁坏了。
只有 D 与定罪和被捕无关。

12. “Though they soon will, patients should not have a legal right to see their medical
records. As a doctor, I see two reasons for this. First, giving them access will be time-
wasting because it will significantly reduce the amount of time that medical staff can
spend on more important duties, by forcing them to retrieve and return files. Second, if
my experience is anything to go by, no patients are going to ask for access to their
records anyway.”

Which one of the following, if true, establishes that the doctor’s second reason does not
cancel out the first?

(A) The new law will require that doctors, when seeing a patient in their office, must be
ready to produce the patient’s records immediately, not just ready to retrieve them.

(B) The task of retrieving and returning files would fall to the lowest-paid member of a
doctor’s office staff.

(C) Any patients who asked to see their medical records would also insist on having
details they did not understand explained to them.

(D) The new law does not rule out that doctors may charge patients for extra expenses
incurred specifically in order to comply with the new law.

(E) Some doctors have all allowing their patients access to their medical records, but
those doctors’ patients took no advantage of this policy.

这题的答案是 A,可问题中的 if true, establishes that the doctor’s second reason does not
cancel out the first? 是什么意思呢
cancel out 是补偿抵消的意思吗?

(3) two critical phrases impede the understandingof the question.

46
cancel out = offset, to match in force or effect. for example: His irritability cancelled out
his natural kindness, Osbert Sitwell, MW dictionary.

go by=pass, MW dictionary, It will make life easier if we think it as make an appointment.

now let's rock and roll.

问题:下面的哪一个如果成立将使得医生的第二个理由不能使第一个理由无效。
提干:医生认为虽然病人将会获得这种权利,但是他认为不应该,理由有 2 条:(1)调阅
和归还病历将耗费时间;(2)如果我的 experience 足够,又有哪些病人会要求去看它的
病历?

医生的推理实际上使(2)已经足够使得(1)不成立,因为我的经验足够应付病人所提出
的一切疑问,那病人自然就不会要看病历了。

但是 answer 1 否定了这一推理。不管你 experience 有多高,patient 在你的 office,你就必


须准备好 produce patient's record, 而不是仅仅准备 retrieve. 注意仅仅是不能抵消第一个理
由,也就是说医生还是有可能抱怨第一个理由。

2.俺的理解是这样的,原题问如何能让第二个 reason 不抵消第一个。原题说医生反对给病


人看病历,给俩原因。第一个是医生会浪费大量的时间来诸如取出和放回档案。第二个是医
生说病人不来要他们档案。确实光从这俩条件来看,既然不来要当然不用花医生时间去取出
和放回档案,但是第一个选项发话了,新 law 规定,医生得写档案,所以还是占医生时间
的,即 2 不能掰了 1。

14. The mayor boasts that the average ambulance turnaround time, the time from
summons to delivery of the patient, has been reduced this year for top-priority
emergencies. This is a serious misrepresentation. This “reduction” was produced simply
by redefining “top priority.” Such emergencies used to include gunshot wounds and
electrocutions, the most time-consuming cases. Now they are limited strictly to heart
attacks and strokes.
Which one of the following would strengthen the author’s conclusion that it was the
redefinition of “top priority” that produced the reduction in turnaround time?
(A) The number of heart attacks and strokes declined this year.
(B) The mayor redefined the city’s financial priorities this year.
(C) Experts disagree with the mayor’s definition of “top-priority emergency.”
(D) Other cities include gunshot wound cases in their category o top-priority
emergencies.
(E) One half of all of last year’s top-priority emergencies were gunshot wounds and
electrocution cases.

47
doesn't E, the so-called answer, contradict the author's claim, which is '... are strictly
limited to heart ..."?
这里不是说的是去年的枪伤多嘛、去年包括在 top-priority emergencies 里了,而且是最花
费时间的。今年的枪伤没那么多了,而且枪伤也不算 top-priority emergencies。所以救护车
抢救 top-priority emergencies 的时间就少了

15. In a large residential building, there is a rule that no pets are allowed. A group of pet
lovers tried to change that rule but failed. The rule-changing procedure outlined in the
building’s regulations states that only if a group of tenants can obtain the signatures of 10
percent of the tenants on a petition to change a rule will the proposed change be put to a
majority vote of all the tenants in the building. It follows that the pet lovers were voted
down on their proposal by the majority of the tenants.

The argument depends on which one of the following assumptions?

(A) The pet lovers succeeded in obtaining the signatures of 10 percent of the tenants on
their petition.

(B) The signatures of less than 10 percent of the tenants were obtained on the pet lovers’
petition.

(C) Ninety percent of the tenants are against changing the rule forbidding pets.

(D) The support of 10 percent of the tenants for a rule change ensures that the rule
change will be adopted.

(E) The failure of the pet lovers to obtain the signatures of 10 percent of the tenants on
their petition for a rule change ensures that the rule change will be voted down by a
majority of the tenants.
it follows that the pet lovers were voted down on their proposal by the majority of the
tenants. 是什么意思?我选 B,答案选 A.

As long as you understand the passage, you should fail on this one.

Obtaining signature of 10% of the tenants is a required condition for putting a proposal to
a "referendum". So the process is:

1. Propose a change --> 2. obtain signature 10% --> 3. majority vote --> 4. pass or fail

Now we know that the proposed change failed and the conclusion is that it failed at the

48
thrid link of majority vote. So the assumption here is the proposed change passed the
first 2 links. So answer is A

17 Every photograph ,because it involves the light rays that something emits hitting
film.must in some obvious sense be true.But because it could always have been made to
show things differently than it does.it cannot express the whole truth and in that sense is
false.Therefore nothing can ever be definitively proved with a photograph.

Which one of the following is an assumption that would permit the conclusion above to be
properly drawn?

A Whatever is false in the sense that it cannot express the whole truth cannot furnish
definitive proof.

B The whole truth cannot be known

C It is not ossible to determine the truthfulness of a photograph in any sense

D It is possible to use a photograph as corroborative evidence if there is additional


evidence establishing the truth about the sense photographed

E If something is being photographed ,then it is possible to prove definitively the truth


about it.

I go with A. It is an assumtion that closes the gap in the argument: P does not reveal the
whole truth, so P cannot offer definite prove. A literally makes the statement that the
argument is right.

20. An examination of corruption provides the basis for rejecting the view that an exact
science of society can ever be constructed. As with all other social phenomena that
involve deliberate secrecy, it is intrinsically impossible to measure corruption, and this is
not merely due to the fact that social science has not yet reached its goal, achievable to
be sure, of developing adequate quantifying techniques. If people were ready to answer
question about their embezzlements and bribes, it would their embezzlements and
bribes, it would mean that these practices had acquired the character of legitimate,
taxable activities and had ceased to be corrupt. In other words, corruption must
disappear if it is to be measurable. Which one of the following most accurately states a
hidden assumption that the author must make in order to advance the argument above

49
(A) Some people believe that an exact science of society can be constructed.

(B) The primary purpose of an exact science to quantify and measure phenomena

(C) An intrinsic characteristic of social phenomena that involve deliberate secrecy is that
they cannot be measured.

(D) An exact science of social phenomena that involve deliberate secrecy cannot be
constructed.

(E) An exact science can be constructed only when the phenomena it studies can be
measured.

21. Efficiency and redundancy are contradictory characteristics of linguistic systems:


however, they can be used together to achieve usefulness and reliability in
communication. If a spoken language is completely efficient, then every possible
permutation of its basic language sounds can be an understandable word. However, if
the human auditory system is an imperfect receptor of sounds, then it is not true that
every possible permutation of a spoken language’s basic language sounds can be an
understandable word.

If all of the statements above are true, which one of the following must also be true?

(A) Efficiency causes a spoken language to be useful and redundancy causes it to be


reliable.

(B) Neither efficiency nor redundancy can be completely achieved in spoken language.

(C) If a spoken language were completely redundant, then it could not be useful.

(D) If the human auditory system were a perfect receptor of sounds, then every
permutation of language sounds would be an understandable word.

(E) If the human auditory system is an imperfect receptor of sounds, then a spoken
language cannot be completely efficient.
3. 我认为先用排除法. 这种题目要把思路严格地局限在原文的逻辑关系内, 当自己是个 idiot.

原文中的两个 IF 标出了逻辑关系:
1. "a spoken language is completely efficient"(A)-->"every possible permutation of its

50
basic language sounds can be an understandable word"(B);
2. "the human auditory system is an imperfect receptor of sounds"(C)-->"it is not true that
every possible permutation of a spoken language’s basic language sounds can be an
understandable word"(非 B)

然后看选项:
A. 不对, 原文没有提到这两个逻辑关系
B. 不对, 原文没有提到 complete redundancy 可能性. 原文提到了 complete efficiency, 但是
并 没 有 说 它 是 否 可 能 . 从 原 文 的 第 二 个 逻 辑 关 系 看 , 如 果 听 觉 系 统 不 完 善 , complete
efficiency 才不可能, 言下之意是如果听觉系统完善(即使这个假设是错的, 原文并没有否定
这个假设, 所以不违背原文的逻辑关系),就可以达到 complete efficiency.
C. 不对, 原文没有提到 complete redundancy
D. 不对, 此选项是第二个逻辑关系的否命题, 与原命题不等价,可对可错, 不能从原文推出来.
E. 对. C-->非 B-->非 A

你可以通过相关性排除前三个选项, 再根据逻辑关系找出 D 的错误并验证 E 的正确性.

22. All intelligent people are nearsighted. I am very nearsighted. So I must be a genius.

Which one of the following exhibits both of the logical flaws exhibited in the argument
above?

(A) I must be stupid because all intelligent people are nearsighted and I have perfect
eyesight.

(B) All chickens have beaks. This bird has a beak. So this bird must be a chicken.

(C) All pigs have four legs, but this spider has eight legs. So this spider must be twice as
big as any pig.

(D) John is extremely happy, so he must be extremely tall because all tall people are
happy.

(E) All geniuses are very nearsighted. I must be very nearsighted since I am a genius.

答案是 D,我选 B.我觉得 B 更好,请解释一下?

2. There are two errors in the passage:


Intelligent people -->nearsighted ==> very nearsighted --> very intelligent
A-->B ==> very B--> very A

B does not show the second error, which is very B--> very A

51
D is right as its logic is tall --> happy ==> extremely happy --> extremely tall

Questions 24
In opposing the 1970 Clean Air Act, the United States automobile industry argued that
meeting the act’s standards for automobile emissions was neither economically feasible
nor environmentally necessary. However, the catalytic converter, invented in 1967,
enabled automakers to meet the 1970 standards efficiently. Currently, automaker are
lobbying against the government’s attempt to pass legislation that would tighten
restrictions on automobile emissions. The automakers contend that these new
restrictions would be overly expensive and unnecessary to efforts to curb air pollution.
Clearly, the automobile industry’s position should not be heeded.

Which one of the following most accurately expresses the method used to counter the
automakers’
current position?
(A) The automakers’ premises are shown to lead to a contradiction.
(B) Facts are mentioned that show that the automakers are relying on false information.
(C) A flaw is pointed out in the reasoning used by the automakers to reach their
conclusion.
(D) A comparison is drawn between the automakers’ current position and a position they
held in the past.
(E) Evidence is provided that the new emissions legislation is both economically feasible
and environmentally necessary.

What’s wrong with E? The key is D.

原 文 中 没 有 提 到 e 中 的 the new emissions legislation is both economically feasible and environmentally


necessary
既然是高标准怎么可能是 economically feasible 呢
只有 d 才是对的,因为这里用乐一个类比,把以前的情况和现在的情况做一个比较,以前可以实现的环保
标准现在也应该可以实现,有空你可以看看七宗罪的逻辑错误总结,常见的问题里面都涉及到了

LSAT-Set 3-1

The original purpose of government farm subsidy programs was to provide income
stability for small family farmers. But most farm-subsidy money goes to a few farmers
with large holdings Payments to farmers whose income, before subsidies, is greater than

52
$100,000 a year should be stopped.

Thomas: It would be impossible to administer such a cutoff point. Subsidies are needed
during the planting and growing season, but farmers do not know their income for given
calendar year until tax returns are calculated and submitted the following April.

Which one of the following, if true, is the strongest counter Rita can make to Thomas'
objection?

(A) It has become difficult for small farmers to obtain bank loans to be repaid later by
money from subsidies.

(B) Having such a cutoff point would cause some farmers whose income would otherwise
exceed $100,000 to reduce their plantings.

(C) The income of a farmer varies because weather and market prices are not stable
from year to year.

(D) If subsidy payments to large farmers were eliminated the financial condition of the
government would improve.

(E) Subsidy cutoffs can be determined on the basis of income for the preceding year.

4.A survey was recently conducted among ferry passengers on the North Sea. Among
the results was this: more of those who had taken anti-seasickness medication before
their trip reported symptoms of seasickness than those who had not taken such
medication. It is clear, then that despite claims by drug companies that clinical tests show
the contrary, people would be better off not taking anti-seasickness medications.
Which one of the following, if true, would most weaken the conclusion above?
(A) Given rough enough weather, most ferry passengers will have some symptoms of
seasickness.
(B) The clinical tests reported by the drug companies were conducted by the drug
companies’ staffs.
(C) People who do not take anti-seasickness medication are just as likely to respond to a
survey on seasickness as people who do.
(D) The seasickness symptoms of the people who took anti-seasickness medication
would have been more severe had they not taken the medication.
(E) People who have spent money on anti-seasickness medication are less likely to
admit symptoms of seasickness than those who have not.
答案是 D

53
conclusion: people would be better off NOT taking anti-seasickness medications.

What D says is : the medication works actually. it improves the symptoms even though the sign of seasickness
still exits.

The seasickness symptoms of the people who took anti-seasickness medication would have been more severe had
they not taken the medication.
是个常见的语法现象,== if they had not ...

c,support
e,ng
People who have spent money on anti-seasickness medication
more of those who had taken anti-seasickness medication
any difference??

Questions 6-7

Rotelle: You are too old to address effectively the difficult issues facing the country, such
as nuclear power, poverty, and pollution.

Sims: I don't want to make age an issue in this campaign. so i will not comment on your
youth and inexperience.

6. Sims does which one of the following?

(A) demonstrates that Rotelle's claim is incorrect

(B) avoids mentioning the issue of age

(C) proposes a way to decide which issues are important.

(D) shows that Rotelle's statement is self-contradictory

(E) fails to respond directly to Rotelle's claim


答案是 E。

这道题考的就是:在面对逻辑问题的时候不要用生活中的眼光。如果从一般的思维看,R 在
利用年龄问题搞人身攻击,而 S 巧妙的回应。但是用逻辑的思维,S 反驳 R 应该要么说我
还不老,要么说老和能力无关,或其他理由来直接反驳 R。但题目中 S 的说法,就好像:
“你说我有这个问题,我还说你有那个毛病呢!”实际是默认了 R 的话。

54
10. Rhizobium bacteria living in the roots of bean plants or other legumes produce fixed
nitrogen which is one of the essential plant nutrients and which for non-legume crops,
such as wheat normally must be supplied by applications of nitrogen-based fertilizer. So if
biotechnology succeeds in producing wheat strains
whose roots will play host to Rhizobium bacteria. The need for artificial fertilizers will be
reduced.
The argument above makes which one of the following assumptions?
(A) Biotechnology should be directed toward producing plants that do not require
artificial fertilizer.
(B) Fixed nitrogen is currently the only soil nutrient that must be supplied by
artificial fertilizer for growing wheat crops.
(C) There are no naturally occurring strains of wheat or other grasses that have
Rhizobium bacteria living in their roots.
(D) Legumes are currently the only crops that produce their own supply of fixed
nitrogen.
(E) Rhizobium bacteria living in the roots of wheat would produce fixed nitrogen.
answer:E

If NOT B, the conclusion will not be weakened, because the conclusion of the passage is that if biotechnology
succeeds in producing wheat strains whose roots will play host to Rhizobium bacteria,the need for artificial
fertilizers will be reduced, so the focus is nitrogen-based fertilizer, there is no any relationship with other soil
nutrients.

12. Leachate is a solutions, frequently highly contaminated, that develops when water
permeates a landfill site, If and only if the landfill's capacity to hold liquids is exceeded
does the leachates escape into the environment, generally in unpredictable quantities, A
method must be found for disposing of leachate. Most landfill leachate is send directly to
sewage treatment plants, but not all sewage plants are capable of handling the highly
contaminated water.

Which one of the following can be inferred from the passage?

(A) The ability to predict the volume of escaping landfill leachate would help solve the
disposal problem.

(B)If any water permeates a landfill, leachate will escape into the environment.

(C) No sewage treatment plants are capable of handling leachate.

55
(D) Some landfill leachate is send to sewage treatment plants that are incapable of

handling it.

(E) If leachate does not escape from a landfill into the environment, then the landfill's
capacity to hold liquids has not been exceeded.

題目: If and only if the landfill's capacity to hold liquids is exceeded does the leachates
escape into the environment 到底是
<1> leachates escape --> landfill's capacity is exceeded 還是
<2> landfill's capacity is exceeded --> leachates escape
那個是 p, 那個是 q??

我把它想成是<1>了. 如果<1>是真, 那麼答案(e), 非 p-->非 q,


leachate 非 escape --> landfill's capacity 非 exceeded 為什麼對??
p-->q, 不是應該 非 q-->非 p 嗎?

請大俠指教這題如何解.

只提醒你一点,既 “ if and only if ”是充分必要条件,这样 the leachates escape into the


environment 和 landfill's capacity to hold liquids is exceeded 就是等价的。否定一个就是否
定另一个。

第一题(12):

第二句话:if and only if A, then B (当且仅当,等价命题 A <--->B, 非 A<---->非 B)


The landfill's capacity to hold liquids is exceeded <----> the Leachates escape into the
environment, generally in unpredictable quantities
E 项就是这个等价命题的否命题。

原文最后一句话:
Most landfill Leachate is send directly to sewage treatment plants, but not all sewage
plants are capable of handling the highly contaminated water.

之所以选 D 是因为将“not all sewage plants”理解为“不是所有的被接受 L 的污水处理厂”,


但这里的应理解成为“不是所有的污水处理厂”。All sewage plants 是全集,而不是那些有 L
送入的 sewage plants 这个子集。所以,原文里推不出有 L 被送到那些不能处理 highly
contaminated water 的 plants。

LSAT 3-1-14
14. The average level of fat in the blood of people suffering from acute cases of disease

56
W is lower than the average level for the population as a whole. Nevertheless, most
doctors believe that reducing blood-fat levels is an effective way of preventing acute W.
Which one of the following, if true, does most to justify this apparently paradoxical belief?
(A) The blood level of fat for patients who have been cured of W is on average the same
as that for the population at large.
(B) Several of the symptoms characteristic of acute W have been produced in laboratory
animals fed large doses of a synthetic fat substitute, though acute W itself has not been
produced in this way.
(C) The progression from latent to acute W can occur only when the agent that causes
acute W absorbs large quantities of fat from the patient's blood.
(D) The levels of fat in the blood of patients who have disease W respond abnormally
slowly to changes in dietary intake of fat.
(E) High levels of fat in the blood are indicative of several diseases that are just as
serious as W.

the key is c
my question is how the answer C explain the paradoxical belief, I just could conclude that
the answer can explain the former part in the question, but how it explains the latter part
in the question(reducing blood-fat levels is an effective way of preventing acute W)?

C tell us that absoring high level of fat is a required condition to triger the disease,
therefore reducing blood-fat level is an effective way to control the occurrence of the
disease.

LSAT-3-I-15.
Baking for winter holidays is tradition that may have a sound medical basis. In midwinter,
when days are short, many people suffer from a specific type of seasonal depression
caused by lack of sunlight. Carbohydrates, both sugars and starches, boost the brain's
levels of serotonin, a neurotransmitter that improve the mood. In this respect, carbon
hydrates act on the brain in the same way as some antidepressants, Thus, eating holiday
cookies may provide an effective form of self-prescribed medication.

Which one of the following can be properly inferred from the passage?

(A) Seasonal depression is one of the most easily treated forms of depression.
(B) Lack of sunlight lowers the level of serotonin in the brain.
(C) People are more likely to be depressed in midwinter than at other times of the year.
(D) Some antidepressants act by changing the brain's level of serotonin.
(E) Raising the level of neurotransmitters in the brain effectively relieves depression.

The key is D, but what’s wrong with B and E?


(E) Raising the level of neurotransmitters in the brain effectively relieves depression.

57
The article says that Carbohydrates boost the brain’s levels of serotonin, a
neurotransmitter that improve the mood, that means raising the level of N, improve the
mood, and act the same way as some antidepressants.

(B) Lack of sunlight lowers the level of serotonin in the brain.


The article says that many people suffer from a specific type of seasonal depression
caused by lack of sunlight, but carbohydrates boost the brain's levels of serotonin. Can
that mean lack of sunlight lowers the level of serotonin.

in e,u considered"neurotransmitter=serotonin".However,there may be several kinds of


neurotransmitters,and serotonin is just a kind of it.u don't know whether raising other
kinds of neuro- can improve the mood.

in b,carbonhydrates not=sunlight.
even though carbonhydyrates=sunlight,we can't infer that lack of carbon- lower the
serotonin from its function of boosting it.

the correct answer,D,we can infer from 'Carbohydrates, boost the brain’s levels of
serotonin. In this respect, carbohydrates act on the brain in the same way as some
antidepressants'
here the bridge is"carbo- = some antidepressants"

16. the current proposal to give college students a broader choice in planning their own
courses of study should be abandoned. the students who are supporting the proposal will
never be satisfied, no matter what requirements are established. some of these students
have reached their third year without declaring a major. one first-year student has failed
to complete four required courses. several others have indicated a serious indifference to
grades and intellectual achievement.
a flaw in the argument is that it does which one of the following?
(a) avoids the issue by focusing on supporters of the proposal
(b) argues circularly by assuming the conclusion is true in stating the premises
(c) fails to define the critical term “satisfied”
(d) distorts the proposal advocated by opponents
(e) uses the term “student” equivocally

A
典型的诉诸人的逻辑错误. 原文: 因为支持该建议的人不行, 所以该建议不行

LSAT-3-I-17/18
The question whether intelligent life(有智力的生命体) exists elsewhere in the universe is

58
certainly imprecise, because we are not sure how different from use something might be
and still count as "intelligent life." Yet we cannot just decide to define "intelligent life" in
some more precise way since it is likely that we will find and recognize intelligent life
elsewhere in the universe only if we leave our definitions open to new, unimagined
possibilities.

17. The argument can most reasonably be interpreted as an objection to which one of the
following claims?
(A) The question whether intelligent life exists elsewhere in the universe is one that will
never be correctly answered.
(B) Whether or not there is intelligent life elsewhere in the universe, our understanding of
intelligent life is limited.
(C) The question about the existence of intelligent life elsewhere in the universe must be
made more precise if we hope to answer it correctly.
(D) The question whether there is intelligent life elsewhere in the universe is so imprecise
as to be meaningless.
(E) The question whether there is intelligent life elsewhere in the universe is one we
should not spend our time trying to answer.

17.
From:
Yet we cannot just decide to define "intelligent life" in some more precise way since it is likely that we will find
and recognize intelligent life elsewhere in the universe only if we leave our definitions open to new, unimagined
possibilities.

we can choose C
"YET" is important here, it tells us that follow sentence is more important and is what arguer mainly want to say..

原文有两层意思:1。不可能 将该问题( whether intelligent life exists elsewhere in the universe


)定义的精确。2。也不必要精确定义该问题。

问题:原文可以最合理的解释为反对下列哪个断言

答案 C:如果我们希望正确回答该问题,就必须更精确定义该问题

原文说不但不可能,而且也没必要精确定义该问题,正好反驳 C 的断言

18. The passage, if seen as an objection to an antecedent claim. Challenges that claim
by:

(A) showing the claim to be irrelevant to the issue at hand

59
(B) citing examples that fail to fit proposed definition of "intelligent life"

(C) claiming that "intelligent life" cannot be adequately defined.

(D) arguing that the claim, if acted on, would be counterproductive

(E) maintaining that the claim is not supported by the available evidence.

这个题目答案选 D,我选 C.有些想不通请解答.这个题目根本不知道是什么意思?

from "Yet" in the passage, we notice a transition in the meaning..


setence before "Yet" claim that XXX is imprecise...
sentence after "Yet" tell us that if XXX become more precise, negative result will occurs..
SO, D is right.

原文 we will find and recognize intelligent life elsewhere in the universe only if we leave our
definitions open to new, unimagined possibilities。意思为:定义死了,会起反作用。

19. The efficiency of microwave ovens in destroying the harmful bacteria frequently found
in common foods is diminished by the presence of salt in the food being cooked. When
heated in a microwave oven, the interior of unsalted food reaches temperatures high
enough to kill bacteria that cause food poisoning, but the interior of salted food does not.
Scientists theorize that salt effectively blocks the microwaves from that salt effectively
blocks the microwaves from heating the interior.

Which one of the following conclusions is most supported by the information above?

(A) The kinds of bacteria that cause food poisoning are more likely to be found on the
exterior of food than in the interior of food.

(B) The incidence of serious food poisoning would be significantly reduced if microwave
ovens were not used by consumers to cook or reheat food.

(C) The addition of salt to food that has been cooked or reheated in a microwave oven
can increase the danger of food poisoning.

(D) The danger of food poisoning can be lessened if salt is not used to prepare foods that
are to be cooked in a microwave oven.

60
(E) Salt is the primary cause of food poisoning resulting from food that is heated in
microwave ovens.

答案是 D, 为什么 C 不对?

20 Pamela: Business has an interest in enabling employees to care for children, because
those children will be the customers, employees, and managers of the future. Therefore,
businesses should adopt policies, such as day-care benefits that facilitate parenting.

Lee: No individual company, though, will be patronized, staffed, and managed only by its
own employees' children, so it would not be to a company's advantage to provide such
benefits to employees children, so it would not be to a company's advantage to provide
such benefits to employees when other companies do not.

In which one of the following pairs consisting of argument and objection does the
objection function most similarly to the way Lee's objection functions in relation to
Pamela's argument?

(A) new roads will not serve to relieve this area's traffic congestion, because new roads
would encourage new construction and generate additional traffic.

Objection: Failure to build new roads would mean that traffic congestion would strangle
the area even earlier.

(B) Humanity needs clean air to breathe, so each person should make an effort avoid
polluting the air.

Objection: The air one person breathes is affected mainly by pollution caused by others,
so it makes no sense to act alone to curb air pollution.

(C) Advertised discounts on products draw customers' attention to the products. So


advertised discounts benefit sales.

Objection: Customers already planning to purchase a product accelerate buying to take

advantage of advertised discounts, and those subsequent sales suffer.

(D) If people always told lies, then no one would know what the truth was, so people
should always tell the truth.

61
Objection: If people always told lies, then everyone would know that the truth was the
opposite of what was said .

(E) Human social institutions have always changed. So even if we do not know what
those changes will be, we do know that the social institutions of the future will differ from
those of the past.

Objection: The existence of change in the past does not ensure that there will always
be change in the future.

这个题目文章是读懂了,但是如何从原文提取信息作对题目是我现在的弱点,请指点?

20, 的逻辑关系事;因为我们未来的 customer, employees, and managers 就是现在的小孩,


因此公司因该采取措施方便父母。

反对者说:没有任何一个公司他未来的 customer, employee, manager 就是本公司雇员现


在的孩子,为什么我们要干别的公司没有干的事呢?

主要的逻辑关键点就在于:全体和部分的矛盾。B 正好就是。

这种纯逻辑题是很吓人的,提干长,每个选象也长。但是我的感觉是,绝大多数这种题如果
你能把握提干的逻辑思路,直接到选项里找跟该逻辑思路一模一样, 90%是一模一样,
8%,顶多把 if 从句和主句颠倒一下位置。2%例外。这 2%里外里就包括一个我要问的 plant-
tulip-tall tulip-plant are tall, 还一个就是什么 white poodle.

21. Pedro: Unlike cloth diapers, disposable diapers are a threat to the environment.
Sixteen billion disposable diapers are discarded annually, filling up landfills at an alarming
rate. So people must stop buying disposable diapers and use cloth diapers.
Maria: But you forget that cloth diapers must be washed in hot water, which requires
energy. Moreover, the resulting wastewater pollutes our rivers. When families use diaper
services, diapers must be delivered by fuel-burning trucks that pollute the air and add to
traffic congestion.
Maria objects to Pedro’s argument by
(A) claiming that Pedro overstates the negative evidence about disposable diapers in the
course of his argument in favor of cloth diapers
(B) indicating that Pedro draws a hasty conclusion, based on inadequate evidence about
cloth diapers
(C) pointing out that there is an ambiguous use of the word “disposable” in Pedro’s
argument
(D) demonstrating that cloth diapers are a far more serious threat to the environment
than disposable diapers are

62
(E) suggesting that the economic advantages of cloth diapers outweigh whatever
environmental damage they may cause

maria did not say that cloth diapers a far more serious threat to the environment than disposable diapers are. She
just gave evidences that cloth diapers can harm the environment, too. Therefore, pedro's conlcusion is not based
on adequate evidence.
The key is B, but what’s wrong with D?

这个偶也错了. 再想想, 看了 mindfree 哥哥的几个贴子, 觉得好象是这样: 关键看问提先:

Maria objects to Pedro's argument by :


是要求从逻辑推理上回答

在看题目:Pedro:先提结论 B, 再给原因 A, 就是 A----->B

Maria:没说别的就说: But you forget that A1, A2, A3 问题不是 Maria 这里到底认为是哪种 diaper 更污染环境,
关键是 Pedro 的逻辑推理有缺陷,所以 Maria 补充了 3 个 evidence,
Maria 本人的论点不重要, 换句话说, 即使有 A1,A2,A3, maria 也可能认为 disposal diaper 更污染环境. M 是不
同意 P 的推理方法....

可以看看这

http://forum.chasedream.com/dispbbs.asp?boardID=24&ID=2983

22. In an experiment, two-year-old boys and their fathers made pie dough together using
rolling pins and other utensils. Each father-son pair used a rolling pin that was
distinctively different from those used by the other, "father-son pairs, and each father
repeated the phrase "rolling pin" each time his son used it. But when the children were
asked to identify all of the rolling pins among a group of kitchen utensils that included
several rolling pins, each child picked only the one that he had used.

Which one of the following inferences is most supported by the information above?

(A) the children did not grasp the function of rolling pin.

(B) No two children understood the name "rolling pin" to apply to the same object

(C) The children understood that all rolling pins have the same general shape.

(D) Each child was able to identify correctly only the utensils that he had used.

(E) The children were not able to distinguish the rolling pins they used from other rolling

63
pins
答案是 B, 为什么 D 不对?
文 中的 中心 词是 rolling pins , 即孩 子辨 认出 的物 什, 而非 所有 的 utensil ,D 项中 将
identified object by children 范围扩大了。

23. When 100 people who have not used cocaine are tested for cocaine use, on average
only 5 will test positive. By contrast. of every 100 people who have used cocaine 99 will
test positive. Thus, when a randomly chosen group of people is tested for cocaine use.
the vast majority of those who test positive will
be people who have used cocaine.
A reasoning error in the argument is that the argument
(A) attempts to infer a value judgment from purely factual premises.
(B) attributes to every member of the population the properties of the average member of
the population.
(C) fails to take into account what proportion of the population have used cocaine.
(D) ignores the fact that some cocaine users do not test positive.
(E) advocates testing people for cocaine use when there is no reason to suspect that
they have used cocaine.
answer:C my choice:B

B 是倒装,还原后是 attributes the properties of the average member of the population to


every member of the population,似乎这并不是推理的 ERROR,因为结论说 the vast
majority of those who test positive,同整体的 every member of the population 没有什么关
系,是特定人群。
C 指出了一个推理谬误,因为如果整体人群中 have used cocaine 的人数很少的话,例如,
就 100 个,而整体人数是 100000000000,则被 randomly chosen group 的概率变得很低,
也就无法推出 the vast majority of those who test positive will be people who have used
cocaine.

24. If a society encourages freedom of thought and expression, then, during the time
when it does so, creativity will flourish in that society. In the United States creativity
flourished, during the eighteenth century. It is clear, therefore, that freedom of thought
was encouraged in the United States during the eighteenth century.

An error of reasoning of the same kind as one contained in the passage is present in
each of the following arguments EXCEPT:

(A) According to the airline industry, airfares have to rise if air travel is to be made safer;
since airfares were just raised, we can rest assured that air travel will therefore become
safer.

64
(B) We can conclude that the Hillside police department has improved its efficiency,
because crime rates are down in Hillside, and it is an established fact that crime rates go
down when police departments increase their efficiency.
(C) People who are really interested in the preservation of wildlife obviously do not go
hunting for big game; since Gerda has never gone hunting for big game and intends
never gone hunting for big game and intends never to do so. it is clear that she is really
interested in the preservation of wildlife.
(D) If the contents of a bottle are safe to drink, the bottle will not be marked "position" so,
since the bottle is not marked "poison", its contents will be safe to drink.
(E) None of the so-called Western democracies is really democratic, because, for a
country to be democratic, the opinion of each of its citizens must have a meaningful
effect on government, and in none of these countries does each citizen's opinion have
such an effect.

The key is E

65
Set 3-4

3. Scientific research that involves international collaboration has produced papers of


greater influence. As measured by the number of times a paper is cited in subsequent
papers, than has research without any collaboration. Papers that result from international
collaboration are cited an average of seven times, whereas papers with single authors
are cited only three times on average This difference shows that research projects
conducted by international research teams are of greater importance than those
conducted by single researchers.

Which one of the following is an assumption on which the argument depends?

(A) Prolific writers can inflate the number of citations they receive by citing themselves in
Subsequent papers.

(B) It is possible to ascertain whether or not a paper is the product of international


Collaboration by determining the number of citations it has received

(C) The number of citations a paper receives is a measure of the importance of the
Research it reports.

(D) The collaborative efforts of scientists who are citizens of the same country do not
produce papers that are as important as papers that are produced by international
Collaboration.

(E) International research teams tend to be more generously funded than are single

Researchers.

my answer is b, the correct one is C.I wonder why answer B is wrong.

C is right.
The number of citations a paper result from a team receives > the number of citations a
paper with single author receives
Conclusion: the projects(paper) conducted by a team are important than the projects
conducted by a single author.

So the number of citations is a measurement of the importance.

B. whether the paper is conducted by a single author or a team is not a focal point of the
augument. so B is out of scope.

5. Most parents who are generous are good parents, but some self-centered parents are
also good parents. Yet all good parents share one characteristic; they are good listeners.

66
If all of the statements in the passage are true, which one of the following must also be
true?

(A) All parents who are good listeners are good parents.

(B) Some parents who are good listeners are not good parent.

(C) Most parents who are good listeners are generous.

(D) Some parents who are good listeners are self-centered

(E) Fewer self-centered parents than generous parents are good listeners

答案:D

这类题目首先要把题目读清楚, 并将思路局限在题目中所给的关系内.

Must be true 的题目一般可以正推出来, 因为如果一定对的话, 原文必须给出一个真命题, 还


有一个方法就是举反例, 原文没有限制的都可能. 比如说最后一句:all good parents share
one characteristic; they are good listeners. 这句话中的关系就是:只要是 good parents---
>good listeners. 其逆否为等价命题,可以得到:not good listeners-->not good parents.

这里可能正确也可能错的是 :good listeners 都是 good parents;不是 good parents-->not


good listeners. 此二命题分别是逆命题和否命题, 即可能对也可能错. 因为原文没有对此二
命题有限制, 两种情况都可能. 我建议在读题时不要考虑这些引申命题, 在读选项时要考虑.

A. 错, 逆命题, 可错, 如逆命题错, 不成立


B. 错, 逆命题, 可对, 如逆命题对, 不成立
C. 错, most 错, 原文没有给出两者关系
E. 错, 原文没有给出两者关系

Some 表示有一些, 可以是一个. most 表示多数, 要大于 50%. 读题是要注意原文所给的范


围. 可以画图联系. 这类题我一般用排除和正推直接做.

Lsat 3-IV-7
7. A certain retailer promotes merchandise by using the following policy: At all times there
is either a “manger’s sale” or a “holiday sale” or both going on. All sales are run for
exactly one calendar month. In any given month, if a manager wishes to clear out
particular Line of merchandise, then a managers’ sale is declared, If a holiday falls
within the calendar month And there is excess merchandise in the warehouse
never contains excess merchandise
Which one of the following can be concluded from the passage?

67
(A) If a holiday falls within a given month and there is no extra merchandise in the
warehouse that month,then a holiday sale is declared
(B) If a holiday sale is not being run, then it is the month of August.
(C) If a manger’s sale is being run in some month, and then there is no excess
merchandise in the warehouse in that month.
(D) If there is not a manger’s sale being run some month. Then there is holiday sale
being run in that month.
(E) If there is no excess merchandise in the warehouse, then it is the month of August.
answer:D my choice:A

"A certain retailer promotes merchandise by using the following policy: At all times there
is either a “manger’s sale” or a “holiday sale” or both going on. All sales are run for
exactly one calendar month." >> 每个月都会有 SALE, 要么 manager 要么 holiday, 要么
both. 反正最少有一个

"In any given month, if a manager wishes to clear out particular Line of merchandise,
then a managers’ sale is declared, If a holiday falls within the calendar month And there
is excess merchandise in the warehouse never contains excess merchandise"
>> manager sale 发生的条件 和 manager sale 和 holiday sale 同时发生的结果。所以不用
管这段就可以得到答案 D.

A 讲 holiday sale 的条件,错,原文没讲


B, E,无关
C. 根据后句关于 both sale 的结果, 无法判断

"提干我都没读懂?那个粗体字,是否有问题?" >> 没问题

9. A scientific theory is a good theory if it satisfies two requirements, It must accurately


describe a large class of observations in terms of a model that is simple enough to
contain only a few elements, and it must make definite predictions about the results of
future observations, For example, Aristotle’s cosmological theory, which claimed that
everything was made out of four elements---earth, air, fire, and water---satisfied the first
requirement, but it did not make any definite, Thus, Aristotle’s cosmological theory was
not a good theory.
If all the statements in the passage are true, each of the following must also be true
EXCEPT:
(A) Prediction about the results of future observations must be made by any good
scientific theory.
(B) Observation of physical phenomena was not a major concern in Aristotle’s
cosmological Theory
(C) Four elements can be the basis of a scientific model that is simple enough to meet

68
the Simplicity criterion of a good theory.
(D) A scientific model that contains many elements is not a good theory
(E) Aristotle’s cosmological theory described a large class of observations in terms of
only four elements.

10. Millions of irreplaceable exhibits in natural history museums are currently allowed to
decay. Yet without analyses of eggs from museums, the studies linking pesticides with
the decline of Birds of prey would have been impossible Therefore, funds must be raised
to preserve at least Those exhibits that will be most valuable to science in the future

The argument presupposes that:

(A) If a museum exhibit is irreplaceable, its preservation is of an importance that


overrides economic considerations.

(B) The scientific analysis of museum exhibits can be performed in nondestructive way

(C) Eggs of extinct species should be analyzed to increase knowledge of genetic


relationships among species

(D) It can be known at this time what data will be of most use to scientific investigators in
the future

(E) The decay of organic material in natural history exhibits is natural and cannot be
prevented

答案:D,不明白 D 为什么是假设。

原文说我们一定要做 XXXXX, 其中一个假设就是 XXXXX is doable. 这里就是我们可以知道


哪些是最有价值的. 同理, 假设还可以是: we can possibly raise some 或是 the decay can
be prevented.

此题的迷惑选项教多:
A. 错, 因为没有讨论 economic consideration. 如果原文提到没有钱或经济不允许, 此选项
才可能对

B. 错, 无关, 因为原文未提到要在实验中保护标本. 如果提到要永久地保护标本, 就对

C. 错, 无关, genetic relation 不在原文讨论范围内

69
E. 错, 与原文想反.

12. The ancient Egyptian pharaoh Akhenaten, who had a profound effect during his
lifetime on Egyptian art and religion, was well loved and highly respected by his subjects.
We know this from the fierce loyalty show to him by his palace guards, as documented in
reports written during Akhenaten’s reign.
A questionable technique used in the argument is to
(A) introduce information that actually contradicts the conclusion
(B) rely on evidence that in principle would be impossible to challenge
(C) make a generalization based on a sample that is likely to be unrepresentative
(D) depend on the ambiguity of the term “ancient”
(E) apply present-day standards in an inappropriate way to ancient times

个人总结出来的 LSAT 中的 logic fallacy 类型(不完整, 还在整理中):

- 诉诸感情而非逻辑
- 诉诸权威人士
- 人格攻击
- 偷换概念
- 以偏盖全
- (A->B 不能推出 B->A)

这题是属于以偏盖全, 即卫兵并不能代表所有臣民.

13.Physician: The patient is suffering either from disease X or else from disease Y, but
there is no available test for distinguishing X from Y. Therefore, since there is an effective
treatment for Y but no treatment for X, we must act on the assumption that the patient
has a case of Y.
The physician’s reasoning could be based on which one of the following principles?
(A) In treating a patient who has one or the other of two diseases, it is more important to
treat the diseases than to determine which of the two diseases the patient has.
(B) If circumstances beyond a decision maker’s control will affect the outcome of the
decision maker’s actions, the decision maker must assume that circumstances are
unfavorable.
(C) When the soundness of a strategy depends on the truth of a certain assumption, the
first step in putting the strategy into effect must be to test the truth of this assumption.
(D) When success is possible only if a circumstance beyond one’s control is favorable,
then one’s strategy must be based on the assumption that this circumstance is in fact
favorable.

70
(E) When only one strategy carries the possibility of success, circumstances must as
much as possible be changed to fit this strategy.
做的时候选择了 E
但答案是 D
可看了半天也 看不出 D 比 E 有多好

1. E 排除的原因是太 STRONG----绝对词 ONLY


2.D 实际上是原文的重写.原文: 只有假定是 Y 病,才有治好的可能(如果假定是 X 病,没有
效方法治,又不能区分得 Y 或 X 病), 所以,我们必须假定是 Y 病来治. D 选项(本质意思):
当只有假定某种有利情况,才可能成功时,我们必须按这种假定去行动.

iv14. Consumer advocate: tropical oils are high in saturated fats, which increase the risk
of heart disease fortunately, in most prepared food tropical oils can be replaced by
healthier alternatives without noticeably affecting taste. therefore, intensive publicity
about the disadvantage of tropical oils will be likely to result in dietary changes that will
diminish many people’s risk of developing heart disease

nutritionist: the major sources of saturated fat in the average north american diet are
meat, poultry, and dairy products, not tropical oils. thus, focusing attention on the health
hazards of tropical oils would be counterproductive, because it would encourage people
to believe that more substantial dietary changes are unnecessary.

which one of the following is a point at issue between the nutritionist and the consumer
advocate?

(a) whether a diet that regularly includes large quantities of tropical oil can increase the
risk of heart disease

(b) whether intensive publicity campaigns can be effective as means of changing


people’s eating habits

(c) whether more people in north american would benefit from reducing the amount of
meat they consume than would benefit from eliminating tropical oils from their diets.

(d) whether some people’s diets could be made significantly healthier if they replaced all
tropical oils with vegetable oils that are significantly lower in saturated fat

(e) whether conducting a publicity campaign that by focusing on the health hazards of
tropical oils persuades people to replace such oils with healthier alternatives is a good
public-health strategy
好长啊!我选了 b,可是却错了!答案是 e 啊!怎么想啊??

71
15. People who take what others regard as a ridiculous position should not bother to say,
“I mean every word!” For either their position truly is ridiculous, in which case insisting
that they are serious about it only exposes them to deeper embarrassment, or else their
position has merit, in which case they should meet disbelief with rational argument rather
than with assurances of their sincerity.

Which one of the following arguments is most similar in its reasoning to the argument
above?

(A) A practice that has been denounced, as poor practice should not be defended on the
grounds that “this I show we have always done it.” If the practice is a poor one, so much
the worse that is has been extensively used; if it is not poor one, there must be a better
reason for engaging in it than inertia

(B) People who are asked why they eat some of the unusual foods they eat some of the
unusual foods they eat should not answer, “because that is what I like, “This sort of
answer will sound either na?ve or evasive and thus will satisfy no one

(C) People whose taste in clothes is being criticized should not replay, “Every penny I
spent on these clothes I earned honestly.” For the issue raise by the critics is not how the
money was come by but rather whether it was spent wisely.

(D) Scholars who champion unpopular new theories should not assume that the
widespread rejection of their ideas shows that they “must be no the right track.” The truth
is that few theories of any consequence are either wholly right or wholly right or wholly
wrong and thus there is no substitute for patient works in ascertaining which parts are
right.

(E) People who set themselves goals that others denounce as overly ambitious do little to
silence their critics if they say, “I can accomplish this if anyone can. “Rather, those people
should either admit that their critics are right or not dignify the criticism with any reply.

Answer:A.

the line of reason of the passage is that :


we can not defend our position by saying something...becuase do so would not benefit
us in both situation, whether or not our position is correct...

the answer A has similar logic structure..

72
19. smith: meat in the diet is healthy, despite what some people say. after all, most
doctors do eat meat, and who knows more about health than doctors do?

which one of the following is a flaw in smith’s reasoning?

(a) attacking the opponents’ motives instead of their argument

(b) generalizing on the basis of a sample consisting of a typical cases

(c) assuming at the outset what the argument claims to establish through reasoning

(d) appealing to authority, even when different authorities give conflicting advice about an
issue

(e) taking for granted that experts do not act counter to what, according to their expertise,
in their best interest
这道题我选了 c 可是 c 是什么意思呢??答案 是 e

題目: most doctors do eat meat, and who knows more about health than doctors do?
大部份 doctors 吃肉, 誰還會比 doctors 更了解 health (作者假設既然 doctors 最了解 health,
他們都吃肉, -->那肉是 healthy

答案 e: 作者"理所當然地認為" experts (指 doctors) 不會違背專業而作出相反的事.

事實上 doctors 也可以在明知 meat 不利於健康的情況下而吃肉.

21. During construction of the Quebec Bridge in 1907, the bridge’s designer, Theodore
Cooper, received word that the suspended span being built out from the bridge’s
cantilever was deflecting downward by fraction of an inch. Before he could telegraph to
freeze the project, the whole cantilever arm broke off and plunged, along with seven
dozens workers, into the St. Lawrence River. It was the worst bridge construction disaster
in history. As a direct result of the inquiry that followed, the engineering “rules of thumb”
by which thousands of bridges had been built went down with the Quebec Bridge
Twentieth-century bridge engineers would thereafter depend on far more rigorous
applications of mathematical analysis.

Which one of the following statements can be properly inferred from the passage?

(A) Bridges built before about 1907 were built without thorough mathematical analysis
and therefore, were unsafe for the public to use

(B) Cooper’s absence from the Quebec Bridge construction site resulted in the breaking

73
off of the cantilever.

(C) Nineteenth-century bridge engineers relied o their rules of thumb because analytical
methods were inadequate to solve their design problems.

(D) Only a more rigorous application of mathematical analysis to the design of the
Quebec Bridge could have prevented its collapse

(E) Prior to 1907 the mathematical analysis incorporated in engineering rules of thumb
was insufficient to completely assure the safety of bridges under construction.
我觉得选 D 要更好一些呢,虽然答案是 E
文章推理不出 07 年前就是不能用数学方法来计算设计的安全性啊,也可能很早就能用只是
一直没有采用呢

74
Set 4-1

3. Mayor of Plainsville: In order to help the economy of Plainsville, I am using some of


our tax revenues to help bring a major highway through the town and thereby attract new
business to Plainsville. Citizens' group: You must have interests other than our economy
in mind. If you were really interested in
helping our economy, you would instead allocate the revenues to building a new
business park. Since it would bring in twice the business that your highway would.

The argument by the citizens; group relies on which one of the following assumptions?

(A) Plainsville presently has no major highways running through it.


(B) The mayor accepts that a new business park would bring in more new business than
would the new highway.
(C) The new highway would have no benefits for Plainsville other than attracting new
business.
(D) The mayor is required to get approval for all tax revenue allocation plans from the city
council.
(E) Plainsville's economy will not be helped unless a new business park of the sort
envisioned by the citizens' group is built.

这题好像见过,可是没找着。答案是 B,C 不是更好吗?不服。

I have seen this one before. Some said C is the answer and some said B. I would pick B.
Other benefit is out of scope in this problem. The argument is based on ecomic factors
and new businesses. Other benefits in choice C might include cultural and political ones.

B is correct in that if the Mayor used another approach to come down to a totally different
projection that business park would attract fewer new businesses than would the
highway, his decision to build the highways is then fully justufied. the answer can also be
like "There is reason to believe that a business park will indeed attract twice as many
new busnesses as will the highway". In another word, if the Citizen's group's projection is
wrong, the argument is flawed.

5. Recently, highly skilled workers in Eastern Europe have left jobs in record numbers to
emigrate to the West. It is therefore likely that skilled workers who remain in Eastern
Europe are in high demand in their home countries.

Which one of the following, if true, most seriously weakens the argument?

(A) Eastern European factories prefer to hire workers from their home countries rather

75
than to import workers from abroad.

(B) Major changes in Eastern European economic structures have led to the elimination
of many positions previously held by the highly skilled emigrants.

(C) Many Eastern European emigrants need to acquire new skills after finding work in the
West.

(D) Eastern European countries plan to train many new workers to replace the highly
skilled workers who have emigrated.

(E) Because of the departure of skilled workers from Eastern European countries, many
positions are now unfilled.
答案是 B,

b infers that why those who remain have not emganted is not because they are in high
demand but because their positions have not eliminated by economic changes.

4-1-6
6. Historian: Alexander the Great should not be judged by appeal to current notions of
justice. Alexander, an ancient figure of heroic stature, should be judged by the standards
of his own culture. That is, did he live up to his culture's ideals of leadership? Did
Alexander elevate the contemporary standards of justice? Was he, in his day, judged to
be a just and wise ruler?

Student: But you cannot tell whether or not Alexander raised the contemporary standards
of justice without invoking standards other than those of his own culture.

Which one of the following argumentative strategies does the student use in responding
to the historian?

(A) arguing that applying the historian's principle would require a knowledge of the past
that is necessarily inaccessible to current scholarship

(B) attempting to undermine the historian's principle by showing that some of its
consequences are inconsistent with each other

(C) showing that the principle the historian invokes, when applied to Alexander, does not
justify the assertion that he was heroic

(D) questioning the historian's motivation for determining whether a standard of behavior
has been raised or lowered

76
(E) claiming that one of the historian's criteria for judging Alexander is inconsistent with
the principle that the historian has advanced

答案是 e
你如果不问这道题,我还一直以为把这道题中的问题都弄明白了. 不过,放心,现在弄明白了.
这道题的原因在于 contemporary 这个词上面, Marriam Webster 上面的解释有两个: (1)
existing during the same period of time(2) current, 这样的话, 在 historian 的论述中就存在
这样一个问题: contemporary 究竟是前面否定的 current 的意思,还是指"跟 Alexander 同时
代 的 意 思 ". 但 是 , 从 那 个 学 生 的 反 驳 : 你 无 法 判 断 出 Alexander 在 提 及 contemporary
standards 的时候是否不会提到他那个时代以外的一些 standards, 从这里就很明显了,这个
学生所说的 contemporary 是指 current, 所以, 在 historian 的话里面, 前面先说不可以用现
在 的 标 准 去 评 价 Alexander(Alexander the Great should not be judged by appeal to
current notions of justice), 后面在例举中却又说应该看看 A 这个人是否提到现在的标准
( Did Alexander elevate the contemporary standards of justice?), 前后矛盾了.

Two paleontologists, Dr Tyson and Dr. Rees, disagree over the interpretation of certain
footprints that were left among other footprints in hardened volcanic ash at site G. Dr.
Tyson claims they are clearly early hominid footprints since they show human
characteristics: a squarish heel and a big toe immediately adjacent to the next toe.
However, since the footprints indicate that if hominids made those prints they would have
had to walk in an unexpected cross-stepping manner, by placing the left foot to the right
of the right foot. Dr. Rees rejects Dr. Tyson's conclusion.

7. The disagreement between the two paleontologists is over which one of the following?

(A) the relative significance of various aspects of the evidence

(B) the assumption that early hominid footprints are distinguishable from other footprints

(C) the possibility of using the evidence of footprints to determine the gait of the creature
that made those footprints

(D) the assumption that evidence from one paleontologic site is enough to support a
conclusion

(E) the likelihood that early hominids would have walked upright on two feet

答案:A,我选了 C。

77
9. It is not known whether bovine spongiform encephalopathy (BSE), a disease of cattle
invariably deadly to them, can be transmitted directly from one infected animal to another
at all stages of the infection. If it can be, there is now a reservoir of infected cattle
incubating the disease. There are no diagnostic tests to identify infected animals before
the animals show overt symptoms. Therefore, if such direct transmission occurs, the
disease cannot be eradicated by ____

Which one of the following best completes the argument?

(A) removing from the herd and destroying any diseased animal as soon as it shows the
typical symptoms of advanced BSE

(B) developing a drug that kills the agent that cause BSE, and then treating with that drug
all cattle that might have the disease

(C) destroying all cattle in areas where BSE occurs and raising cattle only in areas to
which BSE is known not to have spread

(D) developing a vaccine that confers lifelong immunity against BSE and giving it to all
cattle, destroying in due course all those animals for which the vaccine protection came
too late

(E) developing a diagnostic test that does identify any infected animal and destroying all
animals found to be infected

答案:A,我选 了 C。

cos no test can identify weather the cattle is infected before it shows symptom of bse,
and if direct infection exists ,so if we only kill the cattles showing symptoms of bse, we
may leave some cattles infected but showing no symptoms,so these cattles may infect
others.
因为这种病传染初期没法确认,所以光是把那些表现出病症的全杀光是没有用的,因为很
多得病的还没有发出来,这样是根治不了的
你实在不理解就把那种病换成乙肝想想,呵呵,这个例子应该能解决问题的

10. Auto industry executive: Statistics show that cars that were built smaller after 1977 to
make them more fuel-efficient had a higher incidence of accident-related fatalities than
did their earlier larger counterparts. For this reason we oppose recent guidelines that
would require us to produce cars with higher fuel efficiency.
Which of the following, if true, would constitute the strongest objection to the executive's
argument?
(A) Even after 1977, large automobiles were frequently involved in accidents that caused

78
death or serious injury.
(B) Although fatalities in accidents involving small cars have increased since 1977, the
number of accidents has decreased.
(C) New computerized fuel systems can enable large cars to meet fuel efficiency
standards established by the recent guidelines.
(D) Modern technology can make small cars more fuel-efficient today than at any other
time in their production history.
(E) Fuel efficiency in models of large cars rose immediately after 1977 but has been
declining ever since.

答案 C

executive 的 argument 是 为 了 车 more fuel-efficient, 要 把 车 做 得 小 了 , 造 成 了 higher incidence . 因 此 反 对


produce cars with higher fuel efficiency.executive 的隐含的假设是 if the car is more fuel-efficient,then the car is
smaller size.
而 C 就 weaken 了这个隐含的假设,指出 larger size car can be more fuel-efficient.从而 weaken this argument.

终于想明白,原文等价于:
省油---> 小汽车
小汽车---> 更多交通事故
所以 省油----> 更多交通事故
所以不要省油

WEAKEN: 省油不一定---> 小汽车。 否定前提...

我原来的问题主要在于把原文理解成了:
小汽车---->省油
省油-----> 交通事故
所以不要省油

又学到一点:
do A to achieve B 等价于 B-----> A
A that are to achive B suffer from C effect 是 A-----> C 而不是 B------>C

I think A ensure B ==> A-->B. Ensure indicates that B will happen in the presence of A. I saw this problem before.
What are the choices?

12. Impact craters caused by meteorites smashing into Earth have been found all around
the globe, but they have been found in the greatest density in geologically stable regions.
This relatively greater abundance of securely identified crater in geologically stable

79
regions must be explained by the lower rates of destructive
geophysical processes in those regions.
The conclusion is properly drawn if which one of the following is assumed?
(A) A meteorite that strikes exactly the same spot as an earlier meteorite will obliterate all
traces of the earlier impact.
(B) Rates of destructive geophysical processes within any given region vary markedly
throughout geological time.
(C) The rate at which the Earth is struck by meteorites has greatly increased in
geologically recent times.
(D) Actual meteorite impacts have been scattered fairly evenly over the Earth's surface in
the course of Earth's geological history.
(E) The Earth's geologically stable regions have been studied more intensively by
geologists than have its less stable regions.
answer:D my choice:B

D 选项是排除他因,说明 This relatively greater abundance of securely identified crater in


geologically stable regions 不是由于 meteorite impacts 造成的。

原文逻辑:

This relatively greater abundance of securely identified crater in geologically stable regions
——>the lower rates of destructive in geologically stable regions

B、Rates of destructive geophysical processes within any given region vary markedly
throughout geological time 是个无关选项

15. M: It is almost impossible to find a person between the ages of 85 an 90 who


primarily uses the left hand.
Q: Seventy to ninety years ago, however, children were published for using their left
hands to eat or to write and were forced to use their right hands.

Q's response serves to counter any use by M of the evidence about 85 to 90 year olds in
supports of which one of the following hypotheses?

(A) Being born right-handed confers a survival advantage.


(B) Societal attitudes toward handedness differ at different times.
(C) Forcing a person to switch from a preferred hand is harmless.
(D) Handedness is a product of both genetic predisposition and social pressures.
(E) Physical habits learned in school often persist in old age.

80
答案 A,我选 D。
You misunderstood the question. It asks what M's argument is. D is the opinion and
argument of Q's. A is M's argument.

18. Mary, a veterinary student, has been assigned an experiment in mammalian


physiology that would require her to take a healthy, anesthetized dog and subject it to a
drastic blood loss in order to observe the physiological consequences of shock. The dog
would neither regain consciousness nor survive the experiment. Mary decides not to do
this assignment.

Mary's decision most closely accords with which one of the following principles?

(A) All other things being equal, gratuitously causing any animal to suffer pain is
unjustified.
(B) Taking the life of an animal is not justifiable unless doing so would immediately assist
in saving several animal lives or in protecting the health of a person.
(C) The only sufficient justification for experimenting on animals is that future animal
suffering is thereby prevented.
(D) Practicing veterinarians have a professional obligation to strive to prevent the
unnecessary death of an animal except in cases of severely ill or injured animals whose
prospects for recovery are dim.
(E) No one is ever justified in acting with the sole intention of causing the death of a living
thing, be it animal or human.

答案 B,请帮忙分析一下。

关 键是 原文 的提 示信 息。 The dog would neither regain consciousness nor survive the


experiment.

B 中 Taking the life of an animal 和原文 not survive 一致。

A 中 suffer pain 和原文不是最接近。。


C justification for experimenting on animals 没提到能否让狗免死。
D have a professional obligation to strive to prevent the unnecessary death of an animal
虽然也提到了免死。。但和 B 的区别在于,这里强调的是救护动物。而 B 强调的是不去杀害
动物。所以 B 更接近原文。
E No one is ever justified in acting with the sole intention of causing the death of a living
thing, be it animal or human
其中的 sole intention 和原文不符,原文是做实验。而且加了 human,扩大了范围。

81
19. A tree's age can be determined by counting the annual growth rings in its trunk. Each
ring represents one year, and the ring's thickness reveals the relative amount of rainfall
that year. Archaeologists successfully used annual rings to determine the relative ages of
ancient tombs at Pazyryk. Each tomb was constructed from freshly cut logs, and the
tombs builders were constrained by tradition to use only logs from trees growing in the
sacred Pazyryk Valley.

Which one of the following, if true, contributes most to an explanation of the


archaeologists' success in using annual rings to establish the relative ages of the tombs
at the Pazyryk site?

(A) The Pazyryk tombs were all robbed during ancient times, but breakage of the tombs
seals allowed the seepage of water, which soon froze permanently, thereby preserving
the tombs' remaining artifacts.

(B) The Pazyryk Valley, surrounded by extremely high mountains, has a distinctive yearly
pattern of rainfall, and so trees growing in the Pazyryk Valley have annual rings that are
quite distinct from trees growing in nearby valleys.

(C) Each log in the Pazyryk tombs has among its rings a distinctive sequence of twelve
annual rings representing six drought years followed by three rainy years and three more
drought years.

(D) The archaeologists determined that the youngest tree used in any of the tombs was
90 years old and that the oldest tree was 450 years old.

(E) All of the Pazyryk tombs contained cultural artifacts that can be dated to roughly 2300
years ago.

答案是 C,是可以理解成要去历史纪录里面查找具备这样特征的年份,已确定 tomb 的年代


吗?

only c implies that the rings of logs in P tombs contain characteristics by which can be applied to the method of
determing the ages by the rings in their trunks and of revealing the rain fall by the thickness of their rings.

21. Whenever a major political scandal erupts before an election and voters blame the
scandal on all parties about equally, virtually all incumbents, from whatever party, seeking
reelection are returned to office. However, when voters blame such a scandal on only
one party, incumbents from that party are likely to be defeated by challengers from other
parties. The proportion of incumbents who seek reelection is high and remarkably

82
constant from election to election.
If the voters’ reactions are guided by a principle, which one of the following principles
would best account for the contrast in reactions described above?
(A) Whenever one incumbent is responsible for one major political scandal and another
incumbent is responsible for another, the consequences for the two incumbents should
be the same.
(B) When a major political scandal is blamed on incumbents from all parties, that
judgment is more accurate than any judgment that incumbents from only on party are to
blame.
(C) Incumbents who are rightly blamed for a major political scandal should not seek
reelection, but if they do, they should not be returned to office.
(D) Major political scandals can practically always be blamed on incumbents, but whether
those incumbents should be voted out of office depends on who their challengers are.
(E) When major political scandals are less the responsibility of individual incumbents than
of the parties to which they belong, whatever party was responsible must be penalized
when possible.

答案是 E,我不知道为什么

The question stem asks for a principle that would account for voters' actions. The last
sentence has nothing to do with voters' actions, so it is an essentially filler used to
create wrong answer choices. This is a tough one for many reasons, among them, (1) the
credit one is at (E), (2) the priciple also serves as conclusion in the passage, needs an
assumption which is not explicitly noted in the passage, and (3) as principle questions,
some extent of imprecision is allowed and should be tolerated.

To paraphrase the stem, when voters blame a scandal equally on all parties, almost all
incumbents win reelection. But when a scandal is blamed on a single party, incumbents
from that party tend to lose. The assumption here is "it is not the incumbents who should
be held responsible for the scandal."

(A): This deals with what should happen to individual incumbents who are responsible for
scandals, but the stimulus just describes how voters treat parties or incumbents as
members of their parties.
(B): This makes a judgment about the likely accuracy of the judgments of blame, but
nothing of the kind is mentioned in the passage.
(C): This makes the mistake of concentrating on individuals, instead of parties. Moreover,
(C) runs contrary to the passage. According to (C), in cases where all parties are blamed
for a scandal, all incumbents should be turned out.
(D): The passage doesn't imply that voters evaluate the challengers. On the contrary,
voters are depicted as making choices depending on how they believe about the
incumbents' parties.

83
(E) is correct. According to (E) and the assumption noted at start of it, if Party X is
responsible for a scandal, voters should try to punish Party X by voting out its
incumbents; however, if all the parties are responsible, there's not much voters can do -
whomever they vote for would be a member of one of the offending parties, so they might
as well vote for the incumbents. Thus (E) accounts for the contrast in voter reactions.

Of course, even without the assumption at the beginning of (E), the principle stated here
is still hold to account for the voters' actions.

24. Certain minor peculiarities of language are used unconsciously by poets. If such
peculiarities appear in the works of more than one poet, they are likely to reflect the
language in common use during the poets' time. However, if they appear in the work of
only one poet, they are likely to be personal idiosyncrasies. As such, they can provide a
kind of "fingerprint" that allows scholars, by comparing a poem of previously unknown
authorship to the work of a particular known poet, to identify the poem as the work of that
poet.

For which on of the following reasons can the test described above never provide
conclusive proof of the authorship of any poem?

(A) The labor of analyzing peculiarities of language both in the work of a known poet and
in a poem of unknown authorship would not be undertaken unless other evidence already
suggested that the poem of unknown authorship was written by the known poet.

(B) A peculiarity of language that might be used as an identifying mark is likely to be


widely scattered in the work of a poet, so that a single poem not known to have been
written by that poet might not include that peculiarity.

(C) A peculiarity of language in a poem of unknown authorship could be evidence either


that the poem was written by the one author known to use that peculiarity or that the
peculiarity was not unique to that author.

(D) Minor peculiarities of language contribute far less to the literary effect of any poem
than such factors as poetic form, subject matter, and deliberately chosen wording.

(E) A poet's use of some peculiarities of language might have been unconscious in some
poems and conscious in other poems, and the two uses would be indistinguishable to
scholars at a later date.

答案是 C

24 的大意是说诗人一般在作品中显露出自己的特性,从而可以让人辨别。如果一首以上的

84
诗里有这个特性,那么这种特性一般是当时的大众语言特性。但是如果只有一首诗有这个特
性那么这是个人的语言特性。因此这种特性象指纹一样可以让学者辨认一个不知道作者是谁
的诗歌是不是某个有名的诗人的作品。

问题问下面哪个无法让上文中提及方法判断诗歌的作者是谁。 A peculiarity of language in


a poem of unknown authorship could be evidence either that the poem was written by the
one author known to use that peculiarity or that the peculiarity was not unique to that
author.
就是说当一首没有确定作者的诗的特性要么是这个作者一向喜欢用的风格(容易是大众风
格〕,或者不是作者显著的风格时候就无法判断。

25. Because of the recent transformation of the market. Quore, Inc., must increase
productivity, 10 percent over the course of the next two years, or it will certainly go
bankrupt. In fact, however, Quore’s production structure is such that if a 10 percent
productivity increase is possible, then a 20 percent increase is attainable.
If the statements above are true, which one of the following must on the basis of them
also be true?
(A) It is only Quore’s production structure that makes it possible for Quore to survive the
transformation of the market.
(B) Quore will not go bankrupt if it achieves a productivity increase of 20 percent over the
next two years.
(C) If the market had not been transformed, Quore would have required no productivity
increase in order to avoid bankruptcy.
(D) Because of the transformation of the market, Quore will achieve a productivity
increase of 10 percent over the next two years.
(E) If a 20 percent productivity increase is unattainable for Quore, then it must go
bankrupt.

The answer is E. Why not B?

应该是 非 A->B
其实,由条件 1.Quore, Inc. must increase productivity, 10 percent over the course of the
next two years, or it will certainly go bankrupt。中的‘OR“可以推出 if not increase,it will go
bankupt.that is not A->B
由条件 2 if a 10 percent productivity increase is possible, then a 20 percent increase is
attainable 可以推出 10 percent increase will lead to a 20 percent increase. that is A ->
C ,not C-> not A,
conclude from 1 and 2,not C -> B

85
Section 4-3

5. Animals with a certain behavioral disorder have unusually high level of aluminum in
their brain tissue.Since a silicon-based compound binds to aluminum and prevents it from
affecting the brain tissue. Animals can be cured of the disorder by being treated with the
compound.
The argument is based on which one of the following assumptions?
(A) Animals with the disorder have unusually high but invariable levels of aluminum in
their brain tissue.
(B) Aluminum is the cause of the disorder rather than merely an effect of it.
(C) Introducing the compound into the brain tissue has no side effects.
(D) The amount of the compound needed to neutralize the aluminum in an animal's brain
tissue varies depending upon the species.
(E) Aluminum is never present in normal brain tissue.
answer:B
B)相当于一个“治标治本”的问题,如果 Aluminum 仅仅是个“ merely an effect of ”,即使
用了药控制了 Aluminum ,也不能 cure the disorder,因为 Aluminum 不是病原,只是个症
状而且。换句话说,控制了 Aluminum ,这种 disorder 依然存在,只是可能以别的症状出现
而已。
D)不对,因为文中的结论是“ cured of the disorder ”,不涉及是否有副作用或其它影响。

6. As air-breathing mammals. Whales must once have lived on land and needed hind
limbs capable of supporting the mammals' weight. Whales have the bare remnants of a
pelvis. If animals have a pelvis, we expect them to have hind limbs. A newly discovered
fossilized whale skeleton has very fragile hind limbs that could not have supported the
animal's weight on land. This skeleton had a partial pelvis.
If the statement above are true, which one of the following, if also true, would most
strongly support the conclusion that the fragile hind limbs are remnants of limbs that land-
dwelling whales once had?
(A) Whale bones older than the fossilized hind limbs confirm that ancient whales had full
pelvises.
(B) No skeletons of ancient whales with intact hind limbs capable of supporting the
mammals' weight have ever been found.
(C) Scientists are uncertain whether the apparently nonfunctioning limbs of other early
mammals derived from once-functioning limbs of their ancestors.
(D) Other large-bodied mammals like seals and sea lions maneuver on beaches and
rocky coasts without fully functioning hind limbs.
(E) Some smaller sea-dwelling mammals. Such as modern dolphins. Have no visible
indications of hind limbs.

86
whale 必须有 Full pelvis,才能有支撑体重的 Hind limb,然后 pelvis 退化成 patial pelvis,
limb 变成 fragile hind limb。这是的后腿已经不能支撑鲸鱼庞大的体重了。本题为加强题。所
以答案重要出现 whale 有 Full pelvis 的论据,选项 a 恰好是这个意思。
8. Older United States automobiles have been identified as contributing
disproportionately to global air pollution. The requirement in many jurisdictions that
automobiles pass emission-control inspections has had the effect of taking many such
automobiles out of service in the United States. As they fail inspection and their owners
opt to buy newer automobiles. Thus the burden of pollution such older United States
automobiles contribute to the global atmosphere will be gradually reduced over the next
decade.

Which one of the following, If true. Most seriously weakens the argument?

(A) It is impossible to separate the air of one country or jurisdiction from that of others.

(B) When automobiles that are now new become older, they will, because of a design
change. Cause less air pollution than older automobiles do now.

(C) There is a thriving market for used older Untied States automobiles that are exported
to regions that have no emission-control regulations.

(D) The number of jurisdictions in the United States requiring automobiles to pass
emission-control inspections is no longer increasing.

(E) Even if all the older automobiles in the United States were retired from service. Air
pollution from United States automobiles could still increase if the total number of
automobiles in use should increase significantly.

答案:C,我觉得 B,C,E 都有削弱作用。

仔细看一下题目就会发现文中最后说的是 older United States automobiles 的 contribute


那么只要这些车还在世上跑就一定会有污染,文章并没有指出这个污染是在美国还是其他国家,是指这些
车的污染。
而正好第三世界的兄弟们又没钱买新车,所以这些破车对世界环境的污染更本就不会减轻了。

9. The journalistic practice of fabricating remarks after an interview and printing them
within quotation marks, as if they were the interviewee's own words, has been decried as
a form of unfair misrepresentation. However, people's actual spoken remarks rarely
convey their ideas as clearly as does a distillation of those ideas crafted, after an
interview, by a skilled writer. Therefore, since this practice avoids the more serious
misrepresentation that would occur if people's exact words were quoted but their ideas
only partially expressed, it is entirely defensible.

87
Which one of the following is a questionable technique used in the argument?

(A) answering an exaggerated charge by undermining the personal authority of those


who made that charge .
(B) claiming that the prestige of a profession provides ample grounds for dismissing
criticisms of that profession.
(C) Offering as an adequate defense of a practice an observation that discredits only one
of several possible alternatives to that practice.
(D) concluding that a practice is right on the grounds that it is necessary.
(E) using the opponent's admission that a practice is sometimes appropriate as
conclusiveproof that that practice is never inappropriate.

1.推理为:因为完全引述 INTERVIEWEE 的话这种做法不好,将原话处理这种做法比较好,所


以用处理的方法完全合理的.错误在:只是怀疑一种做法不好,便充分的认为另一种方法完全
好 , 而 排 除 其 它 可 能 更 好 的 方 法 . 即 C. 其 实 C 的 "ADEQUATE DEFENSE" 很 好 对 应 原
文"ENTIRELY DEFENSIBLE"

11. Professor Smith published a paper arguing that a chemical found in minute quantities
in most drinking water had an adverse effect on the human nervous system. Existing
scientific theory held that no such effect was possible because there was no neural
mechanism for bringing it about. Several papers by well-known scientists in the field
followed, unanimously purporting to prove Professor Smith wrong. This clearly shows
that the scientific establishment was threatened by Professor Smith's work and conspired
to discredit it.

Which one of the following is the central flaw in the argument given by the author of the
passage?

(A) The author passes over the possibility that Professor Smith had much to gain should
Professor Smith's discovery have found general acceptance.

(B) The author fails to mention whether or not Professor Smith knew that the existence of
the alleged new effect was incompatible with established scientific theory.

(C) The author fails to show why the other scientists could not have been presenting
evidence in order to establish the truth of the matter.

(D) The author neglects to clarify what his or her relationship to Professor Smith is.

(E) The author fails to indicate what, if any, effect the publication of Professor Smith's
paper had on the public's confidence in the safety of most drinking water.

88
Answer: C

c 的意思是作者没有说明为何科学家们不能为说明事实而举出证据.
文章说,反对一种新的理论就是一种 conspiracy,其实反对者可能是实事求是地说明这个问题的

12. The number of North American children who are obese-that is who have more body
fat than do 85 percent of North American children their age -is steadily increasing,
according to four major studies conducted over the past 15 years.
If the finding reported above is correct, it can be properly concluded that
(A) when four major studies all produce similar results .those studies must be accurate.
(B) North American children have been progressively less physically active over the past
15 years.
(C) The number of North American children who are not obese increased over the past
15 years.
(D) Over the past 15 years ,the number of North American children who are underweight
has declined.
(E) The incidence of obesity in North American children tends to increase as the children
grow older.
这个数学题,我看了几遍觉得没有一个时正确的答案,,
答案是 c

公式如下:胖子数量=胖子在总人口的比例(15%)*总人口,那么瘦子=瘦子在总人口的
比例(85%)*总人口;比例不变,而胖子多了,那么总人口一定也多了,那么瘦子一定也
多了

15. The Scorpio Miser with its special high-efficiency engine costs more to buy than the
standard Scorpio sports car. At current fuel prices , a buyer choosing the Miser would
have to drive it 60,000 miles to make up the difference in purchase price through savings
on fuel .It follows that ,if fuel prices fell ,it would take fewer miles to reach the break-even
point.
Which one of the following arguments contains an error of reasoning similar to that in the
argument above?
(A) The true annual rate of earnings on an interest-bearing account is the annual rate of
interest less the annual rate of inflation drops, the rate of interest can be reduced by an
equal amount without there being a change in the true rate of earnings.
(B) For retail food stores ,the Polar freezer, unlike the Arctic freezer, provides a consistent
temperature that allows the store to carry premium frozen foods. Thus ,if electricity rates
fell ,a lower volume of premium-food sales could justify choosing the Polar freezer .
(C) With the Roadmaker ,a crew can repave a mile of decayed road in less time than with
the competing model, which is ,however, much less expensive. Reduced staffing levels
made possible by the Roadmaker eventually compensate for its higher price .Therefore,
the Roadmaker is especially advantageous where average wages are low.
(D) The improve strain the Northland apple tree bears fruit younger and lives longer than

89
the standard strain .The standard strain .The standard strain does grow larger at maturity
,but to allow for this ,standard trees must be spaced farther apart. Therefore , new
plantings should all be of the improved strain.
(E) Stocks pay dividends ,which vary from year to year depending on profits made.
Bonds pay interest ,which remains constant from year to year . Therefore ,since the
interest earned on bonds does not decrease when economic conditions decline ,
investors decline , investors interested in a reliable income should choose bonds.
这个题目的 error 是什么,是不是题目总的 fewer 有问题,说反了,
碰到这种题目我总是做错,不知道到底怎么比较,能不能说一下正确的答案 c 如何是和题
目的 error 一致的

我觉得原文的错误在于 if fuel prices fell, it would take fewer miles to reach the break-even point.事实上应该是
more miles
C 中 错 在 Therefore, the Roadmaker is especially advantageous where average wages are low. 事 实 上 应 该 是
average wages are high.
感觉上他们犯了同一类逻辑错误。
不过谁能指点一下,这里的共同逻辑错误是什么?

题目中两种机器的差价应该认为是一个定值 每公里节省的油量也应该是一定的,那么如果油价降低,则
需要更多的油量来达到差价,所以题目搞反。
呵呵,我想你找到了共同的错误,都是结论相反

17. Being articulate has been equated with having a large vocabulary. Actually, however,
people with large vocabularies have no incentive for, and tend not to engage in, the kind
of creative linguistic self-expression that is required when no available words seem
adequate. Thus a large vocabulary is a hindrance to using language in a truly articulate
way.

Which one of the following is an assumption made in the argument?

(A) When people are truly articulate, they have the capacity to express themselves in
situations in which their vocabularies seem inadequate.

(B) People who are able to express themselves creatively in new situations have little
incentive to acquire large vocabularies.

(C) The most articulate people are people who have large vocabularies but also are able
to express themselves creatively when the situation demands it.

(D) In educating people' to be more articulate, it would be futile to try to increase the size
of their vocabularies.

(E) In unfamiliar situations, even people with large vocabularies often do not have

90
specifically suitable words available.

答案:A,不明白 A 为什么是假设。

原文的结论是这是一个典型的“能不能的问题”。原文的结论是大的词汇量是清晰表达的障碍,
A 就是当能够说清楚的时候,他们有能力表达....
举个例子这是 gwd 经常说的:joy 在写字。这个结论的 assumption 可以是:joy 有能力写字

Questions 18-19

Dr. Schilling: Those who advocate replacing my country's private health insurance
system with nationalized health insurance because of the rising costs of medical care fail
to consider the high human costs that consumers pay in countries with nationalized
insurance: access to high-technology medicine is restricted. Kidney transplants and
open-heart surgery-rationed. People are denied their right to treatments they want and
need.

Dr. Laforte: Your country's reliance on private health insurance denies access even to
basic, conventional medicine to the many people who cannot afford adequate health
coverage. With nationalized insurance, rich and poor have equal access to life-saving
medical procedures. And people's right to decent medical treatment regardless of income
is not violated.

18. Dr. Schilling's and Dr. Laforte's statements provide the most support for holding that
they would disagree about the truth of which one of the following?

(A) People's rights are violated less when they are denied an available medical treatment
they need because they lack the means to pay for it than when they are denied such
treatment on noneconomic grounds.

(B) Where health insurance is provided by private insurance companies, people who are
wealthy generally receive better health care than do people who are unable to afford
health insurance.

(C) In countries that rely primarily on private health insurance to pay for medical costs,
most people who would benefit from a kidney transplant receive one.

(D) In countries with nationalized health insurance, no one who needs a familiar medical
treatment in order to stay alive is denied that treatment.

(E) Anyone who wants a particular medical treatment has a right to receive that
treatment.

91
答案:A

我是这样理解的,不对的地方请各位逻辑大牛指正:
(1)读完题目,原文的意思是 Dr. Schilling 指出了 nationalized insurance 一些不足,并说 People are denied
their right to treatments they want and need。但是 Dr. Laforte 指出了 private health insurance 一些不足,并说明
了 nationalized insurance 的好处,并得出结论 people's right is not violated。

(2)十八题的意思是两个 D 他们都不同意什么,看到 A 我没有什么感觉,只是和原文的结论直接相关。B


是第一个 d 同意的;C 同 B;D 是第二个 D 同意的;E 同 D;只有 A 比较了 right 但是这个我认为从原文是不
能推出的。so,a.

19. In responding to Dr. Schillihng, Dr. Laforte employs which one of the following
argumentative strategies?

(A) showing that the objections raised by Dr. Schilling have no bearing on the question of
which of the two systems under consideration is the superior system.

(B) Calling into question Dr. Schilling's status as an authority on the issue of whether
consumers' access to medical treatments is restricted in countries with nationalized
health insurance.

(C) Producing counterexamples to Dr. Schilling's claims that nationalized health


insurance schemes extract high human costs from consumers.

(D) Demonstrating that Dr. Schilling's reasoning is persuasive only because of his
ambiguous use of the key word "consumer".

(E) Showing that the force of Dr. Schilling's criticism depends on construing the key
notion of access in a particular limited way

答案:E, 这是什么意思啊?

人在 life-saving 上的理解不同. D 认为那些大手术是 life-saving, 而 S 认为很多基本的 medical care 就应该包


括在 life-saving 的范畴里, 所以 E 对.

22. Nursing schools cannot attract a greater number of able applicants than they
currently do unless the problems of low wages and high-stress working conditions in the
nursing profession are solved. If the pool of able applicants to nursing school does not
increase beyond the current level, either the profession will have to lower its entrance
standards, or there will soon be an acute shortage of nurses. It is not certain, however,

92
that lowering entrance standards will avert a shortage. It is clear that with either a
shortage of nurses or lowered entrance standards of the profession, the current high
quality of health care cannot be maintained.
Which one of the following can be property inferred from the passage?
(A) If the nursing profession solves the problems of low wages and high-stress working
conditions, it will attract able applicants in greater numbers than it currently does.
(B) The nursing profession will have to lower its entrance standards if the pool of able
applicants to nursing school does not increase beyond the current level.
(C) If the nursing profession solves the problems of low wages and high-stress working
conditions, high quality health care will be maintained.
(D) If the nursing profession fails to solve the problems of low wages and high-stress
working conditions, there will soon be an acute shortage of nurses.
(E) The current high quality of health care will not be maintained if the problems of low
wages and high-stress working conditions in the nursing profession are not solved.

答案是 E, 但是 A 为什么不可以

If you picked A, that means you need to know some basic logic reasoning. I suggest that you read other's post and
answers first. Make sure you truly understand some explainations.

25. Situation: In the island nation of Bezun ,the government taxes gasoline heavily in
order to induce people not to drive . It uses the revenue from the gasoline tax to
subsidize electricity in order to reduce prices charged for electricity.
Analysis: The greater the success achieved in meeting the first of these objectives, the
less will be the success achieved in meeting the second .
The analysis provided for the situation above would be most appropriate in which one of
the following situations?
(A) A library charges a late fee in order to induce borrowers to return books promptly. The
library uses revenue from the late fee to send reminders to tardy borrowers on order to
reduce the incidence of overdue books .
(B) A mail-order store imposes a stiff surcharge for overnight delivery in order to limit use
of this option. The store uses revenue from the surcharge to pay the extra expenses it
incurs for providing the overnight delivery service.
(C) The park management charges an admission fee so that a park's users will contribute
to the park's upkeep. In order to keep admission fees low ,the management does mot
finance any new projects from them.
(D) A restaurant adds a service charge in order to spare customers the trouble of
individual tips. The service charge is then shared among the restaurant's workers in order
to augment their low hourly wages.
(E) The highway administration charges a toll for crossing a bridge in order to get
motorists to use other routes .It uses the revenue from that toll to generate a reserve fund
in order to be able one day to build a new bridge.
这一题和上题类似,都是不知道如何相似的,答案是 E,我一个一个地看了半天,不理解!

93
请高手指教一下有没有对付这些题目的高招,我都是一个一个地看,费时很多还是不知道
选哪一个,
不是觉得都不对,就是觉得对了太多,郁闷!

原文意思是说:第一个目标越成功(即鼓励大家不要开车),第二个目标成功的机会越小
(即因为大家不开车,收到的税款越少,所以难于支持电力)
E 的意思一样。。第一个目标越成功(绕道走),第二个目标成功的机会越小(建立第二个桥)
Section 5-2

5-2-3
.In Europe, schoolchildren devote time during each school day to calisthenics. North
American schools rarely offer a daily calisthenics program. Tests prove that North
American children are weaker, slower, and shorter-winded than European children. We
must conclude that North American children can be made physically fit only if they
participate in school calisthenics on a daily basis.

Which one of the following is assumed in the passage?

(A) All children can be made physically fit by daily calisthenics.

(B) All children can be made equally physically fit by daily calisthenics.

(C) Superior physical fitness produces superior health.

(D) School calisthenics are an indispensable factor in European children’s superior


physical fitness.

(E) North American children can learn to eat a more nutritious diet as well as to exercise
daily
答案是 d

我看这道题时首先就排除了 a,b,因为原文中只说了 E 和 NA 的小孩,但并没说他们能代表


全体小孩--这中间有个人推理--就像你说的“就想一个小猪,想要通过训练让它飞上天总是

不可能的吧。道理时对的,但原文没说,我怎么知道呢?---避免读题、 解题过程中自己推理

而 D 恰恰就是排除了 E 小孩棒的其他可能原因。。

请教 mindfree 一 lsat-5-2-4
斑竹还未休息,称此机会讨教一题, 谢谢!

Q4. A work of architecture, if it is to be both inviting and functional for public use, must be
unobtrusive, taking second place to the total environment. Modern architects, plagued by

94
egoism, have violated this precept. They have let their strong personalities take over their
work, producing buildings that are not functional for public use.

Which one of the statements below follows logically from the statements in the passage?

(A) Unobtrusive architecture is both inviting and functional.

(B) Modern architects who let their strong personalities take over their work produce
buildings that are not unobtrusive.

(C) An architect with a strong personality cannot produce buildings that functional well for
the public.

(D) A work of architecture that takes second place to the environment functions well for
public use.

(E) A work of architecture cannot simultaneously express its architect’s personality and
be functional for public use.

答案是 B, 但是按照充分条件必要条件来推理,B 似乎也不对。

原文可简写为两个推理:
(1) if a architecture is both inviting and functional for public useè it is unobtrusive。推理可
表示 both inviting and function for public use 是 unobtrusive 的充分条件。
(2) if modern architects let strong personality take over their worksètheir works will no be
functional for public use. Strong personality take over the work 是 not functional for public
use 的充分条件。

Answer B 说 strong personality take over the workènot unobtrusive 的逻辑在于:strong


personality take over the work==〉not functional for public use==〉not unobtrusive。

我认为在推倒 not functional for public use==〉not unobtrusive 有问题。根据(1) if A and


B, then C.那么能否说 if not B, then not C.原命题的部分否命题似乎不成立。

是我理解有误还是其他问题,请指教。

First of all, you can use POE to eliminate A,C,D,E.

The reasoning in the passage, as you pointed out, are:


"both inviting and function for public use" (1)--> unobtrusive (2)
"strong personality take over the work" (3)-->not functional for public use (not 1)
(2) is required for (1) and (3) is sufficient for Not (1)

From the reasoning above, we cannot get (3)-->(2), as (2) might not be the only reason

95
for (1). In another word, (2) is not sufficient for (1). However, in the passage it says
"Modern architects, plagued by egoism, have violated this precept". So we can safely
assume that the only reason that works from those modern architects who let their strong
personality to take over their work are not functional is that they are unobtrusive. So B is
right.

7. Those who participate in local politics include people who are genuinely interested in
public service and people who are selfish opportunists. Everyone who participates in
local politics has an influence on the community’s values.
If the statements above are true, which one of the following must also be true?
(A) Some selfish opportunists have an influence on the community’s values.
(B) Some persons who are interested in public service do not have an influence on the
community’s values.
(C) All those who have an influence on the community’s values participate in local
politics.
(D) Some of those who influence the community’s values neither are interested in public
service nor are selfish opportunists.
(E) All those who have an influence on the community’s values are either interested in
public service or are selfish opportunists.
answer:A my choice:E

E 的充分和必要关系不对。是选 A。

因为所有参会的人都有影响,而参会的有真正关心的,也有自私的。所以有一些自私的人有
影响。

Although nondairy coffee lighteners made with coconut oil contain 2 grams of saturated
fat per tablespoon, or 7 times more than does whole milk, those lighteners usually
contain no cholesterol. Yet one tablespoon of such lighteners causes the consumer’s
blood cholesterol to rise to a higher level than does an identical amount of whole milk,
which contains 2 milligrams of cholesterol per tablespoon.

8. Which one of the following, if true, contributes most to an explanation of the apparent
discrepancy noted above?

(A) Nutritionists recommend that adults consume as little saturated fat as possible and no
more than 250 milligrams of cholesterol a day.

96
(B) One gram of saturated fat in food has roughly the same effect on blood cholesterol as
25 milligrams of cholesterol in food.

(C) Light cream, a dairy product that contains 5 times more cholesterol than does whole
milk, is often chosen as a lightener by consumers who normally prefer whole milk.

(D) Certain nondairy coffee lighteners made without coconut oil contain less saturated fat
and less cholesterol than does whole milk.

(E) The lower the saturated fat content of dairy products, the less cholesterol they usually
contain.

答案:B,不明白为什么 B 是解释了。

1. 原文指出 coconut milk 中有 SF 而没有 C, 但是在使消费者血液中 C 上升的效果上和含有


C 的 while milk 相同, B 是选项中唯一指出 SF 和 C 在效果上的关系的一个, 应该是正确答案.
我的问题是 B 的数字好象和原文不相符. 不知道题目有没有打错.

9. Manufacturers of coffee lighteners based on coconut oil claim that their products
usually cause the typical consumer’s blood cholesterol to rise to a lower level than does
the use of whole milk as a lighteners. Which one of the following, if true, provides the
most support for the manufacturers’ claim?

(A) Consumers of lighteners made with coconut oil who avoid other high-cholesterol
foods and exercise more than average tend to have lower-than-average blood
cholesterol levels.

(B) Coffee is frequently consumed with pastries and other rich desserts that themselves
result in high blood cholesterol levels.

(C) One popular nondairy coffee lightener that is not based on coconut oil has reduced its
fat content by 20 percent while keeping its cholesterol content at zero.

(D) Consumers typically add to their coffee substantially smaller quantities of coconut-oil-
based lighteners than of whole milk.

(E) Most consumers are convinced that whole dairy products increase blood cholesterol
and that nondairy coffee lighteners do not.

97
答案:D,D 怎么会是加强了呢。

2. 8T 中指出同量的 nondiary lightener 和 whole milk 在使 C 上升的作用相同, 9T 中却说


nondiary 可以使 C 的上升值相对 whole milk 要小, D 指出在使用量上, nondiary lightener 用
的比 whole milk 要少, 例如用一勺 whole milk 的消费者一般只用半勺 nondiary lightener 支
持了结论.

选项 D 中有 substantially smaller ,意即〉1/7,所以可以支持 the manufacturers’ claim。

11. It is often said that people should be rewarded for doing a given job in proportion to
the effort it costs them to do it. However, a little reflection will show that this is, in fact, a
very bad idea, since it would mean that those people with the least skill or natural
aptitude for a particular task would be the ones given the most incentive to do it.

Which one of the following argumentative strategies is used above?

(A) stating a general principle and then presenting reasons in favor of adopting it

(B) providing evidence that where the principle under discussion has been adopted, the
results usually have been undesirable

(C) demonstrating that a consequence that had been assumed to follow from the
principle under consideration need not follow from it

(D) attempting to undermine a general principle by arguing that undesirable


consequences would follow from it

(E) showing that, in practice, the principle under consideration could not be uniformly
applied.

B 和 D 就差个 evidence 嘛?这样来分辨太困难了。


还是 evidence 的范畴有比较确切的定义的?

12. Photovoltaic power plants produce electricity from sunlight. As a result of astonishing
recent technological advances, the cost of producing electric power at photovoltaic power
plants, allowing for both construction and operating costs, is one-tenth of what it was 20
years ago, whereas the corresponding cost for traditional plants, which burn fossil fuels,
has increased. Thus, photovoltaic power plants offer a less expensive approach to
meeting demand for electricity than do traditional power plants.

The conclusion of the argument is properly drawn if which one of the following is
assumed?

98
(A) The cost of producing electric power at traditional plants has increased over the past
20 years.

(B) Twenty years ago, traditional power plants were producing 10 times more electric
power than were photovoltaic plants.

(C) None of the recent technological advances in producing electric power at photovoltaic
plants can be applied to producing power at traditional plants.

(D) Twenty years ago, the cost of producing electric power at photovoltaic plants was
less than 20 times the cost of producing power at traditional plants.

(E) The cost of producing electric power at photovoltaic plants is expected to decrease
further, while the cost of producing power at traditional plants is not expected to
decrease.

答案是 D,可我觉得 D 也不对啊,此提是 assume 的题,应该是必要条件, 然举个例子,


如果 20 年前 photovoltic 是 traditional 的 25 倍, 现在用了新科技, 成 2.5 倍(相对于 20
年 前 的 traditiaonal ) , 但 现 在 的 traditional 确 涨 成 了 原 来 的 3 倍 (whereas the
corresponding cost for traditional plants, which burn fossil fuels, has increased) , 结论不
还是成立吗?

in my LSAT 考古题, D: Twenty years ago, the cost of producing electric power at
photovoltaic plants was less than 10 times the cost of producing power at traditional
plants.

and the answer is D.

14. Pamela: Physicians training for a medical specialty serve as resident staff physicians
in hospitals. They work such long hours – up to 36 consecutive hours – that fatigue
impairs their ability to make the best medical decisions during the final portion of their
shifts.

Quincy: Thousands of physicians now practicing have been trained according to the
same regimen, and records show they generally made good medical decisions during
their training periods. Why should what has worked in the past be changed now?

99
Which one of the following, if true, is the most effective counter Pamela might make to
Quincy’s argument?

(A) The basic responsibilities of resident staff physicians in hospitals have not changed
substantially over the past few decades.

(B) Because medical reimbursement policies now pay for less recuperation time in
hospitals, patients in hospitals are, on the average, more seriously ill during their stay
than in the past.

(C) It is important that emergency-room patients receive continuity of physician care,


insofar as possible, over the critical period after admission, generally 24 hours.

(D) The load of work on resident physicians-in-training varies according to the medical
specialty for which each is being trained.

(E) The training of physicians should include observation and recognition of the signs
indicating a hospitalized patient’s progress or decline over a period of at least 36 hours.
首先确认一下问题的意图:问的是 Pamela 会用那个事实来 counter Quincy’s
答案为 B,想不明白

这题可以理解为削弱题,即削弱 P 的结论。p 的结论是没有必要改变工作时间,它的前提是


以前的记录表明医生可以工作的很好。削弱的一种方法是找到前提的缺陷。选项 B 表明现在
的重病号多了,所以要求医生更多的精力,反过来就是说情况变了,过去的纪录不适用了。
所以前提有缺陷。所以选 b
Quincy contends that fatigue did not affect physicians' dicision in the past. However,
answer B raise an issue that the condition, that patients are more seriously ill, are
different in present time than in past. So Quincy cannot use the fact happened in past to
prove that the same thing will happen in present days.

16. It is repeatedly claimed that the dumping of nuclear waste poses no threat to people
living nearby. If this claim could be made with certainty, there would be no reason for not
locating sites in areas of dense population. But the policy of dumping nuclear waste only
in the more sparsely populated regions indicates, at the very least, some misgiving about
safety on the part of those responsible for policy.

Which one of the following, if true, would most seriously weaken the argument?

(A) Evaluation plans in the event of an accident could not be guaranteed to work perfectly
except where the population is small.

(C) Dumping of nuclear waste poses fewer economic and bureaucratic problems in

100
sparsely populated than in densely populated areas.

正确答案是 C),
但是 A)应该是 SUPPORT 吧, C)应该是 WEAKEN 啊: 指出了不是处于 SAFETY 方面的考
虑, 而是 economic and bureaucratic 方面的考虑. 应该属于它因削弱吧.
恳请 NN 指点迷津.

Questions 18-19

Like a number of other articles, Ian Raghall’s article relied on a recent survey in which
over half the couples applying for divorces listed “money” as a major problem in their
marriages. Raghall’s conclusion from the survey data is that financial problems are the
major problem in marriages and an important factor contributing to high divorce rate. Yet
couples often express other types of marital frustrations in financial terms. Despite
appearances, the survey data do not establish that financial problems are the major
problem in contemporary marriages.

19. In the passage, the author does which one of the following?

(A) undermines a conclusion drawn from statistical data by offering a specific


counterexample

(B) undermines a conclusion drawn from statistical data by offering an alternative


explanation for some of the data.

(C) undermines a conclusion drawn from statistical data by showing that one cannot
prove the presence of an emotion by using statistical methods.

(D) undermines a conclusion drawn from statistical data by criticizing the survey for which
the data was gathered.

(E) undermines a conclusion by showing that couples cannot accurately describe their
own problems.

这题我在 B 和 E 间犹豫,答案是 B,我感觉 E 其实也可以啊,因为文中不是有提到说有一些


couples often express other types of marital frustrations in financial terms 吗?

19 ) Can,never,only 等绝对性的词一般不是答案。E 就是这样的。从推理上看作者显然认为


这些调查的数据的可靠的。也就是这些夫妇能够准确的表达他们的观点和描述她们的问题。
对这种题一定要知道这个推理在告诉我们什么?!!本推理无非是在反驳: Ian Raghall’s
article relied on a recent survey in which over half the couples applying for divorces listed
“money” as a major problem in their marriages

101
20. In Brazil, side-by-side comparisons of Africanized honeybees and the native
honeybees have shown that the Africanized bees are far superior honey producers.
Therefore, there is no reason to fear that domestic commercial honey production will
decline in the United States if local honeybees are displaced by Africanized honeybees.

Each of the following, if true, would weaken the argument EXCEPT:

(A) The honeybees native to Brazil are not of the same variety as those most frequently
used in the commercial beekeeping industry in the United States.

(B) Commercial honey production is far more complicated and expensive with Africanized
honeybees than it is with the more docile honeybees common in the United States.

(C) If Africanized honeybees replace local honeybees, certain types of ornamental trees
will be less effectively pollinated.

(D) In the United States a significant proportion of the commercial honey supply comes
from hobby beekeepers, many of whom are likely to abandon beekeeping with the influx
of Africanized bees.

(E) The area of Brazil where the comparative study was done is far better suited to the
foraging habits of the Africanized honeybees than are most areas of the United States.

后面的 argument 是接收了前面的论据,但是否定了结论,所以选 C

available
的理解有歧异
一个是说可以住的,一个是说可以租的起的
所以结论是即可以接受也可以否定,available 更本就没有说明任何问题嘛

E 是说不能统一的适用,和 B 还是有区别的。

C 是很明显的啊,因为它指的是蜜蜂对果树的影响,而与蜜的产出无关

22. Public reports by national commissions, governors’ conference, and leadership


groups have stressed the great need for better understanding of international affairs by
the citizenry. If the country is to remain a leading nation in an era of international

102
competitiveness, the need is undesirable. If there is such a need for the citizenry to have
a better understanding of international affairs, then all of our new teachers must be
prepared to teach their subject matter with an international orientation.

If all of the statements in the passage are true, which one of the following must also be
true?

(A) If the country is to remain a leading nation in an era of international competitiveness,


then new teachers must be prepared to teach their subject matter with an international
orientation.

(B) If new teachers are prepared to teach their subject matter with an international
orientation, then the country will remain a leading nation in an era of international
competitiveness.

(C) If there is better understanding of international affairs by the citizenry, then the
country will remain a leading nation in an era of international competitiveness.

(D) If the country is to remain a leading nation in an era of international competitiveness,


then there is no need for the citizenry to have a better understanding of international
affairs.

(E) Public reports from various groups and commissions have stressed the need for a
more international orientation in the education of teachers.
Why the answer is A? Note: "the need is undesirable."
应该是 Undeniable,Elsat 打错了,则 A 就顺理成章,a-->b, b-->c, 则 a-->c.

23. “DNA fingerprinting” is a recently-introduced biochemical procedure that uses a


pattern derived from a person’s genetic material to match a suspect’s genetic material
against that of a specimen from a crime scene. Proponents have claimed astronomically
high odds against obtaining a match by chance alone. These odds are based on an
assumption that there is independence between the different characteristics represented
by a single pattern.

Which one of the following, if true, casts the most doubt on the claim of the proponents of
DNA fingerprinting?

(A) The large amount of genetic material that people share with all other people and with
other animals is not included in the DNA fingerprinting procedure.

(B) There is generally accepted theoretical basis for interpreting the patterns produced by
the procedure.

103
(C) In the whole population there are various different subgroups, within each of which
certain sets of genetic characteristics are shared.

(D) The skill required of laboratory technicians performing the DNA fingerprinting
procedure is not extraordinary.

(E) In the investigation of certain genetic diseases, the techniques used in DNA
fingerprinting have traced the transmission of the diseases among the living members of
very large families.

答案是 C。

C disputes the assumption that the different characteristics are independent.

24. Anthropologists assert that cultures advance only when independence replaces
dependence – that is, only when imposition by outsiders is replaced by initiative from
within. In other words, the natives of a culture are the only ones who can move that
culture forward. Non-natives may provide valuable advice, but any imposition of their
views threatens independence and thus progress. If one looks at individual schools as
separate cultures, therefore, the key to educational progress is obvious: ______.

Which one of the following best completes the passage?

(A) individual schools must be independent of outside imposition.

(B) some schools require more independence than others, depending on the initiative of
their staffs and students.

(C) school system officials must tailor their initiatives for change to each individual school
in the system.

(D) outsiders must be prevented from participation in schools’ effort to advance.

(E) the more independent a school is, the more educational progress it will make.

答案:A, 我选了 E。完成句子题真难做,不知有什么好办法吗?

E 的错误和 B 是一样的, 原文没有提到 independence 在量上对 progress 的影响, 我们只知


道从质上 progress 是由 independence 决定的. 如果你要推出 E,你自己要做 assumption, 即
量化 progress 和 independence.这在做题时是不允许的.

做完成题, 要抓住原文的逻辑关系. 如 rongchao 所讲, "题目讲到任何强迫接受的观点都可


能损坏独立,进而影响到文明的进步", 所以 culture 要 progress, 就不能有 imposiiton, 这是大

104
结论. 然后讲学校是 culture,所以结论是 school 不能有 imposition. 举个简单的例子:二班的
学生如果课外学习的话, 成绩会更好. 小笨是二班的, ________. 答案是小笨如果课外学习
成绩会更好, 而不会是谁课外学习时间长,谁的成绩就会更好. 量在这里没有出现.

Set 5-4

1. The translator of poetry must realize that word-for-word equivalents do not exist across
languages, any more than piano sounds exist in the violin. The violin can, however, play
recognizably the same music as the piano, but only if the violinist is guided by the nature
and possibilities of the violin as well as by the original composition.

As applied to the act of translating poetry from one language into another, the analogy
above can best be understood as saying that
(A) poetry cannot be effectively translated because, unlike music, it is composed of
words with specific meanings
(B) some languages are inherently more musical and more suitable to poetic composition
than others
(C) the translator should be primarily concerned with reproducing the rhythms and sound
patterns of the original, not with transcribing its meaning exactly
(D) the translator must observe the spirit of the original and also the qualities of
expression that characterize the language into which the original is translated
(E) poetry is easier to translate if it focuses on philosophical insights or natural
descriptions rather than on subjective impressions

原文的意思就是说了翻译应该不拘于形势了,所以当然 D 最符合题意咯
那段英文的意思大概是说小提琴有比钢琴更多的声音表现形势吧

105
不过这个对题目的理解好像不是最重要

6. When machines are invented and technologies are developed, they alter the range of
choices open to us. The clock, for example, made possible the synchronization of human
affairs, which resulted in an increase in productivity. At the same time that the clock
opened up some avenues, it closed others. It has become harder and harder to live
except by the clock, so that now people have no choice in the matter at all.

Which one of the following propositions is best illustrated by the example presented in
the passage?

(A) New machines and technologies can enslave as well as liberate us.

(B) People should make a concerted effort to free themselves from the clock.

(C) Some new machines and technologies bring us improvement to our lives.

(D) The increase in productivity was not worth our dependence on the clock.

(E) Most new machines and technologies make our lives synchronized and productive.

答案是 a

作者举了一个例子时钟:The clock 如何如何.....,接着说: at the same time...harder....


可以从中得到:The clock 对我们有好处,同时我们又变得依赖于时钟。A 正是这个例子所告诉我们的。我觉
得如果读明白作者在这段论述中目的。C 就不会选。

9. An easy willingness to tell funny stories or jokes about oneself is the surest of supreme
self-confidence. This willingness, often not acquired until late in life, is even more
revealing than is good-natured acquiescence in having others poke fun at one.

Which one of the following inference is most supported by the statements above?

(A) A person who lacks self-confidence will enjoy neither telling nor hearing funny stories
about himself or herself.

(B) People with high self-confidence do not tell funny stories or jokes about others.

(C) Highly self-confident people tell funny stories and jokes in order to let their audience
know that they are self-confident.

106
(D) Most people would rather tell a funny story or joke than listen to one being told.

(E) Telling funny stories or jokes about people in their presence is a way of expressing
one’s respect for them.

这 个 题 目 的 答 案 选 A, 我 选 的 是 E. 但 是 A 中 的 "hearing funny stories about himself or


herself"是原文没有说的无关信息啊!这个题目有一定的疑问

愿意自己开自己玩笑 表明 有极强的自信,比 好脾气默许别人开自己玩笑 还表明(有极强


的自信)。
隐含:愿意自己开自己玩笑 和 好脾气默许别人开自己玩笑 都表明 自信。
所以正确理解 revealing 是关键。

11.Nature constantly adjusts the atmospheric carbon level. An increase in the level
causes the atmosphere to hold more heat, which causes more water to evaporate from
the oceans, which causes increased rain. Rain washes some carbon from the air into the
oceans, where it eventually becomes part of the seabed. A decrease in atmospheric
carbon causes the atmosphere to hold less heat, which causes decreased evaporation
from the oceans, which causes less rain, and thus less carbon is washed into the
oceans. Yet some environmentalists worry that burning fossil fuels may raise atmospheric
carbon to a dangerous level. It is true that a sustained increase would threaten human
life. But the environmentalists should relax – nature will continually adjust the carbon
level.

Which one of the following, if true, would most weaken the argument in the passage?

(A) Plant life cannot survive without atmospheric carbon.

(B) It is not clear that breathing excess carbon in the atmosphere will have a negative
effect on human life.

(C) Carbon is part of the chemical “blanket” that keeps the Earth warm enough to sustain
human life.

(D) Breathing by animals releases almost 30 times as much carbon as does the burning
of fossil fuels.

(E) The natural adjustment process, which occurs over millions of years, allows wide
fluctuations in the carbon level in the short term.

答案 E

107
短 文 的 argument 在 于 反 对 Yet some environmentalists worry that burning fossil fuels may raise atmospheric
carbon to a dangerous level.这种观点。那么如果能 support 这种观点就是 weaken the argument 了。E 选项说短
期内会导致大的波动,那就是说短期内会有 carbon reaches to a dangerous level 的情形,则支持了环保人士
的观点,从而 weaken 了 argument.

12. The more television children watch, the less competent they are in mathematical
knowledge. More than a third of children in the United States watch television for more
than five hours a day; in South Korea the figure is only 7 percent. But whereas less than
15 percent of children in the United States understand advanced measurement and
geometric concept, 40 percent of South Korea children are competent in these areas.
Therefore, if Untied States children are to de well in mathematics, they must watch less
television.

Which one of the following is an assumption upon which the argument depends?

(A) Children in the United States are less interested in advanced measurement and
geometric concepts than are South Korea children.

(B) South Korea children are more disciplined about doing schoolwork than are children
in the United States

(C) Children who want to do well in advanced measurement and geometry will watch less
than television.

(D) A child’s ability in advanced measurement and geometry increases if he or she


watches less than one hour of television a day.

(E) The instruction in advanced measurement and geometric concepts available to


children in the United States in not substantially worse than that available to South Korea
children.

假设题目,当时作题的时候我没有分开 C 选项和 E.答案是 E.我觉得 C 是假设,E 是一个加


强。

我认为答案应该是 E
文中认为美国儿童看电视多是造成其数学能力差的原因,文中的暗含假设就是没有其他的
原因造成美国儿童数学能力差。
答案 E 指出在教育上美国儿童不比韩国儿童差,该选项排出了他因,所以正确。
而 C 我认为最好也就是个加强。想要在数学方面最的很好的儿童将会看很少的电视。只能说
明看电视可能是儿童数学差的原因,所以只能是个加强。

108
请教 LSAT-5-VI-13,22
The only way that bookstores can profitably sell books at below-market prices is to get
the books at a discount from publishers. Unless bookstores generate a high sales
volume, however, they cannot get discounts from publishers. To generate such volume,
bookstores must either cater to mass tastes or have exclusive access to a large
specialized market, such as medical market, or both.

13. Which one of the following can be properly inferred from the passage?

(A) If a bookstore receives discounts from publishers, it will profitably sell books at below-
market prices.

(B) A bookstore that caters to mass tastes or has exclusive access to a large specialized
market will have a high sales volume.

(C) A bookstore that profitably sells books at below-market prices gets discounts from
publishers.

(D) A bookstore that does not sell books at below-market prices does not get discounts
from publishers.

(E) A bookstore that not only caters to mass tastes but also has exclusive access to a
large specialized market cannot profitably sell books at below-market prices.

答案:C

原文的思路:
1) below maket price==> get discount
2)discount ==> high sales volume
3)generate high volume==> cater to mass taste or have exclusive access to a large
specilaized makret
还 是 cater to mass taste or have exclusive access to a large specilaized
makret===>generate high volume

A 与 1)是逆命题,错
B.不知 3)是什么推理,是前者还是后者
C.与 1)一致,对
D.与 1)是否命题,错
E.与 2)相反,错

能否帮助看看我的思路,对吗?另外,3)是什么样的推理,是前者还是后者?

109
这种归纳题,需要读出推理的出程,我有时会做反了.

The third one is: generate high volume==> cater to mass taste or have exclusive access
to a large specilaized makret

I think that you can clearly sort out the logic links now. One of my concerns is that this
type of problem is rarely seen in GMAT. So you might spend too much time linking
reasons and results, while in GMAT POE (point of elimination) is the best tool. You might
want to do some GMAT problems at the same time, just for the sake of familiarizing
yourself with GMAT.

16. The public is aware of the possibility of biases in the mass media and distrusts the
media as too powerful. The body of information against which the public evaluates the
plausibility of each new media report comes, however, from what the public has heard of
through the mass media.
If the view above is correct, it provides a reason for accepting which one of the following
conclusions?

The public is aware of the possibility of biases in the mass media and distrusts the media
as (the mass media is) too powerful.
公众了解大众传媒的偏见, 而且由于其过于强势, 公众不信任大众传媒.
The body of information against which the public evaluates the plausibility of each new m
edia report
comes, however, from what the public has heard of through the mass media.
然而, 大众用于评价每条新消息的信息实体又是来自大众传媒.

You should understand the first sentence. Public does not trust mass media. But when
public evaluate a particular report to see if it is biased not, they use mass media as the
benchmark.

20. Politician: Homelessness is a serious social problem, but further government


spending to provide low-income housing is not the cure for homelessness. The most
cursory glance at the real-estate section of any major newspaper is enough to show that
there is no lack of housing units available to rent. So the frequent claim that people are
homeless because of a lack of available housing is wrong.

That homelessness is a serious social problem figures in the argument in which one of
the following ways?

110
(A) It suggests an alternative perspective to the one adopted in the argument.

(B) It sets out a problem the argument is designed to resolve.

(C) It is compatible either with accepting the conclusion or with denying it.

(D) It summarizes a position the argument as a whole is directed toward discrediting.

(E) It is required in order to establish the conclusion.

逻辑描述题目.答案选 c
作 者 也 承 认 HOMELESS is a serious social problem 要 辩 论 的 是 : 是 什 么 引 起 such
serious social problem(作者认为不是 lack of hourse)
所 以, homeless is aserious social problem 不 是全 文要 argue 的东 西, 而是 compatible
with 支持或者反对作者的观点,即都成立的。
在这里并没有提出如何解决无家可归者的问题,仅仅提出政府拨款是不能解决这个问题的

21. Leona: If the average consumption of eggs in the United States were cut in half, an
estimated 5,000 lives might be saved each year.

Thomas: How can that be? That would mean that if people adopt this single change in
diet for ten years, the population ten years from now will be greater by 50,000 people
than it otherwise would have been.

Which one of the following is a statement that Leona could offer Thomas to clarify her
own claim and to address the point he has made?

(A) It is possible for the population to grow by 5,000 people for every year if the base
year chosen for purposes of comparison is one with unusually low population growth.

(B) It is accurate to say that 5,000 lives have been saved as long as 5,000 people who
would have died in a given year as a result of not changing their diet, did not do so–even
if they died for some other reason.

(C) If egg consumption were reduced by more than half, the estimated number of lives
saved each year could be even more than 5,000.

(D) The actual rate of population growth depends not only on the birth rate, but also on
changes in life expectancy.

(E) For the average consumption of eggs to be cut by half, many individual consumers

111
would have to cut their own consumption by much more than half.

答案是 B。虽然我选对了,但还是一头雾水,不知道题目在讲什么的?

Leona 的观点是:an estimated 5,000 lives might be saved each year if ......
Thomas 的观点是可以这么看: 如果按照 Leona 的说法,10 年内 多于 50,000 lives ......
(Thomas 暗含:10 年内 不可能有 50,000lives.......)
问 题 是 : 下 面 哪 个 statement Leona 可 以 让 Thomas 去 澄 清 它 的 观 点 ( 可 以 认 为 让
Thomas 重新想想自己的观点 ),同时 Leona 可以陈述自己的 point?
这是个对话反驳题。问题可以简单归结是如何让 Leona 自己的观点是合理的。

B 的意思是:我们可以准确的说 5000 lives have been saved 只要这些本来这些因为 not


changing their diet 应该死的 5000 lives 没有这么做而即使因为其他原因而死去( forgive
me for my poor english)
可以简单把 B 看成:Leona 对这个 5000 这个数目的不同理解。也就是这些 5000 个因为改
变饮食而没有死亡的人也只能算成 5000 中的。不是因此而新增加的人数(Thomas 是这么
理解的)。

All other choices are not even close. They are all out of scope.

B basically said that even though the 5000 are saved by the diet, they may still die from
other causes.

22. The United States Food and Drug Administration (FDA) regulates the introduction of
new therapeutic agents into the marketplace. Consequently, it plays a critical role in
improving health care in the United States. While it is those in the academic and
government research communities who engage in the long process of initial discovery
and clinical testing of new therapeutic agents, it is the FDA’s role and responsibility to
facilitate the transfer of new discoveries from the laboratory to the marketplace. Only
after the transfer can important new therapies help patients.

Which one of the following statements can be inferred from the passage?

(A) The FDA is responsible for ensuring that any therapeutic agent that is marketed is
then regulated.

(B) Before new therapeutic agents reach the marketplace they do not help patients.

(C) The research community is responsible for the excessively long testing period for
new drugs, not the FDA.

(D) The FDA should work more closely with researchers to ensure that the quality of
therapeutic agents is maintained.

112
(E) If a new medical discovery has been transferred from the laboratory to the
marketplace, it will help patients.

答案:B,我选 E

A,原文无,错
B.是原文最后一句的否命题,错
C.对
D,原文无
E.与原文最后一句,同,对

我什么地方不对了呢?谢谢.

The last sentence points out that new therapies help patients-->reach marketplace. Thus
B is correct and E is not.

"Only after" and "only if" are used in the same way. Only if (only after) A, then B==>B--
>A.

24. It has been claimed that an action is morally good only if it benefits another person
and was performed with that intention; whereas an action that harms another person is
morally bad either if such harm was intended or if reasonable forethought would have
shown that the action was likely to cause harm.

Which one of the following judgments most closely confirms to the principle cited above?

(A) Pamela wrote a letter attempting to cause trouble between Edward and his friends;
this action of Pamela’s was morally bad, even though the letter, in fact, had an effect
directly opposite from the one intended.
(B) In order to secure a promotion, Jeffery devoted his own time to resolving a backlog of
medical benefits claims; Jeffrey’s action was morally good since it alone enabled Sara’s
claim to be processed in time for her to receive much-needed treatment.
(C) Intending to help her elderly neighbor by clearing his walkway after a snowstorm,
Teresa inadvertently left ice on his steps; because of this exposed ice, her neighbor had
a bad fall, thus showing that morally good actions can have bad consequences.
(D) Marilees, asked by a homeless man for food, gave the man her own sandwich;
however, because the man tried to talk while he was eating the sandwich, it caused him
to choke, and thus Marilees unintentionally performed a morally bad action.
(E) Jonathan agreed to watch his three-year-old niece while she played but, becoming
engrossed in conversion, did not see her run into the street where she was hit by a
bicycle; even though he intended no harm, Jonathan’s action was morally bad.
-------------------------------------------------------------

113
答案 E,看晕了,A,C,D 为何错?请大家指教.

whereas an action that harms another person is morally badeither if such harm was
intended or if reasonable forethought would have shown that the action was likely to
cause harm.

这个action 要 harms another,才能说是 morally bad.


A.he intend to write a letter to cause trouble, but the letter had an effect directly opposite
from the one intended.
it means the letter didn't cause any trouble to his friends . it doesn't do harm. it's not
morally bad.

E. the key
his three-year-old niece got hit by a car ( harm), his action is morally bad if either -----
or if reasonable forethought would have shown that the action was likely to cause harm.
E是后面这种情况,he 应考录到事故会发生。his action is morally bad

Set 6-2

5. The museum's night security guard-maintains that the thieves who stole the portrait did
not enter the museum at any point at or above ground level. Therefore, the thieves must
have gained access to the museum from below ground level.

The flawed pattern of reasoning in the argument above is most similar to that in which
one of the following?

(A) The rules stipulate the participants in the contest be judged on both form and
accuracy. The eventual winner was judged highest in neither category , so there must be
a third criterion that judges were free to invoke.
(B) The store's competitors claim that the store in selling off the shirts at those prices,
neither made any profit nor broke even. Consequently, the store's customers must have
been able to buy shirts there at less than the store's cost.
(C) If the census is to be believed, the percentage of men who are married is higher than
the percentage of women who are married. Thus, the census must show a higher
number of men than of women overall.
(D) The product label establishes that this insecticide is safe for both humans and pet.

114
Therefore, the insecticide must also be safe for such wild mammals as deer and rabbits.
(E) As had generally been expected, not all questionnaires were sent in by the official
deadline. It follows that plans must have been made for the processing of questionnaires
received late.
-------------------
答案 B
排出他因不一定对

7. High-technology medicine is driving up the nation's health care costs. Recent


advances in cataract surgery illustrate why this is occurring. Cataracts are a major cause
of blindness, especially in elderly people. Ten years ago, cataract surgery was painful
and not always effective. Thanks to the new technology used in cataract surgery, the
operation now restores vision dramatically and is less expensive. These two factors have
caused the number of cataract operations performed to increase greatly, which has, in
turn, drive up the total amount spent on cataract surgery.

7. Each of the following, if true, would support a challenge to the author's explanation of
the increase in the number of cataract operations EXCEPT:

(A) The overall population of the nation has increased from what it was ten years ago

(B) Any one individual's chance of developing cataracts is greater than it was ten years
ago.

(C) The number of older people has increased during the last ten years.

(D) Today, health insurance covers cataract surgery for more people than it did ten years
ago.

(E) People who have had unsuccessful cataract surgery are left with more seriously
impaired vision than they had before the surgery.

- Should be E. ABCD each point out a reason that it might not be the technology
improvement that caused the increase in number of surgery performed or the hiked bill. D
says that because of the broader coverage of insurance, many poeple who otherwise
would not be covered 10 years ago for their surgeries will be covered now. So it is the
insurance policy that make the surgery affordable to more people, who receive the
surgery. E is not relevant in countering the reasoning in the argument.

10. All of John's friends say they know someone who has smoked 40 cigarettes a day for
the past 40 years and yet who is really fit and well. John does not know anyone like that

115
and it is quite certain that he is not unique among his friends in this respect.

If the statements in the passage are true, then which one of the following must also be
true?

(A) Smoking often lie about how much they smoke.

(B) People often knowingly exaggerate without intending to lie.

(C) All John's friends know the same lifelong heavy smoker.

(D) Most of John's friends are not telling the truth.

(E) Some of John's friends are not telling the truth.

答案:E

纯粹的逻辑题:
JOHN 并不知道有这样的朋友:大烟枪---一天 40 根,40 年。。
同时 JOHN 知道至少在他朋友中,他不是唯一这样子的(not unique)。。说明他的朋友也有人
不认识这样的烟鬼。。而开头又说 ALL OF his friends says that 他们认识这样的烟鬼。
那肯定是 E:至少有朋友撒谎了。

12. By dating fossils of pollen and beetles, which returned after an Ice Age glacier left an
area, it is possible to establish an approximate date when a warmer climate developed.
In one glacial area, it appears from the insect record that a warm climate developed
immediately after the melting of the glacier. From the pollen record, however, it appears
that the warm climate did not develop until long after the glacier disappeared.

Each one of the following, if true, helps to explain the apparent discrepancy EXCEPT:

(A) Cold-weather beetle fossils can be mistaken for those of beetles that live in warm
climates.
(B) Warm-weather plants cannot establish themselves as quickly as can beetles in a new
environment.
(C) Beetles can survive in a relatively barren postglacial area by scavenging.
(D) Since planes spread unevenly in a new climate, researchers can mistake gaps in the
pollen record as evidence of no new overall growth.
(E) Beetles are among the oldest insect species and are much older then many warm-
weather plants.
------------------
答案 E

116
Answer C means that beetles lived longer than plants after the glacier
disappeared(postglacial ), and there was a relative shorter period from extinction of cool
beetles to thriving of warm ones than what happened to different plants.

16. The ancient Romans understood the principles of water power very well and in some
outlying parts of their empire they made extensive and excellent use of water as an
energy sources. This makes it all the more striking that the Romans made do without
water power in dominated by large cities.
Which one of the following, if true , contributes most to an explanation of the difference
described above in the Romans use of water power?
(A) The ancient Romans were adept at constructing and maintaining aqueducts that
could carry quantities of water sufficient to supply large cities over considerable distances
(B) In the areas in which water power was not used water flow in rivers and streams was
substantial throughout the year but nevertheless exhibited some seasonal variation
(C) Water power was relatively vulnerable to sabotage but any damage could be quickly
and inexpensively repaired
(D) In most areas to which the use of water power was not extended other more
traditional sources of energy continued to be used
(E) In heavily populated areas the introduction of water power would have been certain to
cause social unrest by depriving large numbers of people of their livelihood

答案为 E,
要找的就是为什么水力不能使用的原因,而 ABC 都说的是水力的使用是没问题的,D 说的是没用水力而
用了其他能源,这没解释水力不能使用的原因,只有 E

17. From a book review: The authors blithely claim that there are "three basic ways to
store energy: as heat, as electricity or as kinetic energy" However, I can not call to mind
any affective ways to store energy as electricity, whereas any capable student of physics
could readily suggest a few more ways to store energy : chemical , gravitational, nuclear

The reviewer makes which one of the following criticisms of a claim that appears in the
book under review?

(A) There is no reason to consider any particular way to store energy any more basic
than any other

(B) The list given of ways to store energy is possibly inaccurate and certainly not
exhaustive

117
(C) It is overly limiting to treat basic ways to store energy as a question unrelated to the
question of effective ways to use energy

(D) What needs to be considered is not whether various ways to store energy are basic
but whether they are effective

(E) Except possibly for electricity all ways to store energy are equally effective and
therefore equally basic

答案:B,我选了 D,另外想问 C 是什么意思。

A 说有三种方式存能量
作者嘲笑: 我不知道有用电存能量,但是有本事的学生知道有更多方式存能量,又举了一
堆别的方式。。。

(B) The list given of ways to store energy is possibly inaccurate and certainly not
exhaustive
正象 B 说的,作者认为 A 说的可能不对,即使对也不完整。

(C) It is overly limiting to treat basic ways to store energy as a question unrelated to the
question of effective ways to use energy
“认为 A 和 B 两个问题是不相干的”这种看法很肤浅
A 问题是:存储能量的基本方式
B 问题是: 使用能量的有效方式
The reviewer makes which one of the following criticisms of a claim that appears in the
book under review?
translate into Chinese, it means: "作者的批驳是啥意思?"

注意:不要加入不应该的比较和额外信息。。原文唯一的比较是:
a few more ways....
从这里可以可以看出:作者的意思和 B 一样:作者认为 A 说的可能不对,即使对也不完整。

A: 原文根本没有 BASIC 的比较。。。排除

C:(C) It is overly limiting to treat basic ways to store energy as a question unrelated to
the question of effective ways to use energy
原文根本没有考虑到两个问题是否相干,只是说有更多方式。。

D (D) What needs to be considered is not whether various ways to store energy are basic
but whether they are effective
原文根本没有什么应该考虑,什么不应该考虑。。没有这种比较。。排除。。

118
18.There is no mystery as to why figurative painting revived in the fate 1970s.People
want to look at recognizable images. Sorting out art theories reflected in abstract
paintings is no substitute for the sense of empathy that comes from looking at a realistic
painting of a figure in a landscape. Perhaps. members of the art because they felt that its
lack of realistic subject matter was a rejection of the viewers and their world.

Which one of the following most accurately expresses the main point of the passage?

(A) Abstract paintings often include or forms that are suggestive of real objects or
emotions.

(B) The art-viewing public wished to see traditional subjects treated in a nontraditional
manner.

(C) Paintings that depict a recognizable physical world rather than the emotional world of
the artist's life require more artistic talent to create.

(D) The general public is unable to understand the theories on which abstract painting is
based.

(E) The artistic preferences of the art-viewing public stimulated the revival.

答案: E

The reasoning in the passage is that figurative painting was revived because people in
the 70's preferred figurative painting to abstract paining for two reasons: people rather
like the sense of empathy ... from figurative paining than sorting out what is in the
abstract paining; there is little realistic subjects in the abstract paining, which counters the
view of the public (it means that people can not find in the abstract painting what they
normally see around them). So E is right.

The main point is what the author wants to prove. In another word, it asks for the
conclusion. A,B,C,D are not what the author wants to prove here. And they are not
mentioned in the passage either.

19. Valitania’s long-standing practice of paying high salaries to its elected politicians has
had a disastrous effect on the level of integrity among politicians in that country. This is
because the prospect of earning a high salary is always attractive to anyone whose
primary aim in life is to make money, so that inevitably the wrong people must have been
attracted into Valitanian politics: people who are more interested in making money than in
serving the needs of the nation
Which one of the following, if true, world weaken the argument?

119
(A) Many Valitanian candidates for elected office spend some of their own money to
finance their campaigns.
(B) Most Valitanian elective offices have four-year terms.
(C) No more people compete for elected office when officeholders are paid well than
when they are paid poorly.
(D) Only politicians who rely on their offices for income tend to support policies that
advance their own selfish interests.
(E) Most of those who are currently Valitanian politicians could have obtained better-paid
work outside politics.
选的 3,答案给 5

3 的错误在于也许人数没变,但换了另外一群,全是禄蠹的人

Policy Adviser: Freedom of speech is not only a basic human right; it is also the only
rational policy for this government to adopt. When ideas are openly aired ,good idea
flourish, silly proposals are easily recognized as such, and dangerous ideas can be
responded to by forcing citizens to disseminate their thoughts in secret.
20. The policy adviser's method of persuasion in recommending a policy of free speech
to the following?
(A) a circular justification of the idea of free speech as an idea that flourishes when free
speech is allowed
(B) advocating respect for basic rights of citizens for its own sake
(C) a coupling of moral ideals with self-interest
(D) a warning about the difficulty of suppressing the truth
(E) a description of an ideal situation that cannot realistically be achieved.

答案为 C,

22. The trustees of the Avonbridge summer drama workshop have decided to offer
scholarships to the top 10 percent of local applicants and the top 10 percent of nonlocal
applicants as judged on the basis of a qualifying audition. They are doing this to ensure
that only the applicants with the most highly evaluated auditions are offered scholarships
to the program.

Which one of the following points out why the trustees' plan might not be effective in
achieving its goal?
(A) The best actors can also apply for admission to another program and then not enroll
in the Avonbridge program
(B) Audition materials that produce good results for one actor may disadvantage another,
resulting in inaccurate assessment
(C) The top 10 percent of local and nonlocal applicants might not need scholarships to

120
the Avonbridge program
(D) Some of the applicants who are offered scholarships could have less highly
evaluated auditions than some of the applicants who are not offered scholarships
(E) Dividing applicants into local and nonlocal groups is unfair because it favors nonlocal
applicants
参 考 答 案 是 D, 欧 选 B, 欧 觉 得 D 不 一 定 削 若 , 因 为 local applicants 和 non-local
applicants 的人数/drama 水平可能相差很大;极端假设有 1000 个 local applicants, 100 个
non-local applicants, 这 100 个 non-local appliants 的 individual drama level, 都可以进入
top 50 of 1000 local applicants.... 这样 D 就不足为奇了。。。。

B 确实是个 ATTRACTIVE 的答案,但错了,如果你明白计划的目标是什麽的话。计划的目


标是只有 the applicants with the most highly evaluated auditions 能得奖学金。这些人是
most highly evaluated auditions ,即面试时评价最高的,不是实际水平最高的。D 对的
原因是该计划将面试者分两类,LOCAL/NONLOCAL。每类的前 10%得奖学金,这样就存
在其中一类有人面试时表现好于另一类的 10%内,但在本类中排名是 10%之外而得不到奖
学金。而实际他比另一类的 10%内的人表现好。所以存在 D 所说的有奖学金的比没奖学金的
面试差的情况。

此题是削弱。原文结论为:“They are doing this to ensure that only the applicants with the
most highly evaluated auditions are offered scholarships to the program.” 请注意 “ only”这
个词。按照题意,10%的 local 与 10%的 nonlocal 将获奖学金。很可能出现的情况之一是
90%的 local 中有人会强于那 10%的 nonlocal 中某人。

24. Someone's benefiting from having done harm to another person is morally justifiable
only if the person who was harmed knew that what was done could cause that harm but
consented to its being done anyway

Which of the following judgments most closely conforms to the principle above?

(A) Attempting to avoid being kept after school as punishment for breaking a window.
Sonia falsely claimed that her brother had broken it; Sonia's action was morally
unjustifiable since it resulted in both children being kept after school for something only
Sonia had done

(B) Since Ned would not have won the prize for best model airplane if Penny's brother
had not inadvertently damaged her entry while playing with it. Ned is morally unjustified in
accepting his prize

(C) Wesley, a doctor, persuade Max to take part in a medical experiment in which a new
drug was being tested: since Wesley failed to warn Max about the serious side effects of
the drug and the drug proved to have no other effects Wesley was morally unjustified in
using the results obtained from Max in his report.

121
(D) Because Roger's mother suffered severe complications as a result of donating a
kidney to him for lifesaving kidney transplant, it was morally unjustifiable for Roger to
receive the transplant, even though his mother, herself a doctor, had been eager for the
transplant to be performed.

(E) For James, who was convicted of having defrauded a large number of people out of
their savings and wrote a book about his scheme while in prison, to be denied the profits
from his book would be morally unjustifiable since he was already been punished for his
crime.

答案:C
为 什 么 呢 ? C 不 是 说 : “ proved to have no other effects” 吗 ? 而 原 文 中 讨 论 是 否
“justifiable"的前提不是 “having done harm" 吗?

No other effects here indicate that the drugs only had "serious side-effect", which is bad.
That is the harm.

25. Certain governments subsidize certain basic agricultural products in order to


guarantee an adequate domestic production of them. But subsidies encourage more
intensive farming, which eventually leads to soil exhaustion and drastically reduced
yields.
The situation above is most nearly similar to which one of the following situations with
respect to the relationship between the declared intent of a government practice and a
circumstance relevant to it?
(A) Certain governments subsidize theaters in order to attract foreign tourists. But tourists
rarely choose a destination for the theatrical performances it has to offer.
(B) Certain governments restrict imports in order to keep domestic producers in business.
But, since domestic producers do not have to face the full force of foreign competition,
some domestic producers are able to earn inordinately high profits.
(C)Certain governments build strong armed forces in order to forestall armed conflict. But
in order to maintain the sort of discipline and morale that keeps armed forces strong,
those forces must be used in actual combat periodically.
(D)Certain governments reduce taxes on business in order to stimulate private
investment. But any investment is to some extent a gamble, and new business ventures
are not always as successful as their owners hoped.
(E) Certain governments pass traffic laws in order to make travel safer. But the
population-driven growth in volumes of traffic often has the effect of making travel less
safe despite the passage of new traffic laws.
问题:原题目选 C,可感觉国家建立武装力量的目的是 discipline 和 morale,这个与经常
打仗并不矛盾。E 好像更像一些。
Now let's look at "the relationship between the declared intent of a government practice
and a circumstance relevant to it" in the stem:

122
Action: governments subsidize agricultural products
Intention: to guarantee adequate production
Circumstance: subsidies encourage more intensive farming, which eventually leads to
soil exhaustion
Result: drastically reduced yields (contradicting to the stated purpose)
In C, we can see a similar self-contradiction:
Action: governments build strong armed forces
Intention: in order to forestall armed conflict
Circumstance: in order to maintain discipline and morale that keeps armed forces strong,
those forces must be used in actual combat periodically.
Result: not necessarily less armed conflict (contradicting to the stated purpose)
So, C is the best answer to the question.

Set 6-3

lsat-6-3-6

Four randomly chosen market research companies each produced population estimated
for three middle-sized cities; the estimates of each company were then compared with
those of the other companies. Two of the cities had relatively stable populations, and for
them estimates of current population and of projected population in five years varied little
from company to company. However, for the third city, which was growing rapidly,
estimates varied greatly from company to company.

6. Which one of the following, if true, would best help explain why estimates of the
current population of the rapidly growing city varied more than did current population
estimates for the two other cities?

123
(A) Population changes over time are more uniform from one district to another in the
rapidly growing city than in the two other cities.

(B) The population of the rapidly growing city is increasing largely as a result of a high
birth rate.

(C) The population of the rapidly growing city has a lower average age than the
populations of either of the two other cities.

(D) All population estimates of the rapidly growing city were produced first by estimating
the current populations of the city’s districts and then by adding those estimates.(E)

(E) Whereas the companies used different methods for estimating the current population
of the rapidly growing city, the companies used the same method for the two other cities.

我选的 d,答案是 e

e 很明显吧, d 说了一种计算方法,如果这几家公司都用这个方法的话,应该不会有很多
difference.
e 是一种解释,题目说是 best help explain,e 在 5 个里面是最好的解释.

7 Head injury is the most serious type of injury sustained in motorcycle accidents. The
average cost to taxpayers for medical care for nonbelmeted motorcycle-accident victims
is twice that for their helmeted counterparts. Jurisdictions that have enacted motorcycle-
helmet laws have reduced the incidence and severity of accident-related head injuries,
thereby reducing the cost to taxpayers. Therefore, to achieve similar cost reductions,
other jurisdictions should enact motorcycle-helmet laws. For the same reason
jurisdictions should also require helmets for horseback riders, since horseback-riding
accidents are even more likely to cause serious head injury than motorcycle accidents
are.
7.Which one of the following is an assumption upon which the author's conclusion
concerning helmets for horseback riders depend?
(A) Medical care for victims of horseback-riding accidents is financial drain on tax funds.
(B) The higher rate of serious head injury suffered by victims of horseback-riding
accidents is due to the difference in size between horses and motorcycles.
(C) The medical costs associated with treating head injuries are higher than those for
other types of injury.
(D) Most fatalities resulting from horseback-riding and motorcycle accidents could have
been prevented if the victims had been wearing helmets.
(E) When deciding whether to enact helmet laws for motorcyclists and horseback riders,
the jurisdiction's primary concerns is the safety of its citizens.
Answer: A
D 为什么不对?

124
我还查了一下 mindfree 的解释:
"
1. 需 要 理 解 assumption 的 相 关 性 . 原 文 其 实 是 一 个 parallel reasoning. 前 面 说 因 为
motorcycle helmet 可以减轻 medical care 对纳税人的负担,所以应该通过立法. 后面说对
horserider 也应该同样立法要求 helmet. 这里后面省略的是和前面部分相同的成分, 补充后
应该是因为 helmet 立法可以减轻 houserider medical care 对 taxpayer 的负担, 所以应该通
过立法. 省略的就是 assumption, 也是原文的必要条件, 说明 motorcycle 和 horseriding 在
helmet 上的一致性. 如果没有此 assumption, 则两者不可比, 从 motorcycle 不能推导出
horseriding 的结论.

D 是无关选项. 它在意思上是对的, 但是在此问题中的作用就好比是说"太阳是热的", 和原文


的逻辑毫无关系. 原文没有提到 fatality, 而是围绕着 tax, medical care cost. 所以答案一定
要将原文的 elements 联系在一起.
"

10.Anyone who fails to answer a patient’s questions cannot be a competent physician.


That is why I feel confident about my physician’s competence: she carefully answers
every one of my questions, no matter how trivial.
Which one of the following most closely parallels the flawed reasoning in the argument
above?
(A) Anyone who grows up in a large family is accustomed to making compromises.
Meredith is accustomed to making compromises, so she might have grown up in a large
family.
(B) Anyone who is not in favor of this proposal is ill informed on the issue. Jeanne
opposes the proposal, so she is ill informed on the issue.
(C) No one who likes music misses a performance of the symphony. Paul likes music, yet
last week he missed a performance of the symphony.
(D) Anyone who works two or more jobs is unable to find a balance between professional
and personal life. Maggie has only one job, so she can find a balance between her
professional and personal life.
(E) No one who is hot-tempered and strong-willed will succeed in this business. Jeremy
is strong-willed, so he will not succeed in this business.
答案是 D

only D's form matchs that of the stimulus.

flawed reasoning in stimus' : not A, not B. my physician A ( answer everyones Q), so she
B ( competitive)
D: 2 jobs , or NOt 1 job .... unable to balance ..., M has only 1 job , so M can
balance--.

原题 的 premise not A---->not B , conclusion A----->B


D 的 premise 也是 not A----->not B, conclusion A------>B

125
所以都错在了 用 否命题 而非 逆否命题来 作出推论 :]
方便大家以后研究 其他选项
A) Anyone who grows up in a large family is accustomed to making compromises.
Meredith is accustomed to making compromises, so she might have grown up in a large
family. A---->B B----->A
(B) Anyone who is not in favor of this proposal is ill informed on the issue. Jeanne
opposes the proposal, so she is ill informed on the issue.notA----->B not A----->B
(C) No one who likes music misses a performance of the symphony. Paul likes music, yet
last week he missed a performance of the symphony.A---->not B A-----> B
(D) Anyone who works two or more jobs is unable to find a balance between professional
and personal life. Maggie has only one job, so she can find a balance between her
professional and personal life. notA--->not B , A------>B
(E) No one who is hot-tempered and strong-willed will succeed in this business. Jeremy
is strong-willed, so he will not succeed in this business.A&C ---->not B , C---->not B

请教 LSAT-6-III-13,17,23
Arguing that there was no trade between Europe and East Asia in the early Middle Ages
because there are no written records of such trade is like arguing that the yeti, an apelike
creature supposedly existing in the Himalayas, does not exist because there have been
no scientifically confirmed sightings. A verifiable sighting of the yeti would prove that the
creature does exist, but the absence of sightings cannot prove that it does not.

13. Which one of the following considerations, if true, best counters the argument?

(A) Most of the evidence for the existence of trade between Europe and East Asia in the
early Middle Ages is archaeological and therefore does not rely on written records.

(B) Although written records of trade in East Asia in the early Middle Ages survived, there
are almost no Europe documents from that period that mention trade at all.

(C) Any trade between Europe and East Asia in the early Middle Ages would necessarily
have been of very low volume and would have involved high-priced items, such as
precious metals and silk.

(D) There have been no confirmed sightings of the yeti, but there is indirect evidence,
such as footprints, which if it is accepted as authentic would establish the yeti's
existence.

(E) There are surviving European and East Asian written records from the early Middle
Ages that do not mention trade between the two regions but would have been very likely
to do so if this trade had existed.

答案:E,E 为什么会是削弱呢?

126
Have to answer in English.

1. After you finish reading the passage, you should know that the author drew a parallel
reasoning between the trade and yeti (a kind of primate I think, or snowman). If No record
--> no trade, then no sighting --> no yeti; But a sighting can prove that yeti exists. So
there might have been trade, just not on the record. If trade and yeti are not comparable
in this regard, the argument is flawed.

E is right in that it states that if there was trade, it would be very likely that the written
record will mention it. Therefore, no mention, likely no trade. This is the difference
between the case of yeti and that of trade. Yeti is living in area uninhabited by human, so
no sighting cannot rule out the possibility of its existence. But there would be record if
there was trade.

15.Twenty years ago the Republic of Rosinia produced nearly 100 million tons of
potatoes, but last year the harvest barely reached 60 million tons. Agricultural
researchers, who have failed to develop new higher yielding strains of potatoes, are to
blame for this decrease, since they have been concerned only with their own research
and not with the needs of Rosinia.

Which one of the following is an assumption on which the argument depends?

(A) Any current attempts by agricultural researchers to develop higher-yielding potato


strains are futile.

(B) Strains of potatoes most commonly grown in Rosinia could not have produced the
yields last year that they once did.

(C) Agricultural researchers often find concrete solutions to practical problems when
investigating seemingly unrelated questions.

(D) Wide fluctuations in the size of the potato crop over a twenty-year period are not
unusual.

(E) Agricultural research in Rosinia is funded by government grants.

这题我想来想去的就是认为 A 对。但是答案是 B 呀

assumption 就是隐含的必要条件. 如果对其取非,否定此条件,那么就不能推出原文的


conclusion. 即 A-->B ==> 非 B--非 A, B 在此为 assumption.

127
就此题来讲, 将 A 取非, 得到"Not any attempt ...". 如果 A 为 assimption,那么取非原结论没
有根据. 即 Not any attempt ...--> agricultural researchers can produce high-yield strains.

可以看出上述推理不成立, 因为造成没有 high-yield 品种地原因不明,原题没有给,可以说是


out of scope. 我们可以猜到的有时间不足(即使所有的努力都是有成效的,也许也要 10 年后
才能生产出).

这道题你也可以换个角度想: futile attempt 只是造成 failure to produce high-yield strain 的


一个原因, arguer 也许认为是其它原因造成的 .比如原文所说的:"since they have been
concerned only with their own research and not with the needs of Rosinia". 如果答案把解
释局限于 one specific, 一般都错.

如果 A 为"Due to some reasons, agricutural researchers failed to developed a high-yield


strain", 就对了.

anyway, 所谓充分必要,顾名思义,充分就是某个条件,如果其成立,足以推出某个结论.举例:
天黑了{A},(所以)我们不能打球了{B}. "天黑了"就是充分条件. 其逻辑关系为有 A 一定有 B.
注意到充分条件可以不只一个,比如说"下雨了"也是一个充分条件.

必要条件既如果没有此条件,推不出结论,或某个结论成立,此条件必然成立. 举例:吃饭{A}才
能饱{B}. A 为必要条件,既不吃饭{非 A},就一定不饱{非 B}. 其逻辑关系为有 B 一定有 A. 注意
A 不是充分条件,因为 A 本身不足以推出 B,还要吃地够多才行.

还有一种是充分必要条件: A<-->B

17. Samples from the floor of a rock shelter in Pennsylvania were dated by analyzing the
carbon they contained. The dates assigned to samples associated which human
activities formed a consistent series, beginning with the present and going back in time, a
series that was correlated with the depth from which the samples came. The oldest and
deepest sample was dated at 19,650 years before the present, plus or minus 2,400
years. Skeptic, viewing that date of human migration into North America, suggested that
the samples could have been contaminated by dissolved "old carbon" carried by
percolating groundwater from nearby coal deposits.

Which one of the following considerations, if true, argues most strongly against the
suggestion of the skeptics?

(A) No likely mechanism of contamination involving percolating groundwater would have


affected the deeper samples from the site without affecting the uppermost sample.

(B) Not every application of the carbon-dating procedure has led to results that have
been generally acceptable to scientists.

128
(C) There is no evidence that people were using coal for fuel at any time when the
deepest layer might have been laid down.

(D) No sample in the series, when retested by the carbon-dating procedure, was
assigned an earlier date than that assigned to a sample from a layer above it.

(E) No North American site besides the one in Pennsylvania has ever yielded a sample to
which the carbon-dating procedure assigned a date that was comparably ancient.

答案:A,不明白 A 为什么是削弱?

2. As long as you understand the passage and the choices, the answer stands out itself.
The passage says that a series of layers shows the date of huamn presence, from the
earliest to present. The deepest represents the earliest and the top the present. Now the
critics suspect that the layer was contaminated by coal mine from somewhere else. From
this sentecne "The oldest and deepest sample was dated at 19,650 years before the
present, plus or minus 2,400 years. Skeptic, viewing that date of human migration into
North America, ..." you should know that the skeptics suspect that the deepest layer was
contaminated as the history of human might not date back to 19650 years before minus
or plus 2400 years. And that is why they suspect that the layer was contaminated by old
coal mine, which apparently contained carbon of old age.

Now you should know why A is correct. If the deepest layer is contaminated, the top layer
should be contaminated too. But there is no sign of that according to the passage.

20. Psychotherapy has been described as a form of moral coercion. However, when
people are coerced, their ability to make choices is restricted, and the goal of
psychotherapy is to enhance people's ability to make choices. Hence, psychotherapy
cannot possibly be a form of coercion.

Which one of the following describes a flaw in the argument?

(A) The position being augured against is redefined unfairly in order to make it n easier
target.

(B) Psychotherapy is unfairly criticized for having a single goal, rather than having many
complex goals.

(C) No allowance is made for the fact that the proactive or results of psychotherapy might
run counter to its goals.

(D) The goals of psychotherapy are taken to justify any means that are used to achieve

129
those goals.

(E) It offers n argument to show that moral coercion is always undesirable.

答案选 C 是什么意思?是如何反应原文的?d 为什么不对

22. The true scientific significance of a group of unusual fossils discovered by the
paleontologist Charles Walcott is more likely to be reflected in a recent classification than
it was in Walcott's own classification, Walcott was, after all, a prominent member of the
scientific establishment. His classifications are thus unlikely to have done anything but
confirm what established science had already taken to be true.

Which one of the following most accurately describes a questionable technique used in
the argument?

(A) It draws conclusions about the merit of a position and about the content of that
position from evidence about the position's source.

(B) It cites two prices of evidence, each of which is both questionable and unverifiable,
and uses this evidence to support its conclusions.

(C) It bases a conclusion on two premises that contradict each other and minimizes this
contradiction by the vagueness of the terms employed.

(D) It attempts to establish the validity of a claim, which is otherwise unsupported, by


denying the truth of the opposite of that claim.

(E) It analyzes the past on the basis of social and political categories that properly apply
only to the present and uses the results of this analysis to support its conclusion.

这个题目的答案是 A,但是 A 是什么意思是如何反应原文的?

23. Anthony: It has been established that over 80 percent of those who use heirom have
a history of having used marijuana. Such evidence would seem to prove that smoking
marijuana definitely leads to heroin use.

Judith: Maybe smoking marijuana does lead to heroin use, but it is absurd to thins that
citing those statistics proves that it does. After all, 100 percent of the people who take up
heroin had a previous history of drinking water.

Judith's reply to Anthony's argument relies on which one of the following argumentative

130
strategies?

(A) offering evidence suggesting that the statistics Anthony cites in support of his
conclusion are inaccurate

(B) understanding the credibility of his conclusion by showing that it is a statement from
which absurd consequences can be derived

(C) providing an example to show that not everything that promotes heroin use is unsafe

D) demonstrating that Anthony's line of reasoning is flawed by showing such reasoning


can lead to clearly false conclusions

(E) calling into question the possibility of ever establishing causal connections solely on
the basis of statistical evidence .

答案:D, 我选了 B,不知 B/D 有什么区别,我用逻辑技巧题时,经常会在两个选项中难以选择,


拿不定主意.不知有什么好办法?

3. B is wrong because it is the reasoning of Anthony that leads to absurd conclusion, not
his conclusion that leads to an absurd conclsion. Read the choice B again

B. understanding the credibility of his conclusion by showing that it is a statement from


which absurd consequences can be derived

15. Twenty years ago the Republic of Rosinia produced nearly 100 million tons of
potatoes, but last year the harvest barely reached 60 million tons. Agricultural
researchers, who have failed to develop new higher yielding strains of potatoes, are to
blame for this decrease, since they have been concerned only with their own research
and not with the needs of Rosinia.
Which one of the following is an assumption on which the argument depends?
(A) Any current attempts by agricultural researchers to develop higher-yielding potato
strains are futile.
(B) Strains of potatoes most commonly grown in Rosinia could not have produced the
yields last year that they once did.
(C) Agricultural researchers often find concrete solutions to practical problems when
investigating seemingly unrelated questions.
(D) Wide fluctuations in the size of the potato crop over a twenty-year period are not
unusual.
(E) Agricultural research in Rosinia is funded by government grants.
答案是 B。

对 B 取非: Strains of potatoes most commonly grown in Rosinia could have produced the yields last year than they

131
once did. 说明去年土豆产量下降是别的原因造成的, 错不在 strains of potatoes, 自然错也就不在 researcher,
与原文结论相反。

对 D 取 非 , Wide fluctuations in the size of the potato crop over a twenty-year period are unusual 。 说 明
common strains of potatoes 本身一般不会出现产量大幅减少问题,那么并不能排除是 Agricultural researcher
的问题呀,应该不是 weaken 吧。

17. Samples from the floor of a rock shelter in Pennsylvania were dated by analyzing the
carbon they contained. The dates assigned to samples associated which human
activities formed a consistent series, beginning with the present and going back in time, a
series that was correlated with the depth from which the samples came. The oldest and
deepest sample was dated at 19,650 years before the present, plus or minus 2,400
years. Skeptic, viewing that date of human migration into North America, suggested that
the samples could have been contaminated by dissolved "old carbon" carried by
percolating groundwater from nearby coal deposits.

Which one of the following considerations, if true, argues most strongly against the
suggestion of the skeptics?

(A) No likely mechanism of contamination involving percolating groundwater would have


affected the deeper samples from the site without affecting the uppermost sample.

(B) Not every application of the carbon-dating procedure has led to results that have
been generally acceptable to scientists.

(C) There is no evidence that people were using coal for fuel at any time when the
deepest layer might have been laid down.

(D) No sample in the series, when retested by the carbon-dating procedure, was
assigned an earlier date than that assigned to a sample from a layer above it.

(E) No North American site besides the one in Pennsylvania has ever yielded a sample to
which the carbon-dating procedure assigned a date that was comparably ancient.

答案:A,不明白 A 为什么是削弱?

As long as you understand the passage and the choices, the answer stands out itself.
The passage says that a series of layers shows the date of huamn presence, from the
earliest to present. The deepest represents the earliest and the top the present. Now the
critics suspect that the layer was contaminated by coal mine from somewhere else. From
this sentecne "The oldest and deepest sample was dated at 19,650 years before the
present, plus or minus 2,400 years. Skeptic, viewing that date of human migration into

132
North America, ..." you should know that the skeptics suspect that the deepest layer was
contaminated as the history of human might not date back to 19650 years before minus
or plus 2400 years. And that is why they suspect that the layer was contaminated by old
coal mine, which apparently contained carbon of old age.

Now you should know why A is correct. If the deepest layer is contaminated, the top layer
should be contaminated too. But there is no sign of that according to the passage.

20. Psychotherapy has been described as a form of moral coercion. However, when
people are coerced, their ability to make choices is restricted, and the goal of
psychotherapy is to enhance people's ability to make choices. Hence, psychotherapy
cannot possibly be a form of coercion.

Which one of the following describes a flaw in the argument?

(A) The position being augured against is redefined unfairly in order to make it n easier
target.

(B) Psychotherapy is unfairly criticized for having a single goal, rather than having many
complex goals.

(C) No allowance is made for the fact that the proactive or results of psychotherapy might
run counter to its goals.

(D) The goals of psychotherapy are taken to justify any means that are used to achieve
those goals.

(E) It offers n argument to show that moral coercion is always undesirable.

答案选 C 是什么意思?是如何反应原文的?d 为什么不对

虽然表面上 coercion 会削弱人们 make decision 的能力,therapy 的目标却是增强这种能力,


但是作者并没有说明 therapy 作为一种 moaral coersion 最终不能实现使人们增强做决定的
能力。
换言之,therapy 或许可以通过 coercion 最终达到增强 make decision 的能力。

22. The true scientific significance of a group of unusual fossils discovered by the
paleontologist Charles Walcott is more likely to be reflected in a recent classification than
it was in Walcott's own classification, Walcott was, after all, a prominent member of the
scientific establishment. His classifications are thus unlikely to have done anything but
confirm what established science had already taken to be true.

133
Which one of the following most accurately describes a questionable technique used in
the argument?

(A) It draws conclusions about the merit of a position and about the content of that
position from evidence about the position's source.
(B) It cites two prices of evidence, each of which is both questionable and unverifiable,
and uses this evidence to support its conclusions.
(C) It bases a conclusion on two premises that contradict each other and minimizes this
contradiction by the vagueness of the terms employed.
(D) It attempts to establish the validity of a claim, which is otherwise unsupported, by
denying the truth of the opposite of that claim.
(E) It analyzes the past on the basis of social and political categories that properly apply
only to the present and uses the results of this analysis to support its conclusion.
---------------------------------------------------
答案 A, 为何选 A?请大家指教.

试着解释,请大家补充。

被古生物学家 Charles Walcott 发现的一组特殊化石的真正科学意义能更好的被最新的分类


反映而不是被他自己的分类,毕竟 Charles Walcott 只是一个科学机构的杰出成员,因此它的
分类除了能证实现存科学的真实性,没有其他作用.
那个最能描述在论断中使用的问题方法?
Walcott was, after all, a prominent member of the scientific establishment. 是作者用于支
持自己观点的 evidence.

passage mentioned "a true significient scientific discovery" and then mentioned "was
reflected" more of discovery of somebody else rather than that of Walcott's own. In the
last sentence, the author inferred that Walcott's discovery is not important(has not done
anything but confirmed...).

one of the questionable techniques at issue is not to evaluate the discovery on the
discovery itself but on its source. that's what answer A says.

one of the similiar mistakes is to criticize the personality of a position instead of the
position itself.

hope it will be of some help.

23. Anthony: It has been established that over 80 percent of those who use heroin have a
history of having used marijuana. Such evidence would seem to prove that smoking
marijuana definitely leads to heroin use.

134
Judith: Maybe smoking marijuana does lead to heroin use, but it is absurd to think that
citing those statistics proves that it does. After all, 100 percent of the people who take up
heroin had a previous history of drinking water.

Judith's reply to Anthony's argument relies on which one of the following argumentative
strategies?

(A) offering evidence suggesting that the statistics Anthony cites in support of his
conclusion are inaccurate

(B) undermining the credibility of his conclusion by showing that it is a statement from
which absurd consequences can be derived

(C) providing an example to show that not everything that promotes heroin use is unsafe

(D) demonstrating that Anthony's line of reasoning is flawed by showing such reasoning
can lead to clearly false conclusions

(E) calling into question the possibility of ever establishing causal connections solely on
the basis of statistical evidence .

答案是 D

B is wrong because it is the reasoning of Anthony that leads to absurd conclusion, not his
conclusion that leads to an absurd conclusion. Read the choice B again

B. understanding the credibility of his conclusion by showing that it is a statement from


which absurd consequences can be derived

Proposals for extending the United States school year to bring it more in line with its
European and Japanese counterparts are often met with the objection that curtailing the
school's three-month summer vacation would violate an established United States
tradition dating from the nineteenth century. However, this objection misses its mark.
True, in the nineteenth century, the majority of schools closed for three months every
summer, but only because they were in rural areas where successful harvests depended
on children labor. If any policy could be justified by those appears to tradition. It would be
the policy of determining the length of the school year according to the needs of the
economy.

25. Which one of the following principles, if accepted, would provide the strongest

135
justification for the conclusion?

(A) That a given social policy has traditionally been in force justifies maintaining that
policy only if doing so does not conflict with more pressing social needs.

(B) Appeals to us own traditions cannot excuse a country from the obligation to bring its
practices in line with the legitimate expectations of the rest of the world.

(C) Because appeals to tradition often serve to mask the real interests the real interests
at issue, such appeals should be disregarded.

(D) Traditional principles should be discarded when they no longer serve the needs of the
economy.

(E) The actual tradition embodied in a given practice can be accurately identified only by
reference to the reasons that originally prompted that practice.

这个题目我选的是 D,答案是 E.但是这两个选项我有些分不清楚.我的想法是这样的注意原


文的转折,特别是让步之后的转折.原文的"but only because they were in rural areas where
successful harvests depended on children labor."

作者的反驳方式是先承认“尊重传统”(If any policy could be justified by those appears to


tradition),然后指出即使这样,那么也该是尊重传统之所以被建立的原因,以此来说明
在新情况下,这一原因已经发生改变,所以。。也改改变,作者并没有说要摈弃传统
(D),而恰恰采取了欲擒故纵的方式,先退一步,承认了传统。
如果要加强,那么答案应该是 E

26. The argument counters the objection by

(A) providing evidence to show that the objection rallies on a misunderstanding about the
amount of time each year United States schools traditionally have been closed

(B) calling into question the relevance of information about historical practices to current
disputes about proposed social change

(C) arguing for an alternative understanding of the nature of the United States tradition
regarding the length of the school year

(D) showing that those who oppose extending the school year have no genuine concern
for tradition

(E) demonstrating that tradition justifies bringing the United States school year in line with
that of the rest of the industrialized world

136
C 这个题目的题干是什么意思?和 25 题的区别是什么呢?它的突破点是什么?

C。WEAKEN 方法。。我觉得主要是理解原文的论证过程。。论据是以何方式 SUPPORT OR WEAKEN the


conclusion 的。从原文可以看出,作者通过给了 TRATION 一个历史原因解释,来支持它的结论的。。

Set 7-1

3. Harry: Airlines have made it possible for anyone to travel around the world in much
less time than was formerly possible.
Judith: That is not true. Many flights are too expensive for all but the rich.
Judith’s response shows that she interprets Harry’s statement to imply that
(A) the majority of people are rich
(B) everyone has an equal right to experience world travel

137
(C) world travel is only possible via routes serviced by airlines
(D) most forms of world travel are not affordable for most people
(E) anyone can afford to travel long distances by air

问的是 Judith 不同意 Harry 的什么观点。


Harry 认为人人都有钱坐飞机,Judith 认为不是。

Lsat 7-1-4
题目讨论的是 on which to focus,注意是“focus”
题目“focus”的是比例
要说它 misleading,那就是不“focus”比例啦
当然选 e

lsat 7 (1) questions


Coherent solutions for the problem of reducing health-care costs cannot be found within
the current piecemeal system of paying these costs. The reason is that this system gives
health-care providers and insurers every incentive to shift, wherever possible, the costs
of treating illness onto each other or any other party, including the patient. That clearly is
the lesson of the various reforms of the 1980s; push in on one part of this pliable
spending balloon and an equally expensive bulge pops up elsewhere. For example,
when the government health-care insurance program for the poor cut costs by
disallowing payments for some visits to physicians, patients with advanced illness later
presented themselves at hospital emergency rooms in increased numbers.

7. The argument proceeds by

(A) showing that shifting costs onto the patient contradicts the premise of health-care
reimbursement

(B) attribution without justification fraudulent intent to people

(C) employing an analogy to characterize interrelationships

(D) denying the possibility of a solution by disparaging each possible alternative system

(E) demonstrating that cooperation is feasible by citing an instance

138
8. The argument provides the most support for which one of the following?

(A) Under the conditions in which the current system operates, the overall volume of
health-care costs could be shrunk, if at all, only by a comprehensive approach

(B) Relative to the resources available for health-care funding, the income of the higher-
paid health-care professionals is too high.

(C) Health-care costs are expanding to meet additional funds that have been made
available for them.

(D) Advances in medical technology have raised the expected standards of medical care
but have proved expensive.

(E) Since unfilled hospital beds contribute to overhead charges on each patient’s bill, it
would be unwise to hold unused hospital capacity in reserve for large-scale emergencies.

A
这个题目根本没有读懂,请教!

第一句话怎不看完全呢? 现有的系统是没有办法的,所以要有更综合的系统才能找到解决
的办法。Coherent solutions for the problem of reducing health-care costs cannot be found
(within the current piecemeal system of paying these costs.〔

回小美人鱼:
A) Under the conditions in which the current system operates, the overall volume of
health-care costs could be shrunk, if at all, only by a comprehensive approach
"only by a comprehensive approach" 是指一个更综合的系统? 还是仅仅一个 solution? 如
果是后者, 则还是在原系统中打转, 不可能有有效解决的办法; 而如果是指前者, 1. approach
意为方法, 手段, 可以指系统吗? 2, A 中也没有和原系统的比较, 更综合从何而说?

至于第一题,说的是因为没有办法在现行的系统下解决费用的问题,然后给出了为什么不
能的原因,然后 wherever possible, the costs of treating illness onto each other or any
other party, including the patient 就是说大家都互相推。如果是用比较综合的办法,比如每
个 party 都负责一些,就应该解决这个问题了。这个是我延伸出来。为的是让你们更 明白。
逻辑题和阅读不同。不需要给你解释的头头是道,因为不是现象解释,而是逻辑。所以,就
是里面有你没说清楚的漏洞,对于题目无损也无妨。

15. Eight years ago hunting was banned in Greenfield County on the grounds that
hunting endangers public safety. Now the deer population in the county is six times what

139
it was before the ban. Deer are invading residential areas. Damaging property and
causing motor vehicle accidents that result in serious injury to motorists. Since there
were never any hunting=related injuries in the county, clearly the ban was not only
unnecessary but has created a danger to public safety that would not otherwise exist.
Which one of the following, if true, provides the strongest additional support for the
conclusion above?
(A) In surrounding counties, where hunting is permitted, the size of the deer population
has not increased in the last eight years.
(B) Motor vehicle accidents involving deer often result in damage to the vehicle, injury to
the motorist, or both.
(C) When deer populations increase beyond optimal size, disease and malnutrition
become more widespread among the deer herds.
(D) In residential areas in the county, many residents provide food and salt for deer.
(E) Deer can cause extensive damage to ornamental shrubs and trees by chewing on
twigs and saplings.
Answer: A 但我觉得 B 好象更对

1。B 错的原因是:问题要 additional support ,B 所说的 Motor vehicle accidents 在原文已


作为证据提出,所以不是 additional
2。A 对的原因:作对比论证,说明 DEER 数量增加的原因是 BAN,而原文已知 DEER 数
量的增加已对公共安全造成危害。故为答案。

16. Comets do not give off their own light but reflect light from other sources, such as the
Sun. Scientists estimate the mass of comets by their brightness: the greater a comet’s
mass, the more light that comet will reflect. A satellite probe, however, has revealed that
the material of which Halley’s comet is composed reflects 60 times less light per unit of
mass than had been previously thought.

The statements above, if true, give the most support to which one of the following?

(A) Some comets are composed of material that reflects 60 times more light per unit of
mass than the material of which Halley’s comet is composed.
(B) Previous estimates of the mass of Halley’s comet which were base on its brightness
were too low.
(C) The total amount of light reflected from Halley’s comet is less than scientists had
previously thought.
(D) The reflective properties of the material of which comets are composed vary
considerably from comet to comet.
(E) Scientists need more information before they can make a good estimate of the mass
of Halley’s comet.
----------------------
答案 B

140
LSAT-7-1-19
When Alicia Green borrowed a neighbor’s car without permission, the police merely gave
her a warning. However, when Peter Foster did the same thing, he was charged with
automobile theft. Peter came to the attention of the police because the car he was driving
was hit by a speeding taxi. Alicia was stopped because the car she was driving had
defective taillights. It is true that the car Peter took got damaged and the car Alicia took
did not, but since it was the taxi that caused the damage this difference was not due to
any difference in the blameworthiness of their behavior. Therefore, Alicia should also
have been charged with automobile theft.

18. The statement that the car Peter took got damaged and the car Alicia took did not
plays which one of the following roles in the argument?

(A) It presents a reason that directly supports the conclusion.

(B) It justifies the difference in the actual outcome in the two cases.

(C) It demonstrates awareness of a fact on which a possible objection might be based.

(D) It illustrates a general principle on which the argument relies.

(E) It summarizes a position against which the argument is directed.

这个题目的答案选 C,

19. If all of the claims offered in support of the conclusion are accurate, each of the
following could be true EXCEPT:
(A) The interests of justice would have been better served if the police had released
Peter Foster with a warning.
(B) Alicia Green had never before driven a car belonging to someone else without first
securing the owner’s permission.
(C) Peter Foster was hit by the taxi while he was running a red light, whereas Alicia
Green drove with extra care to avoid drawing the attention of the police to the car she
had taken.
(D) Alicia Green barely missed hitting a pedestrian when she sped through a red light ten
minutes before she was stopped by the police for driving a car that had defective
taillights.(C)
(E) Peter Foster had been cited for speeding twice in the preceding month, whereas
Alicia Green had never been cited for a traffic violation.
答案是 C.

141
1.原文无法得出各人以前情况,故 B 有可能,这次是 A 的第一次,怎么不可能,凡事总有
第一次。不违背原文的就有可能

2。选 C 的原因是原文说 DAMAGE 是 TAXI 造成车的,P 完全没责任(所以应跟 A 一样)。


而 C 说 P 是闯红灯时被 DAMAGE 的,故有责任。和原文相反。故必错。为答案

22. There is a widespread belief that people can predict impending earthquakes from
unusual animal behavior. Skeptics claim that this belief is based on selective
coincidence: people whose dogs behaved oddly just before an earthquake will be
especially likely to remember that fact. At any given time, the skeptics say, some of the
world’s dogs will be behaving oddly.
Clarification of which one of the following issues would be most important to an
evaluation of the skeptics’ position?
(A) Which is larger, the number of skeptics or the number of people who believe that
animal behavior can foreshadow earthquakes?
(B) Are there means other than the observation of animal behavior that nonscientists can
use to predict earthquakes?
(C) Are there animals about whose behavior people know too little to be able to
distinguish unusual from everyday behavior?
(D) Are the sorts of behavior supposedly predictive of earthquakes as pronounced in
dogs as they are in other animals?
(E) Is the animal behavior supposedly predictive of earthquakes specific to impending
earthquakes or can it be any kind of unusual behavior?
怎么会选 E 呢

题干说一般认为可以根据动物在地震发生前的异常反应来预测地震。有些对此持怀疑态度的
人认为这是基于某些有选择性的巧合,比如说某个人的狗在地震前有反常行为,这个人是
因为狗的反常行为所以特别容易记住这件事。言下之意是说狗的反常行为并不是地震的预兆,
是人人为地把它和地震联系起来。所以这些怀疑者说任何时候,狗都可能有反常行为。
答案 E 说的就是要区分狗的反常行为究竟是地震预兆还是那只不过是一种反常行为。

23. Defendants who can afford expensive private defense lawyers have a lower
conviction rate than those who rely on court-appointed public defenders. This explains
why criminals who commit lucrative crimes like embezzlement or insider trading are more
successful at avoiding conviction than are street criminals.

The explanation offered above would be more persuasive if which one of the following
were true?

142
(A) Many street crimes, such as drug dealing, are extremely lucrative and those
committing them can afford expensive private lawyers.

(B) Most prosecutors are not competent to handle cases involving highly technical
financial evidence and have more success in prosecuting cases of robbery or simple
assault.

(C) The number of criminals convicted of street crimes is far greater than the number of
criminals convicted of embezzlement or insider trading.

(D) The percentage of defendants who actually committed the crimes of which they are
accused is no greater for publicly defended than for privately defended defendants.

(E) Juries, out of sympathy for the victims of crimes, are much more likely to convict
defendants accused of violent crimes than they are to convict defendants accused of
“victimless” crimes or crimes against property.

原文逻辑:负的起私人律师的被告的定罪率小于公家律师的被告.所以经济案件的被告逃
避定罪的可能性大于公家律师的被告.

A, B 反对.D 说真正犯罪的被告选择公家律师的比例不大于选择私人律师的比例.如果真
正犯了事的人都去找公家律师,而那些没犯事的人都找私人律师.恐怕定罪率的差别和律
师贵不贵就没关系了吧.

Set 7-4

LSAT-7-IV-6,8
The government provides insurance for individuals? band deposits, but requires the
banks to pay the premiums for the insurance. Since it is depositors who primarily benefit
from the security this insurance provides, the government should take steps to ensure

143
that depositors who want this security bear the cost of it and thus should make depositors
pay the premiums for insuring their own accounts.

6. Which of the following is assumed by the argument?

(A) Banks are not insured by the government against default on the loans the banks
make.

(B) Private insurance companies do not have the resources to provide banks or individual
with deposit insurance.

(C) Banks do not always cover the cost of the deposit-insurance premiums by paying
depositors lower interest rates on insured deposits than the banks would on uninsured
deposits.

(D) The government limits the insurance protection it provides by insuring accounts up to
a certain legally defined amount only.

(E) The government does not allow banks to offer some kinds of accounts in which
deposits are not insured.

答案:C,不明白 C 为什么是假设,它出现了原文没有的比较点,我一下子就排除了。

关于 6T, C 选项的意思是: 银行并没有通过降低付给存款人利息的方式来 cover premium


costs. 对其取非, 既银行 always 通过降低利率来 cover costs -->实际上 depositors 一直是
在 cover premium costs, 因为他们得到的利息相比没有 insurance 的情况要低 . Do not
cover the cost 不意味亏损.

When you make judgment on scope, the first think you should make sure is relevance. If
the information is out of scope and is irrelevant, it cannot be right. But here D is relevant
as uninsured deposits are just the opposite of insured deposits. If D is true, when the
loan is uninsured, the bank will pay higher interest than it does on insured deposits, it
means that bank normally forfeits part of the interest on insured deposits to compensate
the premium.

8. George: Some scientists say that global warming will occur because people are
releasing large amounts of carbon dioxide into the atmosphere by burning trees and
fossil fuels. We can see, though, that the predicted warming is occurring already. In the
middle of last winter, we had a month of springlike weather in our area, and this fall,
because of unusually mild temperatures, the leaves on our town 抯 trees were three
weeks late in turning color.

144
Which one of the following would it be most relevant to investigate in evaluating the
conclusion of George 抯 argument?

(A) whether carbon dioxide is the only cause of global warming

(B) when leaves on the trees in the town usually change color

(C) what proportion of global emissions of carbon dioxide is due to the burning of trees by
humans

(D) whether air pollution is causing some trees in the are to lose their leaves

(E) whether unusually warm weather is occurring elsewhere on the globe more frequently
than before

答案:E, 这种评价题有什么思路吗?我觉得 A,B,C 都相关啊。

13. The National Association of Fire Fighters says that 45 percent of homes now have
smoke detectors, whereas only 30 percent of homes had them 10 years ago. This makes
early detection of house fires no more likely, however, because over half of the domestic
smoke detectors are either without batteries or else inoperative for some other reason.
In order for the conclusion above to be properly drawn, which one of the following
assumptions would have to be made?
(A) Fifteen percent of domestic smoke detectors were installed less than 10 years ago.
(B) The number of fires per year in homes with smoke detectors has increased.
(C) Not all of the smoke detectors in homes are battery operated.
(D) The proportion of domestic smoke detectors that are inoperative has increased in the
past ten years.
(E) Unlike automatic water sprinklers, a properly functioning smoke detector cannot by
itself increase fire safety in a home.

这里并不是要将 D 否定。而是,在这里要找到桥梁推出结论。
现在的 smoke detector 数量增加了,但是并没有起到作用,为什么呢? 作者说很多 sd 都
是没有电池或者没有按照正确办法设置。D 中说这种没有电池或者没有按照正确方法设置的
SD 增加了。正好是原因、

1. no more likely 是省略的比较。NO MORE LIKGELY 的意思为:可能性没有更大。是将现


在 DETECTION 的可能性和 10 年前比。

2.这题绕来绕去,把我搞晕了。我举的例正好反了。将 D 取非,变成这 10 年来,不能用的


DETECTORS 比例没有增加。如现在是 50%不能用,10 年前不能用的 DETECTORS 比例

145
应该是比 50%大,而不是小,如果小,则说明不能用的 DETECTORS 增加了。也就是 10
年前能用的小于 50%(或者别的,反正能用的比例小于现在能用的比例),又因为 30%小
于 45%,所以 10 年前能用的 DETECTORS 少于现在。说明现在比以前好(和结论相反)。
否定结论。

解释比找答案难多了。

17. Since the introduction of the Impanian National Health Scheme, Impanians (or their
private insurance companies) have had to pay only for the more unusual and
sophisticated medical procedures. When the scheme was introduced, it was hoped that
private insurance to pay for these procedures would be available at modest cost, since
the insurers would no longer be paying for the bulk of health care costs, as they had
done previously. Paradoxically, however, the cost of private health insurance did not
decrease but has instead increased dramatically in the years since the scheme 抯
introduction.

Which one of the following, if true, does most to explain the apparently paradoxical
outcome?

(A) The National Health scheme has greatly reduced the number of medical claims
handled annually by Impania's private insurers, enabling these firms to reduce overhead
costs substantially.

(B) Before the National Health scheme was introduced, more than 80 percent of all
Impanian medical costs were associated with procedures that are now covered by the
scheme.

(C) Impanians who previously were unable to afford regular medical treatment now use
the National Health scheme, but the number of Impanians with private health insurance
has not increased.

(D) Impanians now buy private medical insurance only at times when they expect that
they will need care of kinds not available in the National Health scheme.

(E) The proportion of total expenditures within Impania that is spent on health care has
declined since the introduction of the National Health scheme.

答案:D

保险公司原本是想 pay only for the more unusual and sophisticated medical procedures.
这样就可以减少一些保费支出
结果却是增加了,

146
作为解释,找出一个理由就好
D 说那些买保险的人直到要出事了才买,那么保险公司肯定导老霉了。保费再多也不会有看
病的钱多啊,所以这就就可以很好的解释原文了

原文意思为引进某个保险计划,该计划保险公司支付异常的医疗费,不像以前异常还是不
异常都付。本来这样会使保险金(买某个保险的价格,这和利润挂钩)下降,但矛盾的是保
险金反而增加。D 提出解释说引入该计划后,人们只在有需要时才买保险。这样保险公司付
医疗费的几率就增加,羊毛出在羊身上,这样保险金当然也随之上涨。

NATIONAL SCHEM 提供的事普通医疗。西方国家有政府买单。但特殊的医疗费需个人买保


险,因为医疗费贵得你付不起。

18. In clinical trials of new medicines, half of the subjects receive the drug being tested
and half receive a physiologically inert substance – a placebo. Trials are designed with
the intention that neither subjects nor experimenters will find out which subjects are
actually being given the drug being tested. However, this intention is frequently frustrated
because ____.

Which one of the following, if true, most appropriately completes the explanation?

(A) often the subjects who receive the drug being tested develop symptoms that the
experimenters recognize as side effects of the physiologically active drug
(B) subjects who believe they are receiving the drug being tested often display
improvements in theirconditions regardless of whether what is administered to them is
physiologically active or not
(C) in general, when the trial is intended to establish the experimental drug’s safety rather
than its effectiveness, all of the subjects are healthy
(D) when a trial runs a long time, few of the experimenters will work on it from inception
to conclusion
(E) the people who are subjects for clinical trials must, by law, be volunteers and must be
informed of the possibility that they will receive a placebo

答案 A,我选 E:intention 是不让 experimenter 知道,只有 E 符合。而 A 只是一种试验现


象。

18. 我不 知道 你为 什么 要选 E,E 是无 关选 项。 就是 说 the people who are subjects for


clinical trials 是不是自愿或者 must be informed of the possibility that they will receive a
placebo。原文中根本没有提到这些信息。但是 A 选项很好,说 experimenter 能认识那些因
为被注射了那种试验的 drug 而得的 symptoms,并把这些 symptoms 当成是 side effects
of the physiologically active drug。而 a placebo 是 a physiologically inert substance。这样

147
把 placebo 与那种试验的 drug 区分开来。

Active and Inert 区分。

19. It takes 365.25 days for the Earth to make one complete revolution around the sun.
Long ?standing convention makes a year 365 days long, with an extra day added every
fourth year, and the year is divided into 52 seven-day weeks. But since 52 times 7 is only
364, anniversaries do not fall on the same day of the week each year. Many scheduling
problems could be avoided if the last day of each year and an additional day every fourth
year belonged to no week, so that January 1 would be a Sunday every year.

The proposal above, once put into effect, would be most likely to result in continued
scheduling conflicts for which one of the following groups?

(A) people who have birthdays or other anniversaries on December 30 or 31

(B) employed people whose strict religious observances require that they refrain from
working every seventh day

(C) school systems that require students to attend classes a specific number of days
each year

(D) employed people who have three-day breaks from work when holidays are
celebrated on Mondays or Fridays

(E) people who have to plan events several years before those events occur

答案:B,我选了 E

B 是说 employed people 被限制在每隔 7 天做一件事情。假如从 January 1 Sunday 开始,


一年后应该是星期 1(364+1)。但按照新的方法,又是 Sunday,明显冲突()。简单可以
这样想。新的方法不是按 7 循环(7 循环*52 再做一个 1 天的调整),而 B 要求按 7 循环

23. The workers at Bell Manufacturing will shortly go on strike unless the management
increases their wages. As Bell's president is well aware, however, in order to increase the
worker's wages, Bell would have to sell off some of its subsidiaries. So, some of Bell's
subsidiaries will be sold.
The conclusion above is properly drawn if which one of the following is assumed?
(A) Bell Manufacturing will begin to suffer increased losses.
(B) Bell's management will refuse to increase its worker's wages.
(C) The workers at Bell Manufacturing will not be going on strike.
(D) Bell's president has the authority to offer the workers their desired wage increase.

148
(E) Bell's workers will not accept a package of improved benefits in place of their desired
wage increase.

这道题目我觉得是一个逻辑推导方向的问题:
如果其中的 in order to 是做为必要条件,那也就是:
increase the worker's wages—》Bell would have to sell off some of its subsidiaries
C 作为答案一点问题没有。
可是,我觉得在很多题目中,作为目的介词(比如, in order to , to , so as to ,
etc),所采取的措施却是做为目的的充分条件,比如该道题目也可以这么理解:
Bell would have to sell off some of its subsidiaries—》increase the worker's wages
(这种推导在 GMAT 的 RC 中,weaken 的思路大部分就是削弱措施没有办法达到目的,
也就是断桥)
可是,如果这边这么理解,既 Bell would have to sell off some of its subsidiaries 是作为
increase the worker's wages 的充分条件,那么 C 是否就有问题呀?
因为,前面的推导也能推出:
The workers at Bell Manufacturing will not be going on strike.— 》increase the worker's
wages
两个都是充分条件,如何判断 C 就为 assumption 呀?
我反而觉得 E 更妥当一点,无论对 in order to 的推导方向是如何理解的,E 取非后对原文
都是一种 weaken。
请指教!

题干前提:no increase wage——》strike


increase wage——》sell subsidirary
结论:sell subsidiary
A 取非,will go on strike
E 为什么错?confusing
其实你自己已经把逻辑关系分析得很清楚了,为使结论成立,必须 increase wage,
那么如何 increase wage 呢,只有加入 not strike 这个条件。所以这是个 assumption

24. One sure way you can tell how quickly a new idea – for example, the idea of
“privatization” – is taking hold among the population is to monitor how fast the word or
words expressing that particular idea are passing into common usage. Professional
opinions of whether or not words can indeed be said to have
passed into common usage are available from dictionary editors, who are vitally
concerned with this question.

The method described above for determining how quickly a new idea is taking hold relies
on which one of the following assumptions?

(A) Dictionary editors are not professionally interested in words that are only rarely used.

149
(B) Dictionary editors have exact numerical criteria for telling when a word has passed
into common usage.
(C) For a new idea to take hold, dictionary editors have to include the relevant word or
words in their dictionaries.
(D) As a word passes into common usages, its meaning does not undergo any severe
distortions in the process.
(E) Words denoting new ideas tend to be used before the ideas denoted are understood.

答案 D

The statement doesn't mention anything about including words into dictionary. Actually,
the term "dictionary editors" may give you such a wrong feeling.

the author wants to say people can decide how quickly a new idea passes to the
population by deciding if the word or words representing the idea are passed to common
usage. but the bottom line is the meaning is not distorted when the words pass by,
otherwise the conclusion is wrong

Set 8-2

150
3. Citizen of Mooresville: Mooresville's current city council is having a ruinous effect on
municipal finances. Since a majority of the incumbents are running for reelection, I am
going to campaign against all these incumbents in the upcoming city council election.
The only incumbent I will support and vote for is the one who represents my own
neighborhood, because she has the experience necessary to ensure that our
neighborhoods interests are served. If everyone in Mooresville would follow my example,
we could substantially change the council's membership.
Assuming that each citizen of Mooresville is allowed to vote only for a city council
representative from his or her own neighborhood, for the council's membership to be
changed substantially, it must be true that
(A) at least some other voters in Mooresville do not make the same exception for their
own incumbent in the upcoming election
(B) most of the eligible voters in Mooresville vote in the upcoming election
(C) few of the incumbents on the Mooresville city council have run for reelection in
previous elections
(D) all of the seats on the Mooresville city council are filled by incumbents whose terms
are expiring
(E) none of the cha1lengers in the upcoming election for seats on Mooresville's city
council are better able to serve the interests of their neighborhoods than were the
incumbents
answer:a
I can’t understand the meaning of a, why not b?

lsat 8-I-3:每个选民只能决定自己社区(Neighborhood)议员的去留,如果他们都象发言
的人一样保留自己社区的现任议员,那么整个议会组成不会有任何变化。(A)是答案,但
不具有充分性。(B) 无关,如果他们投现任议员,再多的人去投票也无济于事。

8. During the 1980s the homicide rate in Britain rose by 50 percent. The weapon used
usually was a knife. Potentially lethal knives are sold openly and legally in many shops.
Most homicide deaths occur as a result of unpremeditated assaults within the family.
Even if these are increasing, they would probably not result in deaths if it were not for the
prevalence of such knives. Thus the blame lies with the permissiveness of the
government that allows such lethal weapons to be sold.
Which one of the following is the strongest criticism of the argument above?
(A) There are other means besides knives, such as guns or poison, that can be used to
accomplish homicide by a person who intends to cause the death of another.
(B) It is impossible to know how many unpremeditated assaults occur within the family,
since many are not reported to the authorities.
(C) Knives are used in other homicides besides those that result from unpremeditated
assaults within the family.
(D) The argument assumes without justification that the knives used to commit homicide
are generally purchased as part of a deliberate plan to commit murder or to inflict

151
grievous harm on a family member.
(E) If the potentially lethal knives referred to are ordinary household knives, such knives
were common before the rise in the homicide rate; but if they are weaponry, such knives
are not generally available in households.

E informs that if the knives is prevalent, the prevalence is not caused by permissiveness
of government, but by its essential trait. Otherwise it will not be prevalent.

9. Nutritionist: Vitamins synthesized by chemists are exactly the same as vitamins that
occur naturally in foods. therefore, it is a waste of money to pay extra for brands of
vitamin pills that are advertised as made of higher-quality ingredients or more natural
ingredients than other brands are.
the nutritionist's advice is based on which one of the following assumptions?
A. it is a waste of money for people to supplement their diets with vitamin polls.
B. brands of vitamin pills made of natural ingredients always cost more money than
brands that contain synthesized vitamins.
C. all brands of vitamin polls contain some synthesized vitamins
D. some producers of vitamin pills are guilty of false advertising
E. there is no nonvitamin ingredient in vitamin pills whose quality makes one brand worth
more money than another brand.
答案是 e。b 为什么不对,题干中 higher-quality 是指 natura ingredients 吗。

10. Most people are indignant at the suggestion that they are not reliable authorities
about their real wants. Such self-knowledge, however, is not the easiest kind of
knowledge to acquire. Indeed, acquiring it often requires hard and even potentially risky
work. To avoid such effort, people unconsciously convince themselves that they want
what society says they should want.
The main point of the argument is that
(A) acquiring self-knowledge can be risky
(B) knowledge of what one really wants is not as desirable as it is usually thought to be
(C) people cannot really want what they should want
(D) people usually avoid making difficult decisions
(E) people are not necessarily reliable authorities about what they really want,
答案是 E,但我觉得是 B

Because some student demonstrations protesting his scheduled appearance have


resulted in violence, the president of the Imperialist Society has been prevented from
speaking about politics on campus by the dean of student affairs. Yet to deny anyone the
unrestricted freedom to speak is to threaten everyone's right to free expression. Hence

152
the dean's decision has threatened everyone's right to free expression.
The pattern of reasoning displayed above is most closely paralleled in which one of the
following?
(A) Dr. Pacheco saved a child's life by performing emergency surgery. But surgery rarely
involves any risk to the surgeon. Therefore, if an act is not heroic unless it requires the
actor to take some risk. Dr. Pacheco's surgery was not heroic.
(B) Because anyone who performs an act of heroism acts altruistically rather than
selfishly, a society that rewards heroism encourages altruism rather than pure self-
interest.
(C) In order to rescue a drowning child, Isabel jumped into a freezing river. Such acts of
heroism performed to save the Life of one enrich the lives of all. Hence. Isabel's action
enriched the lives of all.
(D) Fire fighters are often expected to perform heroically under harsh conditions. But no
one is ever required to act heroically. Hence, fire fighters are often expected to perform
actions they are not required to perform.
(E) Acts of extreme generosity are usually above and beyond the call of duty. Therefore.
most acts of extreme generosity are heroic, since all actions that are above and beyond
the call of duty are heroic
answer: c 我不明白这道题的推理模式,看哪个都象答案?

lsat 8-I-13:这是以前学过的“稻草人(Straw Man)”逻辑模型: 即将一个观点无限上纲到


已经背离原观点的真实含义;然后将对扭曲后观点的定性,强加在原来的观点上面。明白这
一点,从选项中再找出来一个“稻草人”就行了。(C)

original behavior: Isabel's action of jumping into a freezing river to rescue a drowning
child
straw man: such acts of heroism
从 Such acts of heroism performed to save the Life of one enrich the lives of all.
推出 Isabel's action enriched the lives of all

14. Professor: Members of most species are able to communicate with other members of
the same species, but it is not true that all communication can be called "language." The
human communication system unquestionably qualifies as language. In fact, using
language is a trait without which we would not be human.

Student: I understand that communication by itself is not language, but how do you know
that the highly evolved communication systems of songbirds, dolphins, honeybees, and
apes, for example, are not languages?

The student has interpreted the professor's remarks to mean that

(A) different species can have similar defining traits

153
(B) every human trait except using language is shared by at least one other species

(C) not all languages are used to communicate

(D) using language is a trait humans do not share with any other species

(E) humans cannot communicate with members of other species

答案是 D,但学生本身是在提问,并没有对老师的话作出什么解释啊,这其中有什么逻辑
关系呢

17. Biographer: Arnold's belief that every offer of assistance on the part of his colleagues
was a disguised attempt to make him look inadequate and that no expression of
congratulations on his promotion should be taken at face value may seem irrational, in
fact, this belief was a consequence of his early experiences with an admired older sister
who always made fun of his ambitions and achievements. In light of this explanation,
therefore, Arnold's stubborn belief that his colleagues were duplicitous emerges as
clearly justified.
The flawed reasoning in the biographer's argument is most similar to that in which one of
the following?
(A) The fact that top executives generally have much larger vocabularies than do their
subordinates explains why Sheldon's belief, instilled in him during his childhood, that
developing a large vocabulary is the way to gel to the top in the world of business is
completely justified.
(B) Emily suspected that apples are unhealthy ever since she almost choked to death
while eating an apple when she was a child. Now, evidence that apples treated with
certain pesticides can be health hazards shows that Emily's long-held belief is fully
justified.
(C) As a child. Joan was severely punished whenever she played with her father's prize
Siamese cat. Therefore, since this information makes her present belief that cats are not
good pets completely understandable, that belief is justified.
(D) Studies show that when usually well-behaved children become irritable, they often
exhibit symptoms of viral infections the next day. The suspicion, still held by many adults,
that misbehavior must always be paid for is thus both explained and justified.
(E) Sumayia's father and mother were both concert pianists, and as a child. Sumayia
knew several other people trying to make careers as musicians. Thus Sumayia's opinion
that her friend Anthony lacks the drive to be a successful pianist is undoubtedly justified.
Answer :C, I CHOICE E,这题的推理漏洞在哪儿?

lsat 8-I-17:又是一个 Parallel_Flaw; 逻辑模式是“Two Wrongs Make a Right”, 即用一个


错误来解释另外一个错误,使其具有合理性,注意这两个错误之间必须相关联。只有 (C)
才是答案。

154
再问 LSAT Section 8 - 19
Oxygen.18 is a heavier-than-normal isotope of oxygen. In a rain cloud, water molecules
containing oxygen-18 are rarer than water molecules containing normal oxygen. But in
rainfall, a higher proportion of all water molecules containing oxygen-18 than of all water
molecules containing ordinary oxygen descends to earth. Consequently, scientists were
surprised when measurements along the entire route of rain clouds' passage from above
the Atlantic Ocean, the site of their original formation, across the Amazon forests, where
it rains almost daily, showed that the oxygen-18 content of each of the clouds remained
fairly constant.

19. Which one of the following statements, if true, best helps to resolve the conflict
between scientists' expectations, based on the known behavior of oxygen-18, and the
result of their measurements of the rain clouds' oxygen-IS content?
(A) Rain clouds above tropical forests are poorer in oxygen-18 than rain clouds above
unforested regions.
(B) Like the oceans, tropical rain forests can create or replenish rain clouds in the
atmosphere above them.
(C) The amount of rainfall over the Amazon rain forests is exactly the same as the
amount of rain originally collected in the clouds formed above the Atlantic Ocean.
(D) The amount of rain recycled back into the atmosphere from the leaves of forest
vegetation is exactly the same as the amount of ram in river runoffs that is not recycled
into the atmosphere.
(E) Oxygen-18 is not a good indicator of the effect of tropical rain forests on the
atmosphere above them.

以 前 的 回 贴 http://forum.chasedream.com/dispbbs.asp?
boardID=24&replyID=20636&ID=3700&skin=1
——————————————————————————————————————
Oxygen.18 is a heavier-than-normal isotope of oxygen. In a rain cloud, water molecules
containing oxygen-18 are rarer than water molecules containing normal oxygen.
O18 (含 Oxygen.18 的水分子) 的比例很低,与 O (含普通 Oxygen.的水分子), 句题多多少
不知道设为 X

But in rainfall, a higher proportion of all water molecules containing oxygen-18 than of all
water molecules containing ordinary oxygen descends to earth.
降雨时,

所有 O18 降下的比例 (proportion of all water molecules containing oxygen-18)

高于

所有 O 降下的比例 (proportion of all water molecules containing ordinary oxygen)

155
换句话说,所有水分子中, 分为两组 O18 和 O, 下雨时 O18 组 下去的比例要高于 O 组
下去的比例,例如 在所有 O18 有 50%下去了, 在所有中 O 只有 20%下去了

所以 如果下雨 O18 在 rain cloud 中的比例 因该比以前 X 更低, 所以引出下面的科学家的


困惑

Consequently, scientists were surprised when measurements along the entire route of
rain clouds' passage from above the Atlantic Ocean, the site of their original formation,
across the Amazon forests, where it rains almost daily, showed that the oxygen-18
content of each of the clouds remained fairly constant.

所有的云中的 O18 的比例不变?? 所以一种可能是在移动的过程中没下雨,这正好是 C,


答案确是 B

(B) Like the oceans, tropical rain forests can create or replenish rain clouds in the
atmosphere above them.

(C) The amount of rainfall over the Amazon rain forests is exactly the same as the
amount of rain originally collected in the clouds formed above the Atlantic Ocean.

B 只能所提供了可能性,而且这种可能还需要在补从的雨云中, O18 的比例要高于 the


clouds formed above the Atlantic Ocean.

我认为 B 提供了一种不肯定的解释,不应该正确

To jacy: " The clouds rainfall 不等于 the raifall.我想你是看错了." 我觉得问题不在这两个词


的区别,应该是一样的。

To wubin:
But in rainfall, a higher proportion of all water molecules containing oxygen-18 than of all
water molecules containing ordinary oxygen descends to earth.

我想你的意思是关键是对 proportion 的理解。你的理解是: 这里的 proportion 是指 掉到地


上(rainfall)的水分子/原来的 raincloud 的水分子。。。而比较的是: 含 oxygen-18 水分子中 掉
到地上(rainfall)的水分子/原来的 raincloud 的水分子 和 ordinary oxygen 水分子中 掉到地
上(rainfall)的水分子/原来的 raincloud 的水分子的这个 proportion
而我原来的理解是: 在 rainfall 中,掉到地上的 oxygen-18 的比例>ordinary oxygen 的比例,
所以我就理解成了 oxygen-18> ordianary oxygen..

这句话也许等价于:
But in rainfall, the proportion of all water molecules containing oxygen-18, which
descends to the earth, is higher than the proportion of all water molecules containing

156
ordinary oxygen, which descends to the earth.

我的理解对吗?

这句话好复杂,如果考试我估计还错.

albet 正如 mindfree 曾经所说逻辑阅读和阅读理解不同,每句话都很重要(一般情况下是


这样)一定要仔细阅读理解,我刚看到你的问题时也是一愣,但仔细看这句话 —— a
higher proportion of all water molecules containing oxygen-18 than of all water molecules
containing ordinary oxygen descends to earth——我们会发现他翻了费费所说的比较对象
错误,句中说是含氧 18 的所有水分子的一个比例和含通常氧原子的所有水分子的一个比例
相 比 前 一 个 比 例 大 , 但 并 不 能 推 出 The clouds rainfall contains more oxygen-18 than
ordinary oxygen , 因 为 可 能 The clouds rainfall contains less oxygen-18 than ordinary
oxygen,但
含 oxygen-18 的水分子中大部分将落到了地面。不知你理解了吗?举个数学例子:10 个水
分子(相当于 all water molecules containing oxygen-18 ),100 个水分子(相当于 all
water molecules containing ordinary oxygen )100〉10,即 rainfall 中 The clouds rainfall
contains less oxygen-18 than ordinary oxygen , 但最 终 10 个水 分子 全降 落到 earth 为
100%,而 100 个水分子中只有 20 各水分子降落到了 earth 为 20%,100%〉20%,好了我
只能讲到这里了;

20. Which one of the following inferences about an individual rain cloud is supported by
the passage?

对 inference 的疑问:根据 OG,infer 指文章里面没有讲,但是根据文章的逻辑关系可以推


出来的答案。答案 A。但 A 时文章里面说的。

(A) Once it is formed Over the Atlantic, the rain cloud contains more ordinary oxygen than
oxygen-18. 原文复述:In a rain cloud, water molecules containing oxygen-18 are rarer
than water molecules containing normal oxygen.

(B) Once it has passed over the Amazon, the rain cloud contains a greater-than-normal
percentage of oxygen-18. 原文不可推出。

(C) The clouds rainfall contains more oxygen-18 than ordinary oxygen. 原文复述:But in
rainfall, a higher proportion of all water molecules containing oxygen-18 than of all water
molecules containing ordinary oxygen descends to earth.

(D) During a rainfall, the cloud must surrender the same percentage of its ordinary
oxygen as of its oxygen-18. 原文不可推出。

(E) During a rainfall, the cloud must surrender more of its oxygen-l8 than it retains. 原文
没讲,但是可以从一下 2 句话中推出 :(1)In a rain cloud, water molecules containing

157
oxygen-18 are rarer than water molecules containing normal oxygen. ( 2 ) But in
rainfall, a higher proportion of all water molecules containing oxygen-18 than of all water
molecules containing ordinary oxygen descends to earth.因为在 rain cloud 中,O-18 比常
态 少 , 而 在 rainfall 中 , O-18 又 比 常 态 多 , 因 此 the cloud must surrender more of its
oxygen-l8 than it retains。

基数不知,只是比例大。以上黄色。

21. It is very difficult to prove today that a painting done two or three hundred years ago,
especially one without a signature or with a questionably authentic signature, is
indubitably the work of this or that particular artist. This fact gives the traditional
attribution of a disputed painting special weight, since that attribution carries the
presumption of historical continuity. Consequently, an art historian arguing for a
deattribution will generally convince other art historians only if he or she can persuasively
argue for a specific reattribution.

Which one of the following, if true, most strongly supports the position that the traditional
attribution of a disputed painting should not have special weight?

(A) Art dealers have always been led by economic self-interest to attribute any unsigned
paintings of merit to recognized masters rather than to obscure artists.

(B) When a painting is originally created, there are invariably at least some eyewitnesses
who see the artist ac work, and thus questions of correct attribution cannot arise at that
time.

(C) There are not always clearly discernible differences between the occasional interior
work produced by a master and the very best work produced by a lesser talent.

(D) Attribution can shape perception inasmuch as certain features that would count as
marks of greatness in a master's work would be counted as signs or inferior artistry if a
work were attributed to a minor artist.

(E) Even though some masters had specialists assist them with certain detail work, such
as depicting lace, the resulting works are properly attributed to the masters alone.

答案:A,我选 C

23. Teachers are effective only when they help their students become independent
learners. Yet not until teachers have the power to make decisions in their own
classrooms can they enable their students to make their own decisions. Students'
capability to make their own decisions is essential to their becoming independent

158
learners. Therefore, if teachers are to be effective, they must have the power to make
decisions in their own classrooms.
According to the argument, each of the following could be true of teachers who have
enabled their students to make their own decisions EXCEPT:
(A) Their students have not become independent learners.
(B) They are not effective teachers.
(C) They are effective teachers.
(D) They have the power to make decisions in their own classrooms.
(E) They do not have the power to make decisions
先把几个概念抽象出来(不要和选项的 ABC 混淆)
A. Teachers are effective B. they help their students become independent learners
C. teachers have the power to make decisions in their own classrooms
D. they enable their students to make their own decisions
E . Students' capability to make their own decisions F.Students' becoming independent
learners
A->B,B 是 A 的必要条件,D->C,C 是 D 的必要条件,F-〉E,E 是 F 的必要条件。而且隐含
D-〉E, D 是 E 的充分条件,B-〉F, B 是 F 的充分条件。
结论 A-〉 C,C 是 A 的必要条件。所以这个结论是有问题的,因为 D 和 E 的关系有问题,A-
>B->F->E , D->C, 而 D 和 E 没有提到 E-〉D 的关系。如果考 weaken,只要说 E-〉D 是错的,
要 support/strengthen,只要说 E->D 是对的.但是这一题没有考这个结论.
题目的逻辑关系复杂,而且推导错误有疑问,但是问题极其简单,只考了 C,D 之间的关系。
这是考阅读的题目。
According to the argument, each of the following could be true of teachers who have
enabled their students to make their own decisions EXCEPT:
问:D 成立,哪一个不可能发生:选项 E :C 不成立。因为 D-〉C,所以选项 E 的内容不可
能发生.所以 E 正确。
请 NN 指正。

1.这样的逻辑关系对吗?

arguement:1.teacher are effective-->when they help their students become


independent learners.--> teachers have the power to make decisions in their own
classrooms

students become independent learners--> Students' capability to make their own


decisions

--> teachers have the power to make decisions in their own classrooms

我觉得你说的逻辑关系是对的,问题就是问上面红色部分成立,下面哪个不可能成立。

159
A,B ,C,D 都是有可能成立的,虽然不一定成立,但 E 是肯定不能成立的,你看:when
they help their students become independent learners.--> teachers have the power to
make decisions in their own classrooms,E 正好说反了。

24. Dr. Ruiz: Dr. Smith has expressed outspoken antismoking views in public. Even
though Dr. Smith is otherwise qualified, clearly she cannot be included on a panel that
examines the danger of secondhand cigarette smoke. As an organizer of the panel, I
want to ensure that the panel examines the issue in an unbiased manner before coming
to any conclusion.
Which one of the following, if true, provides the strongest basis for countering Dr. Ruiz'
argument that Dr. Smith should not be included on the panel?
(A) A panel composed of qualified people with strong but conflicting views on a particular
topic is more likely to reach an unbiased conclusion than a panel composed of people
who have kept their views, if any, private.
(B) People who hold strong views on a particular topic tend to accept new evidence on
that topic only if it supports their views.
(C) A panel that includes one qualified person with publicly known strong views on a
particular topic is more likely to have lively discussions than a panel that includes only
people with no well-defined views on that topic.
(D) People who have expressed strong views in public on a particular topic are better at
raising funds to support their case than are people who have never expressed strong
views in public.
(E) People who have well-defined strong views on a particular topic prior to joining a
panel are often able to impose their views on panel members who are not committed at
the outset to any conclusion.

这道题在 A 和 C 之间犹豫的半天,还是选了 C,答案是 A。

You need to know the reasoning and conclusion of the argument.

No Dr. Smith because the purpose of the panel is to examine the issue in an unbiased
manner.

The purpose is not to have a lively discussion.

A indicates that with people like Dr. Smith, the purpose of the panel can be better served.

C indicates that with Dr. Smith the panel will have lively discussion, whose relationship to
the purpose of the panel is not dicussed anywhere. So C is wrong.

160
Set 8-4

5. The ends of modern centuries have been greeted with both apocalyptic anxieties and
utopian fantasies. It is not surprising that both reactions have consistently proven to be
misplaced. After all, the precise time when a century happens to end cannot have any
special significance, since the Gregorian calendar, though widely used, is only one
among many that people have devised.

Which one of the following, if true, could be substituted for the reason cited above while
still preserving the force of the argument

(A) it is logically impossible for both reactions to be correct at the same time.

(B) What is a utopian fantasy to one group of people may well be for another group of
people, a realization of their worst fears.

(C) The number system based on the number ten, in the absence of which one hundred
years would not have the appearance of being a significant period of time, is by no
means the only one that people have created.

(D) The firm expectation that something extraordinary is about to happen can make
people behave in a manner that makes it less likely that something extraordinary will
happen.

(E) Since a century far exceeds the normal human life span people do not live long
enough to learn from mistakes that they themselves made one hundred years before.

答案是 C

6. People who listen to certain recordings of music are in danger of being unduly
influenced by spoken messages that have been recorded backwards on the records or
tapes.
A consequence of the view above is that
(A) the spoken messages must be louder than the music on the recordings
(B) backwards messages can be added to a recording while still preserving all of the
musical qualities of the recorded performance
(C) the recordings on which such messages appear are chosen for this purpose either
because they are especially popular or because they introduce a trancelike state

161
(D) if such messages must be comprehended to exert influence, then people must be
able to comprehend spoken messages recorded backwards
(E) when people listen to recorded music, they pay full attention to the music as it plays

答案 D

8. In Malsenia sales of classical records are soaring. The buyers responsible for this
boom are quite new to classical music and were drawn to it either by classical scores
from television commercials or by theme tunes introducing major sports events on
television. Audiences at classical concerts, however, are continually shrinking in
Malsenia. It can be concluded from this that the new Malsenian converts to classical
music, having initially experienced this music as recorded music, are most comfortable
with classical music as recorded music and really have no desire to hear live
performances.
The argument assumes which one of the following?
(A) To sell well in Malsenia, a classical record must include at least one piece familiar
from television.
(B) At least some of the new Malsenian buyers of classical records have available to
them the option of attending classical concerts.
(C) The number of classical concerts performed in Malsenia has not decreased in
response to smaller audiences.
(D) The classical records available in Malsenia are, for the most part, not recordings of
actual public concerts.
(E) Classical concerts in Malsenia are not limited to music that is readily available on
recordings.

以原题为准,黄皮书和电子版都有很多错误没有更正。答案绝对是 B

(C) 上面的理解有误;取反后并没有给观众减少提供另外一种解释,相反,in response to


表示由于观众的减少才导致演出数量的降低,因果关系倒置了。同意吗?

9. Brain scans of people exposed to certain neurotoxins reveal brain damage identical to
that found in people suffering from Parkinson's disease. This fact shows not only that
these neurotoxins cause this type of brain damage, but also that the brain damage itself
causes Parkinson's disease. Thus brain scans can be used to determine who is likely to
develop Parkinson's disease.

The argument contains which one of the following reasoning errors?

(A) It fails to establish that other methods that can be used to diagnose Parkinson's
disease are less accurate than brain scans.
(B) It overestimates the importance of early diagnosis in determining appropriate

162
treatments for people suffering from Parkinson's disease.
(C) It mistakes a correlation between the type of brain damage described and
Parkinson's disease for a causal relation between the two.
(D) It assumes that people would want to know as early as possible whether they were
likely to develop Parkinson's disease.
(E) It neglects to specify how the information provided by brain scans could be used
either in treating Parkinson's disease or in monitoring the progression of the disease.
--------------------
答案 B,为何不是 C,请大家指教。

1. 同意 C. midfree

10. Almost all of the books published in the past 150 years were printed on acidic paper.
Unfortunately, every kind of acidic paper gradually destroys itself due to its very acidity.
This process of deterioration can be slowed if the books are stored in a cool, dry
environment. Techniques, which are now being developed, to deacidify books will
probably be applied only to books with historical significance.
If all of the statements in the passage above are true, which one of the following must
also be true!
(A) If a book was published in the past 50 years and is historically insignificant, it will
probably deteriorate completely.
(B) Almost all of the books published in the past 150 years will gradually destroy
themselves.
(C) Almost all of the books that gradually deteriorate are made of acidic paper.
(D) If a book is of historical significance and was printed before 150 years ago, it will be
deacidified.
(E) Books published on acidic paper in 1900 should now all be at about the same state of
deterioration.

答案是: A but I think B is ok as well


因为原文:
his process of deterioration can be slowed if the books are stored in a cool, dry
environment.
说明了毁坏过程可以被减慢,言下之意毁坏过程还是在继续的,只是变慢了。。
所以和 (B) Almost all of the books published in the past 150 years will gradually destroy
themselves 吻合。

15. Goodbody, Inc., is in the process of finding tenants for its newly completed Parrot
Quay commercial development, which will make available hundreds of thousands of

163
square feet of new office space on what was formerly derelict property outside the
financial center of the city. Surprisingly enough, the coming recession though it will hurt
most of the city's businesses, should help Goodbody to find tenants.
Which one of the following, if true, does most to help resolve the apparent paradox?
(A) Businesses forced to economize by the recession will want to take advantage of the
lower rents available outside the financial center.
(E) The recession is likely to have the most severe effect not on service industries, which
require a lot of office space, but on manufacturers.
答案是A,我理解。但E好象也对吧?

Dr. Kim: Electronic fetal monitors, now routinely used in hospital delivery rooms to check
fetal heartbeat, are more intrusive than ordinary stethoscopes and do no more to improve
the chances that a healthy baby will be born. Therefore, the additional cost of electronic
monitoring is unjustified and such monitor-jog should be discontinued.

Dr. Anders: I disagree. Although you and I know that both methods are capable of
providing the same information, electronic monitoring has been well worth the cost.
Doctors now know the warning signs they need to listen for with stethoscopes, but only
because of what was learned from using electronic monitors.

17. As a reply to Dr. Kim's argument, Dr. Anders' response is inadequate because it

(A) misses the point at issue

(B) assumes what it sets out to prove

(C) confuses high cost with high quality

(D) overestimates the importance of technology to modem medicine

(E) overlooks the fact that a procedure can be extensively used without being the best
procedure available

答案:A,我选了 E。

18. Professor Hartley's new book on moral philosophy contains numerous passages that
can be found verbatim in an earlier published work by Hartley's colleague, Professor
Lawrence. Therefore in view of the fact that these passages were unattributed in
Hartley's book. Hartley has been dishonest in not acknowledging the intellectual debt
owed to Lawrence.
Which one of the following is an assumption on which the argument is based?
(A) Hartley could not have written the new book without the passages in question.

164
(B) While writing the new book, Hartley had access to the manuscript of Lawrence s
book.
(C) A book on moral philosophy should contain only material representing the author's
own convictions.
(D) Lawrence did not get the ideas in the passages in Question or did not get their
formulations originally from Hartley.
(E) Hartley considered the passages in question to be the best possible expressions of
the ideas they contain.

Answer: D but I think B


因为如果对B取非,在写书的时候 hartley 根本没看过 Lawrence 写的书,也就不存在抄袭。
也就无所谓:intellectual debt owed to Lawrence.

D 的意思是劳伦斯没有从哈特利得到启发云云,也就是说,劳伦斯的书虽然在前,但他也
有可能是从哈特利学来的,而哈特利才是原作者。D 就是排除了这种原因。

19. People who receive unsolicited advice from someone whose advantage would be
served if that advice is taken should regard the proffered advice with skepticism unless
there is good reason to think that their interests substantially coincide with those of the
advice giver in the circumstance in question.

This principle, if accepted, would justify which one of the following judgments?

(A) After learning by chance that Harriet is Looking for a secure investment for her
retirement savings. Floyd writes to her recommending the R&M Company as an
especially secure investment. But since Floyd is the sole owner of R&M, Harrier should
reject his advice out of hand and invest the savings elsewhere.

(B) While shopping for a refrigerator. Ramon is approached by a salesperson who, on the
basis of her personal experience, warns him against the least expensive model.
However, the salesperson's commission increases with the price of the refrigerator sold,
so Ramon should not reject the least expensive model on the salesperson's advice
alone.

(C) Mario wants to bring pastry to Yvette's party, and when he consults her Yvette
suggests that he bring his favorite chocolate fudge brownies from the local bakery.
However, since Yvette also prefers those brownies to any other pastry, Mario would be
wise to check with others before following her recommendation.

(D) Sara overhears Ron talking about a course he will be teaching and interrupts to
recommend a textbook for his course. However, even though Sara and Ron each wrote a
chapter of' this textbook, since the book's editor is a personal friend of Sara's, Ron
should investigate further before deciding whether it is the best textbook for his course.

165
(E) Mel is buying fish for soup. Joel, who owns the fish market where Mel is a regular and
valued customer, suggests a much less expensive fish than the fish Mel herself prefers.
Since if Mel follows Joel's advice, Joel will make less profit on the sale than he would
have otherwise. Mel should follow his recommendation.

大家看选什么,我选 A
答案是 B

A 错。
原题说:如果某个人的建议对提建议的人有好处,你得怀疑这个建议(但不是拒绝)。除非这
个建议确实对你有好处。

A 说了这个建议对提建议的人确实有好处,所以应该拒绝。。原文只是说怀疑,所以不一样。

C 错在和原文的 unsolicited advice 不一致。。这个 advice 不是 unsolicited advice

D 错在这个建议对 Ron 也有好处,因为他是作者。和原文的 unless 不符

E 错在信息不能从原文中给出。。E 说某建议如果对提建议的人没好处,应该听取。原文无法
推出

好,我们再看 B:
销售人员建议我不要买便宜商品。这个建议当然对销售人员有好处。而对我没好处。那我当然
要怀疑这个建议了(但没有拒绝)--即 B 结论: should not reject the least expensive model
on the salesperson's advice alone.

22.Doctors in Britain have long suspected that patients who wear tinted eyeglasses are
abnormally prone to depression and hypochondria. Psychological tests given there to
hospital patients admitted for physical complaints like heart pain and digestive distress
confirmed such a relationship. Perhaps people whose relationship to the world is
psychologically painful choose such glasses to reduce visual stimulation, which is
perceived as irritating. At any rate, it can be concluded that when such glasses are worn,
it is because the wearer has a tendency to be depressed or hypochondriacal

The argument assumes which one of the following?

(A) Depression is not caused in some cases by an organic condition of the body.
(B) Wearers do not think of the tinted glasses as a means of distancing themselves from
other people.
(C) Depression can have many causes, including actual conditions about which it is
reasonable for anyone to be depressed.
(D) For hypochondriacs wearing tinted glasses, the glasses serve as a visual signal to
others that the wearer's health is delicate.

166
(E) The tinting does not dim light to the eye enough to depress the wearer's mood
substantially.
-----------------------
答案 E,请大家给点解题思路。谢谢。

22)E 如果是因为色彩不调和 depress the wearer's mood。那么就不能是因为 weare 有沮


丧的倾向而戴 glasses 来 depress the wearer's mood,所以结论不能成立。B 只是戴眼镜的
人不是因为.......而拉远与他人的距离,与心情无关。

23. Each of the following, if true, weakens the argument EXCEPT:


(A) Some people wear tinted glasses not because they choose to do so but because a
medical condition of their eyes forces them to do so.
(B) Even a depressed or hypochondriacal person can have valid medical complaints, so
a doctor should perform all the usual objective tests in diagnosing such persons.
(C) The confirmatory tests were not done for places such as western North America
where the usual quality of light differs from that prevailing in Britain.
(D) Fashions with respect to wearing tinted glasses differ in different parts of the world.
(E) At the hospitals where the tests were given, patients who were admitted for conditions
less ambiguous than heart pain or digestive distress did not show the relationship
between tinted glasses and depression or hypochondria.
Answer: B
为什么是B啊,我怎么看其他几个除了A都不象 WEAKEN

25. George: A well-known educator claims that children who are read to when they are
very young are more likely to enjoy reading when they grow up than are children who
were not read to. But this claim is clearly false. My cousin Emory was regularly read to as
a child and as an adult he seldom reads for pleasure, whereas no one read to me and
reading is now my favorite form of relaxation.

Ursula: You and Emory prove nothing in this case. Your experience is enough to refute
the claim that all avid adult readers were read to as children, but what the educator said
about reading to children is not that sort of claim.

Which one of the following describes a flaw in Georges reasoning?


(A) He treats his own experience and the experiences of other members of his own
family as though they have more weight as evidence than do the experiences of other
people.

(B) He does not distinguish between the quality and the quantity of the books that adults
read to Emory when Emory was a child.

(C) He overlooks the well-known fact that not all reading is equally relaxing.

167
(D) He fails to establish that the claim made by this particular educator accurately reflects
the position held by the majority of educators.

(E) He attempts to refute a general claim by reference to nonconforming cases, although


the claim is consistent with the occurrence of such cases.

这道题答案为什么不是 A,而是 E?明明 George 举例的 case 和 general claim 是不一致的


啊?

他试图用一个反例来反驳一个 general claim,虽然这个 general claim 容许这样的反例存在。


Set 9-1

7. In a study of the effect of radiation from nuclear weapons plants on people living in
areas near them, researchers compared death rates in the areas near the plants with
death rates in areas that had no such plants. Finding no difference in these rates, the
researchers concluded that radiation from the nuclear weapons plants poses no health
hazards to people living near them.
Which one of the following, if true, most seriously weakens the researchers’ argument?
(A) Nuclear power plants were not included in the study.
(B) The areas studied had similar death rates before and after the nuclear weapons
plants were built.
(C) Exposure to nuclear radiation can cause many serious diseases that do not
necessarily result in death.
(D) Only a small number of areas have nuclear weapons plants.
(E) The researchers did not study the possible health hazards of radiation on people who
were employed at the nuclear weapons plants if those employees did not live in the study
areas.

Answer: C

1. 你是不是看错问题了? 问题是 weaken. B 是 strengthen. 如果前后的 death rate 相同, 说


明 nuclear weapon plant 并没有对 death rate 造成影响, 所以 B 加强了原文的逻辑.

C 对, 因为原文从 death rate 相同-->pose no threat to health.将 death rate 和健康相等, 忽


略了 C 中的情况.

9. Complaints that milk bottlers take enormous markups on the bottled milk sold to
consumers are most likely to arise when least warranted by the actual spread between
the price that bottlers pay for raw milk and the price at which they sell bottled milk. The
complaints occur when the bottled-milk price rises, yet these price increases most often
merely reflect the rising price of the raw milk that bottlers buy from dairy farmers. When
the raw-milk price is rising, the bottlers’ markups are actually smallest proportionate to
the retail price. When the raw-milk price is falling, however, the markups are greatest.

168
If all of the statements above are true, which one of the following must also be true on the
basis of them?
(A) Consumers pay more for bottled milk when raw-milk prices are falling than when
these prices are rising.
(B) Increases in dairy farmers’ cost of producing milk are generally not passed on to
consumers.
(C) Milk bottlers take substantially greater markups on bottled milk when its price is low
for an extended period than when it is high for an extended period.
(D) Milk bottlers generally do not respond to a decrease in raw-milk prices by
straightaway proportionately lowering the price of the bottled milk they sell.
(E) Consumers tend to complain more about the price they pay for bottled milk when
dairy farmers are earning their smallest profits.

Answer: D. how about C? 我认为 C 好象也对.

D 对.

从原文你应该知道 markup 其实是不变的,或其变化相对 cost 的变化要小, 既然 markup 是在


cost 之上, 如果 cost 越高, 根据我前面所讲 , markup 相对 cost 的比例就越低 . cost 低,
markup 相对就越高. 所以 D 对, 既 bottler 没有成比例地调低价格, 即没有成比例地调低
markup.

形象的举例, 如果最初 cost 是 10, markup 是 2. 如果 cost 上涨至 15, markup 涨到 2.5;如果
cost 下降至 5, markup 下降至 1.5. 在此例中, markup 的变化幅度相对 cost 的变化幅度要小,
所以和原文的现象一致. 当然如果 markup 始终不变也一样.

Questions 10-11

If the public library shared by the adjacent towns of Redville and Glenwood were
relocated from the library’s current, overcrowded building in central Redville to a larger,
available building in central Glenwood, the library would then be within walking distance
of a larger number of library users. That is because there are many more people living in
central Glenwood than in central Redville, and people generally will walk to the library
only if it is located close to their homes.

10 Which one of the following, if true, most strengthens the argument?

(A) The public library was located between Gienwood and Redville before being moved
to its current location in central Redville.

(B) The area covered by central Glenwood is approximately the same size as that
covered by central Redville.

169
(C) The building that is available in Glenwood is smaller than an alternative building that
is available in Redville.

(D) Many of the people who use the public library do not live in either Glenwood or
Redville.

(E) The distance that people currently walk to get to the library is farther than what is
generally considered walking distance.

答案:B, 我不明白 B 为什么是加强了,是无关啊?不过,其它几个选项都不行.

11. Which one of the following, if true, most seriously weakens the argument?

(A) Many more people who currently walk to the library live in central Redville than in
central Glenwood.

(B) The number of people living in central Glenwood who would use the library if it were
located there is smaller than the number of people living in central Redville who currently
use the library.

(C) The number of people using the public library would continue to increase steadily if
the library were moved to Glenwood.

(D) Most of the people who currently either drive to the library or take public
transportation to reach it would continue to do so if the library were moved to central
Glenwood.

(E) Most of the people who currently walk to the library would remain library users if the
library were relocated to central Glenwood.

答案:B,请问 A,B 有什么区别吗?都行啊.都是削弱.

move 的目的是方便更多的人。

1 面积相等,人口多,密度大

2 G 使用图书馆的人少,R 多。

13. Five years ago, during the first North American outbreak of the cattle disease CXC,
the death rate from the disease was 5 percent of all reported cases, whereas today the
corresponding figure is over 18 percent. It is clear, therefore, that during these past 5
years, CXC has increased in virulence.

170
Which one of the following, if true, most substantially weakens the argument?

(A) Many recent cattle deaths that have actually been caused by CXC have been
mistakenly attributed to another disease that mimics the symptoms of CXC.

(B) During the first North American outbreak of the disease, many of the deaths reported
to have been caused by CXC were actually due to other causes.

(C) An inoculation program against CXC was recently begun after controlled studies
snowed inoculation to be 70 percent effective in preventing serious cases of the illness.

(D) Since the first outbreak, farmers have learned to treat mild cases of CXC and no
longer report them to veterinarians or authorities.

(E) Cattle that have contracted and survived CXC rarely contract the disease a second
time.

我的问题答案是 d,它是如何削弱的???

premises: 18%death among REPORTED CXC cases, in contrast to 5% during the first
outbreak; conclusion:CXC 毒性增强
D weaken. 因为很多农民自己料理了,根本就没 report --> 统计的基数小了 -->18%不是因为
毒性大.

LSAT9/1/15
Economist: Some policymakers believe that our country’s continued economic growth
requires a higher level of personal savings than we currently have. A recent legislative
proposal would allow individuals to set up savings accounts in which interest earned
would be exempt from taxes until money is withdrawn from the account. Backers of this
proposal claim that its implementation would increase the amount of money available for
banks to loan at a relatively small cost to the government in lost tax revenues. Yet, when
similar tax-incentive programs were tried in the past, virtually all of the money invested
through them was diverted from other personal savings, and the overall level of personal
savings was unchanged.
15. The author criticizes the proposed tax-incentive program by
(A) challenging a premise on which the proposal is based
(B) pointing out a disagreement among policymakers
(C) demonstrating that the proposal’s implementation is not feasible
(D) questioning the judgment of the proposal’s backers by citing past cases in which they
had advocated programs that have proved ineffective
(E) disputing the assumption that a program to encourage personal savings is needed

171
答案是 A,D 为什么不对呢?

The first half is D is right. From "...in which they advocated..." it is wrong. In the question
it says" similiar programs". We do not know if the backers "advocate" those programs or
not.

A is right. The premise of the argument is that "...its implementation would increase the
amount of money available for banks to loan ...". The author then argues that the
increase in fund in savings account is from other personal savings, so that the overall
personal savings will not change. So the amoun tof money available will not change.

Pay particular attention to look-like choices such as D. It is too much like an answer and
you need to pay extra attention to it.

我认为 D 错在最后一部分,画蛇添足。
(D) questioning the judgment of the proposal’s backers by citing past cases in which they
had advocated programs (that have proved ineffective )
there is no evdience to prove that such programs are ineffective. In fact, tax-incentive
program indeed keep money in account for a longer time and indirectly benefit loans, but
it just cannot increase the total amount of money.
所以,d 选项是错误的。

Saunders: Everyone at last week’s neighborhood association meeting agreed that the
row of abandoned and vandalized houses on Cariton Street posed a threat to the safety
of our neighborhood. Moreover, no one now disputes that getting the houses torn down
eliminated that threat. Some people tried to argue that it was unnecessary to demolish
what they claimed were basically sound buildings, since the city had established a fund
to help people in need of housing buy and rehabilitate such buildings. The overwhelming
success of the demolition strategy, however, proves that the majority, who favored
demolition, were right and that those who claimed that the problem could and should be
solved by rehabilitating the houses were wrong.

20. Which one of the following principles, if established would determine that demolishing
the houses was the right decision or instead would determine that the proposal
advocated by the opponents of demolition should have been adopted?
(A) When what to do about an abandoned neighborhood building is in dispute, the course
of action that would result in the most housing for people who need it should be the one
adopted unless the building is believed to pose a threat to neighborhood safety.
(B) When there are two proposals for solving a neighborhood problem, and only one of
them would preclude the possibility of trying the other approach if the first proves

172
unsatisfactory, then the approach that does not foreclose the other possibility should be
the one adopted.
(C) If one of two proposals for renovating vacant neighborhood buildings requires
government funding whereas the second does not, the second proposal should be the
one adopted unless the necessary government funds have already been secured.
(D) No pain for eliminating a neighborhood problem that requires demolishing basically
sound houses should be carried out until all other possible alternatives have been
thoroughly investigated.
(E) No proposal for dealing with a threat to a neighborhood’s safety should be adopted
merely because a majority of the residents of that neighborhood prefer that proposal to a
particular counterproposal.

answer: B 怎么得出来的?我对原文的辨证过程有里模糊。。

我对于题目 3 的理解:
题目问哪个原则可以确定:demolish 是对的或者决定 rehabilitate 是对的。

(A) 原则 When what to do about an abandoned neighborhood building is in dispute, the


course of action that would result in the most housing for people who need it should be
the one adopted unless the building is believed to pose a threat to neighborhood safety.
根据该原则,
房子对 public 构成了威胁只是 the course action(rehabiltate) 不可接受的必要条件。但不是
充分条件,所以无法确定 rehabiltate 是否 ok

(B) When there are two proposals for solving a neighborhood problem, and only one of
them would preclude the possibility of trying the other approach if the first proves
unsatisfactory, then the approach that does not foreclose the other possibility should be
the one adopted.
根据原则,保持房子是 unsatifactory 的,可以确定 demolish 是可接受的
选B

(C) If one of two proposals for renovating vacant neighborhood buildings requires
government funding whereas the second does not, the second proposal should be the
one adopted unless the necessary government funds have already been secured.
根据 C 原则,政府提供资金只是 不需要政府资金的方案 (demolish)的不可行性的必要条
件。。
同 A 一样,不是充分条件。。所以无法确定何钟方案优秀。
(D) No pain for eliminating a neighborhood problem that requires demolishing basically
sound houses should be carried out until all other possible alternatives have been
thoroughly investigated.
all other possible alternatives investigated 是 demolish 成立的必要条件...
所以和 A 一样。。无法确定
(E) No proposal for dealing with a threat to a neighborhood’s safety should be adopted
merely because a majority of the residents of that neighborhood prefer that proposal to a

173
particular counterproposal.
E 的意思是仅仅因为大多数人喜欢某个意见并不是某个涉及安全的方案(demolish)成立的充
分条件。所以也无法判断...

所以,本题考的是充分条件和必要条件。。。ACDE 都是必要条件
Midfree- 2. 我认为答案是 A. 其它选项如果成立, 应选择的方案就应该是 rehabilitate, 和问
题不符.

B. 如果一个方案在试过不成功后, 无法再试第二种方案,则要先试第二种方案.在此题中, 如
果 demolish, 房子就没了, 自然不能再试 rehabilitate.所以是支持 rehabilitate, 错.
C. 如果要 government funding 就不用, 除非 funding 已经有了, 也是支持 rehabilitate, 错.
D. 反对 demolishing, 错.
E. 反对 demolishing, 错.

21. Saunders’ reasoning is flawed because it

(A) relies on fear rather than on argument to persuade the neighborhood association to
reject the policy advocated by Saunders’ opponents

(B) fails to establish that there is anyone who could qualify for city funds who would be
interested in buying and rehabilitating the houses

(C) mistakenly equates an absence of vocal public dissent with the presence of universal
public support

(D) offers no evidence that the policy advocated by Saunders’ opponents would not have
succeeded if it had been given the chance

(E) does not specify the precise nature of the threat to neighborhood safety supposedly
posed by the vandalized houses

right answer is d), why c) is wrong? the argument said 'no one now disputes that getting
the houses torn down eliminated that threat'-----treat this as 'everyone agrees with it' , just
what c) says.
Anyone can tell me why C) is wrong? Thx!

共有两种观点:demolish 房子已防贼,not demolish。


而“no one now disputes that getting the houses torn down eliminated that threat.” 这里的“ getting the houses torn
down eliminated that threat.” ”只是一个 subconclusion。所以不能说反对意见是 absenct

认为还是 D, 没有错. 原文在最后陈词之前提到了两种不同的观点, 要拆房子和不拆,分给别人住. C 却说 an


absence of vocal public dissent , 即"没有/缺乏反对者的意见. 有点牵强. 另外, 两种观点陈述完了, 主要看作者

174
S 这个人是怎样支持自己观点的(这正是题目的问题), S 说: 很多拆房子的成功证明了"要拆房子而不要分给
别人住". 如果是我, 我也会对这个结论提出反驳: "你为什么不举出不拆房子而成功解决 thief 问题的例子?"

23. Mayor Smith, one of our few government officials with a record of outspoken,
informed, and consistent opposition to nuclear power plant construction projects, has
now declared herself in favor of building the nuclear power plant at Littletown. If someone
with her past antinuclear record now favors building this power plant, then there is good
reason to believe that it will be safe and therefore should be built.

The argument is vulnerable to criticism on which one of the following grounds?

(A) It overlooks the possibility that not all those who fail to speak out on issues of nuclear
power are necessarily opposed to it.

(B) It assumes without warrant that the qualities enabling a person to be elected to public
office confer on that person a grasp of the scientific principles on which technical
decisions are based.

(C) It fails to establish that a consistent and outspoken opposition is necessarily an


informed opposition.

(D) It leads to the further but unacceptable conclusion that any project favored by Mayor
Smith should be sanctioned simply on the basis of her having spoken out in favor of it.

(E) It gives no indication of either the basis of Mayor Smith’s former opposition to nuclear
power plant construction or the reasons for her support for the Littletown project.

E. 原文没有说明 opposition 和 support 的原因, 推不出和 safe 与否的关系.

175
LSAT-9-4

8. The director of a secondary school where many students were having severe
academic problems impaneled a committee to study the matter. The committee reported
that these students were having academic problems because they spent large amounts
of time on school sports and too little time studying. The director then prohibited all
students who were having academic problems from taking part in sports in which they
were active. He stated that this would ensure that such students would do well
academically.
The reasoning on which the director bases his statement is not sound because he fails to
establish that
(A) some students who spend time on sports do not have academic problems
(B) all students who do well academically do so because of time saved by not
participating in sports
(C) at least some of the time the students will save by not participating in sports will be
spent on solving their academic problems
(D) no students who do well academically spend time on sports
(E) the quality of the school’s sports program would not suffer as a result of the ban

答案:D
B 为什么不对?

This is typical ETS CR. B is not the concern in the argument and therefore irrelevant. The
conclusion is that these people need to spend more time on study to do well, but they
spend most time on sports instead of study. The problem with the order is that it does not
necessarily guarantee enough time for study at all. So C is not.

B includes all students, which is beyond the scope here. If some students who do well do
not save time by not doing sports, it does not create any conflict with the argument,

176
because the concern here is only this specific group of students who lose study time by
doing sports.

Put B into the argument and think again..

11.A distemper virus has caused two-thirds of the seal population in the North Sea to die
since May 1988. The explanation for the deaths cannot rest here, however. There must
be a reason the normally latent virus could prevail so suddenly: clearly the severe
pollution of the North Sea waters must have weakened the immune system of the seals
so that they could no longer withstand the virus.
11. Which one of the following, if true, most strongly supports the explanation given in the
argument?
(A) At various times during the last ten years, several species of shellfish and seabirds in
the North Sea have experienced unprecedented steep drops in population.
(B) By reducing pollution at its source, Northern Europe and Scandinavia have been
taking the lead in preventing pollution from reaching the waters of the North Sea.
(C) For many years, fish for human consumption have been taken from the waters of the
North Sea.
(D) There are two species of seal found throughout the North Sea area, the common seal
and the gray seal.
(E) The distemper caused by the virus was a disease that was new to the population of
North Sea seals in May 1988, and so the seals’ immune systems were unprepared to
counter it.

1. 这道题目答案确实不明显, 可能因为是加强题目,只要有一点加强作用就可. 用排除法可以


排除 CDE
C. 毫无关系. 原文没有提到饮用水和污染或病毒的关系
D. 无关,原文没有区别 seal
E. 相反, 是 weaken

B 错在没有指出控制污染的效果如何,不能说明现在是否还污染, 对原文起不到作用.

A 对是因为最初的观点是认为 seal 数量的下降是因为 seal 的一种病毒,隐含意思是因为


seal 自身的因素(seal 中的病毒), 现在认为原因是污染,实际上扩大了范围, 说不是 seal 本身
病毒的原因. A 证明了是其它非 seal 特质的原因在起作用,造成了数种动物的数量下降.

12. It is clear that none of the volleyball players at yesterday’s office beach party came to
work today since everyone who played volleyball at that party got badly sunburned and
no one at work today is even slightly sunburned.
Which one of the following exhibits a pattern of reasoning that most closely parallels that
in the argument above?

177
(A) Since everyone employed by TRF who was given the opportunity to purchase dental
insurance did so and everyone who purchased dental insurance saw a dentist, it is clear
that no one who failed to see a dentist is employed by TRF.
(B) Since no one who was promoted during the past year failed to attend the awards
banquet, evidently none of the office managers attended the banquet this year since they
were all denied promotion.
(C) Since the Donnely report was not finished on time, no one in John’s group could have
been assigned to contribute to that report since everyone in John’s group has a
reputation for getting assignments in on time.
(D) Everyone with an office on the second floor works directly for the president and, as a
result, no one with a second floor office will take a July vacation because no one who
works for the president will be able to take time off during July.
(E) Since all of the people who are now on the MXM Corporation payroll have been
employed in the
same job for the past five years, it is clear that no one who frequently changes jobs is
likely to be hired by MXM.

肯请大家指点,D 和原文平行在哪里?看了半天,愣是没看出来。

none of the volleyball players at yesterday’s office beach party came to work today since
everyone who played volleyball at that party got badly sunburned and no one
把 D 改写下,再看看吧,只不过稍微在句子顺序前后的小把戏
no one with a second floor office will take a july vacation, since everyone with an office
on the second floor
works directly for the president and, as a result, no one who works for the president will
be able to take time off
during july.

vb player got sunburned


nobody at work today is sunburned
-------------------------------------------------
none of the vb player comes to work today

everyone with an office on the second floorworks directly for the president
no one who works for the president will be able to take time offduring july.
--------------------------------------------------------------------------------------------------------
no one with a second floor office will take a july vacation
两者都是严密的三断论,就 para 了吧
c 选项容易混淆,符合三断论的形式,但不能推出结论,因为有 reputation 不代表一定完成
assignment,证据不支持结论

17. Certain items—those with that hard-to-define quality called exclusivity—have the odd

178
property, when they become available for sale, of selling rapidly even though they are
extremely expensive. In fact, trying to sell such an item fast by asking too low a price is a
serious error, since it calls into question the very thing---exclusivity---that is supposed to
be the item’s chief appeal. Therefore, given that a price that will prove to be right is
virtually impossible for the seller to gauge in advance, the seller should make sure that
any error in the initial asking price is in the direction of setting the price too high.
The argument recommends a certain pricing strategy on the grounds that
(A) this strategy lacks a counterproductive feature of the rejected alternative
(B) this strategy has all of advantages of the rejected alternative, but fewer of its
disadvantages
(C) experience has proven this strategy to be superior, even though the reasons for this
superiority elude analysis
(D) this strategy does not rely on prospective buyers estimates of value
(E) the error associated with this strategy, unlike the error associated with the rejected
alternative, is likely to go unnoticed
依次选择了 C、D,但答案是 A。
提问的这种问法是问假设,还是问一个削弱?
提问的翻译为:文章在。。。的基础上推荐了一种价格策略。可以这样理解嘛?

Discussed before and I explained before. In short, in order to sell something fast, if you
start by selling high, at least you create excusivity; If you start by selling low, customers
will question the chief appeal and would not buy.
我以前讲过此题. 重要是理解原文和选项的意思. 有些东西越贵卖得越快(exclusivity), 如果
为了卖得快而定价低了, 反而会犯错误,导致人们怀疑它的 exclusivity. 因为事先不太可能知
道合理的价格,所以最初定价时要定得贵.

A 的 意 思 解 释 了 这 个 原 则 . 因 为 定 价 低 ( 既 rejected alternative) 会 导 致 人 们 的 怀 疑
(conterproductive).

原文说的是
我对题干的理解是大意是某些东东很特别,你得要巨高价。因为你价要低了,顾客反而怀疑
这东东不特别了
结论是如果定价有错误的话就一定是定的太高了

实际上这个结论是没有任何道理的,因为这个错误的价格也可能是定的太低了啊,所以这
就正是 a 中所说的 lacks a counterproductive feature of the rejected alternative

请教 LSAT-9-4-21
So-called environmentalists have argued that the proposed Golden Lake Development
would interfere with bird-migration patterns. However, the fact that these same people
have raised environmental objections to virtually every development proposal brought
before the council in recent years indicates that their expressed concern for bird-
migration patterns is nothing but a mask for their antidevelopment, antiprogress agenda.

179
Their claim, therefore, should be dismissed without further consideration.

For the claim that the concern expressed by the so-called environmentalists is not their
real concern to be properly drawn on the basis of the evidence cited, which one of the
following must be assumed?
(A) Not every development proposal opposed in recent years by these so-called
environmentalists was opposed because they believed it to pose a threat to the
environment.
(B) People whose real agenda is to block development wherever it is proposed always try
to disguise their true motives.
(C) Anyone who opposes unrestricted development is an opponent of progress.
(D) The council has no reason to object to the proposed Golden Lake Development other
than concern about the development’s effect on bird-migration patterns.
(E) When people say that they oppose a development project solely on environmental
grounds, their real concern almost always lies elsewhere.

答案是 A。
话说为了证明这些所谓环境保护者实际上是反进步反发展的落后分子,前提是 (A 中说):
并不是每一个被他们反对的都被反对了。感觉不够强劲。如果说所有他们反对的最后都实施
了,还可以吧。B 和 E 呢?大家觉得?
多谢指教!

这段话的推理是, 因为这帮家伙总是拿对环境有害为理由,反对几乎所有的提案,所以他
们真的的目的不是环境问题,而是反对发展。所以不要理他们。

假如对环境有害的确是真的理由的话,那下面的都不成立了。也就是对 A 取非的观点。B 和
E 都不相关。

24. Most disposable plastic containers are now labeled with a code number (from 1 to 9)
indicating the type or quality of the plastic. Plastics with the lowest code numbers are the
easiest for recycling plants to recycle and are thus the most likely to be recycles after use
rather than dumped in landfills. Plastics labeled with the highest numbers are only rarely
recycled.
Consumers can make a significant long-term reduction in the amount of waste that goes
unrecycled, therefore, by refusing to purchase those products packaged in plastic
containers labeled with the highest code numbers.
Which one of the following, if true, most seriously undermines the conclusion above?
(A) The cost of collecting, sorting, and recycling discarded plastics is currently higher
than the cost of manufacturing new plastics from virgin materials.
(B) Many consumers are unaware of the codes that are stamped on the plastic
containers.
(C) A plastic container almost always has a higher code number after it is recycled than it
had before recycling because the recycling process causes a degradation of the quality

180
of the plastic.
(D) Products packaged in plastics with the lowest code numbers are often more
expensive than those packaged in the higher-numbered plastics.
(E) Communities that collect all discarded plastic containers for potential recycling later
dump in landfills plastics with higher-numbered codes only when it is clear that no
recycler will take them.
答案是 C:可以理解。
B:为什么不对?如果人们无法意识到号码的作用,也就不会去选择。

The conclusion is that consumers can make the contribution by doing that. It is equal to
"If consumer ..., they can contribute..." Based on the reasoning in the passage, the
conclusion is correct. Give you another example: Rats carried germs that cause sickness
in human being. So if all the rats are eliminated from the earth, human being will suffer
much less than now. There is nothing wrong in the reasoning and you cannot counter the
argument by saying "it is impossible to kills all the rats on earth"
LSAT-10-2

3. Tall children can generally reach high shelves easily. Short children can generally
reach high shelves only with difficulty. It is known that short children are more likely than
are tall children to become short adults. Therefore, if short children are taught to reach
high shelves easily, the proportion of them who become short adults will decrease.
A reasoning error in the argument is that the argument
A.attributes a characteristic of an individual member of a group to the group as a whole
B. presupposes that which is to be proved
C.refutes a generalization by mean of an exceptional case
D.assumes a causal relationship where only a correlation has be indicated
E.take lack of evidence for the existence of a state of affairs as evidence that there can
be no such state of affairs
answer: d, I choose e

LAST10-2
3. Balance is particularly important when reporting the background of civil wars and
conflicts. Facts must not be deliberately manipulated to show one party in a favorable
light, and the views of each side should be fairly represented. This concept of balance,
however, does not justify concealing or glossing over basic injustices in an effort to be
even-handed. If all the media were to adopt such a perverse interpretation of balanced
reporting, the public would be given a picture of a world where each party in every
conflict had an equal measure of justice on its side, contrary to our experience of life and,
indeed, our common sense.
Which one of the following best expresses the main point of the argument?
(A) Balanced reporting presents the public with a picture of the world in which all sides to
a conflict have equal justification.

181
(B) Balanced reporting requires impartially revealing injustices where they occur no less
than fairly presenting the views of each party in a conflict.
(C) Our experience of life shows that there are indeed cases in which conflicts arise
because of an injustice, with one party clearly in the wrong.
(D) Common sense tells us that balance is especially needed when reporting the
background of civil wars and conflicts.
(E) Balanced reporting is an ideal that cannot be realized, because judgments of balance
are necessarily subjective.
答案 B,E 为何不对,B 为何对。

B 的意思和原文很好的吻合,而 E 你是无法推出 that cannot be realized 的

4. Data form satellite photographs of the tropical rain forest in Melonia show that last year
the deforestation rate of this environmentally sensitive zone was significantly lower than
in previous years. The Melonian government, which spent millions of dollars last year to
enforce laws against burning and cutting of the forest, is claiming that the satellite data
indicate that its increased efforts to halt the destruction are proving effective.

Which one of the following, if true, most seriously undermines the government’s claim?

(A) Landowner opposition to the government’s antideforestation efforts grew more violent
last year in response to the increased enforcement.

(B) Rainfall during the usually dry 6-month annual burning season was abnormally heavy
last year.

(C) Government agents had to issue fines totaling over 59 million to 3,500violators of
burning-and-cutting regulations.

(D) The inaccessibility of much of the rain forest has made it impossible to confirm the
satellite data by direct observation from the field.

(E) Much of the money that was designated last year for forest preservation has been
spent on research and not on enforcement.
我选 d,答案是 b,我觉得 b 是无关的。

B 原文中有"environmentally sensitive",足以说明问题. D 只是不能 confirm,不能 refute the


evidence

5. Advertisement: Northwoods Maple Syrup, make the old-fashioned way, is simply tops
for taste. And here is the proof: in a recent market survey, 7 out of every 10 shoppers
who expressed a preference said that Northwoods was the only maple syrup for them, no

182
ifs, ands, or buts.

Of the following, which one is the strongest reason why the advertisement is potentially
misleading?

(A) The proportion of shoppers expressing no preference might have been very small.

(B) Other brands of maple syrup might also be made the old-fashioned way.

(C) No market survey covers more than a sizable minority of the total population of
consumers.

(D) The preference for the Northwoods brand might be based on such a factor as an
exceptionally low price.

(E) Shoppers who bu7y syrup might buy only maple syrup.
答案是 d,我选的是 c,请解释!

你 要找 到原 文的 逻辑 推理 过程 : survey result-->tops taste. D survey result-->low price,


weaken

7. Waste management companies, which collect waste for disposal in landfills and
incineration plants, report that disposable plastics make up an ever-increasing
percentage of the waste they handle. It is clear that attempts to decrease the amount of
plastic that people throw away in the garbage are failing.

Which one of the following, if true, most seriously weakens the argument?

(A) Because plastics create harmful pollutants when burned, an increasing percentage of
the plastics handled by waste management companies are being disposed of in landfills.

(B) Although many plastics are recyclable, most of the plastics disposed of by waste
management companies are not.

(C) People are more likely to save and reuse plastic containers than containers made of
heavier materials like glass or metal.

(D) An increasing proportion of the paper, glass, and metal cans that waste management
companies used to handle is now being recycled.

(E) While the percentage of products using plastic packaging is increasing, the total
amount of plastic being manufactured has remained unchanged.

183
这个题目的答案选 D 让我有些想不通。

这实际上是个数学问题,D 从反面暗示出:因为别的 waste 减少了从而导致 plastics 的所


占 的 比 重 增 加 了 , 可 能 实 际 在 数 量 上 没 有 变 , 甚 至 下 降 了 , 因 此 undermine the
conclusion.

The number of aircraft collisions on the ground is increasing because of the substantial
increase in the number of flights operated by the airlines. Many of the fatalities that occur
in such collisions are caused not by the collision itself, but by an inherent flaw in the
cabin design of most aircraft, in which seats, by restricting access to emergency exits,
impede escape. Therefore, to reduce the total number of fatalities that result annually
from such collisions, the airlines should be required to remove all seats that restrict
access to emergency exits.
10. Which one of the following proposals, if implemented together with the proposal
made in the passage, would improve the prospects for achieving the stated objective of
reducing fatalities?

(A) The airlines should be required, when buying new planes, to buy only planes with
unrestricted access to emergency exits.

(B) The airlines should not be permitted to increase further the number of flights in order
to offset the decrease in the number of seats on each aircraft.

(C) Airport authorities should be required to streamline their passenger check-in


procedures to accommodate the increased number of passengers served by the airlines.

(D) Airport authorities should be required to refine security precautions by making them
less conspicuous without making them less effective.

(E) The airlines should not be allowed to increase the ticket price for each passenger to
offset the decrease in the number of seats on each aircraft.

这个题目我举棋不定最后选了 d,答案是 b,请指正。

你是不是打错了, 我没有 LSAT 的题目不能确认.B 中应该是 flights 而不是 lights.你选 E 没有


道理,票价如何和 fatality 有关系? 造成事故的原因有二: increase in number of flights and
seat blocking exits. 解决了 seat 的问题,同时又避免第一个问题加重.所以 B. 迷惑选项应为
A, 但是 A 只是解决了原文已解决的问题,不如 B.

11. Recently discovered fossil evidence casts doubt on the evolutionary theory that

184
dinosaurs are more closely related to reptiles than to other classes of animals. Fossils
show that some dinosaurs had hollow bones-a feature found today only in warm-blooded
creatures, such as birds, that have a high metabolic rate. Dinosaurs had well-developed
senses of sight and hearing, which is not true of present-day cold-blooded creatures like
reptiles. The highly arched mouth roof of some dinosaurs would have permitted them to
breathe while eating, as fast-breathing animals, such as birds, need to do. Today, all fast-
breathing animals are warm-blooded. Finally, fossils reveal that many dinosaurs had a
pattern of growth typical of warm-blooded animals.

The argument in the passage proceeds by

(A) attempting to justify one position by demonstrating that an opposing position is based
on erroneous information

(B) establishing a general principle that it then uses to draw a conclusion about a
particular case

(C) dismissing a claim made about the present on the basis of historical evidence

(D) assuming that if all members of a category have a certain property then all things with
that property belong to the category

(E) presenting evidence that a past phenomenon is more similar to one rather than the
other of two present-day phenomena

我选 d,答案是 e,请指教?

因为题目就是在拿 dinosaurs of past 和 bird and reptile of today 作比较,但更倾向于 bird,


所以答案就是 E 了.

12. Purebred dogs are prone to genetically determined abnormalities. Although such
abnormalities often can be corrected by surgery, the cost can reach several thousand
dollars. Since nonpurebred dogs rarely suffer from genetically determined abnormalities,
potential dog owners who want to reduce the risk of incurring costly medical bills for their
pets would be well advised to choose nonpurebred dogs.

Which one of the following, if true, most seriously weakens the argument?

(A) Most genetically determined abnormalities in dogs do not seriously affect a dog’s
general well-being.

(B) All dogs, whether purebred or nonpurebred, are subject to the same common
nongenetically determined diseases.

185
(C) Purebred dogs tend to have shorter natural life spans than do nonpurebred dogs.

(D) The purchase price of nonpurebred dogs tends to be lower than the purchase price of
purebred dogs.

(E) A dog that does not have genetically determined abnormalities may nevertheless
have offspring with such abnormalities.

这题我怎么觉得答案应该是 E 呢?LSAT 答案是 A

12 题,,A 中,既然这种遗传病并不影响狗狗们的健康,狗狗的主人也无须给它作手术(即
不必花钱),这种清况下纯种狗还是比非纯种狗有优势的。

D E 加强原题结论,,,B C 无关,

13. Criticism that the press panders to public sentiment neglects to consider that the
press is a profit-making institution. Like other private enterprises, it has to make money to
survive. If press were not profit-making, who would support it? The only alternative is
subsidy and, with it, outside control. It is easy to get subsidies for propaganda, but no
one will subsidize honest journalism.

It can be properly inferred from the passage that if the pres is

(A) not subsidize, it is in no danger of outside control

(B) not subsidized, it will not produce propaganda

(C) not to be subsidized, it cannot be a profit-making institution

(D) to produce honest journalism, it must be profit-making institution

(E) to make a profit, it must produce honest journalism

答案:D

报社要生存...这是前提...生存有两个办法: 1. make profit 2 if not making profit, you should


get subsidize, SUBSIDIZE 的必要条件就是 LYING...
所以要生存,要么: 1. MAKE PROFIT 2. 要么 NOT BE HONEST
D 正是这个意思

Criticism that the press panders to public sentiment neglects to consider that the press is

186
a profit-making institution. Like other private enterprises, it has to make money to survive.
If press were not profit-making, who would support it? The only alternative is subsidy
and, with it, outside control. It is easy to get subsidies for propaganda, but no one will
subsidize honest journalism.
It can be properly inferred from the passage that if the pres is
(A)not subsidize, it is in no danger of outside control
(B)not subsidized, it will not produce propaganda
(C)not to be subsidized, it cannot be a profit-making institution
(D)to produce honest journalism, it must be profit-making institution
(E)to make a profit, it must produce honest journalism
没有搞懂这提的推理过程?
----------------------------------------------------------------------------
这里是一个两层推导:
1、If not profit-making => Have to be subsidized (to survie)
2、If honest journalism => No way to be subsidized

第二个命题与第一个推论的逆反命题相结合即推出答案。(D)

16. Some people take their moral cues from governmental codes of law; for them, it is
inconceivable that something that is legally permissible could be immoral.
Those whose view is described above hold inconsistent beliefs if they also believe that
(A) law does not cover all circumstances in which one person morally wrongs another
(B) a legally impermissible action is never morally excusable
(C) governmental officials sometimes behave illegally
(D) the moral consensus of a society is expressed in its laws
(E) some governmental regulations are so detailed that they are burdensome to the
economy
答案 A
code of law->moral cues
非(legally permissible)->非(moral)
所以我选 D

17. Certain instruments used in veterinary surgery can be made either of stainless steel
of nylon. In a study of such instruments, 10 complete sterilizations of a set of nylon
instruments required 3.4 times the amount of energy used to manufacture that set of
instruments, whereas 50 complete sterilizations of a set of stainless steel instruments
required 2.1 time the amount of energy required to manufacture that set of instruments.

If the statements above are true, each of the following could be true EXCEPT:

(A) The 50 complete sterilizations of nylon instruments used more energy than did the 50

187
complete sterilizations of the stainless steel instruments.

(B) More energy was required for each complete sterilization of the nylon instruments
than was required to manufacture the nylon instruments.

(C) More nylon instruments than stainless steel instruments were sterilized in the study.

(D) More energy was used to produce the stainless steel instruments than was used to
produce the nylon instruments.

(E) The total cost of 50 complete sterilizations of the stainless steel instruments was
greater than the cost of manufacturing the stainless steel instruments.

答案:B

18. A local group had planned a parade for tomorrow, but city hall has not yet acted on its
application for a permit. The group had applied for the permit well in advance, had made
sure their application satisfied all the requirements, and was clearly entitled to a permit.
Although the law prohibits parades without a permit, the group plans to proceed with its
parade. The group’s leader defended its decision by appealing to the principle that
citizens need not refrain from actions that fail to comply with the law if they have made a
good-faith effort to comply but are prevented from doing so by government inaction.
Which one of the following actions would be justified by the principle to which the leader
of the group appealed in defending the decision to proceed?
(A) A chemical-processing company commissioned an environmental impact report on its
plant. The report described foul odors emanating from the plant but found no hazardous
wastes being produced. Consequently, the plant did not alter its processing practices.
(B) A city resident applied for rezoning of her property so that she would build a bowling
alley in a residential community. She based her application on the need for recreational
facilities in the community. Her application was turned down by the zoning board, so she
decided to forgo construction.
(C) The law requires that no car be operated without a certain amount of insurance
coverage. But since the authorities have been unable to design an effective procedure for
prosecuting owners of cars that are driven without insurance, many car owners are
allowing their insurance to lapse.
(D) a real-estate developer obtained a permit to demolish a historic apartment building
that had not yet been declared a governmentally protected historic landmark. Despite the
protests of citizens’ groups, the developer then demolished the building.
(E) A physician who had been trained in one country applied for a license to practice
medicine in another country. Although he knew he met all the qualifications for this
license, he had not yet received it one year after he applied for it. He began to practice
medicine without the license in the second country despite the law’s requirement for a
license.

188
答案 E,C 为何不对。

A university should not be entitled to patent the inventions of its faculty members.
Universities, as guarantors of intellectual freedom, should encourage the free flow of
ideas and the general dissemination of knowledge. Yet a university that retains the right
to patent the inventions of its faculty members has a motive to suppress information
about a potentially valuable discovery until the patent for it has been secured. Clearly,
suppressing information concerning such discoveries is incompatible with the university’s
obligation to promote the free flow of ideas.

19. Which one of the following is an assumption that the argument makes?

(A) Universities are the only institutions that have an obligation to guarantee intellectual
freedom.

(B) Most inventions by university faculty members would be profitable if patented.

(C) Publication of reports on research is the only practical way to disseminate information
concerning new discoveries.

(D) Universities that have a motive to suppress information concerning discoveries by


their faculty members will occasionally act on that motive.

(E) If the inventions of a university faculty member are not patented by that university,
then they will be patented by the faculty member instead.

答案:D

本题的结论句在第一句,而不是 clearly,后的 sub-conlusion.


结论:学校不应拥有申请专利的权利。
证据:
(1)学校应该保证信息和知识的自由传播。
(2)但是拥有申请专利权的学校 have motive 去压制关于这些专利发明的知识的传播。

Sub-conclusion: “压制关于这些专利发明的知识的传播” 与( 1)相矛盾 --->全文结论:


学校不应拥有申请专利的权利。

这里,sub-conclusion 中的“压制关于这些专利发明的知识的传播” 与( 2)have motive


去“压制关于这些专利发明的知识的传播”之间有一个 gap. 有动机压制≠压制。

D 选项在“有动机压制”与“压制”之间架桥,说明有压制的动机会导致压制。将 D 取非,有
动机也不会压制,则原文的 sub-conclusion 就被 weaken 了,从最后的结论也不成立。

189
C 项提到的 puclication 没有涉及到与本文逻辑推理相关的元素。  

20. The claim that a university should not be entitled to patent the inventions of its faculty
members plays which one of the following roles in the argument?

(A) It is the conclusion of the argument.

(B) It is a principle from which the conclusion is derived.

(C) It is an explicit assumption.

(D) It is additional but nonessential information in support of one of the premises.

(E) It is a claim that must be demonstrated to be false in order to establish the


conclusion.

这个题目我在 a/b 之间犹豫结果选了 b,答案是 a,如何区分 a/b?/

22. Politician: From the time our party took office almost four years ago the number of
people unemployed city-wide increased by less than 20 percent. The opposition party
controlled city government during the four preceding years, and the number of
unemployed city residents rose by over 20 percent. Thus, due to our leadership, fewer
people now find themselves among the ranks of the unemployed, whatever the
opposition may claim.

The reasoning in the politician’s argument is most vulnerable to the criticism that

(A) the claims made by the opposition are simply dismissed without being specified

(B) no evidence has been offered to show that any decline in unemployment over the
past four years was uniform throughout all areas of the city

(C) the issue of how much unemployment in the city is affected by seasonal fluctuations
is ignored

(D) the evidence cited in support of the conclusion actually provides more support for the
denial of the conclusion

(E) the possibility has not been addressed that any increase in the number of people
employed is due to programs supported by the opposition party.
此题我的答案是 C,LAST 答案是 D.

190
22 题,,题目要求找出 most vulnerable to the criticism 。C 也是一个原因但 D 是最好的答
案。
因为原文中说 From the time our party took office almost four years ago the number of
people unemployed city-wide increased by less than 20 percent,说明失业人数增长的没
有以前快了,但还是在增长。结论又说 fewer people now find themselves among the ranks
of the unemployed,明显是茅盾的,所以 D 最好。

23. A poor farmer was fond of telling his children: “In this world, you are either rich or
poor, and you are either honest or dishonest. All poor farmers are honest. Therefore, all
rich farmers are dishonest.”

The farmer’s conclusion is properly drawn if the argument assumes that

(A) every honest farmer is poor


(C) everyone who is dishonest is a rich farmer
(D) everyone who is poor is honest

when peopl say all A is B does not necessarily mean every B is A. So answer A is to say
All honest farmer is poor. That "All honest farmer is poor" plus "All poor farmer is honest"
means honest farmer is congruent with poor farmer.

because the farmer has already stipulated that people either rich or poor, either honest or
dishonest. therefore, the farmer says "all rish farmers are dishonest"

one step further, we can also say "every (or all) dishonest farmer is rich", can we?!

25. Medical research findings are customarily not made public prior to their publication in
a medical journal that has had them reviewed by a panel of experts in a process called
peer review. It is claimed that this practice delays public access to potentially beneficial
information that, in extreme instances, could save lives. Yet prepublication peer review is
the only way to prevent erroneous and therefore potentially harmful information from
reaching a public that is ill equipped to \evaluate medical claims on its own. Therefore,
waiting until a medical journal has published the research finding s that have passed peer
review is the price that must be paid to protect the public from making decisions based
on possibly substandard research.

The argument assumes that

(A) unless medical research findings are brought to peer review by a medical journal,
peer review will not occur

191
(B) anyone who does not serve on medical review panel does not have the necessary
knowledge and expertise to evaluate medical research finding

(C) the general public does not have access to the medical journals in which research
findings are published.

(D) all medical research findings are subjected to prepublication peer review

(E) peer review panels are sometimes subject to political and professional pressures that
can make their judgments less than impartial

答案:A

25. A
原文说 findings-->medical journal-->peer review 是对大众负责,因为 peer review 是必要的.
所以要 medical journal. See, peer review 必要推出 medical journal 必要.

25 题:因为 A---->B, 所以 A---->C。其假设为 B---->C

具体到原文:A 为 prevent erroneous and therefore potentially harmful information from


reaching a public that is ill equipped to \evaluate medical claims on its own.(前提中) 。
即 protect the public from making decisions based on possibly substandard research.
(结论中)

B 为 prepublication peer review (前提中),即 peer review will occur (A 选项中)

C 为 a medical journal has published the research finding s that have passed peer
review(结论中),即 medical research findings are brought to peer review by a medical
journal(A 选项中)

192
LSAT-10-4

8. A translation invariably reflects the writing style of the translator. Sometimes when a
long document needs to be translated quickly, several translators are put to work on the
job, each assigned to translate part of the document. In these cases, the result is usually
a translation marked by different and often incompatible writing styles. Certain computer
programs for language translation that work without the intervention of human translation
can finish the job faster than human translators and produce a stylistically uniform
translation with an 80 percent accuracy rate. Therefore, when a long document needs to
be translated quickly, it is better to use a computer translation program than human
translators.

Which one of the following issues would be LEAST important to resolve in evaluating the
argument?

(A) whether the problem of stylistic variety in human translation could be solved by giving
stylistic guidelines to human translators

(B) whether numerical comparisons of the accuracy of translations can reasonably be


made

193
(C) whether computer translation programs, like human translators, each have their own
distinct writing style

(D) whether the computer translation contains errors of grammar and usage that
drastically alter the meaning of the text

(E) how the accuracy rate of computer translation programs compares with that of human
translators in relation to the users’ needs.

答案:C,请问这种评价题的解题思路。

我的意思是:
首先这题的结论是:使用一种 computer program 比 human translators 好。这里要注意的
是一种 computer program,而不是多种。因此对每一种 computer program 是否 have their
own distinct writing style 的回答对结论没有影响,也就是最不重要。EACH

Questions 9-10
Myrna: People should follow diets in which fat represents no more than 30 percent of
total calories, not the 37 percent the average diet in this country contains.
Roland: If everyone in the country followed you recommendation during his or her entire
life, just 0.2 percent would lengthen their live at all, and then only by an average of 3
months. Modifying ort diet is not worthwhile. A lifetime of sacrifice spent eating an
unappealing low-fat diet is too high a price to pay for the chance of extending that
sacrifice for 3 months.
Myrna: But for everyone who dies early from a high-fat diet, many more people suffer
from serious chronic diseases because they followed such diets.
9. Myrna responds to Roland by
(A) disputing the correctness of the facts cited by Roland and offering facts that she
considers correct
(B) showing that the factors considered by Roland are not the only ones relevant in
evaluating her recommendation
(C) demonstrating that the statistics used by Roland to dispute her recommendation are
inaccurate
(D) suggesting that Roland’s evidence derives from unreliable sources
(E) pointing out that Roland’s argument assumes the very proposition it sets out to prove

10. Roland’s argument assumes that


(A) it is desirable to live in such a way as to length life as much as possible
(B) a low-fat diet cannot readily be made appealing and satisfying to a person who
follows it regularly

194
(C) diet is the only relevant factor to consider in computing influences on length of life
(D) the difference in tastiness between a diet in which fat represents 30 percent of total
calories and one in which it represents 37 percent is not noticeable
(E) not everyone in the country eats the average diet

11. Some critics claim that it is unfair that so many great works of art are housed in huge
metropolitan museums, since the populations served by these museums already have
access to a wide variety of important artwork. But this criticism is in principle unwarranted
because the limited number of masterpieces makes wider distribution of them
impractical. Besides, if a masterpiece is to be fully appreciated, it must be seen alongside
other works that provide a social and historical context for it.

Which one of the following, if established, could most logically serve as the principle
appealed to in the argument countering the critics’ claim?

(A) In providing facilities to the public, the goal should be to ensure that as many as
possible of those people who could benefit from the facilities are able to do so.

(B) In providing facilities to the public, the goal should be to ensure that the greatest
possible number of people gain the greatest benefit possible from them.

(C) It is unreasonable to enforce a redistribution of social goods that involves depriving


some members of society of these goods in order to supply others

(D) For it to be reasonable to criticize an arrangement as unfair, there must be a more


equitable arrangement that is practically attainable.

(E) A work of art should be displayed in conditions resembling as closely as possible


those in which the work was originally intended to be displayed.

According the argument, critics claim that great woks of art should not be stored in
metropoliton museums by which people are served already have access to various art
works.To rebut this criticism, the arguer point out that being housed in metropoliton
museums is necessary for master pieces to distribute wider and be fully
appreciated.Thus,
if there are not other equitable arrangements as metropoliton museums available, it will
not be reasonable to criticize housing art works in metropoliton museums as unfair.So,
the best answer is D.

13. This summer, Jennifer, who has worked at KVZ Manufacturing for just over three

195
years, plans to spend with her family the entire four weeks of paid vacation to which she
is entitled this year. Anyone who has worked at KVZ Manufacturing for between one and
four years is automatically entitled to exactly three weeks paid vacation each year but
can apply up to half of any vacation time that remains unused at the end of one year to
the next year’s vacation.
If the statements above are all true, which one of the following must also be true on the
basis of them?
(A) Jennifer did not use two weeks of the paid vacation time to which she was entitled
past year.
(B) If Jennifer continues to work for KVZ Manufacturing, she will only be entitled to three
weeks paid vacation next year.
(C) The majority of KVZ’s employees use each year all of the paid vacation time to which
they are entitled.
(D) Last year Jennifer took only one week of the paid vacation time to which she was
entitled.
(E) KVZ Manufacturing sometimes allows extra vacation time to employees who need to
spend more time with their families.

18-19. Marcus: For most ethical dilemmas the journalist is likely to face, traditional
journalistic ethics is clear, adequate, and essentially correct. For example, when
journalists have uncovered newsworthy information, they should go to press with it as
soon as possible. No delay motivated by the journalists’ personal or professional
interests is permissible.

Anita: Well, Marcus, of course interesting and important information should be brought
before the public-that is a journalist’s job. But in the typical case, where a journalist has
some information but is in a quandary about whether it is yet important or “newsworthy,”
this guidance is inadequate.

18. The point made by Anita’s statements is most accurately expressed by which one of
the following?

(A) Marcus’s claim that traditional journalistic ethics is clear for most ethical dilemmas in
journalism is incorrect.

(B) A typical case illustrates that Marcus is wrong in claiming that traditional journalistic
ethics is essentially correct for most ethical dilemmas in journalism.

(C) The ethical principle that Marcus cites does not help the journalist in a typical hind of
situation in which a decision needs to be made.

(D) There are common situations in which a journalist must make a decision and in which

196
no principle of journalistic ethics can be of help.

(E) Traditional journalistic ethics amounts to no more than an unnecessarily convoluted


description of the journalist’s job.

I choose (b), the right answer is (c). But answer (b) and (c) is the same in the e-version I
downloaded. So what is the original (C)?

19. In order to conclude properly from Anita’s statements that Marcus’ general claim
about traditional journalistic ethics is incorrect, if would have to be assumed that

(A) whether a piece of information is or is not newsworthy can raise ethical dilemmas for
journalists.

(B) there are circumstances in which it would be ethically wrong for a journalist to go to
press with legitimately acquired, newsworthy information.

(C) the most serious professional dilemmas that a journalist is likely to face are not
ethical dilemmas

(D) there are no ethical dilemmas that a journalist is likely to face that would not be
conclusively resolved by an adequate system of journalistic ethics

(E) For a system of journalistic ethics to be adequate it must be able to provide guidance
in every case in which a journalist must make a professional decision

答案:A,我选了 D

MARCUS: 传统 ethics 好<----newsworthy information go to press fast


ANITA: newsworthy 与否,有时难判断。

gap 存在于认定 traditional ethics 好坏和能否判断 newsworthy 之间。A,填了这个 gap.

e 所说的并不是不对,只是作为假设,它只能是结论成立的必要条件而非其它,文章很明白的
说了所谓前提是在"记者所面临的大部分道德两难问题的时候",而马库斯认为传统职业道德
是足以提供解决方法的,而安妮塔的意见是,在面临判断是否有新闻价值时,这个职业道德就
无法用.我们看到他们说的似乎是不同的两个事情,一个说的是 dilemma,一个说的是判断是
否有新闻价值,如果要使得安妮塔的理论成立,那么一定要弥合 gap,所以 a 是答案.E 的错误
在于它的范围超过了文章所说的内容,比较 general 并非是必要条件.

Questions 20-21

197
Of every 100 burglar alarms police answer, 99 are false alarms. This situation causes an
enormous and dangerous drain on increasingly scarce public resources. Each false
alarm wastes an average of 45 minutes of police time. As a result police are consistently
taken away from responding to other legitimate calls for service, and a disproportionate
share of police service goes to alarm system users, who are mostly businesses and
affluent homeowners. However, burglar alarm systems, unlike car alarm systems, are
effective in deterring burglaries, so the only acceptable solution is to fine burglar alarm
system owners the cost of 45 minutes of police time for each false alarm their systems
generate.
20. The statement that burglar alarm systems, unlike car alarm systems, are effective in
deterring burglaries plays which one of the following roles in the argument?
(A) It justifies placing more restrictions on owners of burglar alarms than on owners of car
alarms.
(B) It provides background information needed to make plausible the claim that the
number of burglar alarms police are called on to answer is great enough to be a drain on
public resources.
(C) It provides a basis for excluding as unacceptable one obvious alternative to the
proposal of fining owners of burglar alarm systems for false alarms.
(D) It gives a reason why police might be more inclined to respond to burglar alarms than
to car alarms.
(E) It explains why a disproportionate number of the burglar alarms responded to by
police come from alarm systems owned by businesses.

21. On the basis of the premises advanced, which one of the following principles, if
established, would provide the most justification for the concluding recommendation?
(A) No segment of a community should be permitted to engage in a practice that has
been shown to result in a disproportionate share of police service being devoted to that
segment of the community.
(B) When public resources are in short supply, any individual who wants special services
from public agencies such as police and fire departments should be required to pay for
those services if he or she can afford to do so.
(C) Police departments are not justified in improving service to one segment of the
community at the expense of other segments of the community unless doing so reduces
the crime level throughout the entire area served.
(D) Anyone who directly benefits from a service provided by public employees should be
required to reimburse the general public fund an amount equivalent to the average cost
providing that service.
(E) If receipt of a service results in the waste of scarce public resources and people with
other legitimate needs are disadvantaged in consequence, the recipient of that service
should compensate the public for the resources wasted.

198
22. When butterfat was considered nutritious and healthful, a law was enacted requiring
that manufacturers use the term “imitation butter” to indicate butter whose butterfat
content had been diminished through the addition of water. Today, it is known that the
high cholesterol content of butterfat makes it harmful to human health. Since the public
should be encouraged to eat foods with lower rather than higher butterfat content and
since the term “imitation” with its connotations of falsity deters many people from
purchasing products so designated, manufactures who wish to give reduced-butterfat
butter the more appealing name of “lite butter” should be allowed to do so.
Which one of the following, if true, most seriously undermines the argument?
(A) The manufactures who prefer to use the word “lite” instead of “imitation” are
motivated principally by the financial interest of their stock holders.
(B) The manufacturers who wish to call their product “lite butter” plan to change the
composition of the product so that it contains more water than it now does.
(C) Some individuals who need to reduce their intake of cholesterol are not deterred from
using the reduced-butterfat product by the negative connotations of the term “imitation.”
(D) Cholesterol is only one of many factors that contribute to the types of health problems
with which the consumption of excessive amounts of cholesterol is often associated.
(E) Most people deterred from eating “imitation butter” because of its name choose
alternatives with a lower butterfat content than this product has.

当 butterfat 被 认 为 是 好 东 西 时 , 法 律 要 求 butterfat 含 量 低 的 Butter 标 明 为 “ imitation


butter”。现在 butterfat 被认为不是好东西。因为应鼓励吃 butterfat 含量低的食品,而且
“imitation butter”的说法吓住了大家,所以应允许生产商用“ lite butter”的名去标 butterfat 含
量低的 butter。

原 文 的 意 思 是 , 由 于 1) 需 要 鼓 励 公 众 吃 BUTTERFAT 含 量 低 的 食 品 , 2) 一 些 人 被
IMITATION 这个词吓住了, 因为它含有"虚假"之意, 所以不买. 所以要换名字
C 的意思是一些刻意注意吃低脂肪含量的人不会被 IMITATION 这个名字吓住,所以不需要
换名字
C 不选是因为: C 其实没有削弱,因为它没有否定原因 2, 刻意注意的人没有被吓住, 但不
排除一种可能,既被吓住的人都是那些不刻意注意降脂的人,而他们也属于需要被鼓励的
大众范围内. 也就是说, C 没有削弱 2)
E 刚好,被吓住的人反而因此去选 FAT 含量更低的产品, 多好啊,还改什么名呀?

23. Farm animals have certain behavioral tendencies that result from the evolutionary
history of these species. By imposing on these animals a type of organization that
conflicts with their behavioral tendencies, current farm-management practices cause the
animals more pain and distress than do practices that more closely conform to the
animals’ behavioral tendencies. Because the animals tend to resist this type of
organization, current practices can also be less efficient than those other farm-
management practices.
If the statements above are true, which one of the following can be properly inferred from
them?

199
(A) Some of the behavioral tendencies of farm animals can be altered by efficient farm-
management practices.
(B) In order to implement efficient farm-management practices, it is necessary to be
familiar with the evolutionary history of farm animals.
(C) In order to create farm-management practices that cause less pain and distress to
farm animals, a significant loss of efficiency will be required.
(D) Farm-management practices that cause the least amount pf pain and distress to farm
animals are also the most efficient management practices.
(E) Some changes in farm-management practices that lessen the pain and distress
experienced by farm animals can result in gains in efficiency.

24. It now seems clear that the significant role initially predicted for personal computers in
the classroom has not become fact. One need only look to the dramatic decline in sales
of computers for classroom use in the past year for proof that the fad has passed.
Which one of the following arguments contains flawed reasoning parallel to that in the
argument above?
(A) Clearly government legislation mandating the reduction of automobile emissions has
been at least partially successful, as is demonstrated by the fact that the air of the 20
largest cities now contains smaller amounts of the major pollutants mentioned in the
legislation than it did before the legislation was passed.
(B) Mechanical translation from one language into another, not merely in narrow contexts
such as airline reservations but generally, is clearly an idea whose time has come. Since
experts have be4en working on the problem for 40 years, it is now time for the
accumulated expertise to achieve a breakthrough.
(C) Sales of computers for home use will never reach the levels optimistically projected
by manufacturers. The reason is that home use was envisioned as encompassing tasks,
such as menu planning and checkbook reconciliation, that most homemakers perform in
much simpler ways than using a computer would require.
(D) It is apparent that consumers have tired of microwave ovens as quickly as they
initially came to accept this recent invention. In contrast to several years of increasing
sales following the introduction of microwave ovens, sales of microwave ovens flattened
last year indicating that consumers have found relatively little use for these devices.
(E) Creating incentives for a particular kind of investment inevitably engenders boom-
and-bust cycles. The evidence is in the recent decline in the value of commercial real
estate, which shows that, although the government can encourage people to put up
buildings, it cannot guarantee that those buildings will be fully rented or sold.

200
Set 11-1

1. Educational television is a contradiction in terms. While a classroom encourages


social interaction, television encourages solitude. School is centered on the
development of language, but television depends upon constantly changing visual
images. And in a classroom, fun is merely a means to an end, but on television it is
the end in itself.
Upon which one of the following assumptions does the author rely in the passage?
(A) The classroom should not be a place where anyone has fun.
(B) Only experiences that closely resemble what takes place in the school
environment can be educational.
(C) Television programs reinforce some of the values of the school environment.
(D) Educational television programs are qualitatively better than most other television
programs.
(E) The potential of television as a powerful learning tool has not yet been realized.
--------------------------
答案 B, 我觉得 E 也能作为假设,为何 E 不对?
in a classroom, fun is merely a means to an end, but on television it is the end in

201
itself. 这句话该如何理解?请大家指教。

文中主要讲了电视教育作为一种教育的方式和课堂相比的问题。E 是说电视作为可以用来进
行教育的一种工具没有发挥其潜能,主体被换掉了,另外,E 也只是一个结果,不是前提。
中文意思:教室中,乐趣是达到目的的一个手段,而在电视教育中,乐趣成了终极目的。

5. ( E ) Some people have questioned why the Homeowners Association is


supporting Cooper’s candidacy for mayor. But if the Association wants a mayor who will
attract more businesses to the town, Cooper is the only candidate it could support. So,
since the Association is supporting Cooper, it must have a goal of attracting more
businesses to the town.
The reasoning in the argument is in error because
(A) the reasons the Homeowners Association should want to attract more businesses to
the town are not given
(B) the Homeowners Association could be supporting Cooper’s candidacy for reasons
unrelated to attracting businesses to the town
(C) other groups besides the Homeowners Association could be supporting Cooper’s
candidacy
(D) the Homeowners Association might discover that attracting more businesses to the
town would not be in the best interest of its members(B)
(E) Cooper might not have all of the skills that are needed by a mayor who wants to
attract businesses to a town

答案:B
error of the reasoning: mistaken reversal
if association wants a mayor who will attract business, then it will support Cooper; (A---B)
it is supporting Cooper, so it has a goal of attracing more business.(B---A)
So the association may be supporting Cooper for a reason other than attracting business.
E talks about the skill of Cooper, which is never mentioned in the argument. it is therefore
irrelevant.

lsat 11-11
11.The fire that destroyed the Municipal Building started before dawn this morning, and
the last fire fighters did not leave until late this afternoon. No one could have been
anywhere in the vicinity of a fire like that one and fail to notice it. Thomas must have seen
it, whatever be now says to the contrary. He admits that, as usual, he went from his
apartment to the library this morning, and there is no way for him to get from his
apartment to the library without going past the Municipal Building.
The main conclusion of the argument is that
(A) Thomas was in the vicinity of the fire this morning
(B) Thomas claimed not to have seen the fire
(C) Thomas saw the fire this morning
(D) Thomas went directly from his apartment to the library this morning

202
(E) Thomas went by the Municipal Building this morning
答案:C,为什么 E 不行?

13. Editorial: In rejecting the plan proposed by parliament to reform the electoral process,
the president clearly acted in the best interests of the nation. Anyone who thinks
otherwise should remember that the president made this decision knowing it would be
met with fierce opposition at home and widespread disapproval abroad. All citizens who
place the nation’s well-being above narrow partisan interests will applaud this
courageous action.
The reasoning in the editorial is in error because
(A) it confused a quality that is merely desirable in a political leader with a quality that is
essential to effective political decision-making
(B) it fails to distinguish between evidence concerning the courage required to make a
certain decision and evidence concerning the wisdom of making that decision
(C) it ignores the likelihood that many citizens have no narrow partisan interest in the
proposed
(D) it overlooks the possibility that there was strong opposition to the parliament’s plan
among members of the president's among members of the president's own party
(E) it depends on the unwarranted assumption that any plan proposed by a parliament
will necessarily serve only narrow partisan interests

答案:B

原文的推理是:因为 President 有勇 气 经受住反对,所以他是以国家最好利益为行为准则,


每个市民都该支持他。这个 GAP 就在有勇气不一定决定就智慧。即 B

Question15-16
A favored theory to explain the dinosaurs, together with many other species, has been
the globally catastrophic collision of a large asteroid with the Earth: Supporting evidence
is an extraterrestrial chemical element in a layer of dust found worldwide at a geological
level laid down contemporaneously with the supported event. A new competing theory
contends that any asteroid impact was irrelevant, because it was massive volcanic
activity that caused the extinctions by putting enough dust into the atmosphere to cool
the planet. The Decean region of India contains extensive volcanic flows that occurred
within the same time period as the supposed asteroid impact and the extinctions.
16. Which one of the following, if true, most of strongly indicates that the asteroid-impact
theory is at least incomplete, if not false?
(A) Large concentrations of dinosaur nests with fossil eggs found in Alberta indicate that
at least some species of dinosaurs congregated in large groups during some part of their
lives.
(B) Dinosaur remains indicate that some species of dinosaur could have migrated in
herds over wide ranges, so that they could have traveled to escape the local effects of

203
certain catastrophes.
(C) Legends from many cultures, such as the Creex legend that Cudmus raised an army
by sowing dragons’ teeth in the ground, show that various accident peoples worldwide
were familiar with the fossils of dinosaurs.
(D) In the Gobi desert in China, where now only small animals can eke out an existence,
fossil dinosaur skeletons 27 feet long were found in circumstances indicating that the
climate there was as dry when the dinosaurs lived as it is now.
(E) The fossil record in Montano from below the layer of extraterrestrial dust shows a
diminution over time in dinosaur species from 35 to 13, and dinosaur teeth found above
the dust layer show a diminution in species from 13 to 5.
key's E。没有找到讲这一题 E 的帖子。
请问 E 是不是说在行星碰撞之后还有恐龙的存在,所以削弱了这个理论?
还有 B,里面的 catastrophes 是指 collision 还是指 volcanic activity?如果是指 volcanic,
那 就 是 削 弱 volcanic 理 论 了 , 如 果 是 指 collision , 同 样 也 就 削 弱 了 collision 理 论 了 。
puzzled.

Lawyer: 1。你对 E 的理解是对的。以灰层上下分撞击前和后。而上下都有恐龙。说明没在撞击


中灭绝。
2。16 题再将第一个理论,跟第二个理论无关,故灾难当然是 COLLISION。B 错的原因是原
文说 globally catastrophic collision,所以跑得了初一,跑不了十五。

原 文 : asteroid-impact theory 声 称 恐 龙 的 灭 绝 是 行 星 撞 地 球 引 起 的 环 球 大 灾 难 (the


globally catastrophic collision)。
B 选项说明幸免的恐龙是因为迁移而逃离了地域性的灾难。这不也证明了 asteroid-impact
theory 是不正确或不完整的,因为如果有环球性的灾难,迁移是救不了恐龙的。

Mindfree: "could have" indicates that these species did not travel and did not escape the
disaster. it therefore support the collision theory in that the catastrophy is global.

17. A contract, whether expressed or unexpressed, exists when two parties engage with
each other for the reciprocal transfer of benefits. Thus, in accepting support from public
funds, an artist creates at least an unexpressed contract between himself or herself and
the public, and the public can rightly expect to benefit from the artist’s work.
Which one of the following most accurately describes an error in reasoning in the
passage?
(A) attempting to justify a rule of conduct on the grounds that it confers benefits on all of
the parties involved.
(B) concluding that a definition is fully applicable to a situation when it is know only that
the situation conforms partially to that definition
(C) speaking only in a abstract terms about matters that involve contingencies and that
must be judged on a case-by-case basis
(D) confusing the type of mental or emotional activity in which an individual can engage
with the mental or emotional states that can characterize groups of individuals

204
(E) treating an issue that requires resolution through political processes as if it were
merely a matter of opinion
答案:B

18. People cannot be morally responsible for things over which they have no control.
Therefore, they should not be held morally responsible for any inevitable consequences
of such things, either. Determining whether adults have any control over the treatment
they are receiving can be difficult. Hence in some cases it can be difficult to know
whether adults bear any moral responsibility for the way they are treated. Everyone,
however, sometimes acts in ways that are an inevitable consequently of treatment
received as an infant and infants clearly cannot control, and so are not morally
responsible for the treatment they receive.
Anyone making the claims above would be logically committed to which one of the
following further claims.
(A) An infant should never be held morally responsible for an action that infant has
performed.
(B) There are certain commonly performed actions for which no one performing those
actions should ever be held morally responsible.
(C) Adults who claim that they have no control over the treatment they are receiving
should often be held at least partially responsible for being so treated.
(D) If a given action is within a certain person’s control that person should be held
morally.
(E) No adult should be held morally responsible for every action he or she performs.
这道题我为什莫觉得都不对呢 ? 答案为 E,应该如何考虑?

由 People cannot be morally responsible for things over which they have no control. 推出
结 论 1Therefore, they should not be held morally responsible for any inevitable
consequences of such things, either.
再由 Determining whether adults have any control over the treatment they are receiving
can be difficult.推出结论 2Hence in some cases it can be difficult to know whether adults
bear any moral responsibility for the way they are treated

此 处 有 一 个 小 转 折 everyone, however, sometimes acts in ways that are an inevitable


consequently of treatment received as an infant and infants clearly cannot control, and so
are not morally responsible for the treatment they receive.注意这里提到了 everyone 综上
两个结论,推出第三个结论。就是答案 E。

an infant does not have control over the treatment he or she receives
|
Everyone acts in ways that are consequences of the treatment he or she received as an
infant (that he/she cannot control)
|
Since "People cannot be morally responsible for things/and their consequences over
which they have no control. "

205
(E) No adult should be held morally responsible for ever action he or she performs.

Everyone, however, sometimes acts in ways that are an inevitable consequence of treat
ment received as an infant and infants clearly cannot control, and so are not morally resp
onsible for, the treatment they receive.

然而,每个人有时都会象婴儿一样接受无法自己控制的治疗,因此也就不需要为这种治疗
负责。因此答案是 No adult should be held morally responsible for every action he or she p
erforms

19. Fares on the city-run public buses in Greenville are subsidized by city tax revenues,
but among the beneficiaries of the low fares are many people who commute from outside
the city to jobs in Greenville. Some city councilors argue that city taxes should be used
primarily to benefit the people who pay them, and therefore that bus fares should be
raised enough to cover the cost of the service.

Each of the following, if true, would weaken the argument advanced by the city councilors
EXCEPT:

(A) Many businesses whose presence in the city is beneficial to the city’s taxpayers
would relocate outside the city if public-transit fare were more expensive.

(B) By providing commuters with economic incentives to drive to work, higher transit
fares would worsen air pollution in Greenville and increase the cost of maintaining the
city’s streets.

(C) Increasing transit fares would disadvantage those residents of the city whose low
incomes make them exempt from city taxes, and all city councilors agree that these
residents should be able to take advantage of city-run services.

(D) Voters in the city, many of whom benefit from the low transit fares are strongly
opposed to increasing local taxes.

(E) People who work in Greenville and earn wages above the nationally mandated
minimum all pay the city wage tax of 5 percent.

The answer is D, but I think it should be E.

D 是典型的不相关选项. 重点是原文的逻辑关系, 你要在原文的逻辑关系内找答案.

D 错在并没有 weaken 原文的逻辑关系:tax money should benefit people who pay taxes--
>tax money should not be used for people living outside the city. weaken argument 是指

206
削弱了论述. 既能指出论述和逻辑上的错误. D 和此逻辑没有关系.

E 是 weaken 因为它指出了原文逻辑的一个漏洞:people living outside also pay taxes. 实际


上否定了原文的一个别 assumption:people living outside the city do not pay taxes. 所以对.

20. Government official: Clearly, censorship exists if we, as citizens, are not allowed to
communicate what we are ready to communicate at our own expense or if other citizens
are not permitted access to our communications at their own expense. Public
unwillingness to provide funds for certain kinds of scientific, scholarly, or artistic activities
cannot, therefore, be described as censorship.
The flawed reasoning in the government official’s argument is most parallel to that in
which one of the following?
(A) All actions that cause unnecessary harm to others are unjust: so if a just action
causes harm to others, that action must be necessary.
(B) Since there is more to good manners than simply using polite forms of address, it is
not possible to say on first meeting a person whether or not that person has good
manners.
(C) Acrophobia, usually defined as a morbid fear of heights, can also mean a morbid fear
of sharp objects. Since both fears have the same origin.
(D) There is no doubt that a deed is heroic if the doer risks his or her own life to benefit
another person. Thus an action is not heroic if only thing it endangers is the reputation of
the doer.
(E) Perception of beauty in an object is determined by past and present influences on the
mind of the beholder. Thus on object can be called beautiful, since not everyone will see
beauty in it.

答案:D

如 果 我 花 自 己 的 钱 去 COMMUNICATE 或 者 花 自 己 的 钱 去 ACCESS 别 人 的
COMMUNICATIONS, 你不让, 这叫 CENSORSHIP
所以, 公众拒绝为一些活动(别人)掏自己的腰包就不能说是 CENSORSHIP

Say, I want to pay 100000000000RMB to put up a commercial at CCTV, saying that


China has no freedom of speech.
of course nobody would allow me to communicate this view,(though it is so so true)
This is cencorship.

Finally I understood.
at own expense to communicate,but are not allowed to ------censorship
unwillingness to provide funds(not at own expense)------- not censorship
The conclusion is : A---B, ===== negA---ne
Thanks chelseayang!

原 文 : 如 果 不 能 自 己 掏 钱 搞 活 动 ( 如 果 由 PUBLIC 出 钱 ) , 则 存 在 审 查 制 度 。 所 以 ,

207
PUBLIC 不出钱搞活动,则不存在审查制度。

21. The Japanese haiku is defined as a poem of three lines with five syllables in the first
line, seven syllables in the second line, and five syllables in the third line. English poets
tend to ignore this fact. Disregarding syllable count, they generally call any three-line
English poets have little respect for foreign traditions, even those from which some of
their own poetry derives.
The reasoning is flawed because it
(A) confuses matters of objective fact with matters of subjective feeling
(B) draws a conclusion that is broader in scope than is warranted by the evidence
advanced
(C) relies on stereotypes instead of presenting evidence
(D) overlooks the possibility that the case it cites is not unique
(E) fails to acknowledge that ignoring something implies a negative judgment about that
thing
答案是 b
have little respect for 是什么意思啊
还有就是答案从何而来啊,谢谢先了

上文只说忽略了音节,下文得到结论是忽视传统,这明显扩大了范畴

Questions 22-23

No one knows what purposes, if any, dreams serve, although there are a number of
hypotheses. According to one hypothesis, dreams are produced when the brain is
erasing "parasitic connections" ( meaningless, accidental associations between ideas),
which accumulate during the day and which would otherwise clog up our memories.
Interestingly, the only mammal that does not have rapid eye movement sleep, in which
we humans typically have our most vivid dreams, is the spiny anteater, which has been
seen as anomalous in that in it has a very large brain relative to the animal’s size. This
fact provides some confirmation for the parasitic-connection hypothesis, since the
hypothesis predicts that for an animal that did not dream to have an effective memory
that animal would need extra memory space for the parasitic connections.

22. The parasitic-connection hypothesis, if true, most strongly supports which one of the
following?

(A) The animals with the smallest brains spend the most time sleeping.

(B) Immediately after a person awakens from normal sleep, her or his memory contains
virtually no accidental associations between ideas.

208
(C) When a mammal that would normally dream is prevented from dreaming, the
functioning of its memory will be impaired.

(D) Insofar as a person’s description of a dream involves meaningful associations


between ideas, it is an inaccurate description.

(E) All animals other than the spiny anteater dream

答案:D

23. The reasoning in the argument most closely conforms to which one of the following
principles?

(A) Facts about one species of animal can provide confirmation for hypothesis about all
species that are similar in all relevant respects to the particular species in question.

(B) A hypothesis from which several predictions can be drawn as logical conclusions is
confirmed only when the majority of these predictions turn out to be true

(C) A hypothesis about the purpose of an action or object is confirmed when it is shown
that the hypothesized purpose is achieved with the help of the action or object and could
not be achieved without that action or object.

(D) A hypothesis is partially confirmed whenever a prediction derived from that


hypothesis provides an explanation for an otherwise unexplained set of facts.

(E) When several competing hypotheses exist, one of them is confirmed only when it
makes a correct prediction that its rivals fail to make.

答案:D

这两题题干我没读懂,读了 N 遍了,能不能讲讲,谢谢。

观点解释型
No one knows what purposes, if any, dreams serve, although there are a number of
hypotheses. 废话,可不看

According to one hypothesis, dreams are produced when the brain is erasing "parasitic
connections" ( meaningless, accidental associations between ideas), which accumulate
during the day and which would otherwise clog up our memories.
观点:梦用来收集垃圾,否则占内存

209
Interestingly, the only mammal that does not have rapid eye movement sleep, in which
we humans typically have our most vivid dreams, is the spiny anteater, which has been
seen as anomalous in that in it has a very large brain relative to the animal’s size.
现象:有个冬冬叫 sa 不寻常,睡觉时眼睛不动(暗示不作梦),其大脑相对体型而言比例
特大。

This fact provides some confirmation for the parasitic-connection hypothesis, since the
hypothesis predicts that for an animal that did not dream to have an effective memory
that animal would need extra memory space for the parasitic connections.
解释:证明了前文观点。因为原观点预测:没有梦的动物(是可怜的。编者注)必须有个特
别大的内存(即大脑)来存放垃圾(有梦的动物通过梦把垃圾收集了)。

提出观点,展示论据,总结。正评价。

题目,就不用说了吧。

24. The body of anyone infected by virus X will, after a week, produce antibodies to fight
the virus: the antibodies will increase in number for the next year or so. There is now a
test that reliably indicates now many antibodies are present in a person’s body. If
positive, this test can be used during the first year of infection to estimate to within a
month how long that person has had the virus.

Which one of the following conclusions is best supported by the statements above?

(A) Antibodies increase in number only until they have defeated the virus.

(B) Without the test for antibodies, there is no way of establishing whether a person has
virus X.

(C) Antibodies are produced only for viral infections that cannot be fought by any other
body defenses.

(D) If a person remains infected by virus X indefinitely, there is no limit to the number of
antibodies that can be present in the person’s body.

(E) Anyone infected by virus X will for a time fail to exhibit infection if tested by the
antibody test

答案 E,我选 B,为什么呢?

为这个 test 是通过测抗体来得知感染的情况,而抗体要在感染之后几周在出现(The body


of anyone infected by virus X will, after a week, produce antibodies to fight the virus),

210
自然就是 E:如果病人是通过 antibody test,在感染之后的一段时间内是表现不出感染症
状的

Lawyer:
1. 如果呈阳性,可用这个实验去估计那个人感染病毒的时间,精确度可在一个月内
2. E 对的原因:原文是 after a week 才产生 produce antibodies ,而 the antibody test 是靠
测 antibodies 来显示是否感染病毒。故有 for a time (你的答案中 not a time 是错的)测不到
感染

25. Large inequalities in wealth always threaten the viability of true democracy, since
wealth is the basis of political power, and true democracy depends on the equal
distribution of political power among all citizens.
The reasoning in which one of the following arguments most closely parallels the
reasoning in the argument above?
(A) Consumer culture and an emphasis on technological innovation are a dangerous
combination, since together they are uncontrollable and lead to irrational excess.
(B) If Sara went to the bookstore every time her pocket was full, Sara would never have
enough money to cover her living expenses, since books are her love and they are
getting very expensive.
(C) It is very difficult to write a successful science fiction novel that is set in the past,
since historical fiction depends on historical accuracy, whereas science fiction does not.
(D) Honesty is important in maintaining friendships. But sometimes honesty can lead to
arguments, so it is difficult to predict the effect a particular honesty act will have on a
friendship.
(E) Repeated encroachments on one’s leisure time by a demanding job interfere with the
requirements of good health. The reason is that good health depends on regular
moderate exercise, but adequate leisure time is essential to regular exercise.
答案:E

原文推理:true democracy-------->the equal distribution of political power。Large


inequalities in wealth 导致 Large inequalities distribution of political power----->Not true
democract(即 threaten the viability of true democracy)

该推理本质是是通过 true democracy depends on the equal distribution of political power


among all citizens 的逆否命题来实现推理的。原文除 E 用到逆否命题作为推理的基础

明白了,原文的前提和结论其实可以看作原命题和逆否命题,本来就是成立的,只是中间
通过“since wealth is the basis of political power”来替换成 political power;而 E,也一样,
是用“adequate leisure time is essential to regular exercise”来替换,其实 basis 和
essential 从友好词汇上都能看出来:)

211
LSAT-11-4

Lsat-Set 11, SectIV, T9,19,21,22


9. There is relatively little room for growth in the overall carpet market, which is tied to the
size of the population. Most who purchase carpet do so only once or twice, first in their
fifties or sixties. Thus as the population ages, companies producing carpet will be able to
gain market share competitors, and not through more aggressive marketing.
Which one of the following, if true, casts the most doubt on the conclusion above?
(A) Most of the major carpet producers market other floor coverings as well.
(B) Most established carpet producers market several different brand names and
varieties, and there is no remaining niche in the market for new brands to fill.
(C) Two of the three mergers in the industry’s last ten years led to a decline in profits and
revenues for the newly merged companies.

212
(D) Price reductions, achieved by cost-cutting in production, by some of the dominant
firms in the carpet market are causing other producers to leave the market altogether.
(E) The carpet market is unlike most markets in that consumers are becoming
increasingly resistant to new patterns and styles.

理解:B、E 都可以 weaken 原题结论。?

first, e did not weaken the conclusion ,and on the contrary,it strengthens the conclusion in
some way,cos it suggest that the old style will be popular for ever and the companies
need not make any change.and b is an answer but it is not as good as d,for this is an
example of how the aggressive maketing strategy gain shares in the market.

LSAT-11-4-10
答案是 A,但是,我不明白。
另:命题的最后一句话是什么意思?
谢谢!

10. Decision makers tend to have distinctive styles. One such style is for the decision
maker to seek the widest possible input from advisers and to explore alternatives while
making up his or her mind. In fact, decision makers of this sort will often argue vigorously
for a particular idea, emphasizing its strong points and downplaying its weaknesses, not
because they actually believe in the idea but because they want to see if their real
reservations about it are idiosyncratic or are held independently by their advisers.

Which one of the following is most strongly supported by the statement above?

(A) If certain decision makers’ statements are quoted accurately and at length; the
content of the quote could nonetheless be greatly at variance with the decision eventually
make.

(B) Certain decision makers do not know which ideas they do not really believe in until
after they have presented a variety of ideas to their advisers

(C) If certain decision makers dismiss an idea out of hand, it must be because its
weaknesses are more pronounced than any strong points it may have.

(D) Certain decision makers proceed in a way that makes it likely that they will frequently
decide in favor of ideas in which they do not believe.

(E) If certain decision makers’ advisers know the actual beliefs of those they advise,
those advisers will give better advice than they would if they did not know those beliefs.

213
做决定者对于某个论题,强烈支持一方而贬低另一方。并不是因为他们真的支持这一方,只
是因为他们想看看他们支持的这个观点是否只有他们才会支持,还是他们的 ADVISER 也
会支持。

A 这是这个意思。

1.最后的话:这种做决定的人常常为某个主意强烈辩护,强调它的好处,故意忽略它的坏
处。他们这样做并不是因为他们相信这个主意,而是他们想看看他们对这个主意的保留意见
(reservation)只有自己有,还是提建议的人也独立持有这个保留意见。其实原文说的是这
种人口是心非,强烈为之辩护的观点,其实自己也有保留。

A。如果做决定的人的表述被精确和详尽的引述,这种表述可能和最后的决定大大不同。

是 Must be true 的题,不是加强题。

LSAT 11-IV-12 的思路


The foreign minister of Zeria announced today that her country was severing diplomatic
relations with Nandalo because of Nandalo’s flagrant violations of human rights. But
Zeria continues to maintain diplomatic relations with many countries that the minister
knows to have far worse human-rights records than Nandalo does. Therefore, despite the
foreign minister’s claim, this latest diplomatic move cannot be explained exclusively by
Zeria’s commitment to upholding human rights.
12. The argumentative structure of which one of the following most closely parallels that
of the argument in the passage?
(A) Henry’s parents insist that he eat breakfast before leaving for school because not
doing so would be bad for his health. But his parents themselves almost never eat
breakfast, so their insistence cannot be completely explained by their concern for his
health.
(B) Professor Walsh says that only typed term papers will be accepted because most
handwriting is difficult to read. But since she lectures from handwritten notes, her policy
cannot be exclusively explained by any difficulty she has with handwritten maternal.
(C) James claims that he stole only because he was hungry. But although hunger could
account for stealing if food could not be readily obtained in any other way, in this case
food was otherwise readily available, and so James theft cannot be completely explained
by his hunger.
(D) Armand declined Helen’s invitation to dinner on the grounds that socializing with
coworkers is imprudent. But since Armand went to a movie with another coworker, Maria,
that same evening, his expressed concern for prudence cannot fully explain his refusal.
(E) It is often asserted that there are fewer good teachers than there used to be because
teachers’ salaries have reached a new low. But teachers have always been poorly paid;

214
so low salaries cannot fully explain this perceived decline in the effectiveness of
teachers.

为什莫是 D,感觉答案好像都对是的,对这类题总是翻晕,那位给指点一二?

This question is structured to deny the premise in the form of:

1: A => B
2: A => -B
----------------
A can not cause B

Given this, we can see D is the only one in line with the structure.
(A) His parents never eat breakfast, but we don't know whether they are healthy or not.
(B) What the professor meant for is others' handwriting, not his own.
(C) If we are told that other people were hungry but they didn't steal, we can pick it up,
who care the availability of food.
(E) It doesn't offer us the same situation here, "a new low" means a new situation, not the
same as before.

Let's look at (D)

Imprudence => Decline invitation of Coworker


Imprudence => Accept invitation of Coworker
----------------------------------------------------------------
Imprudence is not the reason of declining invition. (Must have some other reason)

14. In a learning experiment a researcher ran rats through a maze. Some of the rats were
blind, others deaf, others lacked a sense of smell, and others had no sensory
deficiencies: yet all the rats learned the task in much the same amount of time. Of the
senses other than sight, hearing, and smell, only kinesthesia had not previously been
shown to be irrelevant to maze-learning. The researcher concluded on the basis of these
facts that kinesthesia, the sensation of bodily movement, is sufficient for maze-learning.
The researcher’s reasoning is most vulnerable to which one of the following criticisms?
(A) The small differences in proficiency found by the researcher did not appear to fall into
a systematic pattern by group.
(B) The possibility that the interaction of kinesthesia with at least one other sense is
required for maze-learning cannot be ruled out on the basis of the data above.
(C) It can be determined from the data that rats who are deprived of one of their sources
of sensory stimulation become more reliant on kinesthesia than they had been, but the
data do not indicate how such a transference takes place.
(D) It can be determined from the data that rats can learn to run mazes by depending on

215
kinesthesia alone, but the possibility that rats respond to non-kinesthetic stimulation is
not ruled out.
(E) It can be determined from the data that maze-learning in rats depends on at least two
sources of sensory stimulation, one of which is kinesthesia, but which of the remaining
sources must also be employed is not determinable.

答案:B

的确不太明白答案再说什么?:( 可能是阅读的问题:(
能不能也帮我分析一下其他选项错在哪里,多谢了:)

原文:原文试验老鼠,它们每种的感官各缺一种,还有一种是三种感官都好。但它们的某种
能 力 一 样 。 因 为 所 有 感 官 中 , 只 有 kinesthesia 没 有 证 明 和 这 种 能 力 无 关 , 所 以 说
kinesthesia 充分导致了这种能力有关。它没有排除的可能性就是 kinesthesia 和三种感官中
的至少一种共同导致这能力(因为尽管每种老鼠缺一种感官,却还有别的两种)。即 B。其
它容易排除。

15. New legislation would require a seven-day waiting period in the sale of handguns to
private individuals, in order that records of prisons could be checked and the sale of
handguns to people likely to hurt other people thereby prevented. People opposed to this
legislation claim that prison records are so full of errors that the proposed law would
prevent as many law-abiding citizens as criminals from having access to handguns.
If the claim made by people opposed to the new legislation is true, which one of the
following is a principle that, if established, would do the most to justify opposition to the
new legislation on the basis of that claim?
(A) The rights of law-abiding citizens are more worthy of protection than are the rights of
criminals.
(B) Nothing should be done to restrict potential criminals at the cost of placing restrictions
on law-abiding citizens.
(C) Legislation should not be enacted if no benefit could accrue to society as a result of
that legislation.
(D) No restrictions should be placed on the sale of merchandise unless sale of that
merchandise could endanger innocent people.
(E) Even citizens who are neither fugitives nor felons should not be permitted to own a
handgun unless they have received adequate training.
答案:B

19. Many people change their wills on their own every few years, in response to
significant changes in their personal or financial circumstances. This practice can create
a problem for the executor when these people are careless and do not date their wills:
the executor will then often know neither which one of several undated wills is the most
recent, nor whether the will drawn up last has ever been found. Therefore, people should

216
not only date their wills but also state in any new will which will it supersedes, for then
there would not be a problem to begin with.
The reasoning in the argument is flawed because the argument
(A) treats a partial solution to the stated problem as though it were a complete solution.
(B) Fails to distinguish between prevention of a problem and successful containment of
the adverse effects that the problem might cause.
(C) Proposes a solution to the stated problem that does not actually solve the problem
but merely makes someone else responsible for solving the problem.
(D) Claims that a certain action would be a change for the better without explicitly
considering what negative consequences the action might have.
(E) Proposes that a certain action be based on information that would be unavailable at
the time proposed for that action.

为什么选 A?

Explained before. Read carefully what the problem is: "...the executor will then often
know neither which one of several undated wills is the most recent, nor whether the will
drawn up last has ever been found". And think if the second part of the problem is
addressed by the solution.
我的意见, 供参考.答案非确认标准答案,请 1stzhang 给出标准答案.

1. A 原文的结论为问题解决了. 所以削弱题的答案很可能是问题没有解决. 果然 A 就是. 因


为即使表明日期, 可以分清楚已发现 will 的先后, 但很多情况下还是不知道是不是好有没有
找到最新的 will. 所以只是 partial solution.
处于某些原因, 人们常常几年就改一次遗嘱, 结果是给遗嘱执行人造成很大的麻烦. 原因是
搞不清几个没标日期的遗嘱中哪个是最新的, 或是最新的遗嘱是不是已经找到了. 解决办法
是在每份遗嘱上表明日期, 并表明前一份遗嘱的日期, 这样问题就全解决了.

选择答案, 我的方法是读问题, 看文章, 然后 1. 首先要自己想想答案可能是什么. 2. 想想答


案不可能是什么. 3. 看选项 4. 迅速排除无关选项, 包括有新名词, 第三者等. 4. 排除 out of
scope 选项. 5. 找到可能的正确答案, 验证, 如 assumption 和 infer 的取非.

我认为多多练习是必要的, 在做题的过程中找到 ETS 的思路. 我在这次考试中能发现很明显


的干扰项, 如果我没有理解 ETS, 很可能会选错.

20. Some flowering plant species entirely dependent on bees for pollination, lure their
pollinators with abundant nectar and pollen, which are the only source of food for bees.
Often the pollinating species is so highly adapted that it can feed from-and thus pollinate-
only a single species of plant. Similarly, some plant species have evolved flowers that
only a single species of bee can pollinate-an arrangement that places the plant species
at great risk of extinction. If careless applications of pesticides destroy the pollinating bee
species, the plant species itself can no longer reproduce.

217
The information above, if true, most strongly supports which one of the following?

(A) The earliest species of flowering plants appeared on Earth contemporaneously with
the earliest bee species.

(B) If the sole pollinator of a certain plant species is in no danger of extinction, the plant
species it pollinates is also unlikely to become extinct.

(C) Some bees are able to gather pollen and nectar from any species of plant.

(D) The blossoms of most species of bees and do not attract others.

(E) The total destruction of the habitat of some plant species could cause some bee
species to become extinct.

答案:E

E 从原文找逻辑关系. 某些花和 pollinator 是一对一的关系. 还说 nector 和 pollen 是 only


source of food for bees. 所以 E 中的 some...could 就对了. 其它答案都不能从原文推出来,
不知道你所谓的混淆答案是哪个.

21. The proper way to plan a scientific project is first to decide its goal and then to plan
the best way to accomplish that goal. The United States space station project does not
conform to this ideal. When the Cold War ended, the project lost its original purpose, so
another purpose was quickly grafted onto the project that of conducting limited-gravity
experiments, even though such experiments can be done in an alternative way. It is,
therefore, abundantly clear that the space station should not be built.
The reasoning in the argument is flawed because the argument
(A) attacks the proponents of a claim rather than arguing against the claim itself.
(B) Presupposes what it sets out to prove.
(C) Faults planners for not foreseeing a certain event, when in fact that event was not
foreseeable.
(D) Contains statements that lead to a self-contradiction.
(E) Concludes that a shortcoming is fatal having produced evidence only of the existence
of that shortcoming.

C is not right. Read the argument carefully and see if the writer ever blame the planners
for not being able to forecast the end of cold war? Think within the frame of the
argument, never go out of it.

C 不是答案, 因为原文并没有指责 planner failed to predict the end of Cold War. 原文的逻
辑是:做一个 project, 要先找出 goal, 然后再找到实现 goal 的最佳途径. 太空站用于研究失重

218
不是经过这个 planning process,所以太空站不应该建立 (隐含意思:太空站不是最好的途径).
所以 E 是答案, 因为存在一个弱点, 就说这个弱点是致命的. 举一个简单的例子说明:正常的
任命过程是由 500 名代表选举出最合格的人选 , 现在由 president 任命了 A, 所以不应该选
A(意思是 A 不是最合格的). 逻辑上的错误就是由一个缺陷否定了大前提.

22. Only an expert in some branch of psychology could understand why Patrick is
behaving irrationally. But no expert is certain of being able to solve someone else’s
problem. Patrick wants to devise a solution to his own behavioral problem.
Which one of the following conclusions can be validly drawn from the passage?
(A) Patrick does not understand why he is behaving in this way.
(B) Patrick is not an expert in psychology.
(C) Patrick is not certain of being able to devise a solution to his own behavioral problem.

(D) Unless Charles is an expert in some branch of psychology, Charles should not offer a
solution to Patrick’s behavioral problem.
(E) If Charles is certain of being able to solve Patrick’s behavioral problem, then Charles
does not understand why Patrick is behaving in this way.

E 中的 Patrick’s behavioral 改为 someone else 就对了。现在感觉也不对。


D 为什么错?should not 改变为 could not 的话,D 也是可选答案?

Explained before. You probably would not find this type of CR in GMAT. It is sheer logic.
And you cannot reply on "sense" or "instinct" to solve this type. You need to sort out the
line of reasoning.
"understand why Patrick is behaving irrationally"(A) --> "an expert in some branch of
psychology" (B)
"certain of being able to solve someone else’s problem" (C) --> not an expert (Not B)
Patrick wants to (not certain of) C.

Now you use the reasoning to find the answer. BTW, you mentioned changing "Patrick's"
to "someone else's". That is not acceptable. "someone else" is not mentioned in the
argument. You can not add your own assumtpion to the argument. It is out of scope.

E 还是逻辑关系和充分必要条件 . 原文中的逻辑关系有 : understand behavior (A)--> an


expert (B); certain of soving problem (C)--> no expert ( 非 B). E 选项: Charles certain of
solving (C) --> not understand behavior (非 A) 符合原文逻辑推理: C-->非 B-->非 A

23. Throughout European history famines have generally been followed by period of
rising wages, because when a labor force is diminished, workers are more valuable in
accordance with the law of supply and demand. The Irish potato famine of the 1840s is
an exception; it resulted in the death or emigration of half of Ireland’s population, but
there was no significant rise in the average wages in Ireland in the following decade.

219
Which one of the following, if true, would LEAST contribute to an explanation of the
exception to the generalization?
(A) Improved medical care reduced the mortality rate among able-bodied adults in the
decade following the famine to below prefamine levels.
(B) Eviction policies of the landowners in Ireland were designed to force emigration of the
elderly and infirm, who could not work, and to retain a high percentage of able-bodied
workers.
(C) Advances in technology increased the efficiency of industry and agriculture, and so
allowed maintenance of economic output with less demand for labor.
(D) The birth rate increased during the decade following the famine, and this
compensated for much of the loss of population that was due to the famine.
(E) England, which had political control of Ireland, legislated artificially low wages to
provide English-owned industry and agriculture in Ireland with cheap labor.

答案是 D,但是我怎莫觉得是 B 呢? D 中的事实明明可以解释 the exception 呀?

出生率的提高跟劳动力的供给没有关系,刚出生的人等二十年以后吧,才会成劳动力呢。

b 的意思是解释了总人口的减少是由于驱逐了不能工作的人,所以虽然总人口下降了,劳
动力市场上的供给没有什么变化。

24. When the rate of inflation exceeds the rate of return on the most profitable investment
available, the difference between those two rates will be the percentage by which, at a
minimum, the value of any investment will decline. If in such a circumstance the value of
a particular investment declines by more than that percentage. It must be true
that________

Which one of the following logically completes the argument?

(A) The rate of inflation has risen.

(B) The investment in question is becoming less profitable.

(C) The investment in question is less profitable than the most profitable investment
available.

(D) The rate of return on the most profitable investment available has declined.

(E) There has been a change in which particular investment happens to be the most
profitable available.
-----------------------
答案 C,(C 不是在说废话吗?investment 和 the most profitable investment 比)
提干中说的关系不太明白,谁和谁比,什么百分比?

220
the difference between those two rates will be the percentage by which, at a minimum,
the value of any investment will decline.该怎样理解? 请大家指教.

文中讲,这个 pecentage 最少也是通涨率超过最赚钱的投资的回报率的部分,如果一个投


资的回报率少于最赚钱的回报率,则它的回报率和通涨率之间的差距一定会大于那个
PECENTAGE 所以跌价的幅度就更大。

LSAT-12-2

4。原文讨论的是计算机网络的立法要以什麽为版本。原文说立法版本必须限于 telephone
system 和 legislation regulating a public broadcasting service。每种立法版本各有特点。但
(YET)计算机网络必须具备这两种立法版本的特点,不能只具备其中一种立法版本的特
点。其实 B 就是原文结论 THUS 后那句话的另一种说法。PAST LEGISTRATION 就是指原文
的那两种立法版本。注意原文第一句话提到。

221
lsat 12-7
Every adult male woolly monkey is larger than even the largest female woolly monkey. In
colonies of woolly monkeys, any adult male will dominate any female.
If the statements above are true, which one of the following must on the basis of them be
true of woolly monkeys in colonies?
(A) Size is the primary determinant of relations of dominance among woolly monkeys.
(B) Some large adolescent male woolly monkeys dominate some smaller females of the
species.
(C) If a male woolly monkey is larger than a female of the species, that male will
dominate that female.
(D) If a female woolly monkey dominates a male of the species. the dominated male
monkey is not an adult.
(E) An adult male woolly monkey can dominate a female of the species only if that female
is also an adult.
答案是:D,为什么不是 A?

It just cannot be A. Typically A is what you conclude based on your assumption and D is
absolutely correct. Remember, CR is to test your logic reasoning, not how you can rely
on the evidence and your own assumption to reach a conclusion.

In this one, it is like saying: X, Y happens at the same time. Your conclusion (A) is: X
causes B. The conclusion is unwarranted. Something else can cause Y, or as a matter of
fact Y causes X. You never know for sure.

What is definite is that if A --> B, then not B --> not A. Basic logic reasoning tested in
GMAT.

12-2-11
The labeling of otherwise high-calorie foods as "sugar-free," based on the replacement of
all sugar by artificial sweeteners, should be prohibited by law. Such a prohibition is
indicated because many consumers who need to lose weight will interpret the label
"sugar-free" as synonymous with "low in calories" and harm themselves by building
weight-loss diets around foods labeled "sugar-free." Manufacturers of sugar-free foods
are well aware of this tendency on the part of consumers.

11. Which one of the following, if true, provides the strongest basis for challenging the
conclusion in the passage?

(A) Food manufacturers would respond to a ban on the label "sugar-free' by reducing the
calories in sugar-free products by enough to be able to promote those products as diet
foods.

222
(B) Individuals who are diabetic need to be able to identify products that contain no sugar
by reference to product labels that expressly state that the product contains no sugar.

(C) Consumers are sometimes slow to notice changes in product labels unless those
changes are themselves well advertised.

(D) Consumers who have chosen a particular weight-loss diet tend to persist with this
diet if they have been warned not to expect very quick results.

(E) Exactly what appears on a product label is less important to consumer behavior than
is the relative visual prominence of the different pieces of information that the label
contains.

原文逻辑:这种做法(将高能量外的食物标作 SUGAR-FREE)会造成误解,引起伤害,
所以不好,应禁止。B 说对糖尿病的人就不会误解,所以 WEAKEN 结论。A 为无关项,原文
举例的目的是说明这种标法引起误解而导致伤害,从而需要禁止,而不是这种产品造成伤
害而要禁止。

Conclusion: The labeling of otherwise high-calorie foods as "sugar-free," based on the


replacement of all sugar by artificial sweeteners, should be prohibited by law.

Facts: many consumers who need to lose weight will interpret the label "sugar-free" as
synonymous with "low in calories" and harm themselves by building weight-loss diets
around foods labeled "sugar-free." Manufacturers of sugar-free foods are well aware of
this tendency on the part of consumers.

this is actually asking you to weaken the conclusion


B-- if individual who is diabetic, it is possible to tell whether the products are "low in
calories" or "sugar-free" and also they know what is the difference between them.
therefore, the prohibition law is not necessary because those manufacturers cannot fool
their customers by saying "sugar-free" as to imply "low calories".

12. In the Centerville Botanical Gardens, all tulip trees are older than any maples. A
majority, but not all, of the garden’s sycamores are older than any of its maples. All the
garden’s maples are older than any of its dogwoods.
If the statements above are true, which one of the following must also be true of trees in
the Centerville Botanical Gardens?

223
(A) Some dogwoods are as old as the youngest tulip trees.
(B) Some dogwoods are as old as the youngest sycamores.
(C) Some sycamores are not as old as the oldest dogwoods.
(D) Some tulip trees are not as old as the oldest sycamores.(E)
(E) Some sycamores are not as old as the youngest tulip trees
All Tulip trees >any maple trees > any dogwoods
Majority Sycamores trees > any maple trees > any dogwoods
是怎么得出 E 的?

12 题,本来画图最清楚。这里无法画图。

原文有两个大小关系:T>M。多数 S>M>D。注意不包括等于。C 的情况是可能,不是 MUST


BE。而 E 是 MUST BE (必须)。由于所以 T 比 M 老,即最 YONG 的 T 也比最老的 M 老。
而肯定有一些 S 比最老的 M 还 YONG(majority)。所以肯定(must be)有一些 S 比最
YONG 的 T 年轻。即 E

13. Emissions from automobiles that burn gasoline and automobiles that burn diesel fuel
are threatening the quality of life on our planet, contaminating both urban air and global
atmosphere. Therefore, the only effective way to reduce such emissions is to replace the
conventional diesel fuel and gasoline used in automobiles with cleaner-burning fuels,
such as methanol, that create fewer emissions.
Which one of the following is an assumption on which the argument depends?
(A) Reducing the use of automobiles would not be a more effective means to reduce
automobile emissions than the use of methanol.
(B) There is no fuel other than methanol that is cleaner-burning than both diesel fuel and
gasoline.
(C) If given a choice of automobile fuels, automobile owners would not select gasoline
over methanol.
(D) Automobile emissions constitute the most serious threat to the global environment.
(E) At any given time there is a direct correlation between the level of urban air pollution
and the level of contamination present in the global atmosphere.
答案 A, 做题时感觉应该是它, 可是注意到原文划线部分是本文结论部分 ,其中的 the only
effective 觉得在 A 中找不到很好的 assumption 对应, 如果把 only 改成 most, 那么答案 A 我
就能理解了.不知道我这么理解对不对?

第一题我认为 B 不好,因为 in automobiles with cleaner-burning fuels, such as methanol,


that create fewer emissions。这里面 methanol 仅是一个例子。注意用的是 such as.
如果题目是用 only methanol,或是去掉 such as .则 B 对。指出唯一性。

15. A standard problem for computer security is that passwords that have to be typed on
a computer keyboard are comparatively easy for unauthorized users to steal or guess. A

224
new system that relies on recognizing the voices of authorized users apparently avoids
this problem. In a small initial trial, the system never incorrectly accepted someone
seeking access to the computer's data. Clearly, if this result can be repeated in an
operational setting, then there will be a way of giving access to those people who are
entitled to access and to no one else.

The reasoning above is flawed because it

(A) makes a faulty comparison, in that a security system based on voice recognition
would not be expected to suffer from the same problems as one that relied on passwords
entered from a keyboard

(8) bases a general conclusion on a small amount of data

(C) fails to recognize that a security system based on voice recognition could easily have
applications other than computer security

(D) ignores the possibility that the system sometimes denies access to people who are
entitled to access

(E) states its conclusion in a heavily qualified way

答案:D,我觉得 A,B 都可以,D 不明白为何是错误?

个人感觉这种“密码”系统题应该考虑两个方面,评价一个“密码”系统是否完备,要考虑两
个方面的情况:(1) 拒绝冒牌货的情况 (2) 接受正牌货的情况;

这与检测某种病的仪器是一个道理,要关注两方面情况: (1)检测某病 positive 的正确率(2)


检测某病 passive 的正确率(GMAT 真题里有一题)依稀记得 Lsat 题里还有一道讲安检的
题,也同此理。

(D)正是指出了原文推理只关注了 voice system 拒绝冒牌货的情况,但是如果也同样拒


绝正牌货,这个系统你说有用吗?

(A)说两种密码系统不可比。两种系统的方式不可比,但效果是可比的。
(B)说 data is small.其实 Data 是否 small 不是关键,关键在于 data 是否 representative.
如果取样科学,small data 也是可以 representative 的。所以 B 不对。

18 . The format of network television news programs generally allows advocates of a


point of view only 30 seconds to convey their message. Consequently, regular watchers
become accustomed to thinking of issues in terms only of slogans and catch phrases,
and so the expectation of careful discussion of public issues gradually disappears from
their awareness. The format of newspaper stories, on the other hand, leads readers to

225
pursue details of stories headed by the most important facts and so has the opposite
effect on regular readers—that of maintaining the expectation of careful discussion of
public issues. Therefore, in contrast to regular newspaper reading, regular watching of
network television news programs increases the tendency to think of public issues in
oversimplified terms.
18. The argument assumes which one of the following?
(A) Viewers of network television news programs would be interested in seeing
advocates of opposing views present their positions at length.
(B) Since it is not possible to present striking images that would symbolize events for
viewers, and since images hold sway over words in television, television must
oversimplify.
(C) It is not possible for television to present public issues in a way that allows for the
nuanced presentation of diverse views and a good-faith interchange between advocates
of opposing views.
(D) In network television news reports, it is not usual for a reporter to offer additional
factual evidence and background information to develop a story in which opposing views
are presented briefly by their advocates.
(E) Television news reporters introduce more of their own biases into news stories than
do newspaper reporters.
理解:
C 也可以说对,但语气太绝对。对 C 取非并不能保证原结论不成立。因为不可能的反面,只
要有一个反例就可以了。但还是少数,也许还是导致了原理的 tendency。
原题目是说 tendency,并非绝对;所以 D 的 not usual 正好符合。
是这么样理解?

"not possible" is incorrect..the passage is talking about the usualness but not possibility

原文对电视 NEWS REPORT 的结论是建立在与"Newspaper is able to "lead readers to


pursue details of stories headed by the most important facts and so has the opposite
effect on regular readers"这一对比之上的
so the assumption should relate to "details of news stories and facts", rather than
"diverse views" and "interchange between advocates of opposing views"(c)

22. Oil company representative: We spent more money on cleaning the otters affected by
our recent oil spill than has been spent on any previous marine mammal rescue project.
This shows our concern for the environment.

Environmentalist: You have no such concern. Your real concern is evident in your
admission to the press that news photographs of oil-covered otters would be particularly
damaging to your public image, which plays an important role in your level of sales.

The environmentalist's conclusion would be properly drawn if it were true that the

226
(A) oil company cannot have more than one motive for cleaning the otters affected by the
oil spill

(B) otter population in the area of the oil spill could not have survived without the cleaning
project

(C) oil company has always shown a high regard for its profits in choosing its courses of
action

(D) government would have spent the money to clean the otters if the oil company had
not agreed to do it

(E) oil company's efforts toward cleaning the affected otters have been more successful
than have such efforts in previous projects to clean up oil spills

答案:A,我奇怪 A 为何会支持了结论?

假设题

原文推理:
O 论点: 原因(A) ――>spent more money on XX than before

E 论点: O 公司(非 A)
E 证据:原因(B)――>spent more money on XX than before
E 的论证有一个暗含的假设,导致 O 公司做善事的动机只有一个,非此即彼。是 B 就不能
是 A。

如果把(A)取非,O 公司有多种动机做善事,则有 B 因不能说明无 A 因。Weaken 了 E 的论


点:O 公司做善事不是因为 A 因。所以 A 项应该是假设。

(A) oil company can have more than one motive for cleaning the otters affected by the oil
spill 把 NOT 去掉,倒有可能.

以前的一个解释如下:
“题目问的是假设,即必要条件,不是充分条件。如果 A 不成立,环境雪茄的推理不可能正确。C 仅是前提
的改写,未能提供假设。”
A 成立即“石油公司只有一个动机这么做”就可以推出“石油公司清洗 otter 只是为了自己的形象”这个结论
吗?

这种问法“would be properly drawn if it were true that”是问必要条件还是充分?觉得该是充分条件。如果是问


必要条件的话该是“。。。only if… were true…”。

yes. I think it is a sufficient condition..But in this question, it is a type of "SUPPORT" question

227
You have no such concern.
(this is conclusion)

Your real concern is evident in your admission to the press that news photographs of oil-
covered otters would be particularly damaging to your public image, which plays an
important role in your level of sales.
(this is evidence)

we notice that there is a GAP between evidence and conclusion...evidence is saying they
are concering other factor...conclusion is saying that they don't have such concern...

to draw the conclusion safely, we have to add choice A as one addition information....

24. Since Mayor Drabble always repays her political debts as soon as possible, she will
almost certainly appoint Lee to be the new head of the arts commission. Lee has wanted
that job for a long time, and Drabble owes Lee a lot for his support in the last election.

Which one of the following is an assumption on which the argument depends?

(A) Mayor Drabble has no political debt that is both of longer standing than the one she
owes to Lee and could as suitably be repaid by an appointment to be the new head of the
arts commission.

(B) There is no one to whom Mayor Drabble owes a greater political debt for support in
the last election than the political debt she owes to Lee.

(C) Lee is the only person to whom Mayor Drabble owes a political debt who would be
willing to accept an appointment from her as the new head of the arts commission.

(D) Whether Lee is qualified to head the arts commission is irrelevant to Mayor Drabble's
decision.

(E) The only way that Mayor Drabble can adequately repay her political debt to Lee is by
appointing him to head the arts commission.

答案:A,虽然做对了,是蒙的.觉得 A,B,C,E 都可以.

谢谢.

假设题
原文推理如下:
结论:MD repays her political debts as soon as possible――>MD will appoint L to the

228
arts commission
论据:L want to be appointed and MD own political debts to B

原文推理有一个 gap,那就是没有别的人排在 L 前面。如果有一个人 X 与 L 在其它方面一样


且等候提名的时间比 L 还长,按照原则,MD 就应该提句 X 而不是 L。要使结论成立,需要
有一个假设条件:MD 欠 L 的 political debts 时间最长。

(B) In the last election 中,MD 欠 L 的 political debts 最长,不排除在更早一些的 election


中,MD 欠了别人的债没还。

(C) 只有 L 会 accept,与结论中说 MD appoint L 没有关系。

(E) MD 唯一能够充分报答 L 的方式是提名 L。这个假设条件有点过分,如果将 E 取非,MD


还可以用其它方式报答 L,那么 MD 依然有可能提名 L 为 head of the arts commission. 否
定条件 E 不能 weaken 原文结论,则 E 不是假设条件。

25. The fact that tobacco smoke inhaled by smokers harms the smokers does not prove
that the much smaller amount of tobacco smoke inhaled by nonsmokers who share living
space with smokers harms the nonsmokers to some degree. Many substances, such as
vitamin A, are toxic in large quantities but beneficial in small quantities.
In which one of the following is the pattern of reasoning most similar to that in the
argument above?
(A) The fact that a large concentration of bleach will make fabric very white does not
prove that a small concentration of bleach will make fabric somewhat white. The effect of
a small concentration of bleach may be too slight to change the color of the fabric.
(B) Although a healthful diet should include a certain amount of fiber, it does not follow
that a diet that includes large amounts of fiber is more healthful than one that includes
smaller amounts of fiber. Too much fiber can interfere with proper digestion.
(C) The fact that large amounts of chemical fertilizers can kill plants does not prove that
chemical fertilizers are generally harmful to plants. It proves only that the quantity of
chemical fertilizer used should be adjusted according to the needs of the plants and the
nutrients already in the soil.
(D) From the fact that five professional taste testers found a new cereal product tasty, it
does not follow that everyone will like it.
Many people find broccoli a tasty food, but other people have a strong dislike for the taste
of broccoli.
(E) Although watching television for half of every day would be a waste of time, watching
television briefly every day is not necessarily even a small waste of time. After all, it
would be a waste to sleep half of every day, but some sleep every day is necessary.

答案:E

B 哪里错了呢?

229
26. Why should the government, rather than industry or universities, provide the money
to put a network of supercomputers in place? Because there is a range of problems that
can be attacked only with the massive data-managing capacity of a supercomputer
network. No business or university has the resources to purchase by itself enough
machines for a whole network, and no business or university wants to invest in a part of a
network if no mechanism exists for coordinating establishment of the network as a whole.
Which one of the following indicates a weakness in the argument?
(A) It does not furnish a way in which the dilemma concerning the establishment of the
network can be resolved.
(B) It does not establish the impossibility of creating a supercomputer network as an
international network.
(C) It fails to address the question of who would maintain the network if the government,
rather than industry or universities, provides the money for establishing it.
(D) It takes for granted and without justification that it would enhance national
preeminence in science for the government to provide the network.(E)
(E) It overlooks the possibility that businesses or universities, or both, could cooperate to
build the network.
答案为什么是 e 呢?题目最后一句话不是说 and no business or university wants to invest
in a part of a network if no mechanism exists for coordinating establishment of the
network as a whole.不是已经考虑到并排除了合作的可能性了吗?

原文说的是没说协调机制存在(这机制是由政府建立),E 说的是商业和大学自己合作。

LSAT-12-4

230
10. Lydia: Each year, thousands of seabirds are injured when they become entangled in
equipment owned by fishing companies. Therefore, the fishing companies should
assume responsibility for funding veterinary treatment for the injured birds.

Jonathan: Your feelings for the birds are admirable. Your proposal, however, should not
be adopted because treatment of the most seriously injured birds would inhumanely
prolong the lives of animals no longer able to live in the wild, as all wildlife should.

Jonathan uses which one of the following techniques in his response to Lydia?

(A) He directs a personal attack against her rather than addressing the argument she
advances.

(B) He suggests that her proposal is based on self-interest rather than on real sympathy
for the injured birds.

(C) He questions the appropriateness of interfering with wildlife in any way, even if the
goal of the interference is to help.

(D) He attempts to discredit her proposal by discussing its implications for only those
birds that it serves least well.

(E) He evades discussion of her proposal by raising the issue of whether her feelings
about the birds are justified.

答案:D,我选了 C,C 为何不对呢?

M: You need to understand the passage, instead of using your own reasoning. J never
question the interference with wildlife. From his argument you can see that his point is
that treated bird will not live in the wild any more and thus the treatment is inhumane. In
another word, if the treatment can make the birds live in the wild, it might be humane and
adopted. So interference is not the problem here.

You really need to understand the meaning of premises and conclusion correctly before
you do the problem

11. Logging industry official: Harvesting trees from old-growth forests for use in
manufacture can reduce the amount of carbon dioxide in the atmosphere, since when
large old trees die in the forest they decompose., releasing their stored carbon dioxide.
Harvesting old-growth forests would, moreover, make room for rapidly growing young
trees, which absorb more carbon dioxide from the atmosphere than do trees in old-
growth forests.

231
Which one of the following, if true, most seriously weakens the official's argument?

(A) Many old-growth forests are the home of thousands of animal species that would be
endangered if the forests were to be destroyed.

(B) Much of the organic matter from old-growth trees, unusable as lumber, is made into
products that decompose rapidly.

(C) A young tree contains less than half the amount of carbon dioxide that is stored in an
old tree of the same species.

(D) Much of the carbon dioxide present in forests is eventually released when wood and
other organic debris found on the forest floor decompose.

(E) It can take many years for the trees of a newly planted forest to reach the size of
those found in existing old-growth forests.

答案:B,我选了 D,我想了半天,也不明白为什么发 B 是削弱?

Discussed before. In short, the conclusion is that the lumber from old-growth wood will
stop it from decomposing and releasing CD to the atmosphere. B states that it is not true,
and it will still decompose and release CD.

D is not right. It somehow supports the argument. If all the woods on the floor will
decompose and release CD, those old-growth woods should be harvested and used,
instead of be left to die and decompose.

A,E 显然是无关选项,因为此题讨论的主要是吸入二氧化碳,以及树死后进行分解产生二氧化碳对自然有害
的问题.
这道题要从二氧化碳,以及分解入手,文中 OFFICIAL 宣称砍老树可以减少空气中的二氧化碳,我们就应该从
砍掉老树不能够减少空气中的二氧化碳入手 ,B 选项恰恰说明了这一点:老树不仅 unusable,而且被制成分解
迅速的产品,因此迅速的分解仍然对空气有害

15. A certain experimental fungicide causes no harm to garden plants, though only if it is
diluted at least to ten parts water to one part fungicide. Moreover, this fungicide is known
to be so effective against powdery mildew that it has the capacity to eliminate it
completely from rose plants. Thus this fungicide, as long as it is sufficiently diluted,
provides a means of eliminating powdery mildew from rose plants that involves no risk of
harming the plants.

232
Which one of the following is an assumption on which the argument depends?

(A) There is not an alternative method, besides application of this fungicide. for
eliminating powdery mildew from rose plants without harming the plants.

(B) When the fungicide is sufficiently diluted. it does not present any risk of harm to
people, animals. or beneficial insects.

(C) Powdery mildew is the only fungal infection that affects rose plants.

(D) If a fungicide is to be effective against powdery mildew on rose plants, it must


eliminate the powdery mildew completely.

(E) The effectiveness of the fungicide does not depend on its being more concentrated
than one part in ten parts of water.

答案:E,虽然我猜对了答案,但不明白为什么 E 是假设。

Premises:
a. 10:1 no harm;
b. effective in eliminating PM, no mention of concentration
conclusion: no harm and effective at the same time. So assumtion is effective when 10:1.

If effective only 3:1, then it can not be harmless and effective.

Teacher: Journalists who conceal the identity of the sources they quote stake their
professional reputations on what may be called the logic of anecdotes. This is so
because the statements reported by such journalists are dissociated from the precise
circumstances in which they were made and thus will be accepted for publication only if
the statements are high in plausibility or originality or interest to a given audience—
precisely the properties of a good anecdote.

Student: But what you are saying, then, is that the journalist need not bother with sources
in the first place. Surely, any reasonably resourceful journalist can invent plausible,
original, or interesting stories faster than they can be obtained from unidentified sources.

17. The student’s response contains which one of the following reasoning flaws?
(A) confusing a marginal journalistic practice with the primary work done by journalists
(B) ignoring the possibility that the teacher regards as a prerequisite for the publication of
an unattributed statement that the statement have actually been made(选项意思没看懂)
(C) confusing the characteristics of reported statements with the characteristics of the

233
situations in which the statements were made
(D) judging the merits of the teacher’s position solely by the most extreme case to which
the position applies
(E) falsely concluding that if three criteria, met jointly, assure an outcome, then each
criterion, met individually, also assures that outcome
答案 B, 还请大家帮忙, 题目的 teacher 和 student 的逻辑关系我看懂了, 可是 B 的意思没看

懂, B 中的 ignore the possibility...对应的是文中的什么地方啊? 如果问题过于愚蠢,还请

大家多包涵!

teacher:说 only if the statements are high in plausibility or originality or interest to a given audience—precisely
the properties of a good anecdote。注意这是个必要条件。但学生理解成充要条件了。 this is so because the
statements reported by such journalists are dissociated from the precise circumstances in which they were made 。
这句话回答了答案 B 中的可能性。
B 的意思是学生忽视了一种可能,(老师认为报道已经真是发生的是一个没有依附的报道发表的前提)。
不知我解释的是否正确。

Lawyer 以前的翻译是在报道关于 anecdotes 的逻辑时,记者没说明引用的来源这种做法是对他


们的职业信誉冒险。因为它和其背景隔离开来,所以只有在满足 3 个条件之一时才会被接受:
high in plausibility or originality or interest to a given audience。

对题目基本理解了,但 B 正确的原因是不是把必要条件和充分条件搞错了?如何理解呢?

的确可以从"必要条件和充分条件"这个角度进行考虑

the teacher takes "high in plausibility or originality or interest to a given


audience" as three necessary conditions, whereas the student mistakens
these three conditions for sufficient ones.

B actually presents another necessary condition.

You have a very good point

18. Which one of the following, if true, most strengthens the teacher’s argument?

234
(A) A journalist undermines his or her own professional standing by submitting for
publication statements that, not being attributed to a named source, are rejected for
being implausible, unoriginal, or dull.
(B) Statements that are attributed to a fully identified source make up the majority of
reported statements included by journalists in stories submitted for publication.
(C) Reported statements that are highly original will often seem implausible unless
submitted by a journalist who is known for solid, reliable work.
(D) Reputable journalists sometimes do not conceal the identity of their sources from
their publishers but insist that the identity of those sources be concealed from the public.
(A)
(E) Journalists who have special access to sources whose identity they must conceal are
greatly valued by their publishers.
原文有点没读懂,能不能麻烦讲一讲,谢谢!!!

在报道关于 anecdotes 的逻辑时,记者没说明引用的来源这种做法是对他们的职业信誉冒


险。因为它和其背景隔离开来,所以 只 有 在满 足 3 个条 件 之 一时 才 会 被接 受 : high in
plausibility or originality or interest to a given audience。(结论)。A 其实是举个例。
所以支持

19. The proposal to extend clinical trials, which are routinely used as systematic tests of
pharmaceutical innovations, to new surgical procedures should not be implemented. The
point is that surgical procedures differ in one important respect from medicinal drugs: a
correctly prescribed drug depends for its effectiveness only on the drug's composition,
whereas the effectiveness of even the most appropriate surgical procedure is
transparently related to the skills of the surgeon who uses it.

The reasoning in the argument is flawed because the argument

(A) does not consider that new surgical procedures might be found to be intrinsically
more harmful than the best treatment previously available

(B) ignores the possibility that the challenged proposal is deliberately crude in a way
designed to elicit criticism to be used in relining the proposal

(C) assumes that a surgeon's skills remain unchanged throughout the surgeon's
professional life

(D) describes a dissimilarity without citing any scientific evidence for the existence of that
dissimilarity

(E) rejects a proposal presumably advanced in good faith without acknowledging any
such good faith .

235
答案:A,我选了 C,不明白 A 为何是错误 

A points out that the clinical trial is to identify the nature and possible risk involved in the procedure. C is not
mentioned or implied.

T2 argue that due to the dependce on skills of different surgeon, surgical procedure should not rely on surgical
trial.

However, the arguer ignore the fact that an immature surgical procedure lack of the trial , as a rule, will be more
harmful than a mature sugrical with full trial and efficiecy proof. That is the meaning of answer A

21. When the supply of a given resource dwindles, alternative technologies allowing the
use of different resources develop, and demand for the resource that was in short supply
naturally declines. Then the existing supplies of that resource satisfy whatever demand
remains. Among the once-dwindling resources that are now in more than adequate
supply are flint for arrowheads, trees usable for schooner masts, and good mules.
Because new technologies constantly replace old ones, we can never run out of
important natural resources.

Which one of the following, if true, most seriously undermines the conclusion?

(A) The masts and hulls of some sailing ships built today are still made of wood.

(B) There are considerably fewer mules today than there were 100 years ago.

(C) The cost of some new technologies is often so high that the companies developing
them might actually lose money at first.

(D) Dwindling supplies of a natural resource often result in that resource's costing more
to use.

(E) The biological requirements for substances like clean air and clean water are
unaffected by technological change.

答案:E,我选了 C,为什么 E 是削弱呢?

M: My suggestion to you is not to do more exercise, but think more and understand the
problem. After reading your recent posts, I think you are still making the same mistakes
as you did before. In another word, you have not learned from your past. It sounds cruel
but you must realize it.

For example, 1T is a typical GMAT CR. Your mistake is also typical in problems we

236
discussed before. You need to know what is the premise and what is the conclusion.

P: If technology make demand for natural resources in short supply dwindle


C: we will not run out of NR

Cost is not a consideration here, unless it is a prohibitive factor.

Let me give you another example: Solar power technology is developed, we will rely on
crude oil less and less. Here cost is not a factor. No matter it is expensive or cheap to
use solar power, the reasoning holds. But if I argue that solar energy is prohibitively
expensive and will be so for a long time, then the reasoning is flawed, as the cost renders
the technology useless for a long time.

BTW, if the reasoning is that: "if we use solar energy soly, we do not need crude oil", then
cost and any other factor that make solar energy unusable cannot weaken the argument.
The reason is that the argument assumes that solar energy is usable. So it is irrelevant if
it is usable in reality.

23. A study of adults who suffer from migraine headaches revealed that a significant
proportion of the study participants suffer from a complex syndrome characterized by a
set of three symptoms. Those who suffer from the syndrome experienced excessive
anxiety during early childhood. As adolescents, these people began experiencing
migraine headaches. As these people approached the age of 20, they also began to
experience recurring bouts of depression. Since this pattern is invariant always with
excessive anxiety at its beginning, it follows .hat excessive anxiety in childhood is one of
the causes of migraine headaches and depression in later life.

The reasoning m the argument is vulnerable to criticism on which one of the following
grounds'?

(A) It does not specify the proportion of those in the general population who suffer from
the syndrome.

(B) It fails to rule out the possibility that all of the characteristic symptoms of the
syndrome have a common cause.

(C) It makes a generalization that is inconsistent with the evidence.

(D) it fails to demonstrate that the people who participated in the study are representative
of migraine sufferers.

(E) It does not establish why the study of migraine sufferers was restricted to adult

237
participants.

答案:B,我选了 D

M: The study is not on migraine. And the conclusion has nothing to do with whether these
people are typical migraine sufferers or not. In another word, they do not have to
represent migraine sufferers.

For example, people who hated study when young hated word when older, then hated life
when even older. So the conclusion is that hate-->hate-->hate. It does not matter if there
people who hated work represent all the people who hated work. It is irrelevant.

B is typical answer. You think about it...

24. Mainstream economic theory holds that manufacturers. in deciding what kinds of
products to manufacture and what form those products should have, simply respond to
the needs and desires of consumers. However, most major manufacturers manipulate
and even create consumer demand. as anyone who watches television knows. Since
even mainstream economic theorists watch television, their motive in advancing this
theory must be something other than disinterested concern for scientific truth.

The claim that manufacturers manipulate and create consumer demand plays which one
of the following roles in the argument?

(A) It is one of the claims on which the conclusion is based.

(B) It is the conclusion of the argument.

(C) It states the position argued against.

(D) It states a possible objection to the argument's conclusion.

(E) It provides supplementary background information.

答案是 A),那么哪一句是结论呢?
我个人认为选 D), 结论是第一句话, 这句话是对结论的反驳

The conclusion is "...their motive in advancing this theory must be something other than disinterested concern for
scientific truth. "

When I read the question, I felt that each sentence was the starting sentence of a paragraph in an RC article, and

238
each expressed the central meaning of the paragraph. The first sentecne gave you the commonly seen statement.
Then different claims were made and evidence were presented (not in this CR though), then the original statement
was countered and new conclusion was drawn.

I think your problem lies in understanding. Without understanding, you cannot crack CR or RC. So read the
sentence and get to understand what it means

the last sentence begins with 'since...' seems to make explanation, more like a cause than a conclusion.
By the way, when I re-read the paragraph, i found maybe the conclusion is not within this paragraph, seems the
author makes 2 claims, the first sentence and the second, the 3rd sentence is just more detailed
description/explanation about the 2nd claim.
How do u think about this? Anyway, THX!

You are right: since A, B. Simply there are a premise and a conclusion in one sentence. So you do not understand a
whole sentence by reading the first word.

The first claim (first sentence) is not by the writer. It is a "mainstream theory", which means it is a commonly held
opinion. As you read on, you at least should realize that the write is trying to counter this opinion.

The last one (conclusion) is based on the second sentence. So it is not a more detailed description. Similar example
showing the same relationship is: only stupid people are watching TV (like second sentence). Since (!) he watches
TV (premise), he is stupid (conclusion)

239
LSAT-13-1

lsat 13 (1.1) questions


Oscar: I have been accused of plagiarizing the work of Ethel Myers in my recent article.
But that accusation is unwarranted. Although I admit I used passages from Myers’s book
without attribution. Myers gave me permission in private correspondence to do so.

Millie: Myers cannot give you permission to plagiarize. Plagiarism is wrong, not only
because it violates author’s rights to their own words, but also because it misleads
readers: it is fundamentally a type of lie. A lie is no less a lie if another person agrees to
the deception.

6. Millie uses which one of the following argumentative strategies in contesting Oscar’s
position?

(A) analyzing plagiarism in a way that undermines Oscar’s position

(B) invoking evidence to show that Oscar did quote Myers’ work without attribution

(C) challenging Oscar’s ability to quote Myers’ work without attribution

(D) citing a theory of rights that prohibits plagiarism and suggesting that Oscar is
committed to that theory

(E) showing that Oscar’s admission demonstrates his lack of credibility

这个题目的 A 和 D 我没有分开。答案是 D

10. Stage performances are judged to be realistic to the degree that actors
reproduce on stage the behaviors generally associated by audiences with the
emotional states of the characters portrayed. Traditional actors imitate those
behaviors, whereas Method actors, through recollection of personal experience, actually
experience the same emotions that their characters are meant to be experiencing.
Audiences will therefore judge the performances of Method actors to be more realistic
than the performances of traditional actors.

Which one of the following is an assumption on which the argument depends?

240
(A) Performances based on an actor’s own experience of emotional states are more
likely to affect an audience’s emotions than are performances based on imitations of the
behaviors generally associated with those emotional states.

(B) The behavior that results when a Method actor fells a certain emotion will conform to
the behavior that is generally associated by audiences with that emotion.

(C) Realism is an essential criterion for evaluating the performances of both traditional
actors and Method actors.

(D) Traditional actors do not aim to produce performances that are realistic
representations of a character’s emotional states.

(E) In order to portray a character, a Method actor need not have had experiences
identical to those of the character portrayed.

这个假设题目相关的选项太多,答案是 B.(1)请详细解释 (2)黑体字部分如何理解呢?

2,提出一点你来考虑,观众既然根据主观上认为这个角色应该在这种情绪下作出的行为来判断是否表演
真实,而体验派的演员因为加入自己的体验而得到观众的认可,则观众根据什么认可的呢?黑体字告诉我
们,观众根据他们认为角色应该作出的反应来判断演员的表演是否和他们思想中的真实一致来决定是不是
真实。可体验派是自己的感受和经验呀?那么一定是他们根据自身的体验而作出的行为和观众认为的真实
一致。

2. 同意 Canberry 的意见. 原文还是有逻辑上的 gap. 观众根据自己对角色在某种情绪下应有的行为来判断是


否 realistic. Method actor 体会角色的情绪作出行为. 结论是观众认为 method actor 更 realistic. 这里 method
actor 的行为和观众认为应有的行为是两个不同的事物, assumption 要将它们联系起来.

A 错在"are more likely to affect an audience’s emotions".这里 audience's emotion 不是判定 realistic 的标准. 可
以说是推理或是原结论的延伸, 做为 assumption 就是 out of scope.无关选项.

C 典型的 out of scope. 完全在原文的逻辑关系之外. 举简单的例子: 鸡吃富含磷和钙的精饲料下蛋多,所以要


想下蛋多, 就要用 A 饲料而不用 B 饲料. 这里的 assumption 就是 A 比 B 富含磷和钙, 而不是下蛋多是衡量鸡
的唯一标准

D.典型错误答案, A 比 B 好, 不能说明 B 不好. 举例: 钻石比蓝宝石值钱, 不能说明蓝宝石不值钱.

E. 无关选项. experience 在这里和 emotion 没有联系起来, 我们不知道有无 experience 对体会 emotion 的影响.
如果建立起来某种联系, 如没有 experience 就没有类似的 emotion, 那么 E 与原文相反.

12. In 1980 health officials began to publicize the adverse effects of prolonged exposure

241
to the sun, and since then the number of people who subathe for extended periods of
time has decreased considerably each year. Nevertheless, in 1982 there was a dramatic
rise in newly reported cases of melanoma, a form of skin cancer found mostly in people
who have had prolonged exposure to the sun.

Which one of the following, if true, helps to resolve the apparent discrepancy in the
information above?

(A) Before 1980 a considerable number of the people who developed melanoma as a
result of prolonged exposure to the sun were over forty years of age.

(B) Before1980, when most people had not yet begun to avoid prolonged exposure to the
sun, sunbathing was widely thought to be healthful.

(C) In 1982 scientists reported that the body’s need for exposure to sunlight in order to
produce vitamin D, which helps prevent the growth of skin cancers, is less than was
previously though.

(D) In 1982 medical researchers perfected a diagnostic technique that allowed them to
detect the presence of melanoma much earlier than had previously been possible.

(E) Since 1980, those people who have continued to sunbathe for extended periods of
time have used sunblocks that effectively screen out the ultraviolet rays that help cause
melanoma.

答案:D

Salmonella is a food-borne microorganism that can cause intestinal illness. The illness is
sometimes fatal, especially if not identified quickly and treated. Conventional Salmonella
tests on food samples are slow and can miss unusual strains of the microorganism. A
new test identifies the presence or absence of Salmonella by the one piece of genetic
material common to all strains. Clearly, public health officials would be well advised to
replace the previous Salmonella tests with the new test.

16. Which one of the following, if true, most substantially weakens the argument?

(A) The new test identifies genetic material from Salmonella organisms only and not from
similar bacteria.

(B) The new test detects the presence of Salmonella at levels that are too low to pose a
health to people.

242
(C) Salmonella is only one of a variety of food-borne microorganism that can cause
intestinal illness.

(D) The new test has been made possible only recently by dramatic advances in
biological science.

(E) Symptoms of Salmonella poisoning are often mistaken for those of other common
intestinal illness.

这个题目我觉得 A 比 B 好,但答案是 B.

A 是非常典型的错误答案. 无关并 out of scope. 原文中提到的用于衡量检测的标准是:是否能测出大多数的 S


细菌, 而不是它是否能检测出其它的细菌. 如果原文提到检测方法应该能检测出大部分 S 细菌和其它的细菌,
A 才为正确答案.

还要注意一点, B 是正确答案是因为原文结论是:"public health officials would be well advised". 如果测出的细


菌菌落数值很低 , 则推出 not well advised. 如果原文的结论改为 : public health official can detect unusual
bacterial with the new method, B 就不是答案.

18. Frieda: Lightning causes fires and damages electronic equipment. Since lightning
rods can prevent any major damage, every building should have one.

Erik: Your recommendation is pointless. It is true that lightning occasionally causes fires,
but faulty wiring and overloaded circuits cause far more fires and damage to equipment
than lightning does.

Erik’s response fails to establish that Frieda’s recommendation should not be acted on
because his response

(A) does not show that the benefits that would follow from Frieda’s recommendation
would be

offset by any disadvantage

(B) does not offer any additional way of lessening the risk associated with lightning

(C) appeals to Frieda’s emotions rather than to her reason

(D) introduces an irrelevant comparison between overloaded circuits and faulty wiring

(E) confuses the notion of preventing damage with that of causing inconvenience

243
答案:A,我选了 D

I used POE to pick A. The question is similar to "because it fails to...". I think that there can be many right answers
for this type of question. For example, the answer can also be "becasue other causes that inflict more damages does
not justify not preventing lightening damages".

So I suggest that you use POE for this type of problems. D is wrong. Where is the comparison between the two?
Joy, you cannot pick a choice because it looks "like" an answer because it mentions the some of the subjects in the
argument. Actually, most of the time these kinds of choices are misleading and therefore wrong. You need to
understand what the answer means.

The reasoning of the original argument is:


A: X causes damages and Y can prevent X. So Y should be used.
B: But Y causes more damages than X. So A is wrong.

You can see that B's argument is lrrelevant. And the answer A points out one of the problems of the argument.

BTW, POE is one of the best ways for CR.

22. At the end of the year, Wilson’s Department Store awards free merchandise to its top salespeople. When
presented with the fact that the number of salespeople receiving these awards has declined markedly over the past
fifteen years, the newly appointed president of the company responded, “In that case, since our award criterion at
present is membership in the top third of our sales force, we can also say that the number of salespeople passed
over for these awards has similarly declined.”

Which one of the following is an regard to hiring salespeople have not become more lax over the past fifteen years.

(A) Policies at Wilson’s with regard to hiring salespeople have not become more lax over the past fifteen years.

(B) The number of salespeople at Wilson’s has increased over the past fifteen years.

(C) The criterion used by Wilson’s for selecting its award recipients has remained the same for the past fifteen
years.

(D) The average total sales figures for Wilson’s salespeople have been declining for fifteen years.

(E) Wilson’s calculates its salespeople’s sales figures in the same way as it did fifteen years ago.

这种题目是什么类型的题目??

244
24. Until recently it was thought that ink used before the sixteenth century did not contain
titanium. However, a new type of analysis detected titanium in the ink of the famous Bible
printed by Johannes Gutenberg and in that of another fifteenth-century Bible known as B-
36, though not in the ink of any of numerous other fifteenth-century books analyzed. This
finding is of great significance, since it not only strongly supports the hypothesis that B-36
was printed by Gutenberg but also shows that the presence of titanium in the ink of the
purportedly fifteenth century Vinland Map can no longer be regarded as a reason for
doubting the map’s authenticity.

The reasoning in the passage is vulnerable to criticism on the ground that

(A) the results of the analysis are interpreted as indicating that the use of titanium as an
ingredient in fifteenth-century ink both was, and was not, extremely restricted

(B) if the technology that makes it possible to detect titanium in printing ink has only
recently become available, it is unlikely that printers ore artists in the fifteenth century
would know whether their ink contained titanium or not

(C) it is unreasonable to suppose that determination of the date and location of a


document’s printing or drawing can be made solely on the basis of the presence or
absence of a single element in the ink used in the document.

(D) both the B-36 Bible and the Binland Map are objects that can be appreciated on their
own merits whether or not the precise date of their creation or the identity of the person
who made them is known.

(E) the discovery of titanium in the ink of the Vinland Map must have occurred before
titanium was discovered in the ink of the Gutenberg Bible and the B-36 Bible .

答案:A,我选了 E,胡做呢。

A indicates the contradictary reasoning of the argument. Because there is Titanium in the ink of a
book by J.G., the author argues that the other bible whose ink also contains titanium should be
written by J.G. too. From this we can reason that the use of titanium ink must be extremely
restricted. In another word, only this guy had access to this type of ink. Then the author argues that
the titanium ink in the Map does not pose doubt on its dating of 15th century. In another word, his
reasoning is that titanium ink should be available during that period.

25. All actors are exuberant people and all exuberant people are extroverts, but
nevertheless it is true that some shy people are actors.
If the statements above are true, each of the following must also be true EXCEPT:
(A) Some shy people are extroverts.

245
(B) Some shy extroverts are not actors.
(C) Some exuberant people who are actors are shy.
(D) All people who are not extroverts are not actors.
(E) Some extroverts are shy.
key - B

LSAT-13-4

2. For the past 13 years, high school guidance counselors nationwide have implemented
an aggressive program to convince high school students to select careers requiring
college degrees. The government reported that the percentage of last year’s high school
graduates who went on to college was 15 percent greater than the percentage of those
who graduated 10 years ago and did so. The counselors concluded from this report that
the program had been successful.

The guidance counselors’ reasoning depends on which one of the following assumptions
about high school graduates?

(A) The number of graduates who went on to college remained constant each year during
the 10-year period.

(B) Any college courses that the graduates take will improve their career prospects.

(C) Some of the graduates who went on to college never received guidance from a high
school counselor.

(D) There has been a decrease in the number of graduates who go on to college without
career plans.

(E) Many of last year’s graduates who went on to college did so in order to prepare for
careers requiring college degrees.

这个题目 a/e 我没有分清楚,答案是 E.

只就 AE 吧。A 说 10 年来上大学的人数没变,结合文章,这个表述不能说明任何问题,比如毕业的人数增
加,上大学的人数没变,说明比例减少了呢。E 是说去年许多上大学的人正是为了给将来的职业打基础。而
文中的 gap(MINDFREE 偶剽窃你的说法了)就存在于,上大学的人数和选择需要大学学历的职业之间的关
系,既然许多人是为了职业选择了大学,说明校方的方法有效。
M: 同意 canberry 的意见. 原文将上大学比例的增加归功于这个 program. E 就是将这两个现象联系起来.举个
例子:有个 program 教育人们多吃菜,少吃肉, 因为吃肉不能去西方极了世界. 在实施了这个 program 后, 吃肉

246
的人少了, 所以说这个 program 是成功的. assumption 就是:那些改不吃肉的人是为了去 XFJLSJ. 其它答案也
可以是:肉都是疯牛疯猪; 吃肉的 chlesterol 高, 一个 program 教育人们不要吃肉以降低 C, 避免心脏病; 等等

3. Insectivorous plants, which unlike other plants have the ability to trap and digest
insects, can thrive in soils that are too poor in minerals to support noninsectivorous
plants. Yet the mineral requirements of insectivorous plants are not noticeably different
from the mineral requirements of noninsectivorous plants.
The statements above, if true, most strongly support which one of the following
hypotheses?
(A) The insects that insectivorous plants trap and digest are especially abundant where
the soil is poor in minerals.
(B) Insectivorous plants thrive only in soils that are too poor in minerals to support
noninsectivorous plants.
(C) The types of minerals required by noninsectivorous plants are more likely than are
the types of minerals required by insectivorous plants to be found in soils poor in
minerals.
(D) The number of different environments in which insectivorous plants thrive is greater
than the number of different environments in which noninsectivorous plants thrive.
(E) Insectivorous plants can get some of the minerals they require from the insects they
trap and digest.

食虫植物的矿物质需求=不食虫植物的需求
这里的不等式明显不对等,我们需要找能够使<变成=的条件。
选项 E 提出了 insect 的养分可以补充给植物的问题。就是食虫植物从 insect 获得的养分肯定>不食虫植物从
insect 获得的养分。
所以这个>弥补了<的不足。从而得出=的结论。

6. Chronic fatigue syndrome is characterized by prolonged fatigue, muscular pain, and


neurological problems. It is not know whether these symptoms are all caused by a single
virus or whether each symptom is the result of a separate viral infection. A newly
synthesized drug has been tested on those who suffer from chronic fatigue syndrome.
Although the specific antiviral effects of this drug are unknown. It has lessened the
severity of all of the symptoms of chronic fatigue syndrome. Thus there is evidence that
chronic fatigue syndrome is, in fact, caused by one virus.

The argument assumes which one of following?

(A) All those who suffer from prolonged fatigue also suffer from neurological problems.

(B) It is more likely that the new drug counteracts one virus than that it counteracts
several viruses.

247
(C) The symptoms of chronic fatigue syndrome are dissimilar to those of any other
syndrome.

(D) Most syndromes that are characterized by related symptoms are each caused by a
single viral infection.

(E) An antiviral medication that eliminates the most severe symptoms of chronic fatigue
syndrome thereby cures chronic fatigue syndrome.

有是一个假设,我选的是 C 答案是 B.

B 是正确答案. 原文的逻辑关系是:因为一种药可以缓解所有的症状, 所以这些症状是一种病毒造成的. 这里


一种药和一种病毒就是 assumption. 我不知道你选 C 的原因, 因为 C 是 out of scope, 其它综合症的症状在此
题中没有讨论.

8. A history book written hundreds of years ago contains several inconsistencies. Some
scholars argue that because the book contains inconsistencies, the author must have
been getting information from more than one source.

The conclusion cited does not follow unless


(A) author generally try to reconcile discrepancies between sources
(B) the inconsistencies would be apparent to the average reader of the history book at
the present time
(C) the history book’s author used no source that contained inconsistencies repeated in
the history book
(D) the author of the history book was aware of the kinds of inconsistencies that can arise
when multiple sources are consulted
(E) the author of the history book was familiar with all of the available source material that
was relevant to the history book .

答案 C。

c eliminates another explanation, that is, the inconsistence resulted from the author of the
history book used a souce that contains incosistence.

so even if the souce is the only source the author have used, there may be inconsistence
exist in his or her history book. That contradict the conclusion cited by these scholors.

248
Household indebtedness, which some theorists regard as causing recession, was high
preceding the recent recession, but so was the value of assets owned by households.
Admittedly, if most of the assets were owned by quite affluent households, and most of
the debt was owed by low-income households, high household debt levels could have
been the cause of the recession despite high asset values: low-income households might
have decreased spending in order to pay off debts while the quite affluent ones might
simply have failed to increase spending. But, in fact, quite affluent people must have
owed most of the household debt, since money is not lent to those without assets.
Therefore, the real cause must lie elsewhere.

10. The argument is structured to lead to which one of the following conclusions?
(A) High levels of household debt did not cause the recent recession.
(B) Low-income households succeeded in paying off their debts despite the recent
recession.
(C) Affluent people probably increased their spending levels during the recent recession.
(D) High levels of household debt have little impact on the economy.(A)
(E) When people borrowed money prior to the recent recession, they did not use it to
purchase assets.

11. Which one of the following, if true, casts the most doubt on the argument?

(A) Prior to the recent recession, middle-income households owed enough debt that they
had begun to decrease spending.

(B) The total value of the economy’s household debt is exceeded by the total value of
assets held by households.

(C) Low-income households somewhat decreased their spending during the recent
recession.

(D) During a recession the affluent usually borrow money only in order to purchase
assets.

(E) Household debt is the category of debt least likely to affect the economy.

这个题目大牛们给我解答的时候,我想问这样几个问题:(1)原文你读完后记下了什么?
我总是读的很晕?(2)答案是 A 是如何削弱的。

M: 读完原文应该对文中的逻辑关系和经济现象之间的关系有个认识 . 原文提出造成 recession 的一个现象,

249
然后解释了为什么这个现象不是造成 recession 的原因,然后说应该是其它原因造成了 recession. 这道题完全
可以扩展为一到阅读题目, 原文的结构很典型, 指出现象, 提出一中解释, 然后否定这个解释, 提出是其它原
因.

A 是对的. 原文通过对 low-income 和 affluent 两种人进行了分析, 进而否定了 indebtedness 是原因, 但是漏掉


了 middle-income, 而 A 指出正是 middle-income 的 indebtedness 造成了 decreased spending-->recession.

13. Police published a “wanted” poster for a criminal fugitive in a medical journal,
because the fugitive was known to have a certain acute noninfectious skin problem that
would eventually require a visit to a doctor. The poster asked for information about the
whereabouts of the fugitive. A physician’s responding to the poster’s request for
information would not violate medical ethics, since physicians are already subject to
requirements to report gunshot wounds to police and certain infectious diseases to health
authorities. These exceptions to confidentiality are clearly ethical.

Which one of the following principles, while remaining compatible with the requirements
cited above, supports the view that a physician’s responding to the request would violate
medical ethics?

(A) Since a physician acts both as a professional person and as a citizen, it is not ethical
for a physician to conceal information about patients from duly constituted law
enforcement agencies that have proper jurisdiction.

(B) Since a patient comes to a physician with the expectation that the patient’s visit and
medical condition will remain confidential, it is not ethical for a physician to share this
information with anyone except personnel within the physician’s office.

(C) Since the primary concern of medicine if individual and public health, it is not ethical
for a physician, except in the case of gunshot wounds, to reduce patients’ willingness to
come for treatment by a policy of disclosing their identities to law-enforcement agencies.

(D) Except as required by the medical treatment of the patient, physicians cannot
ethically disclose to others information about a patient’s identity or medical condition
without the patient’s consent.

(E) Except to other medical personnel working to preserve or restore the health of a
patient or of other persons, physicians cannot ethically disclose information about the
identity of patients or their medical condition.

这个题目我还是想听听思路?

我选 C. 原文说枪伤和传染病要 report. 所以 respond to the poster 并不是 unethical. 问怎么 respond 就 unethical
而其它两个还是 ethical.

250
A 错. 说明 respond 是 ethical
B 错. 说明 gunshot 也是 unethical
C 对. 将 guanshot wound 和此例区分开.respond 会使 fugitive unwilling to visit a doctor.所以 unethical
D 错. 同 B
E 错. 同 B

15. Health insurance insulates patients from the expense of medical care, giving doctors
almost complete discretion in deciding the course of most medical treatments. Moreover,
with doctors being paid for each procedure performed, they have an incentive to
overtreat patients. It is thus clear that medical procedures administered by doctors are
frequently prescribed only because these procedures lead to financial rewards.

The argument uses which one of the following questionable techniques?

(A) assigning responsibility for a certain result to someone whose involvement in the
events leading to that result was purely coincidental

(B) inferring the performance of certain actions on no basis other than the existence of
both incentive and opportunity for performing those actions

(C) presenting as capricious and idiosyncratic decisions that are based on the rigorous
application of well-defined principles

(D) depicting choices as having been made arbitrarily by dismissing without argument
reasons that have been given for these choices(B)

(E) assuming that the irrelevance of a consideration for one participant in a decision
makes that consideration irrelevant for each participant in the decision

到 B 的 “on no basis other than” 就看晕了 我做的时候答案选的 E

中文你没看懂,实际 B 就是说,作者认为有动机有条件做某事的人就会做某事。

16. Chlorofluorocarbons are the best possible solvents to have in car engines for
cleaning the electronic sensors in modern automobile ignition systems. These solvents
have contributed significantly to automakers’ ability to meet legally mandated emission
standards. Now automakers will have to phase out the use of chlorofluorocarbons at the
same time that emission standards are becoming more stringent.

If under the circumstances described above cars continue to meet emission standards,
which one of the following is the most strongly supported inference?

251
(A) As emission standards become more stringent, automakers will increasingly
cooperate with each other in the area of emission control.

(B) Car engines will be radically redesigned so as to do away with the need for cleaning
the electronic ignition sensors.

(C) There will be a marked shift toward smaller, lighter cars that will have less powerful
engines but will use their fuel more efficiently.

(D) The solvents developed to replace chlorofluorocarbons in car engines will be only
marginally less effective than the chlorofluorocarbons themselves.

(E) Something other than the cleansers for electronic ignition sensors will make a
relatively greater contribution to meeting emission standards than at present.

这个加强的题目,我选择的是 B,答案是 E.

E 典型的题目. 原文说 C 是最好的 solvent cleanser.现在规定更严格了, 所以要 phase out C.


选项 A,B,C 都是生产商可能采取的措施. 文章中并没有说会采取何种措施, 所以答案只能是 E, 既生产商会
采取 something different from cleanser. A,B,C 也分别是答案集合的一个 factor.
感觉 inference 题目有时是结论题型, 有时更象 assumption.此题应该算是结论题型. 这里的结论是概括性的,
包含了可能采取的措施, A,B,C 都是这些措施的一种, 既所说的 factor(element). 可以把所有的 solution 想象
成一个 set, A,B,C 分别是这个 set 的一个 element.

如果解决某个问题的措施有超过一个, 比如说有 A,B,C 三个, 那么实际实行时可能用 A,也可能用 B,也可能


是 C. 结论是可能用 A,B,C 的其中一个, 而不会是:会用 A.

19. Jane: According to an article in this newsmagazine, children’s hand-eye coordination


suffers when they spend a great amount of time watching television. Therefore, we must
restrict the amount of time Jacqueline and Mildred are allowed to watch television.

Alan: Rubbish! The article says that only children under three are affected in that way.
Jacqueline is ten and Mildred is eight. Therefore, we need not restrict their television
viewing.

Alan’s argument against Jane’s conclusion makes which one of the following errors in

reasoning?

(A) It relies on the same source that Jane cited in support of her conclusion.

(B) It confuses undermining an argument in support of a given conclusion with showing

252
that the conclusion itself is false.

(C) It does not address the main point of Jane’s argument and focuses instead on a side
issue.

(D) It makes an irrelevant appeals to an authority.

(E) It fails to distinguish the consequences of a certain practice from the causes of the
practice.
B
我理解题目是:问 A 在论证中犯的错误。
读完选项后,觉得一个都不对。

j 的前提是长时间看电视会损坏孩子的眼手协调,结论是要限制两个孩子看电视。
而 a 回答是对前提的质疑(范围),即不是所有的孩子而是仅在一年龄段的才会受此影响。因此并不是反
对结论本身,而是将削弱支持结论的论述 confuse 成了指出结论本身错误(B)。

22. A long-term health study that followed a group of people who were age 35 in 1950
found that those whose weight increased by approximately half a kilogram or one pound
per year after the age of 35 tended, on the whole, to live longer than those who
maintained the weight they had at age 35. This finding seems at variance with other
studies that have associated weight gain with a host of health problems that tend to lower
life expectancy.

Which one of the following, if true, most helps to resolve the apparently conflicting
findings?

(A) As people age, muscle and bone tissue tends to make up a smaller and smaller
proportion of total body weight.

(B) Individuals who reduce their cholesterol levels by losing weight can thereby also
reduce their risk of dying from heart attacks or strokes.

(C) Smokers, who tend to be leaner than nonsmokers, tend to have shorter life spans
than nonsmokers.

(D) The normal deterioration of the human immune system with age can be slowed down
by a reduction in the number of calories consumed.

(E) Diets that tend to lead to weight gain often contain not only excess fat but also
unhealthful concentrations of sugar and sodium.

253
答案是 C,这个答案是如何缓解原文的矛盾的?

C 说明抽烟的人又瘦又短命, 符合文章中说的体重未增长还短命的人. (抽烟真的会短命吗?应该在 CD 做个


调查看谁是烟民)

24. The problem that environmental economics aims to remedy is the following: people
making economic decisions cannot readily compare environmental factors, such as clean
air and the survival of endangered species, with other costs and benefits. As
environmental economists recognize, solving this problem requires assigning monetary
values to environmental factors. But monetary values result from people comparing costs
and benefits in order to arrive at economic decisions. Thus, environmental economics is
stymied by what motivates it.

If the considerations advanced in its support are true, the passage’s conclusion is
supported

(A) strongly, on the assumption that monetary values for environment factors cannot be
assigned unless people make economic decisions about these factors

(B) strongly, unless economic decision-making has not yet had any effect on the things
categorized as environmental factors

(C) at best weakly, because the passage fails to establish that economic decision-makers
do not by and large take adequate account of environmental factors

(D) at best weakly, because the argument assumes that pollution and other effects on
environmental factors rarely result from economic decision-making

(E) not at all, since the argument is circular, taking that conclusion as one of its premises

原 文 的 逻 辑 指 出 了 environmental economics 的 问 题 : 要 compare environmental factors with ..., 就 要 assign


monetary value; 而 monetary value 是通过 compare 得到的. 两者互为前提,则永远无法实现.

就好比两个人交换苹果, A 不会把自己的 PG 给 B,除非 B 先把她的 PG 给 A, 人家 B 也不傻, 才不会先把 PG


给 A, 也要先拿到 A 的 PG, 这样一来 PG 永远不会交换.

A 指出了错误.

原文说的是人们作出决定需要知道环境值多少钱
而 a 中说了,只有当人们作出了决定之后才能确定环境值多少钱
所以 a 就很好的指出了原文的矛盾之所在咯

254
LSAT-14-2

LSAT14-4
Conservationists have established land reserves to preserve the last remaining habitat
for certain species whose survival depends on the existence of such habitat. A grove of
trees in Mexico that provide habitat for North American monarch butterflies in winter is a
typical example of such a land reserve. If global warming occurs as predicted, however,
the temperature bands within which various types of vegetation can grow will shift into
regions that are currently cooler.
If the statements above are true, they provide the most support for which one of the
following?
(A) If global warming occurs as predicted, the conservation land reserves will cease to
serve their purpose
(B) Monarch butterflies will succeed in adapting to climatic change by shortening their
migration
(C) If global warming occurs, it will melt polar ice and so will cause the sea level to rise so
high that many coastal plants and animals will become extinct
(D) The natural world has adapted many times in the past to drastic global warming and
cooling
(E) If global warming occurs rapidly, species of plants and animals now protected in
conservation land reserves will move to inhabit areas that are currently used for
agriculture
为什么选 A?最后一句话什么意思?( If global warming occurs as predicted, however,
the temperature bands within which various types of vegetation can grow will shift into
regions that are currently cooler.)

5. Financial success does not guarantee happiness. This claim is not mere proverbial
wisdom but a fact verified by statistics. In a recently concluded survey, only one-third
of the respondents who claimed to have achieved financial success reported that
they were happy.

255
Which one of the following, if true, most strongly supports the conclusion drawn from
the survey results?
A) The respondents who reported financial success were, for the most part,
financially successful
B) Financial success was once thought to be necessary for happiness but is no
longer considered a prerequisite for happiness
C) Many of the respondents who claimed not to have achieved financial success
reported that they were happy five years ago
D) Many of the respondents who failed to report financial success were in fact
financially successful
E) Most of the respondents who reported they were unhappy were in fact happy.
A 怎么能够支持结论?

7. It is not correct that the people of the United States, relative to comparable countries,
are the most lightly taxed. True, the United States has the lowest tax, as percent of gross
domestic product, of the Western industrialized countries, but tax rates alone do not tell
the whole story. People in the United States pay out of pocket for many goods and
services provided from tax revenues elsewhere. Consider universal health care, which is
an entitlement supported by tax revenues in every other Western industrialized country.
United States government health-care expenditures are equivalent to about 5 percent of
the gross domestic product, but private health-care expenditures represent another 7
percent. This 7 percent, then, amounts to a tax.

The argument concerning whether the people of the United States are the most lightly
taxed is most vulnerable to which one of the following criticisms?
A) It bases a comparison on percentages rather than on absolute numbers
B) It unreasonably extends the application of a key term
C) It uses negatively charged language instead of attempting to give a reason
D) It generalizes from only a few instances
E) It sets up a dichotomy between alternatives that are not exclusive
答案:B "This 7 percent, then, amounts to a tax."什么意思,a key term 又指什么?

那句话的意思是“这 7%也是税。”a key term 的意思是关键词。意思就是既然只说美国的税付轻,那么就不


要把不是税付的东西拉进来讨论。
我的理解是作者把 TAX 这个词含义延伸了,因为前提就是看是否是最轻的税负的国家,那只能就税负问
题谈。
我同意 canberry 的解释. 原文引申了 tax 的涵义. 相对值和绝对值不是此题的错误. tax 本身就是一个
percentage.
9. In a mature tourist market such as Bellaria there are only two ways hotel owners can
increase profits: by building more rooms or by improving what is already there. Rigid
land-use laws in Bellaria rule out construction of new hotels or, indeed, any expansion of
hotel capacity. It follows that hotel owners cannot increase their profits in Bellaria since

256
Bellarian hotels _____.

Which one of the following logically completes the argument?

(A) are already operating at an occupancy rate approaching 100 percent year-round

(B) could not have been sited any more attractively than they are even in the absence of
land-use laws

(C) have to contend with upward pressures on the cost of labor which stem from an
incipient shortage of trained personnel

(D) already provide a level of luxury that is at the limits of what even wealthy patrons are
prepared to pay for

(E) have shifted from serving mainly Bellarian tourists to serving foreign tourists traveling
in organized tour groups

答案竟然是 D,太让我吃惊了。

题目本身为这个什么地方的旅店提供了挣钱的两条路:一是盖更多的店,一是装修,提高档次。现在已经
说了盖房子的不能,结论又是无法挣钱,那么一定是另一条路也不成,即提高档次也不成,只有 D 说出了
这个意思。

Section14-II-10
10. Every political philosopher of the early twentieth century who was either a socialist or
a communist was influenced by Rosa Luxemburg. No one who was influenced by Rosa
Luxemburg advocated a totalitarian state.
If the statements above are true, which one of the following must on the basis of them
also be true?
(A) No early-twenty-century socialist political philosopher advocated a totalitarian state
(B) Every early-twenty-century political philosopher who did not advocate a otalitarian
state was influenced by Rosa Luxemburg
(C) Rosa Luxemburg was the only person to influence every early-twenty-century political
philosopher who was either socialist or communist
(D) Every early-twenty-century political philosopher who was influenced by Rosa
Luxemburg and was not a socialist was a communist
(E) Every early-twenty-century political philosopher who did not advocate a otalitarian
state was either socialist or communist
answer:A my choice:D

由 philosopher-> influenced by RL 得到 Not influenced by RL -> no philosopher

257
:political philosopher(socialist or communist)
B:influenced by Rosa
C:advocate a totalitarian state
A->B->not C

in answer (a), A->not C


in answer (d), seems like B->A

13. From an analysis of broken pottery and statuary, archaeologists have estimated that
an ancient settlement in southwestern Arabia was established around 1000 B.C.
However, new evidence suggests that the settlement is considerably older: tests show
that a piece of building timber recently uncovered at the site is substantially older than
the pottery and statuary.

Which one of the following, if true, most seriously undermines the conclusion drawn from
the new evidence?

(A) The building timber bore marks suggesting that it had been salvaged from an earlier
settlement

(B) The pieces of pottery and fragments of statues that were analyzed come from several
parts of the site

(C) The tests used to determine the age of the pottery and statuary had been devised
more recently than those used to determine the age of the building timber

(D) The site has yielded many more samples of pottery and statuary than of building
timber

(E) The type of pottery found at the site is similar to a type of pottery associated with
civilizations that existed before 1000 B.C.

答案是 A,我怎么觉得 A 是加强啊!

A 是说这个木料是从一个更早的定居点弄来的,这就说明文中关于木料比陶器早,所以定居点要早的说法
有问题了,不是加强。

结论是说这个 settlement 比原来估计的更老,因为里面有个 timber 比发现的 pottery 更老.


A 是直接否定. 就是说这个 timber 上的 mark 说明这个 timber 是属于另外一个 earlier settlement 的, 所以不
是属于第一个 settlement 的. 所以说第一个 settlement 比原来估计的要老这个结论是错误的.

lsat 14 ,sec2 ,18/19

258
Questions 18 – 19
Each year, an official estimate of the stock of cod in the Grand Banks is announced. This
estimate is obtained by averaging two separate estimates of how many cod are available,
one based on the number of cod caught by research vessels during a once-yearly
sampling of the area and the other on the average number of tons of cod caught by
various commercial vessels per unit of fishing effort expended there in the past year – a
unit of fishing effort being one kilometer of net set out in the water for one hour. In
previous decades, the two estimates usually agreed closely. However, for the last decade
the estimate based on commercial tonnage has been increasing markedly, by about the
same amount as the sampling based estimate has been decreasing.
18. If the statements in the passage are true, which one of the following is most strongly
supported by them?
(A) Last year’s official estimate was probably not much different from the official estimate
ten years ago
(B) The number of commercial vessels fishing for cod in the Grand Banks has increased
substantially over the past decade
(C) The sampling-based estimate is more accurate than the estimate based on
commercial tonnage in that the data on which it relies is less likely to be inaccurate
(D) The once-yearly sampling by research vessels should be used as the sole basis for
arriving at the official estimate of the stock of cod
(E) Twenty years ago, the overall stock of cod in the Grand Banks was officially estimated
to be much larger than it is estimated to be today

19. Which one of the following, if true, most helps to account for the growing discrepancy
between the estimate based on commercial tonnage and the research-based estimate?
(A) Fishing vessels often exceed their fishing quotas for cod and therefore often
underreport the number of tons of cod that they catch
(B) More survey vessels are now involved in the yearly sampling effort than were
involved 10 years ago
(C) Improvements in technology over the last 10 years have allowed commercial fishing
vessels to locate and catch large schools of cod more easily
(D) Survey vessels count only those cod caught during a 30-day survey period, whereas
commercial dishing vessels report all cod caught during the course of a year
(E) Because of past overfishing of cod, fewer fishing vessels now catch the maximum
tonnage of cod each vessel is allowed by law to catch

这两道题我实在看不懂题目,做错了,18 题不理解为什么是 a

Q18: (A)

259
"for the last decade the estimate based on commercial tonnage has been increasing
markedly, by about the same amount as the sampling based estimate has been
decreasing."

One figure increased, and the other decreased, at about the same amount

Therefore, the average of the two figures, which is the "estimate", should not show big
difference from previous estimate

Q19: my answer is (c):

Technology improvements allowed commercial fishing vessels to catch more cod. Since
commercial estimate is "the average number of tons of cod caught by various
commercial vessels per unit of fishing effort expended

大意为:平均两组数据来估计 Cod 鱼的存量。一组数据是研究船每年取样时捕的鱼量,一


组是平均去年各商业船的 fishing effort expended 所捕的鱼量(每小时每公里撒网所捕的
鱼)。过去几十年,两组数据较吻合。但在过去的十年,商业船的数据明显增加,而取样船
的数据等数量(和商业船)减少。

Q18 如 chelsea 版主所说。Q19 因为计算单位是每小时捕的鱼量,如果如 C 所说鱼更易定


位和捕捉,则单位时间捕的鱼更多,所以能解释商业船捕鱼量的增加。

原文要解释的是 growing discrepancy ,不需要两边都解释。

20. Pretzels can cause cavities. Interestingly, the longer that a pretzel remains in contact
with the teeth when it is being eaten, the greater the likelihood that a cavity will result.
What is true of pretzels in this regard is also true of caramels. Therefore, since caramels
dissolve more quickly in the mouth than pretzels do, eating a caramel is less likely to
result in a cavity than eating a pretzel is.

The reasoning in the argument is vulnerable to criticism on the grounds that the
argument

(A) treats a correlation that holds within individual categories as thereby holding across
categories as well

(B) relies on the ambiguous use of a key term

(C) makes a general claim based on particular examples that do not adequately
represent the respective classes that they are each intended to represent

260
(D) mistakes the cause of a particular phenomenon for the effect of that phenomenon

(E) is based on premises that cannot all be true

答案是 A,不过原文中有 What is true of pretzels in this regard is also true of caramels
那是为什么呢

原文的意思是在各自的条件下 时间越长 cavity 的可能越大 而结论跨越了各自的条件限制。 可能的是


caramels 停留 10 秒对牙齿的伤害远大于 Pretzels 停留 1 小时对牙齿的伤害 那么结论就是错误的了。

There are two different things: pretzel and caramel. The correlation holds for all pretzels or
all caramel. However, it does not necessarily hold between two different things (pretzel and
caramel).

In plain English, the correlation (the longer... the greater...) holds for everything in the
category of pretzel, or everything in the category of caramel. So one pretzel might cause
cavity more than the other pretzel. But we cannot be sure that such correlation holds
across categories (pretzel vs. caramel).

too mindfree,我来用中文解释吧。从总体上 C 溶解比 P 快,但作为个体的一个 C 就不一定溶解的比一个 P


快。

24. A certain airport security scanner designed to detect explosives in luggage will alert
the scanner's operator whenever the piece of luggage passing under the scanner
contains an explosive. The scanner will erroneously alert the operator for only one
percent of the pieces of luggage that contain no explosives. Thus in ninety-nine out of a
hundred alerts explosives will actually by present.

The reasoning in the argument is flawed because the argument


(A) ignores the possibility of the scanner's failing to signal an alert when the luggage
does contain an explosive
(B) draws a general conclusion about reliability on the basis of a sample that is likely to
be biased
(C) ignores the possibility of human error on the part of the scanner's operator once the
scanner has alerted him or her
(D) fails to acknowledge the possibility that the scanner will not be equally sensitive to all
kinds of explosives
(E) substitutes one group for a different group in the statement of a percentage
结论这句“Thus in ninety-nine out of a hundred alerts explosives will actually by present.

100 个报警中 99 个的行李中实际有 EXPLOSIVES。原文实际是将没 EXPLOSIVE 的行李的报警错误率 1%

261
当作所有行李的报警错误率。从而得出该结论。

14-2-25
25. Unless negotiations begin soon, the cease-fire will be violated by one of the two sides
to the dispute. Negotiations will be held only if other countries have pressured the two
sides to negotiate; an agreement will emerge only if other countries continue such
pressure throughout the negotiations. But no negotiations will be held until international
troops enforcing the cease-fire have demonstrated their ability to counter any aggression
from either side, thus suppressing a major incentive for the two sides to resume fighting.

If the statements above are true, and if negotiations between the two sides do begin
soon, at the time those negotiations begin each of the following must also be true
EXCEPT:

(A) The cease-fire has not been violated by either of the two sides

(B) International troops enforcing the cease-fire have demonstrated that they can counter
aggression from either of the two sides

(C) A major incentive for the two sides to resume hostilities has been suppressed

(D) Other countries have exerted pressure on the two sides to the dispute

(E) The negotiations'reaching an agreement depends in part on the actions of other


countries.

答案是 a
可是原文的第一句话说的和问题里的 if negotiations between the two sides

do begin soon 两句相对应的话,a 也好像是对的啊


谁能帮我把第一句翻译一下
我的理解是除非谈判立即开始,否则就会打起来

(1) Cease fire will NOT be violated  negotiation


(2) Negotiation pressured
Conclusion: But no negotiaton……

262
LSAT-14-4

6. Murray: You claim Senator Brandon has accepted gifts from lobbyists. You are wrong
to make this criticism.That it is motivated by personal dislike is shown by the fact that
you deliberately avoid criticizing other politicians who have done what you accuse
Senator Brandon of doing.
Jan: You are right that I dislike Senator Brandon, but just because I have not
criticized the same failing in others doesn’t mean you can excuse the senator’s
offense.
If Murray and Jane are both sincere in what they say, then it can properly be
concluded that they agree that
(A) Senator Brandon has accepted gifts from lobbyists
(B) It is wrong for politicians to accept gifts from lobbyists
(C) Jane’s criticism of Senator Brandon is motivated only by personal dislike
(D) Senator Brandon should be criticized for accepting gifts from lobbyists
(E) One or more politicians have accepted gifts from lobbyists
answer :E my choice:C

when you deal with "CONCLUSION" in CR, the choice should be the information mentioned by passage..

from the passsage, we know that Senator Brandon have accepted gift from lobbyists, therefore one or more
politician have accepted gifts from lobbyists.

C is incorrect because from the passage we only know that Jan dislike Senator Brandon, But we can't conclude that
his criticism is motivated only by personal dislike...Senator Brandon did commit the blunder after all..

As Albert said, you need to reply solely on the information provided by the argument to draw the conclusion. For
this particular question, both M and J need to admit to the same thing, which is the answer.

A is wrong, as M never affirmatively said that B accepted gifts.

263
B is wrong, as M never said it is wrong.
C is wrong, as J never admitted that it was because she disliked B.
D is wrong, same as A
E is right, as both M and J mentioned in their arguments:
M: "...other politicians who have done what you accuse Senator Brandon of doing"
J: "...the same failing in others..."

LSAT-14-4-9,LSAT-14-4-12,LSAT-14-4-14
Oscar: Emerging information technologies will soon make speed of information
processing the single most important factor in the creation of individual, corporate, and
national wealth. Consequently, the division of the world into northern countries - in
general rich -and southern countries - in general poor -will soon be obsolete. Instead,
there simply will be fast countries and slow countries, and thus a country's economic
well-being will not be a function of its geographical position but just a matter of its relative
success in incorporating those new technologies

Sylvia: But the poor countries of the south lack the economic resources to acquire those
technologies and will therefore remain poor. The technologies will thus only widen the
existing economic gap between north and south.

9. The reasoning that Oscar uses in supporting his prediction is vulnerable to criticism on
the ground that it

(A) overlooks the possibility that the ability of countries to acquire new technologies at
some time in the future will depend on factors other than those countries' present
economic status

(B) fails to establish that the division of the world into rich countries and poor countries is
the single most important problem that will confront the world economy in the future

(C) ignores the possibility that, in determining a country's future wealth, the country's
incorporation of information-processing technologies might be outweighed by a
combination of other factors

(D) provides no reason to believe that faster information processing will have only
beneficial effects on countries that successfully incorporate new information technologies
into their economies

(E) makes no distinction between those of the world's rich countries that are the
wealthiest and those that are less wealthy

其他的答案肯定不对,但是 c/d 我没有分出.答案是 C,我选 D

264
M: D 其实是在意思上有误. D 说没有说明 faster information 对 incorpoate 此项技术的国家只有好处.在这里
有没有坏处不是讨论范围. 举了例子:一个人中$88MM 彩票了, 他就一辈子有花不完的钱了.问逻辑错误.这
里就只能局限于钱上. 比如说他就要交 55%的税,亲戚都来替他花,或是这种人 99%都投资破产等. 答案决不
会是说他可能一高兴,死翘翘了.虽然这是一个危害,但不影响原文的逻辑.

Beverage company representative: the plastic rings that hold six-packs of beverage cans
together pose a threat to wild animals, which often become entangled in the discarded
rings and suffocate as a result. Following our lead, all beverage companies will soon use
only those ring consisting of a new plastic that disintegrates after only three days'
exposure to sunlight. Once we all complete the switchover from the old to the new plastic
rings, therefore, the threat of suffocation that plastic rings pose to wild animals will be
eliminated.

12. Which one of the following, if true, most seriously weakens the representative's
argument?

(A) The switchover to the new plastic rings will take at least two more years to complete

(B) After the beverage companies have switched over to the new plastic rings, a
substantial number of the old plastic rings will persist in most aquatic and woodland
environments

(C) The new plastic rings are slightly less expensive than the old rings

(D) The new plastic rings rarely disintegrate during shipping of beverage six-packs
because most trucks that transport canned beverages protect their cargo from sunlight

(E) The new plastic rings disintegrate into substances that are harmful to aquatic animals
when ingested in substantial quantities by them

我认为 B/E 都是削弱,答案是 B.

E 是典型的错误选项. 还是一个局限的问题. 你要理解原文的逻辑范围. 如果原文是讲总体的危害性, E 就是


正确答案. 比如说把题目改成:改用了新的 ring,对环境和野生动物的危害就会消失.在此题中原文的逻辑关系
是:complete the switchove-->the threat of suffocation will be eliminated. E 中设计的毒害问题就是 out of scope.
不在讨论范围. 既:即使新的 ring 会造成毒害,suffocation 的问题可能被解决了. 这样就起不到 weaken 的作用.

14. Some of the world's most beautiful cats are Persian cats. However, it must be
acknowledged that all Persian cats are pompous, and pompous cats are invariably
irritating.

265
If the statements above are true, each of the following must also be true on the basis of
them EXCEPT:

(A) Some of the world's most beautiful cats are irritating

(B) Some irritating cats are among the world's most beautiful cats

(C) Any cat that is not irritating is not a Persian cat

(D) Some pompous cats are among the world's most beautiful cats

(E) Some irritating and beautiful cats are not Persian cats
我觉得五个答案都对,E 说的不好而已.虽然作对,还是请讲解.

must be true 的题目正确答案都可以根据原文的逻辑关系顺推. 错误选项可以通过举反例来严证. 只要是原


文没有提到的都可能发生(当然也可能不发生), 比如说我们可以假设所有的 irritating cats 都是 persian, 虽然
这是从原文推不出来的, 但是原文并没有否定此假设, it will be safe to make such assumption. 进而推出 E 可
能错.

其它选项都可以从原文逻辑正推出来.

L14-4-15

At Flordyce University any student who wants to participate in a certain archaeological dig is
eligible to do so(结论) but only if the student has taken at least one archaeology course(必要
条件 1) and has shown an interest in the field(必要挑拣 2). Many students who have shown a
n interest in archaeology never take even one archaeology course(没有必要条件 1). Therefor
e, many students who want to participate in the dig will be ineligible to do so(结论取非).

The flawed reasoning of which one of the following arguments is most similar to that of the
argument above?

(A) Theoretically, any jar is worth saving regardless of its size, but only if it has a lid.
Therefore, since some jars are sure not to have lids, there are certain sizes of jar that are
actually not worth saving.

(B) For a horse that is well schooled to be ideal for beginning riders that horse must also be
surefooted and gentle. Many horses that are surefooted are not gentle. Therefore many well-
schooled horses are not ideal for beginning riders.

266
(C) If an author’s first novel has a romantic setting and a suspenseful plot, it will become a
best-seller. Since many authors’ first novels have neither, not many first novels become best-
sellers.

(D) Any automobile that is more than a few years old is eventually sure to need repairs if it is
not regularly maintained. Many automobiles are more than a few years old, but still do not
need repairs. Therefore, many automobiles are regularly maintained.

(E) An expensive new building will prove to be a good investment only if it is aesthetically
pleasing or provides lots of office space. However, since many expensive new buildings are
not aesthetically pleasing, few expensive new buildings will prove to be good investments.

所以 B is right.

16. From the observation that each member of a group could possess a characteristic, it
is fallacious to conclude immediately that it is possible for all the group’s members to
possess the characteristic. An example in which the fallacy is obvious: arguing that
because each of the players entering a tennis tournament has a possibility of winning it,
there is therefore a possibility that all will win the tournament.
Which one of the following commits the fallacy described above?
(A) You can fool some of the people all of the time and all of the people some of the time,
but you cannot fool all of the people all of the time
(B) Each of the candidates for mayor appears at first glance to possess the necessary
qualifications. It would therefore be a mistake to rule out any of them without more careful
examination
(C) Each of the many nominees could be appointed to any one of the three openings on
the committee. Therefore it is possible for all of the nominees to be appointed to the
openings on the committee
(D) If a fair coin is tossed five times, then on each toss the chance of heads being the
result is half. Therefore the chance of heads being the result on all five tosses is also half
(E) It is estimated that ten million planets capable of supporting life exist in our galaxy.
Thus to rule out the possibility of life on worlds other than Earth, ten million planetary
explorations would be needed

我选了 d 答案是 c。 但是我觉得 d 是对的。反而 c 是错的

D 错在用的是肯定的语气( is )。而原文是可能的语气(it is possible )。C 就是可能的语气(it is


possible)。尽管 CD 都和原文一样是部分推整体的逻辑错误。

267
17. Recent research shows that hesitation, shifting posture, and failure to maintain eye
contact are not reliable indicators in discriminating between those who are lying and
those who are telling the truth. The research indicates that behavior that cannot be
controlled is a much better clue, at least when the lie is important to the liar. Such
behavior includes the dilation of eye pupils, which indicates emotional arousal, and small
movements of facial muscles, which indicate distress, fear or anger.

Which one of the following provides the strongest reason for exercising caution when
relying on the "better" clues mentioned above in order to discover whether someone is
lying?

(A) A person who is lying might be aware that he or she is being closely observed for
indications of lying

(B) Someone who is telling the truth might nevertheless have a past history of lying

(C) A practiced liar might have achieved great control over body posture and eye contact

(D) A person telling the truth might be affected emotionally by being suspected of lying or
by some other aspect of the situation

(E) Someone who is lying might exhibit hesitation and shifting posture as well as dilated
pupils

这个题目没有思路?大。答案是 D.

M: 根据原文的意思可以推出. 文中说不应该依赖 hesitation, eye contact 来测谎,而应该根据一些不能控制的


行为如瞳孔扩张和面部肌肉的 movement 来判定. 又近一步指出这些 uncontrollable behavior 反映了 emotion.
问题问为什么要慎用这种方法.

A. 原文没有指出 being closely observed 对文中的概念有任何影响,无关


B. 原文没有解释有撒谎史对文中概念的影响, 无关
C. 加强了原文对此方法的推崇, 错.
D. 说明不撒谎的人在 emotion 上会受影响, 和原文对说明的此方法以来 emotion 的变化直接相关
E. 相关但是不 weaken 原文的推论

Orthodox medicine is ineffective at both ends of the spectrum of ailments. At the more
trivial end, orthodox medicine is largely ineffective in treating aches, pains and allergies,
and, at the other extreme, it has yet to produce a cure for serious, life-threatening
diseases such as advanced cancer and lupus. People turn to alternative medicine when
orthodox medicine fails to help them and when it produces side effects that are
unacceptable to them. One of the reasons alternative medicines is free of such side

268
effects is that it does not have any effects at all.

18. if the statements above are true, which one of the following can be properly inferred
from them?

(A) Practitioners of alternative medicine are acting in bad faith

(B) There are some medical conditions for which no orthodox or alternative treatment is
effective

(C) There are some trivial illnesses that can be treated effectively by the methods of
alternative medicine

(D) There are no effective medical treatments that are free from unacceptable side
effects

(E) Orthodox medicine will eventually produce a solution for the diseases that are
currently incurable

答案:B, 我选 D,D 为何不对?

原文讨论的是某种特定药和他的替代物的副作用比较。。。D 说的任何药品,根本超出了范围

19. The charge made above against alternative medicine is most seriously weakened if it
is true that

(A) predictions based on orthodox medicine have sometimes failed, as when a patient
has recovered despite the judgment of doctors that an illness is fatal

(B) alternative medicine relies on concepts of the body and of the nature of healing that
differ from those on which orthodox medicine is based

(C) alternative medicine provides hope to those for whom orthodox medicine offers no
cure

(D) a patient's belief in the medical treatment the patient is receiving can release the
body's own chemical painkillers, diminish allergic reactions, and promote healing

(E) many treatments used for a time by orthodox medicine have later been found to be
totally ineffective

答案是 D,我觉得 D 有些象无关,也有点象他因.请讲解 D 是如何削弱的??

269
M: 原文说 alternative 没有任何效果, 问 weaken. D 指出(alternative 本身可能没有效果,)病人在知道他们被治
疗时自身会....,达到治疗效果. 所以 weaken.

我的理解是,文中对最后的结论并没有提供什么有利的支持的论据,所以,只要找到一个替代疗法有效的
例子就行了,答案说明了替代疗法的有效的一个例子,B 的意思是说替代疗法是基于和传统疗法不同的理

21. The town of Greenfield recently instituted a substantial supplementary tax on all
households, whereby each household is taxed in proportion to the volume of the trash
that it puts out for trash collectors to pick up, as measured by the number of standard-
sized garbage bags put out. In order to reduce the volume of the trash on which their tax
bill is based, Greenfield households can deliver their recyclable trash to a conveniently
located local commercial recycling center, where such trash is accepted free of charge

The supplementary tax provides some financial incentive to Greenfield households to do


each of the following EXCEPT

(A) Sort out recyclable trash thoroughly from their other trash

(B) Dump nonrecyclable trash illegally at parks and roadsides

(C) Compress and nest items of nonrecyclable trash before putting them out for pickup

(D) Deliver recyclable materials to the recycling center instead of passing them on to
neighbors who want to reuse them

(E) Buy products without packaging or with recyclable rather than nonrecyclable
packaging

答案是 D,那么 B 是如何从原文得出的.


根据问题, 只要能使得居民少付 tax 就不是答案.
B. 如果把垃圾扔到那些地方,居民就不用给 collector,也就不用付 tax.
D. 两种情况对居民 tax 的影响没有区别,答案.

22. In a survey of consumers in an Eastern European nation, respondents were asked


two questions about each of 400 famous Western brands: whether or not they
recognized the brand name and whether or not they thought the products bearing that
name were of high quality. The results of the survey were a rating and corresponding
rank order for each brand based on recognition, and a second rating-plus-ranking based
on approval. The brands ranked in the top 27 for recognition were those actually
available in that nation. The approval rankings of these 27 brands often differed sharply
from their recognition rankings. By contrast, most of the other brands had ratings, and
thus rankings, that were essentially the same for recognition as for approval

270
Which one of the following, if each is a principle about consumer surveys, is violated by
the survey described?

(A) Never ask all respondents a question if it cannot reasonably be answered by


respondents who make a particular response to another question in the same survey

(B) Never ask a question that is likely to generate a large variety of responses that are
difficult to group into a manageable number of categories

(C) Never ask all respondents a question that respondents cannot answer without giving
up their anonymity

(D) It is better to ask the same question about ten different products than to ask ten
different questions about a single product

(E) It is best to ask questions that a respondent can answer without fear of having gotten
the answer wrong

这个题目作对了但是有猜测的原因,请讲解?答案是 A

M: A 对是因为被 survey 的人被问及他们不熟悉的品牌(第一个问题), 然后还被问到对这些品牌的品质(第二


个问题). 第二个问题不可能 reasonably answered. 所以 A 正确.

第二题抽象出来,就是接受调查者的对相同问题的反应不一致。这就是出题点。答案中体现这一点的只有
A.

271
LSAT-15-2

3. Dear mice normally do not travel far from their nests, and dear mice that are moved
more than half a kilometer from their nests generally never find their way back. Yet in one
case, when researchers camped near a dear mouse nest and observed a young dear
mouse for several weeks before moving it to an area over two kilometers away, the dear
mouse found its way back to its nest near their camp in less than two days.

Which one of the followings, if true, most help to explain how the dear mouse might have
found its way back to its nest?

(A) The area to which the dear mouse was moved was dryer and more rocky than the
area in which its nest was located.
(B) The researchers released the dear mouse in a flat area across which their campfire
smoke drifted.
(C) There were very few dear mice in the area to which the dear mouse was moved.
(D) The researchers had moved the dear muse in a small dark box, keeping the mouse
calm before it was released.
(E) Animals that prey on dear mice were common in the area to which the dear mouse
was moved.
B
7. all zebras have stripes, and the most widespread subspecies has the best-defined
stripes. The stripes must therefore be of importance to the species. Since among
these grassland grazers the stripes can hardly function as camouflage, they must

272
serve as some sort of signal for other zebras.

Which one of the following, if true, most strongly supports the conclusion regarding a
signaling function?

(A) The subspecies of zebras with the best-defined strips is also characterized by
exceptional size and vigor.

(B) In certain tall grasses zebras can be harder to spot than grazing animals with a
coat of uniform color.

(C) A visual signal transmitted among the members of a species can consist of a
temporary change of color perceptible to other members of the species.

(D) Zebras react much faster to moving shapes that have stripes than they do to
moving shapes that are otherwise identical but lack stripes

(E) Zebras have a richer repertoire of vocal signals than do similar species such as
horses.

答案是 D,但是我觉得 d 是无关的,我选的是 B.

M: B 是 weaken, 说明 stripes 还是 camouflage, 而不是 signaling. D 是加强,说明 stripes 确实有 signal 的作用,
能使 zebra 反应快

Questions 12
In many languages other than English there is a word for "mother's brother" which is
different from the word for "father's brother." Whereas English uses the word "uncle" for
both. Thus, speakers of these languages evidence a more finely discriminated kinship
system than English speakers do. The number of basic words for colors also caries
widely from language to language. Therefore. Speakers of languages that have fewer
basic words for colors than English has must be perceptually unable to distinguish as
many colors as speakers of English can distinguish.

12. which one of the following, if true, undermines the conclusion concerning words for
colors?

(A) Speakers of English are able to distinguish between lighter and darker shades of the
color they call "blue" for which Russian has two different basic words.

(B) Almost every language distinguishes red from the other colors.

(C) Khmer uses a basic word corresponding to English "blue" for most leaves, but uses

273
its basic word corresponding to English "green" for unripe bananas.

(D) The word "orange" in English has the same origin as the equivalent word in Spanish.

(E) Most languages do not have a basic word that distinguishes gray from other colors,
although gray is commonly found in nature.

答案是 A), 可是: 原文结论是: 如果外国语言的 BASIC WORDS FOR COLOR 比英语少, 表
明说外国语言的人不能识别更多的颜色. 但是 A)是说: RUSSIAN 用 2 个 BASIC WORDS 来
表达英文中的一个颜色单词 BLUE 的不同深浅. 不能说是 UNDERMINE 吧. 除非他说: 英语
用 2 个单词, 而 RUSSIAN 用一个单词.
欢迎大家讨论.

M: A undermines. The question asks for the answer that undermines the conclusion concerning
words for color. The reasoning in the question is that people with fewer basic words for color are
unable to distinguish the colors as people with more basic words.

I do not think the question asks for an answer that undermines the whole conclusion that
"Speakers of languages that have fewer basic words for colors than English has must be
perceptually unable to distinguish as many colors as speakers of English can distinguish".

Zachary: One would have to be blind to the reality of moral obligation to deny that people
who believe a course of action to be morally obligatory for them have both the right and
the duty to pursue that action, and that no one else has any right to stop them form doing
so.

Cynthia: But imagine an artist who feels morally obliged to do whatever she can to
prevent works of art from being destroyed confronting a morally committed
antipornography demonstrator engaged in destroying artworks he deems pornographic.
According to your principle that artist has, simultaneously, both the right and the duty to
stop the destruction and no right whatsoever to stop it.

14. Cynthia's response to Zachary's claim is structured to demonstrate that

(A) the concept of moral obligation is incoherent

(B) the ideas of right and duty should not be taken seriously since doing so leads to
morally undesirable consequences.

(C) Zachary's principle is untenable on its own terms.

(D) Zachary's principle is based on an understanding of moral obligation that is too

274
narrow to encompass the kind of moral obligation artists feel toward works of art

M: 理解题目意思. 原文中 Z 说只要 morally obliged to do sth., 就有 right and duty 去做这件事,其他人不能阻
拦. Cynthtia 的回答说明两种意见完全相反的人都 have both right and duty.nobody can stop them. 指出了 Z 的
argument 的矛盾. C 指出了 Z 的问题,就是在 right and duty 上.

15. Which one of the following, if substituted for the scenario invoked by Cynthia, would
preserve the force of her argument?

(A) a medical researcher who feels a moral obligation not to claim sole credit for work
that was performed in part by someone else confronting another researcher who feels no
such moral obligation

(B) a manufacturer who feels a moral obligation to recall potentially dangerous products
confronting a consumer advocate who feels morally obliged to expose product defects

(C) an investment baker who believes that governments are morally obliged to regulate
major industries confronting an investment banker who holds that governments nave a
moral obligation not to interfere with market forces

(D) an architect who feels amoral obligation to design only energy-efficient buildings
confronting, as a potential client, a corporation that believes its primary moral obligation
is to maximize shareholder profits

(E) a health inspector who feels morally obliged to enforce restrictions on the number of
cats a householder may keep confronting a householder who, feeling morally obliged to
keep every stray that comes along ,has over twice that number of cats.

这两个题目原文读的不是很清楚,请问:大牛们,从原文读出什么信息作题的?答案是 C 和 E

M: 此题要求在举出相同的例子来. E 是符合 Cynthia 的意思,两个意见相反的人同时都 feel morally obliged to


do things that are opposite.

A 错,因为两个相反意见的人中的一个没有 moral obligation


B 错,因为虽然两个意见不同,但不是相对的
C 错,是 government obliged,和两种 banker 无关
D 错, 同 B

16. A county airport, designed to serve the needs of private aircraft owners, planned to
cover its operating expenses in part by charging user fees to private aircraft using the
airport. The airport was unable to pay its operating expenses because the revenue from

275
user fees was lower that expected.
If the statements above are true, which one of the following must also be true?
(A) Most of the country's citizens live a convenient distance from one or another airport
now offering commercial airline services.
(B) Private aircraft owners were unwilling to pay the user fees charged at the airport
(C) The airport's construction was financed exclusively by private funds
(D) The airport's operating expenses were greater than the revenues raised from sources
other than the airport user fees for private planes
(E) The number of owners of private aircraft who use the county's airport facilities will not
change appreciably in the future

答案:D

我选的是 B,B 的错误是不是原文没有提到 owners were unwilling? 那 D 是如何得出的呢?


谢谢!

Set 15-2-17/18
Consumer activist: By allowing major airlines to abandon, as they promptly did, all but
their most profitable routes, the government's decision to cease regulation of the airline
industry has worked to the disadvantage of everyone who lacks access to large
metropolitan airport.

Industry representative: On the contrary, where major airlines moved out, regional
airlines have moved in and ,as a consequence, there are more flights into and out of
most small airports now that before the change in regulatory policy.

17. The industry representative's argument will not provide an effective answer to the
consumer activist's claim unless which one of the following is true?

(A) No small airport has fewer flights now than it did before the change in policy
regarding regulation of the airline industry.

(B) When permitted to do so by changes in regulatory policy, each major airline


abandoned all but large metropolitan airports.

(C) Policies that result in an increase in the number of flights to which consumers have
easy access do not generally work to the disadvantage of consumers.

(D) Regional airlines charge les to fly a given route now that the major airlines charged
when they flew the same route.

(E) Any policy that leads to an increase in the number of competitors in a given field

276
works to the long-term advantage of consumers.
这个题目当时我在 B/C 之间犹豫了很久,最后选择了 C.请讲讲 B 什么地方错了?答案是 C

M: 选 C 之前我也在两个选项中想了一下,不过我是在 A/C 之间. 此题是问 rep 的 assumption. Rep 说航班比以


前更多了, 所以 decision is not to the disadvantage. C 直接将两个联系起来. B 在这里是无关的, 它更象是另一
类问题的答案. 如果原文是说:

consumer advocate: By allowing major airlines to abandon all but their most profitable routes, the government's
decision to cease regulation of the airline industry will ultimately to the disadvantage of everyone who lacks access
to large metropolitan airport.

问 assumption. 就是 B, 即如果 airlines 被允许那么做, 他们就会那么做.

18. Which one of the following is assumption on which the consumer activist’s argument
depends?

(A) Before the recent change in regulatory policy, there was no advantage in having easy
access to large metropolitan airport.

(B) When any sizable group of consumers is seriously disadvantaged by a change in


government policy, that change should be reversed.

(C) Government regulation of industry almost always works to the advantage of


consumers.

(D) At the time of the regulatory change, the major airlines were maintaining their less
profitable routes at least in part be4cause of government requirements.

(E) Regional airlines lack the resources to provides consumers with service of the same
quality as that provided by the major airlines.

这个题目我选 E,答案是 D.

M: 错在 consumer advocate 并没有提及 regional airlines. 原文的 gap 并不在此. D 填补 gap 的作用很明显, 将
decison 和 airlines 联系在一起. 在做此题时就要严格地根据原文的逻辑,局限于原文的因果关系内.

21. In response to high mortality in area hospitals, surgery was restricted to emergency
procedures during a five-week period. Mortality in these hospitals was found to have
fallen by nearly one-third during the period. The number of deaths rose again when
elective surgery (surgery that can be postponed) was resumed. It can be concluded
that ,before the five --week period, the risks of elective surgery had been incurred
unnecessarily often in the area.

277
Which one of the following, if true, most seriously undermines the conclusion above?

(A) The conditions for which elective surgery was performed would in the long run have
been life-threatening, and surgery for them would have become riskier with time.

(B) The physicians planning elective surgery performed before the five-week period had
fully informed the patients who would undergo it of the possible risks of the procedures.

(C) Before the suspension of elective surgery, surgical operations were performed in area
hospitals at a higher rate, per thousand residents of the area, than was usual elsewhere.

(D) Elective surgery is, in general, less risky than is emergency surgery because the
conditions requiring or indicating surgery are often less severe.

(E) Even if a surgical procedure is successful, the patient can die of a hospital-contracted
infection with a bacterium that is resistant to antibiotic treatment.

这个题目我选 C,答案是 A.当时作题的时候觉得 A 是无关的就排除了.A 是如何削弱的?

M: 此题确实有难度,我一上来也是排除了 A, 但是后来发现 BCDE 都不对, 回来又看 A 才选上.

C 貌似正确答案, 但是它的问题是并没有给出两个阶段的不同 . 原文是根据两段时间 death rate 的不同做的


结论, C 中并没有指出后来的情况.还有一个关键的问题是 C 并没有给出 higher rate of surgery 和 death rate 以
及 elective surgery 之间的关系, 如果选 C,那么你要自己做 assumption,这在 GMAT 中是基本不允许的.

A 虽然没有给出两个阶段的不同, 但是"long run"和"riskier with time"解释了 death rate 的变化,所以是正确答


案.

原文应该是,原来 area hospital 既有 emergency procedure 又有 elective surgery,结果由于 high mortality, 决


定停掉 elective surgery 5 周, 结果 mortality rate 下降 1/3。5 周后 elective surgery resume, mortality rate 上去了。

下面这句话看不太懂,有更好的翻译请指教:5 周前 elevtive surgery 的风险被不适当的增加了频率?(我怎


么看都觉得这句话别扭,特别是 incurred unnecessarily often 的这个 often, 他在此时表示 frequency 这一概念
吗?)

我觉得削弱应该是讲 elective surgery 在 5 周以前并没有增加风险。但是我对 A 的了解好像总是不对:长期


看,elective surgery 实施的情况是致命的,而且越晚做越危险。

这句话是不是在表达这样一个意思:elective surgery (我的理解应该是类似切除癌细胞组织,心脏搭桥这一


类手术,emergency surgery 应该是枪伤,骨折一类的手术) 本来就比 emergency surgery 要危险,而且越晚
做越危险。因此否定了 incurred unnecessarily often 这一结论。

278
以上是我自己的解答,当时并不知道已经有人问过词题并且 mindfree 已经给除了回答。这次花了 2 小时找
到了答案,并贴出来,希望能被版主收到总结版中,有利后人。

以下是 mindfree 的解答:此题确实有难度,我一上来也是排除了 A, 但是后来发现 BCDE 都不对, 回来又看


A 才选上.

C 貌似正确答案, 但是它的问题是并没有给出两个阶段的不同 . 原文是根据两段时间 death rate 的不同做的


结论, C 中并没有指出后来的情况.还有一个关键的问题是 C 并没有给出 higher rate of surgery 和 death rate 以
及 elective surgery 之间的关系, 如果选 C,那么你要自己做 assumption,这在 GMAT 中是基本不允许的.

A 虽然没有给出两个阶段的不同, 但是"long run"和"riskier with time"解释了 death rate 的变化,所以是正确答


案.

再问一下,如果问题问的是加强,那么 C 是否能起到加强的作用。
烦请解释一下,谢谢。

Jocy: 我认为加强, c 也不对,


因为,c 只是说明了一种客观状况:该地区的 surgical operations 比其他地方做的更多.
而结论是: 由于 emergency procedure 导致了 elective surgery 的危险性不必要的增加了.
是 emergency and elective surgey 之间的关系.
而 c 的指出与其他地区的比较, 这种关系应该是无关的.

文章的 结论是 elective surgery 手术在五周前增加了不必要的风险


A 的意思是说那些被选择来的手术的病人本来就是有生命危险的,而且会随时间的增加危险增加

24. Between 1951 and 1963, it was illegal in the country of Geronia to manufacture, sell,
or transport any alcoholic beverages. Despite this prohibition, however, the death rate
form diseases during the first five years of the period than it was during the five years
prior to 1951. Therefore, the attempt to prevent alcohol use merely make people want
and use alcohol more than they would have it had not been forbidden.

Each of the following, if true, weakens the argument EXCEPT:

(A) Death form an alcohol-related disease generally does not occur until five to ten years
after the onset of excessive alcohol consumption.

(B) The diseases that can be caused by excessive alcohol consumption can also be
caused by other kinds of behavior that increased between 1951 and 1963.

(C) The death rate resulting from alcohol-related diseases increase just as sharply during
the ten years before and the ten years after the prohibition of alcohol as it did during the
years of prohibition.

279
(D) Many who died of alcohol-related diseases between 1951 and 1963 consumed
illegally imported alcoholic beverages produced by the same methods as those used
within Geronia.

(E) Between 1951 and 1963, among the people with preexisting alcohol-related diseases,
the percentage who obtained lifesaving medical attention declined because of a social
stigma attached to excessive alcohol consumption.

答案:D,不明白.

原文:因为戒酒法律执行期间,因为 VIRUS 而引起的 DEATH 上升。


所以戒酒法律引起更多人喝酒
现象解释 Weaken 的方式:
A 之前引起的,有潜伏期(别的解释)
B 可能别的原因引起的(他因)
C 法律执行前后和期间死亡增长一样猛(通过比较,切断法律和死亡增长的关系)
E 之前有病的人,期间因为医治的少了导致死亡(他因)
D 无法削弱

25. A letter submitted to the editor of a national newsmagazine was written and signed by
a Dr. Shirley Martin who, in the text of the letter, mentions being a professor at a major
North American medical school. Knowing that fewer than 5 percent of the professors
at such schools are women, the editor reasons that the chances are better than 19
to 1 that the letter was written by a man.

Which one of the following involves flawed reasoning most like that used by the editor?

(A) Since 19 out of 20 home computers are purchased primarily for use with computer
games, and the first computer sold today was purchased solely for word processing, the
next 19 computers sold will almost certainly be used primarily for computer games.

(B) Fewer than 1 in 20 of the manuscripts submitted to Argon Publishing Vo. Are
accepted for publication. Since only 15 manuscripts were submitted last week, there is
almost no chance that any of them will be accepted for publication.

(C) Fewer that 5 percent of last year's graduating class took Latin in secondary school.
Howard took Latin in secondary school, so if he had graduated last year, it is likely that
one or the other Latin scholars would not have graduated.

(D) More that 95 percent of the planes built by UBC last year met government standards
for large airliners. Since small planes account for just under 5 percent of UBC 抯 output

280
last year, it is almost certain that all their large planes met government standards.

(E) Since more than 19 out of every 20 animals in the wildlife preserve are mammals and
fewer than 1 out of 20 are birds, there is a greater than 95 percent chance that the animal
Emily saw flying between two trees in the wildlife refuge yesterday morning was a
mammal.

这个题目的原文的黑体没有读懂?请问从原文提取了什么信息来分辨这些长相都差不多的
选项?

黑体字的意思就是说从<5%是女性-->是男性写的可能性是女的 19 倍(由 95:5 得出)


本文的逻辑错误就在于只是从一个泛指的比例直接推出某一个单个事件的可能性。
从五个选项中我们可以看到,E 选项最符合这个状况!
我对于排除其他几个选项的心得是看每个选项从事件 1-->事件 2,事件 1 与事件 2 的关系,在原题中两者
的关系不是很明确,事件 1 是一所指定的学校,事件 2 的概率是泛指的学校。
选项 A,B,C,D 事件 1 和事件 2 都是同一回事,不存在泛指与特指的关系,只有 E 选项是从一个泛指的推出
一个特指的关系。
因此答案应该是 E

flawed reasoning 指明明 Dr. 翠花 Martin 是个女的,还经验老道的认为可能是个男的.


E 中明明看到的是个鸟,还说可能是个非鸟.

281
LSAT-15-4

The number of hospital emergency room visits by heroin users grew more than 25
percent during the 1980s. Clearly, then, the use of heroin rose in that decade.

6. Which one of the following, if true, would account for the statistic above without
supporting the author’s conclusion?

(A) Widespread use of automatic weapons in the drug trade during the 1980s raised the
incidence of physical injury to heroin users.

(B) The introduction of a smokable type of heroin during the 1980s removed the need for
heroin to be injected intravenously and thus reduced the user’s risk of infection.

(C) Many hospital emergency rooms were barely able to accommodate the dramatic
increase in the number of medical emergencies related to drug abuse during the 1980s.

(D) Heroin use increased much more that is reflected in the rate of heroin-linked hospital
emergency room visits.

(E) Viral and bacterial infections, malnourishment, and overdose account for most
hospital emergency room visits linked to heroin.

这个题目的答案是 A,但是我觉得 E 也可以回答原文的问题?

282
M: E 没 有 给 出 80's 和 其 它 阶 段 的 不 同 , 我 们 不 能 自 己 assume 80's 中 Viral and bacterial infections,
malnourishment, and overdose 的 情 况 与 其 它 阶 段 不 同 . 如 果 原 文 指 出 80's 更 多 的 addict 被 感 染 或
malnutrition, 或这个阶段的人比其它阶段更容易被感染, 更容易营养不良, E 才是答案.

8. The year 1917,1937,1956,1968,1979, and 1990 are all notable for the occurrence of
both popular uprisings and near-maximum sunspot activity. During heavy sunspot activity,
there is a sharp rise in positively charged ions in the air people breathe, and positively
charged ions are known to make people anxious and irritable. Therefore, it is likely that
sunspot activity has actually been a factor in triggering popular uprisings.

Which one of the following exhibits a pattern of reasoning most similar to that in the
passage?

(A) The ancient Greeks sometimes attempted to predict the outcome of future events by
watching the flight patterns of birds. Since the events themselves often matched the
predictions, the birds were probably responding to some factor that also influenced the
events.

(B) Martha, Sidney, and Hilary are the city's three most powerful politicians, and all three
graduated from Ridgeview high School. Although Ridgeview never had a reputation for
excellence, it must have been a good school to have produced three such successful
graduates.

(C) Unusually cold weather last December coincided with a rise in fuel prices. When it is
cold, people use more fuel to keep warm: and when more fuel is used, prices rise.
Therefore if prices are high next winter, it will be the result of cold weather.

(D) The thirty healthiest people in a long-term medical study turned out to be the same
thirty whose regular diets included the most vegetables. Since specific substances in
vegetables are known to help the body flight disease, vegetables should be part of
everyone's diet.

(E) Acme's most productive managers are consistently those who occupy the corner
offices, which have more windows than other offices at Acme. Since people are more
alert when they are exposed to abundant natural light, the greater productivity of these
managers is probably at least in part a result of their working in the corner offices.

逻辑相似题,答案是 E.请问从原文得出了哪些本质信息?如何迅速地排除答案的?

题的意思是 A 和 B 同时发生而且 B->c->d->A,所以 B->A


.用这个模式去套,不过要很仔细。一糊涂就错了。还有两题不行了,其他人上

9. Since anyone who supports the new tax plan has no chance of being elected, and

283
anyone who truly understands economics would not support the tax plan, only someone
who truly understands economics would have any chance of being elected.

The reasoning in the argument is flawed because the argument ignores the possibility
that some people who

(A) truly understand economics do not support the tax plan

(B) truly understand economics have no chance of being elected

(C) don not support the tax plan have no chance of being elected

(D) do not support the tax plan do not truly understand economics

(E) have no chance of being elected do not truly understand economics

对于这种题目"ignores the possibility that "应该如何下手呢?答案是 D

M: 首先要找到原文的关系.
Since anyone who supports the new tax plan (A)has no chance of being elected(非 B)==>A-->非 B;
anyone who truly understands economics (C)would not support the tax plan (非 A)==>C-->非 A;
结论是 B-->C. 可以看出结论有错误. B-->非 A-->C. 在这里第二个逻辑是原命题的逆命题,不一定成立,不得
既非 A 可能 C,也可能不能推出 C. 忽略的就是非 A 和非 C 的情况.

以上是一个详细的逻辑分析,在做题时要迅速抓住原文的错误, 找到答案.

Conservative: Socialists begin their arguments with an analysis of history, from which
they claim to derive certain trends leading inevitably to a socialist future. But in the day-
to-day progress of history there are never such discernible trends. Only in retrospect
does inevitability appear, for history occurs through accident, contingency, and individual
struggle.

Socialist: If we thought the outcome of history were inevitable, we would not work so hard
to transform the institutions of capitalist society. But to transform them we must first
understand them, and we can only understand them by an analysis of their history. This
is why historical analysis is important in socialist argument.
12. The socialist’s statements imply a conflict with the conservative’s view of history if the
conservative also holds that

(A) it would have been impossible for anyone to predict a significant period beforehand
that the institutions of capitalist society would take the form that they actually took

(B) the apparent inevitability of historical change is deceptive; all historical events could

284
have occurred otherwise than they actually did

(C) in the past, radical changes in social structures have mostly resulted in a
deterioration of social conditions

(D) since socialism cannot arise by accident or contingency, it can only arise as a result
of individual struggle

(E) because historical changes are mostly accidental, it is impossible for people to direct
their efforts sensibly toward achieving large-scale changes in social conditions

这个题目如何分辨选项,然后抛弃不正确的选项.答案是 E

M: 此题关键在于理解原文的意思. conservative 认为历史是不可预见的, 因为充满了 accident 等,所以 analysis


无用. 而 socialist 认为历史是不可预见的,所以要 transform,通过对了解历史事件对的影响,可以改变历史. 这
里你可以看到一个分歧, 就是 S 认为 accident 等也是历史时间的一部分,通过对它们的了解可以用于改变和
创造历史, 而 C 认为 accident 本身造成无法 analysis. 所以 E 对.

13. "addiction" has been defined as "dependence on and abuse of a psychoactive


substance" Dependence and abuse do not always go hand in hand, however. For
example, cancer patients can become dependent on morphine to relieve their pain, but
this is not abusing the drug. Correspondingly, a person can abuse a drug without being
dependent on it. Therefore, the definition of "addiction" is incorrect.

The relevance of the example of cancer patients to the argument depends on the
assumption that

(A) cancer patients never abuse morphine

(B) cancer patients often become dependent on morphine

(C) cancer patients who are dependent on morphine are addicted to it

(D) cancer patients who abuse a drug are dependent on it

(E) cancer patients cannot depend on morphine without abusing it

------------------------
答案 C, 不太明白问题的意思,好像不是问结论成立所需的假设,请大家指教。

M: 既然说了 cancer patient 的例子反驳关于 addition 的定义, 其前提必然是 cancer patient addited to morphine.

285
否则此例无效.

14. The commissioner has announced that Judge Khalid, who was on the seven-member panel
appointed to resolve the Amlec labor dispute will have sole responsibility for resolving the Simdo labor
dispute. Since in its decision the Amlec panel showed itself both reasonable and fair, the two sides in the
Simdon dispute are undoubtedly justified in the confidence they have expressed in the reasonableness
and fairness of the arbitrator assigned to their case.

Which one of the following contains flawed reasoning most parallel to that contained in the passage?

(A) Representing the school board. Matein Barthes presented to the school's principal a list of recently
elected school board members. Since only an elected member of the school board can act as its
representative. Ms. Barthes's name undoubtedly appears on that list.

(B) Alan Caldalf, who likes being around young children, has decided to become a pediatrician. Since the
one characteristic common to all good pediatricians is that they like young children. Mr. Caldalf will
undoubtedly be a very good pediatrician.

(C) Jorge Diaz is a teacher at a music school nationally known for the excellence of its conducting
faculty. Since Mr. Diaz has recently been commended for the excellence of his teaching, he is
undoubtedly a member of the school's conducting faculty.

(D) Ula Borg, who has sold real estate for Arcande Realty for many years, undoubtedly sold fewer houses
last year that she had the year before since the number of houses sold last year by Arcande Realty is far
lower than the number sold the previous year.

(E) The members of the local historical society unanimously support designating the First National Bank
building a historical landmark. Since Evelyn George is a member of that society, she undoubtedly favors
according landmark status to the city hall as well.

这个题目我的确对原文读完以后,提取不了什么本质信息?特别是不能快速地扔掉选项.

我觉得提干的主要思想是由于元素 A 属于集合 B,那么由于 B 具有 C 特性,A 也具有 C 特性。答案 d 正反


映了这个意思。即 Ula Borg 属于 Arcande Realty ,那么由于 Arcande Realty 具有 the number of houses sold
last yearis far lower than the number sold the previous year.这个特性,Ula Borg 也具有这个特性,即 he sold
fewer houses last year.

15. Magazine article: The Environmental Commissioner's new proposals are called "Fresh Thinking on
the Environment" and a nationwide debate on them has been announced. Well, "fresh thinking" from
such an unlikely source as the commissioner does deserve closer inspection. Unfortunately we
discovered that these proposals are virtually identical to those issued three months ago by Tsarque Inc,
under the heading "new Environmentalism" (Tsarque Inc's chief is a close friend of the commissioner).

286
Since Tsarque Inc's polluting has marked it as an environmental nightmare in our opinion the
"nationwide debate"can end here.

A flaw in the magazine article 抯 reasoning is that it

(A) assumes without any justification that since two texts are similar one of them must be influenced by
the other

(B) gives a distorted version of the commissioner 抯 proposals and then attacks this distorted version

(C) dismisses the proposals because of their source rather than because of their substance

(D) uses emotive language in labeling the proposals

(E) appeals to the authority of Tsarque Inc's chief without giving evidence that this person's opinion
should carry special weight

这种逻辑错误题目我 A/C 没有分开,我觉得选 A 更好,答案是 C.

a 选项的意思是:它没理由的认为由于两个建议相似而一定有一个影响了另一个。但是文中确实给出了理
由,即 Tsarque Inc 三个月前提出了相同的议案,而且两个组织之间有一定联系。姑且不论这个理由是否正
确,它毕竟是一个理由。

16. It is not reasonable to search out "organic" foods--those grown without the application of synthetic
chemicals ?as the only natural foods. A plant will take up the molecules it needs from the soil and turn
them into the same natural compounds, whether or not those molecules come from chemicals applied to
the soil. All compounds made by plants are part of nature , so all are equally natural.

The argument proceeds by

(A) redefining a term in a way that is favorable to the argument

(B) giving a reason why a recommended courses of action would be beneficial

(C) appealing to the authority of scientific methods

(D) showing that a necessary condition for correctly applying the term "organic" is not satisfied

(E) reinterpreting evidence presented as supporting the position being rejected

答案是 A,我的确有些想不通.我选的是 D,D 错在哪里了?

.这里 synthetic 是合成物. 它认为吸收合成物的作用和将自然物质单元自己合成是相同的.

287
所以是 A.

18. According to government official involved in overseeing airplane safety during the last
year, over 75 percent of the voice-recorder tapes taken from small airplanes involved in
relatively minor accidents record the whistling of the pilot during the fifteen minutes
immediately preceding the accident. Even such minor accidents pose some safety risk.
Therefore, if passengers hear the pilot start to whistle they should take safety
precautions, whether instructed by the pilot to do so or not.

The argument is most vulnerable to criticism on the grounds that it

(A) accepts the reliability of the cited statistics on the authority of an unidentified
government official

(B) ignores the fact that in nearly one quarter of these accidents following the
recommendation would not have improved passenger' safety

(C) does not indicate the criteria by which an accident is classified as "relatively minor"

(D) provides no information about the percentage of all small airplane flights during which
the pilot whistles at some time during that flight

(E) fails to specify the percentage of all small airplane flights that involve relatively minor
accidents
-----------------------------
答案 D,不知 A 为何错,感觉 E 也比 D 好,知道 D 但不知 the percentage of all small
airplane flights that involve relatively minor accidents,还是无法判断结论,为何选 D?请
大家指教。

M: 如果 90%的 flight 中 pilot 都 whistle, 远结论就错. 所以此项信息于结论十分关键, 不给不行. A 错在数据


并不是主要问题. E 无关.
原结论为口哨导致 accident. 举个例子: 一年一共 10000 flights, 其中有 100 起 minor accidents. 如果 10000 次
飞行中, 9000 次 pilot 都吹口哨, 原结论就不成立. 但如果就那 75 次 accident 中吹了, 原结论成立. 所以 D 中
的 information is critical in evaluating the conclusion. Without this piece of info, the conclusion is unfounded.

19. When permits for the discharge of chemicals into a waterway are issued, they are
issued in terms of the number of pounds each chemical that can be discharged into the
waterway per day. These figures, calculated separately for each chemical for which a
permit is issued, are based on an estimate of the effect of the dilution of the chemical by
the amount of water flowing through the waterway. The waterway is therefore protected
against being adversely affected by chemicals discharged under the permits.

288
The argument depends on the assumption that

(A) relatively harmless chemicals do not interact with each other in the water to form
harmful compounds

(B) there is a swift flow of water in the waterway that ensures rapid dispersion of
chemicals discharged

(C) those who receive the permits do not always discharge the entire quantity of
chemicals that the permits allow

(D) the danger of chemical pollution of waterways is to be evaluated in terms of human


health only and not in terms of the health of both human beings and wildlife

这个假设题目请大牛们重点讲解,A/B 我的确是没有分开的我觉得两个都是假设.答案是 A.

M: A 是相关选项, 与原文中的 separately 对应, 如果取非,不同的 chemical 合成新的 chemical, permit 的作用就
不存在了, 结论中的 permits --> protection 就不成立. 所以 A 是答案.

B 的问题在于 1. 和原文中的重点问题无关(见上) 2. 非必要条件,而是充分条件. 原文提到 dilution, 所以


chemical 要被冲散,而不是聚集在某处. B 是一个充分条件, 既:有 A,就有 dilution. 但是原文并没有提出 B 这
种情况是唯一的办法. 既还有可能通过其它手段来解决 dilution 的问题.

举一个例子: 为了节约用电, 要求不开灯或不用电器. 如果节约了用电,我们不能推出一定是没开灯或一定是


没用电器.

Monroe, despite his generally poor appetite, thoroughly enjoyed the three meals he ate at
the Tip-Top Restaurant, but, unfortunately, after each meal he became ill. The first time
he ate an extra-large sausage pizza with a side order of hot pepper: the second time he
took full advantage of the all-you-can-eat fried shrimp and hot peppers special and the
third time he had two of Tip-Top's giant meatball sandwiches with hot peppers. Since the
only food all three meals had in common was the hot peppers. Monroe concludes that it
is solely due to Tip-Top's hot peppers that he became ill.
21. If both Monroe's conclusion and the evidence on which he bases it are correct, they
would provide the strongest support for which one of the following?

(A) Monroe can eat any of Tip-Top's daily all-you-can-eat specials without becoming ill as
long as the special does not include the hot peppers.

(B) If, at his third meal at Tip-Top, Monroe had chosen to eat the baked chicken with hot
peppers, he would have become ill after that meal.

(C) If the next time Monroe eats one of Tip-Top's extra-large sausage pizzas he does not

289
have a side order of hot peppers, he will not become ill after his meal.

(D) Before eating Tip-Top's fried shrimp with hot peppers special, Monroe had eaten fried
shrimp without suffering any ill effects.

(E) The only place Monroe has eaten hot peppers has been at Tip-Top.

答案是 B,如何排除 A/C.

M: 还是充分必要条件的问题 . 这里已经确认的是 hot pepper 是问题的根源. 但是原文没有提到的是 hot


pepper 是唯一的原因. 所以 hot pepper 是充分非必要条件. 有 HP 就肯定 sick, 没有 HP 不一定不 sick.

22. "This company will not be training any more pilots in the foreseeable future, since we
have 400 trained pilots on our waiting list who are seeking employment. The other five
major companies each have roughly the same number of trained pilots on their waiting
lists, and since the projected requirement of each company is for not many more than
100 additional pilots, there will be no shortage of personnel despite the current upswing
in the aviation industry".

Which one of the following, if true, casts the most doubt on the accuracy of the above
conclusion?

(A) Most of the trained pilots who are on awaiting list for a job are on the waiting lists of
all the major companies

(B) In the long run, pilot training will become necessary to compensate for ordinary
attrition.

(C) If no new pilots are trained, there will be an age imbalance in the pilot work force.

(D) The quoted personnel projections take account of the current upswing in the aviation
industry.

(E) Some of the other major companies are still training pilots but with no presumption of
subsequent employment.

这个题目答案是 A,我觉得 A 是有关的但是加强,它是怎么削弱的?

M: 是一到数学题. 5 家公司,如果他们的 waiting list 上的 400 人是重复的话, 而且每家公司需要 100 人, 那么


总需求为 500, 而总供给是 400.

23. A car’s antitheft alarm tat sounds in the middle of the night in a crowded city

290
neighborhood may stop an attempted ca theft. On the other hand, the alarm might signal
only a fault in the device, or a response to some harmless contact, such as a tree branch
brushing the car. But whatever the cause, the sleep of many people in the neighborhood
is disturbed. Out of consideration for others, people who have these antitheft alarms on
their cars should deactivate them when they park in crowded city neighborhoods at night.
Which one of the following, if assumed by the author of the passage, would allow her
properly to draw her conclusion that the owners of alarm-equipped cars should
deactivate the alarms when parking in crowded city neighborhoods at night?
(A) The inconvenience of false alarms is small price to pay for the security of a
neighborhood.
(B) In most cases when a car alarm sounds at night, it is a false alarm.
(C) Allowing the residents of a crowded city neighborhood to sleep undisturbed is more
important than preventing car theft.
(D) People who equip their cars with antitheft alarms are generally inconsiderate of
others.
(E) The sounding of car antitheft alarms during the daytime does not disturb the residents
of crowded city neighborhoods.
answer:C my choice:B

In Peru, ancient disturbances in the dark surface material of a desert show up as light-
colored lines that are the width of a footpath and stretch for long distances. One group of
lines branching out like rays from a single point crosses over curved lines that form a
very large bird figure. Interpreting the lines in the desert as landing strips for spaceship-
traveling aliens, an investigator argues that they could hardly have been Inca roads,
asking, "what use to the Inca would have been closely spaced roads that ran parallel?
That intersected in a sunburst pattern? That came abruptly to an end in the middle of an
uninhabited plain".

24. The argumentative strategy of the investigator quoted is to

(A) reject out of hand direct counterevidence to the investigator's own interpretation

(B) introduce evidence newly discovered by the investigator which discredits the
alternative interpretation

(C) support one interpretation by calling into question the plausibility of the alternative
interpretation

(D) challenge the investigative methods used by those who developed the alternative
interpretation

291
(E) show that the two competing interpretations can be reconciled with one another

这个题目的 CD 两个选项我没有分开,答案是 C.这个题目的原文我也没有读的太懂,能给解释一下吗?

M: D 中的 investigation methods used by...是原文没有提到的.

25. For someone who interprets the lines as referring to astronomical phenomena, which one of the following, if
true, most effectively counters an objection that the crossing of the straight-line pattern over the bird figure shows
that the two kinds of line pattern served unrelated purposes?

(A) IN areas that were inhabited by ancient native North American peoples, arrangements of stones have been
found that make places where sunlight falls precisely on the spring solstice, an astronomically determined date.

(B) The straight lines are consistent with sight lines to points on the horizon where particular astronomical events
could have been observed at certain plausible dates, and the figure could represent a constellation.

(C) The straight-line pattern is part of a large connected complex of patterns of straight-line rays connecting certain
points with one another.

(D) Native Central American cultures, such as that of the Maya, left behind elaborate astronomical calendars that
were engraved on rocks.

(E) There is evidence that the bird figure was made well before the straight-line pattern.

题目没有太读懂,这个题目只能猜了,它还居然对了,答案是 B.

M: 最好用排除法. 原文说有两种 line, 一种形成一个鸟的图案,另一种是直线. 问如何反驳那种认为两种线作


用无关的说法. B 说明两种线有关, 直线是指出可以观测到某种天象的方位, 鸟线指出是哪个星系.

292
LSAT-16-2

1. Walter: Although cigarette smoking is legal, it should be banned on all airline flights.
Cigarette smoking in the confines of an aircraft exposes nonsmokers to harmful
secondhand smoke that they cannot avoid.

Which one the following principles, if established, would justify the proposal put forth
by Walter?

(A) People should be prohibited from engaging in an otherwise legal activity would
unavoidable expose others to harm.

(B) An activity should be banned only if most situations in which a person engaged in
that activity would inevitably expose others to harm.

(C) A legal activity that has the potential for causing harm to others in certain
situations should be modified in those situations to render it harmless.

(D) People who regularly engage in an activity that has the potential for harming
others when that activity takes place in certain situations should be excluded from

293
those situations.

(E) If an activity is legal in some situations in which a person's engaging in that


activity could harm others, then that activity should be legal in all situations.

这个 A 中的"otherwise "应该如何理解?作题的时候就是对 otherwise legal activity 的理


解错误导致误选.

let me try to help you to type Chinese。

应该禁止这样一种情况发生:在这种情况下,那些在别的地方合法的行为对其他人有害。

3. A certain stain of bacteria was found in the stomachs of ulcer patients. A medical
researcher with no history of ulcers inadvertently ingested some of the bacteria and
within weeks developed an ulcer. Therefore, it is highly likely that the bacteria strain
induces ulcers.
Which one of the following, if true, most supports the argument above?
(A) People who have the bacteria strain in their stomachs have been found to have no
greater incidence of kidney disease that do people who lack the bacteria strain.
(B) The researcher did not develop any other serious health problems within a year after
ingesting the bacteria strain.
(C) There is no evidence that the bacteria strain induces ulcers in laboratory animals.
(D) The researcher is a recognized expert in the treatment of diseases of the stomach.
(E) A study of 2000 people who do not have ulcers found that none of these people had
the bacteria strain in their stomachs.

答案是 E,LSAT 里原题没有 E 选项。。我想问的是 D 为什么不对?好象也是 SUPPORT 啊,


虽然比较弱的 SUPPORT

a-b, 非 b-非 a

我认为 D 是支持 bacteria 为 ulcer 的必要条件, 而不是支持其为充分条件. 而原文的结论是 bacteria 造成 ulcer.

7. Workers may complain about many things at work, but stress is not high on the list. In
fact, in a recent survey a majority placed boredom at the top of their list of complaints.
The assumption that job-related stress is the most serious problem for workers in the
corporate world is thus simply not warranted.

Which one of the following, if true, most seriously weakens the argument?

(A) Those workers who are responsible for the planning and supervision of long-term
projects are less likely to complain of either boredom or stress

294
(B) Workers who complain of boredom exhibit more stress-related symptoms than do
those who claim their work is interesting,

(C) Workers responding to opinion surveys tend to emphasize those experiences that
have happened most recently

(D) Workers who feel that their salaries are commensurate with the amount of work they
do are less likely to complain of boredom.

(E) Workers are less likely to complain about work if they feel that their jobs are secure

这个题目答案是 B,请解释选项是如何削弱原文的?其他选项为什么又不对?这个题目请重点
讲解.

11. Should a Journalist's story begin with the set phrase "In a surprise development" as
routinely happens? Well, not if the surprise was merely the journalist' s, since journalists
should not intrude themselves into their stories, and not if the surprise was someone
else's, because if some person's surprise was worth mentioning at all, it should have
been specifically attributed. The one possibility remaining is that lots of people were
surprised: in that case, however, there is no point in belaboring the obvious.

Which one of the following most accurately states the conclusion of the argument above?

(A) Journalists should reserve use of the phrase "In a surprise development" for major
developments that are truly unexpected.

(B) The phrase "in a surprise development" is appropriately used only where someone's
being surprised is itself interesting.

(C) The phase "in a surprise development" is used in three distinct sorts of
circumstances.

(D) Journalists should make the point that a development comes as a surprise when
summing up, not when introducing a story.

(E) Introducing stories with the phrase "in a surprise development" is not good journalistic
practice.

这个题目原文读的很晕,重要请讲解一下原文.答案是 E.

12. Individual pyrrole molecules readily join together into larger molecules called

295
polypyrroles. If polypyrroles form from pyrrole in the presence of zeolites, they do so by
attaching to the zeolite either in lumps on the outer surface of the zeolite or in delicate
chains within the zeolite 抯 inner channels. When zeolite changes color from yellow to
black, it means that on or in that zeolite polypyrroles have formed from pyrrole. Yellow
zeolite free of any pyrrole was submerged in dissolved pyrrole. The zeolite, turned black
even though no polypyrroles formed on its outer surface.

If the statements above are true, which one of the following must on the basis of them be
true?

(A) Polypyrroles had already formed on or in the zeolite before it was submerged.

(B) Lumps of polypyrrole attached to the zeolite were responsible for its color change.

(C) At least some of the pyrrole in which the zeolite was submerged formed polypyrrole
chains.

(D) None of the pyrrole in which the zeolite was submerged attached itself to the zeolite.

(E) Little, if any, of the pyrrole in which the zeolite was submerged reached the zeolite 抯
inner channels.

这个题目作对了,但是原文读的不是很清楚.用排除法得到 C,请问原文提取了哪些信息?如何
从原文得到 C.

M: 分析原文,可以得出以下关系: a. 在 Z 存在时,PY 形成 PO 的方式有两种--1.lumps on the outer surface


2.chain within inner channel; b. 当 Z 变色时, 说明 PY 形成 PO(或 inner 或 outer). 现知道 Z 变色,且 outer surface
没有 PO, 可以正推出必然有 PO 在 inner 形成. 既 C

Pedigreed dogs, including those officially classified as working dogs, must conform to standards set by
organizations that issue pedigrees. Those standards generally specify the physical appearance necessary
for a dog to be recognized as belonging to a breed but stipulate nothing about other genetic traits, such
as those that enable breeds originally developed as working dogs to perform the work for which they
were developed. Since dog breeders try to maintain only those traits specified by pedigree organizations,
and traits that breeders do not try to maintain risk being lost, certain traits like herding ability risk being
lost among pedigreed dogs. Therefore, pedigree organizations should set standards requiring working
ability in pedigreed dogs classified as working dogs.

13. Which one of the following principles, if valid, justifies the argument's conclusion that pedigree
organizations should set standards for working ability in dogs?

(A) Organizations that set standards for products or activities should not set standards calling for a
particular characteristic if such standards increase the risk of some other characteristic being lost.

296
(B) Any standard currently in effect for a product or an activity should be rigorously enforced regardless
of when the standard was first set.

(C) Organizations that set standards for products or activities should be responsible for seeing to it that
those products or activities conform to all the specifications called for by those standards

(D) Any standard that is set for a product or an activity should reflect the uses to which that product or
activity will eventually be put.

(E) Organizations that set standards for products or activities should attempt to ensure that those
products or activities can serve the purposes for which they were originally developed.

这个题目原文没有读懂?请重点讲解一下这个题目的原文和解题思路??

I choose E. The passage says that the requirements on work dogs have nothing to do with the dog's ability to fulfill
it duty as it was originally developed to. So the requirement should be set to preserve the generic traits related to
those jobs.

A B and C can be eliminated quickly as none of them deals with what was in the passage. D is a bit tricky. But
"uses to which ....eventually put to" is quite different from "that enable breeds originally developed as working
dogs to perform the work for which they were developed". It is mentioned in the passage that the working dogs
will really work in those jobs.

原文:pedigree 的标准,保证了外形,但是忽略了 working ability 这一 genetic traits。


13. Which one of the following principles, if valid, justifies the argument's conclusion that pedigree organizations
should set standards for working ability in dogs?
评价:P 组织应该设立标准:包含狗的 working ability.
(E) Organizations that set standards for products or activities should attempt to ensure that those products or
activities can serve the purposes for which they were originally developed.
一个 general 的 rule:标准应该确保预期目标得以实现。
具体到狗,预期标准应该是:所有 genetic traits(原文的 assumption),包含外形和 working ability. 现在的
标准只有外形,预期目标无法实现。所以,要修正标准。加强了结论。

这个评价题,我自己觉得费劲的地方是要读出其 assumption,最终评价 assumption。而这个 assumption 是要


经过推理才有的。gwd 估计可以用排除法得到答案,这么复杂的句子,这么多事儿,我记不住,所以就没
有尝试老倌的方法。

16. In their native habit, amaryllis plants go dormant when the soil in which they are growing dries out
during the dry season. Therefore, if amaryllis plants kept as houseplants are to thrive, water should be
withheld from them during part of the year so that the plants go dormant.

297
Which one of the following is an assumption on which the argument depends?

(A) Most kinds of plants go dormant at some time or other during the year.

(B) Amaryllis are more difficult keep as houseplants than other kinds of plants are.

(C) Water should be withheld from amaryllis plants kept as houseplants during the exact time of year
that corresponds to the dry season in their native habitat

(D) Any amaryllis plant that fails to thrive is likely to have been dormant for too short a time.

(E) Going dormant benefits amaryllis plants in their native habitat in some way other than simply
preventing death during overly dry periods.

这个题目答案给的是 E,我选的是 C,

This one is pretty obvious. C is not within the boundary of the logic in the passage. It goes beyond the point with
the logic ends. It is more of a inference from the reasoning in the passage. If the pattern is being followed even
when the plant is in-house, it is more likely that the exact time is followed.

The gap in the passage is "thrive" and "natural'. The plant will go dormant in nature because of the dry season.
Then the passage leaped to the conclusion that the practice is necessary for the plant to "thrive". But the connection
between the natural practice and thriving is unspecified, and therefore, is a assumption.

Another example, when John was poor, he only had one deal a day. So when he is rich, he should eat one meal a
day to stay healthy. The conclusion is wrong if there is no such assumption that John used to eat one meal for the
purpose of staying healthy.

原文说,某种植物在野外旱季要睡觉觉。结论要让它长得好,在室内也要让他睡一段时间。

可能这种植物要睡觉的特性只是为了抗旱呢?

17. Most people believe that yawning is most powerfully triggered by seeing someone else yawn. This
belief about yawning is widespread not only today, but also has been commonplace in many parts of the
world in the past, if we are to believe historians of popular culture. Thus, seeing someone else yawn must
be the most irresistible cause of yawning.

The argument is most vulnerable to which one of the following criticisms?

(A) It attempts to support its conclusion solely by restating that conclusion in other words.

(B) It cites the evidence of historians of popular culture in direct support of a claim that lies outside their

298
area of expertise.

(C) It makes a sweeping generalization about yawning based on evidence drawn from a limited number
of atypical cases.

(D) It supports its conclusion by appealing solely to opinion in a matter that is largely factual.

(E) It takes for granted that yawns have no cause other than the one it cites.

这个题目的答案是 D,d 是如何反映原文的信息?


仅仅以对一件很大程度上是事实的事情的意见作为依据得出结论。原文说的意见是指历史学家的意见(if
we are to believe historians of popular cultur ) , 历 史 学 家 对 大 众 文 化 的 看 法 基 本 上 是 个 事 实
(commonplace)。

19. Without information that could only have come from someone present at the secret
meeting between the finance minister and the leader of the opposition party, the
newspaper story that forced the finance minister to resign could not have been written.
No one witnessed the meeting, however, except the minister's aide. It is clear, therefore,
that the finance minister was ultimately brought down, not by any of his powerful political
enemies, but by his own trusted aide.
The argument commits which one of the following errors of reasoning?
(A) drawing a conclusion on the basis of evidence that provides equally strong support
for a competing conclusion
(B) assuming without warrant that if one thing cannot occur without another thing s
already having occurred, then the earlier thing cannot occur without bringing about the
later thing
(C) confusing evidence that a given outcome on one occasion was brought about in a
certain way with evidence that the same outcome on a different occasion was brought
about in that way
(D) basing its conclusion on evidence that is almost entirely irrelevant to the point at
issue
(E) treating evidence that a given action contributed to bringing about a certain effect as
though that evidence established that the given action by itself was sufficient to bring
about that effect
答案:A,为什么? 我选了 C

M: minister 是和 opposition party 会谈, 所以很可能是 opposition party leader 将消息透露出去, 所以原文的
evidence 同样支持 competing conclusion, 即"brought down by his powerful political enemies"

24. People were asked in a survey how old they felt. They replied, almost unanimously

299
despite a great diversity of ages, with a number that was 75 percent of their real age.
There is, however, a problem in understanding this sort of response. For example,
suppose it meant that a 48-year-old man was claiming to feel as he felt at 36. But at age
36 he would have said he felt like a man of 27, and at 27 he would have said he felt just
over 20, and so on into childhood. And surely, that 48-year-old man did not mean to
suggest that he felt like a child!
Which one of the following techniques of reasoning is employed in the argument?
(A) projecting from responses collected alone time from many individuals of widely
different ages to hypothetical earlier responses of a single individual at some of those
ages
(8) reinterpreting what certain people actually said in the light of what would, in the
circumstances, have been the most reasonable thing for them to say
(C) qualifying an overly sweeping generalization in light of a single well chosen
counterexample
(D) deriving a contradiction from a pair of statements in order to prove that at least one of
those statements is false
(E) analyzing an unexpected unanimity among respondents as evidence, not of a great
uniformity of opinion among those respondents but of their successful manipulation by
their questioners
答案: A 为什么?

LSAT-16-3

Those who support the continued reading and performance of Shakespeare's plays
maintain that in England appreciation for his work has always extended beyond educated
elites and that ever since Shakespeare's own time his plays have always been known
and loved by comparatively uneducated people. Skepticism about this claim is borne out
by examining early eighteen-century editions of the plays. These books, with their fine
paper and good bindings, must have been far beyond the reach of people of ordinary
means.
1. The main point of the argument is to
(A) suggest that knowledge of Shakespeare's play is a suitable criterion for distinguishing
the educated elite from other members of English society
(B) provide evidence that at some time in the past appreciation for Shakespeare's play
was confined to educated elites
(C) prove that early eighteenth-century appreciation for Shakespeare's works rested on

300
aspects of the works that are less appreciated today
(D) demonstrate that since Shakespeare's time the people who have known and loved
his work have all been members of educated elites
(E) confirm the skepticism of the educated elite concerning the worth of Shakespeare's
plays
answer: B why not D?

文中明显的只提到 18 世纪,而 D 则根据这个事实得到了一个 general 的结论,所以不对。

3. Organization president: The stationery and envelopes used in all of the mailings from
our national headquarters are made from recycled paper, and we never put anything but
letters in the envelopes. When the envelopes have windows are also made from recycled
material. Therefore the envelopes and thus these mailings, are completely recyclable.

Which one of the following is an assumption on which the organization president's


argument depends?

(A) All the paper used by the organization for purposes other than mailings is recycled.

(B) The mailing from the organization's national headquarters always use envelopes that
have windows.

(C) The envelope windows made from recycled material are recyclable.

(D) The envelopes and stationery used in the organization's mailings are always
recycled.

(E) The organization sends mailings only from its national headquarters.

答案:C

C 正确是不是因为 made from recycled material 并不一定 recyclable? D 又错在哪里了?别


怪我问这么弱的问题:(

the key word here is "completely"

from his talk, we know that, envelope and letters are made from recycled paper and ,
therefore, must be recyclable. but in adressing of envelop with window, we are not
informed whether window is made from recycled materials.

But , complete mailing include envelop, window and letter.

C provides the guarantee.

Questions4-5

301
The frequently expressed view that written constitutions are inherently more liberal than
unwritten ones is false. No written constitution is more than a paper with words on it until
those words are both interpreted and applied. Properly understood, then, a constitution is
the sum of those procedures through which the power of the state is legitimately
exercised and limited. Therefore, even a written constitution becomes a liberal
constitution only when it is interpreted and applied in a liberal way.
4. The main point of the argument above is that
(A) written constitutions are no more inherently liberal than are unwritten constitutions
(B) the idea of a written constitution, properly understood, is inherently self-contradictory
(C) unwritten constitutions are less subject to misinterpretation than are constitutions that
have been written down
(D) liberal constitutions are extremely difficult to preserve
(E) there are criteria for evaluating the interpretation and application of a constitution.

5. If the statements in the argument are all true, which one of the following must also be
true on the basis of them?
(A) careful analysis of the written text of a constitution can show that the constitution is
not a liberal one.
(B) It is impossible to determine that a written constitution is liberal merely through careful
analysis of the written text.
(C) There are no advantages to having a written rather than an unwritten constitution.
(D) Constitutions that are not written are more likely to be liberal than are constitutions
that are written.
(E) A constitution is a liberal constitution if it is possible to interpret it in a liberal way.

答案:B

我选的是 E,我觉得 E 和文种最后还是挺像的,错在哪里了?B 是怎么推出来的?谢了!

a written constitution becomes a liberal constitution only when it is


interpreted and applied in a liberal way.

constitution is a liberal (A)--------- it is interpreted and applied in a


liberal way(B) A 是 B 的充分条件

(E) A constitution is a liberal constitution if it is possible to interpret


it in a liberal way.

interpret it in a liberal way(B)--------constitution is a liberal (A) B 是


A 的充分条件

302
6. As far as we know, Earth is the only planet on which life has evolved, and all known life forms are
carbon-based. Therefore, although there might exist noncarbon-based life on planets very unlike Earth,
our scientific estimates of the probability of extraterrestrial life should be generated from estimates of
the number of planets like Earth and the likelihood of carbon-based life on those planets.

Which one of the following general principles most strongly supports the recommendation?

(A) There is no good reason to think that unobserved phenomena closely resemble those that have been
observed.

(8) A scientific theory that explains a broad range of phenomena is preferable to a competing theory that
explains only some of those phenomena.

(C) It is preferable for scientists to restrict their studies to phenomena that are observable and forego
making estimates about unobservable things.

(D) A scientific theory that explains observed phenomena on the basis of a few principles that are
independent of each other is preferable to a theory that explains those same phenomena on the basis of
many independent principles.

(E) Estimations of probability that are more closely tied to what is known are preferable to those that are
less closely tied to what is known.

我选 C,觉得 C 说的更加合理。E 是答案是如何反映原文的。

原文说的是“因为地球生命有这种性质,所以我们的研究应该倾向与和地球相似的星球,因为这个可能大”
C 说研究应该局限于可观察的现象。。。原文根本没提到 observable 这个词。。如果你的 observable 是因为地球
is observable 推出来的。。所以原文要观察的对象不是“obserbable"

E Estimations of probability that are more closely tied to what is known are preferable to those that are less
closely tied to what is known.
正是强调了原文的对比

Consumer advocate: The toy-labeling law should require manufacturers to provide explicit safety labels
on toys to indicate what hazards the toys pose. The only labels currently required by law are labels
indicating the age range for which a toy is intended. For instance, a "three and up" label is required on
toys that pose a choking hazard for children under three years of age. Although the current toy-labeling
law has indeed reduced the incidence of injuries to children from toys, parents could prevent such
injuries almost entirely if toy labels provided explicit safety information.

10. Which one of the following, if true, most strengthens the consumer advocate's argument:

(A) Certain types of toys have never been associated with injury to children.

303
(B) Most parents believe that the current labels are recommendations regarding level of cognitive skill.

(C) The majority of children injured by toys are under three years of age.

(D) Many parents do not pay attention to manufacturers' labels when they select toys for their children.

(E) Choking is the most serious hazard presented to children by toys.

这是一个加强题目。我选 E 答案是 B,B 是如何加强原文的?我觉得 B 是一个无关的答案。

(B) Most parents believe that the current labels are recommendations regarding level of cognitive skill.
大多数家长认为产品标签上只是关于认知水平的一个推荐(并不是所谓的安全提示)
正如例子中提到的标签上说明适合 3 岁以上小孩玩(3 岁以上小孩的智力水平)
同时说明 labels have not explictly safety information
因此加强了结论.

14. Although inflated government spending for weapons research encourages waste at
weapons research laboratories, weapons production plants must be viewed as equally
wasteful of taxpayer dollars. After all, by the government's own admission, the weapons
plant it plans to reopen will violate at least 69 environmental, health, and safety laws. The
government has decided to reopen the plant and exempt it from compliance, even though
the weapons to be produced there could be produced at the same cost at a safer facility.
The reasoning in the argument is most vulnerable to criticism on which one of the
following grounds?
(A) It offers no evidence that the "safer'" alternative production site actually complies with
any of the jaws mentioned.
(8) It concedes a point regarding weapons research laboratories that undermines its
conclusion about weapons production plants.
(C) It relies on evidence that does not directly address the issue of wasteful spending.
(D) It confuses necessary expenditures for research with wasteful spending on weapons
(E) It fails to establish that research laboratories and weapons production plants are
similar enough to be meaningfully compared.
Answer: C 为什么?

文章的首句就说明要讨论的是浪费问题,可接下来的文字就偏离对浪费的讨论,开始说合规什么的,c 指
出了这一问题。

Questions15-16
Dr. Godfrey: Now that high school students are allowed to work more than 15 hours per
week at part-time jobs, those who actually do so show less interest in school and get
lower grades thin those who do not work as many hours at part-time jobs. Obviously,
working long hours at part-time jobs during the school year contributes to the academic

304
problems that many of our high school students experience.
Dr. Nash: That's not so. Many of our high school students set out to earn as much money
as they can simply to compensate for their lack of academic success.

15. Dr. Nash responds to Dr. Godfrey's argument by doing which one of the following?
(A) attempting to downplay the seriousness of the problems facing academically troubled
high school students
(B) offering an alternative interpretation of the evidence cited by Dr. Godfrey
(C) questioning the accuracy of the evidence on which Dr. Godfrey bases his conclusion
(D) proposing that the schools are not at fault for the academic problems facing many
high school students
(E) raising the possibility that there is no relationship between academic problems among
high school students and part-time employment
答案:B
我选的 A。我能理解 B,但 A 呢?我觉得 Nash 觉得挣钱可以补偿 academic problem,他不
也就是这不是什么严重的问题吗?帮忙解释一下,谢谢。

Dr. Godfrey: Part-time jobs cause academic problems. 打工在先,导致学业差


Dr. Nash: Part-time jobs compensate for acdemic problems, which means part-time jobs
are the effect of bad acdemic performance. 学业差,所以用打工弥补
It doesn't show whether Dr.Nash downplays the problem's seriousness. Dr.Nash just
wants to correct the reason and the effect which Dr Godfrey claims to be.

16. The answer to which one of the following would be the most helpful in determining
whether the conclusion that Dr. Godfrey draws could be logically defended against Dr.
Nash's counterargument?
(A) whether people who have had academic problems in high school are ultimately less
successful in their careers than people who have not had such problems
(B) whether students are allowed to spend more than 15 hours per week at school-
sponsored nonacademic extracurricular activities such as team sports or clubs
(C) whether the students who work more than 15 hours per week and have academic
problems had such problems before they began to work that many hours
(D) whether employers and high school students typically obey all the laws that regulate
the conditions under which young people may legally be employed
(E) whether high school students who have after-school jobs continue to work at those
jobs after graduating from high school

答案:C
我选的 A。A 不是正好说如果有 academic problems 无论是否当时挣钱,对以后都不利吗?
而 C,既然 Nash 已经不觉得 academic problems 有什么了,Godfrey 再比较做不做 part
time 有没有影响还有意义吗?还有,我觉得 C 就是对原文第一句的重复呀?
谢谢了。

305
(A) whether people who have had academic problems in high school are ultimately less
successful in their careers than people who have not had such problems
在学校学业差的学生是否在以后的职业生涯中 less successful ,比起学业好的学生?这跟
主题无关。
二人是在争论学业好坏和打工,哪个导致哪个的问题。
C 说要看究竟那些打工,学业差的学生在没打工之前是不是也学业差。
如果没打工之前,学业不差,那就说明是打工导致了学业差。
如果没打工之前,学业差,那打工有可能导致学业更差。这样 Dr. Godfrey draws could be
logically defended against Dr. Nash's counterargument。 G 就可以否定 N 的说法了。

18. In experiments in which certain kinds of bacteria were placed in a generous supply of
nutrients, the populations of bacteria grew rapidly, and genetic mutations occurred at
random in the populations. These experiments show that all genetic mutation is random.
Which one of the following, if true, enables the conclusion to be properly drawn?
(A) Either all genetic mutations are random or none are random.
(B) The bacteria tested in the experiments were of extremely common forms.
(C) If all genetic mutations in bacteria are random, then all genetic mutations in every
other life form arc random also.
(D) The kind of environment in which genetic mutation takes place has no effect on the
way genetic mutation occurs.
(E) The nutrients used were the same as those that nourish the bacteria in nature.
Answer: A why not B C D

文中只说了一种细菌在某种条件下的基因变异,然后就得出所有的基因变异是随机的。B 不对,因为此种
细菌是否是普通的,不能作为一个得出 general 结论的依据,试想,人的基因突变和细菌一样吗?C 的错
误在于文中并未能证明是否所有的细菌的基因变异是随机的。 D 只说明环境对此种细菌的基因变异没有影
响,同样不能解决细菌的基因变异和泛泛意义上的基因变异有和关系。

原文有个 GAP: certain kinds of bacteria RANDOM ---> all genetic mutation is random.
特殊例子推出一般结论
B 如果真,可以让这成立。。B 的意思是要么所有的都不是 RANDOM,要么都是 RANDOM 的。

19. Thomas: The club president had no right to disallow Jeffrey's vote. Club rules say that only members
in good standing may vote. You've admitted that club rules also say that all members whose dues are
fully paid are members in good standing. And since, as the records indicate, Jeffrey has always paid his
dues on time, clearly the president acted in violation of club rules.

Althea: By that reasoning my two-year-old niece can legally vote in next month's national election since
she is a citizen of this country, and only citizens can legally vote in national elections.

The reasoning in Thomas' argument is flawed because his argument

(A) fails to take into account the distinction between something not being prohibited and its being

306
authorized

(B) offers evidence that casts doubt on the character of the club president and thereby ignores the
question of voting eligibility

(C) wrongly assumes that if a statement is not actually denied by someone, that statement must be
regarded as true

(D) does not specify the issue with respect to which the disputed vote was cast

(E) overlooks the possibility that Althea is not an authority on the club's rules

这个 A 答案是如何反映原文的逻辑错误的,原文的逻辑错误具体表现在什么地方?

原文错误在把必要条件当充分条件。
只有 good standing 的人可以选,但不是说明 good standing 的人一定可以选。
A 的意思是说把不被禁止和同意混淆了. 有了 good standing 只是不被禁止,但并不表示同意。

23. Asbestos, an almost indestructible mineral once installed as building insulation, poses
no health risk unless the asbestos is disturbed and asbestos fibers are released into the
environment. Since removing asbestos from buildings disturbs it, thereby releasing
asbestos fibers, the government should not require removal of all asbestos insulation.
Which one of the following, if true, most strengthens the argument?
(A) Asbestos poses far less risk to health than does smoking, drug and alcohol abuse,
improper diet or lack of exercise.
(B) Asbestos can post a health threat to workers who remove it without wearing required
protective gear.
(C) Some kinds of asbestos, when disturbed, pose greater health risks than do other
kinds.
(D) Asbestos is inevitably disturbed by building renovations or building demolition.
(E) Much of the time, removed asbestos is buried in landfills and forgotten with no
guarantee that it will not be disturbed again.

答案:E

我选的 D。看过 Lawyer 的解释,很遗憾还是没懂。我认为 D 对的原因是 renovation 和


demolition 我认为是说对 building 整体处理,从而加强结论不需要 removal…(有点牵强)。
但 E 呢?怎么加强了?不还是 removed 了吗?No guarantee that it will not be disturbed
again 又起到什么作用?

disturb + fiber release = risk

307
D , only get disturb, not sure whether fiber will release.

E, removed materials both disturb and release fiber, (Since removing asbestos
from buildings disturbs it, thereby releasing asbestos fibers) , so , causing rish.

remove itself will guarantee the risk.

Question24
When volcanic lava solidifies, it becomes uniformly magnetized in the direction in which
the Earth's magnetic field points. There are significant differences in the direction of
magnetization among solidified lava flows from different volcanoes that erupted at
different times over the past several million years. Therefore, it must be that the direction
of the Earth's magnetic field has changed over time. Since lava flows differing by
thousands of years in age often have very similar directions of magnetization, the change
in the direction of the Earth's magnetic field must take place very gradually over
hundreds of thousands of years.
24. The argument that the direction of the Earth's magnetic field has changed over time
requires the assumption that
(A) only lava can be used to measure the direction of the Earth's magnetic field as it
existed in the distant past
(B) a single volcano can produce lava of differing consistencies during different eruptions
(C) not all solidified lava has changed the direction of its magnetization unpredictably
(D) there are fewer volcanic eruptions now than there were millions of years ago
(E) as lava flows down the side of a volcano, it picks up magnetized rocks
answer: C 为什么?

谢谢!

大前提是岩浆磁场和地球的磁场走向一致,小前提是许多岩浆的走向不一致,结论是地球的磁场走向变化
了。我们测量岩浆的磁场是现在,这就要求在这许多年里,岩浆的磁场没有变化,或没有无法预期的变化。

26. When the manufacturers in a given country are slower to adopt new technologies
than their foreign competitors are, their production costs will fall more slowly than their
foreign competitors' costs will. But if manufacturers' production costs fall less rapidly than
their foreign competitors' costs do, those manufacturers will be unable to lower their
prices as rapidly as their foreign competitors can; and when a country's manufacturers
cannot lower their-prices as rapidly as their foreign competitors can, that country gets
squeezed out of the global market.

If the statements above are true, which one of the following must also be true on the
basis of them?

308
(A) If the manufacturers in one country raise their prices, it is because they have
squeezed their foreign competitors out of the global market.

(B) If manufacturers in one country have been squeezed out of the global market, this
shows that their foreign competitors have adopted new technologies more rapidly than
they have.

(C) If a country's foreign competitors can lower their production costs more rapidly than
the country's own manufacturers can, then their foreign competitors must have adopted
new manufacturing techniques.

(D) If a country's manufacturers adopt new technologies at the same rate as their foreign
competitors, neither group will be able to squeeze the other out of the global market.

(E) If a country's manufacturers can lower their prices as rapidly as their foreign
competitors can, this shows that they adopt new technology at least as fast as their
foreign competitors do.

 
这个题目的确是把我比来比去就给搞晕了!答案是 E

列出逻辑关系就很清楚了:
原文:

1。 fail to adapt new...than ...---> cost fall more slow than ...
2. cost less rapidly than...---> unalbe to low price as quickly as...
3. unalbe to low price as quick as ...--> get out

E) 利用 2 和 1 的逆否命题可以推出。。

其他选项可以一一排除。。注意只有逆否命题等价。。其他选项都使用了否命题或者逆命题。可以排除。

LSAT-17-3

3. Early in this century, Alfred Wegener developed the concept of continental drift . His
ideas were rejected vehemently because he postulated no identifiable force strong
enough to make the continents move. We have come to accept Wegener's theory, not
because we have pinpointed such a force, but because new instruments have finally
allowed continental movement to be confirmed by observation.

The passage best illustrates which one of the following statements about science?

309
(A) The aim of science is to define the manifold of nature within the terms of a single
harmonious theory.

(B) In a accepting a mathematical description of nature, science has become far more
accurate at identifying underlying forces.

(C) The paradox of science is that every improvement in its measuring instruments
seems to make adequate theories harder to work out.

(D) Science employing statistics and the laws of probability, is concerned not with the
single event but with mass behavior.

(E) When the events a theory postulates are detected, the theory is accepted even
without an explanation of how those events are brought about.

Answer is E.
I was totally no idea how to do it. Please help!!!

这道题目主要是阅读理解。。
原文说: 虽然我们还无法知道是什么力量推动了板块。。但是由于观察到了他确实移动,所以我们不得不
接受大陆漂移理论。。

E 当某个理论支持的现象被发现了。。即使无法知道什么产生这个现象,我们也可以接受这个理论。

I think you problem is reading comprehension, do more reading..you will get improvement..

4. The Theory of military deterrence was based on a simple psychological truth, that fear
of retaliation makes a would-be aggressor nation hesitate before attacking and is often
sufficient to deter it altogether from attacking. Clearly, then, to maintain military
deterrence, a nation would have to be believed to have retaliatory power so great that a
potential aggressor nation would have reason to think that it could not defend itself
against such retaliation.

If the statements above are true, which one of the following can be properly inferred?

(A) A would-be aggressor nation can be deterred from attacking only if it has certain
knowledge that it would be destroyed in retaliation by the country it attacks.
(B) A nation will not attack another nation if it believes that its own retaliatory power
surpasses that of the other nation.
(C) One nation's failing to attack another establishes that the nation that fails to attack
believes that it could not withstand a retaliatory attack from the other nation.

310
(D) It is in the interests of a nation that seeks deterrence and has unsurpassed military
power to let potential aggressors against it become aware of its power of retaliatory
attack.
(E) Maintaining maximum deterrence from aggression by other nations requires that a
nation maintain a retaliatory force greater than that of any other nation.

不明白答案 D 的含义。分析:国家利益(?)原文没有提到过,unsurpassed 原文似乎也没有说


这个 D 好像最应该排除 ;)

“retaliatory power so great” that " it could not defend itself against such retaliation" 就 是 说 has unsurpassed
military power。 而且 in the country's interest 不一定就是这具体的国家利益。 它是说 符合这个国家的利益。
或者简单说就是对这个国家有利

原文是说:A 国只要报复的打击>B 国所能抵挡的力量。。则 A 国对 B 国就有威慑力。


E 讲的是有威慑力的条件是:A 国家报复的力量>B 国家报复的力量。。比较的方式和原文不同

M: 此题用排除法可以很快得到答案.

A,C 错在原文中 deterence 是充分条件, AC 中变成了必要条件.


B,E 原文没有 relaliatory power 的比较.
D 中的 interest 是 common sense. 不被侵略当然是 national interest.

5. To the editor:
In 1960, an astronomer proposed a mathematical model for determining whether
extraterrestrial life exists.It was based on the assumptions that life as we know it could
exist only on a planet and that many stars are,like our Sun, orbited by planets, On the
basis that there are nine planets in our solar system and one of them
has life as we know it, the astronomer predicted that there are as many as one million
extraterrestrial civilizations across all solar systems. Yet astronomers to date have not
detected even one planet outside our solar system. This indicates that the astronomer's
model is wrong, and life as we know it exists only on the
plant Earth. Clay Moltz
Which one of the following, if accepted by Clay Moltz would require him to reconsider his
conclusion?
(A) Forms of life other than life as we know it exist on other planets.
(B) There are many stars that are not orbited by planets.
(C) Detecting planets outside our solar system requires more sophisticated instruments
than are currently available.
(D) The soundness of the conclusion reached by applying a mathematical model
depends on the soundness of the assumptions on which the model is based.
(E) Due to sheer distances and expanses of space involved, any extraterrestrial
civilization would have great difficulty communicating with ours.
answer:C my choice:A

311
Clay Moltz 是 拿 没 发 现 planet outside our solar system (life) 当 证据 ,证 明不 存在 planet outside our solar
system(life). 他忽略了可能是因为探测手段不足, 而还没发现。 正是(C)讲的。

8. Fines levied against those responsible for certain environmentally damaging


accidents are now so high that it costs a company responsible for such an accident
more to pay the fine than it would have cost to adopt measures that would have
prevented the accident. Therefore, since businesses value their profits, those that
might have such accidents will now install adequate environmental safeguards

Which one of the following, if true, most seriously weakens the argument?

(A) Businesses generally greatly underestimate the risk of future accidents.

(B) Businesses are as concerned with long-term as they are with short-term
strategies for maximizing profits.

(C) Businesses generally do the environmentally "right" thing only if doing so makes
good business sense.

(D) Businesses treat fines that are levied against them as an ordinary business
expense.

(E) Businesses are learning to exploit the public's environmental awareness in


promoting themselves.

Answer:A
I attacked since businesses value their profits, and chose D. Where i went wrong?
Thanks!

你选择 B 的依据在文中找不到,我想你的逻辑是重视短期利润就不会为环保支出,可这个逻辑你在文中是
读不出的,同时,短期利润就不会因为被罚款而受影响么?什么是短期呢?很模糊的一个表达。

你对于 D 的解释又增加了一个题外的条件,即增加费用支出不会影响利润,方法是提高价格,那么就要
问了,提高价格会都卖的出去吗?又要涉及许多题外的条件,这些都是文中读不出的,所以,请一定不要
自己增加文中不存在的条件的推理。

line of reasoning:
since fine is greater than prevention, company will certainly choose the cheaper one.
A= the actual cost for a business activity by a company may be higher than fine, but the company thought it si
lower than fine. so it would like to be fined rather than do prevention.
D even the fine is a common expense, a company will definitely choose a cheaper cost, right ? so, it will surely
pick prevention. Not weaken.

312
also , it is common sense

(A) Businesses generally greatly underestimate the risk of future accidents.===>

They consider they will pay less fine for future accidents. So, they may take corresponingly
less prevention. However, the actual risk will cost them much more and if they correctly
gauge the risk, they will certainly install adequate environmental safeguards. But, they
actually do not since they underestimate the risk .

12. Some of the most prosperous nations in the world have experienced a pronounced
drop in n
ational savings rates-the percentage of after-tax income an average household saves.
This trend will undoubtedly continue if the average age of these nations' populations
continues to rise, since older people have fewer reasons to save than do younger people.

Which one of the following indicates an error in the reasoning leading to the prediction
above?
(A) It fail to specify the many reasons younger people have for saving money, and it fails
to identify which of those reasons in the strongest.
(B) It assumes that a negative savings rate-the result of the average household's
spending all of its after-tax income as well as some of its existing savings-cannot ever
come about in any nation.
(C) It fails to cite statistics showing that the average age of the population of certain
nations is rising.
(D) It only takes into account the comparative number of reasons older and younger
people, respectively, have for saving, and not the comparative strength of those reasons.
(E) It uses after-tax income as the base for computing the national savings rate without
establishing by argument that after-tax income is a more appropriate base than before-
tax income.
Answer: D

比较的本质,是原因导致的结果,而不是原因数量的多少。老头有 1 万条存钱的理由但只有 1 块钱去存,


小伙只有一条理由,却存了 1 万块。

M: 此题首先用排除法排除 ABCE. D 对是因为原文指出 rate 有一个 pronouced drop. 然后说因为 average age
continues to rise,这个 trend 会继续. 这里将 age 作为原因,导致 pronouced rate drop. D 指出其问题是此原因的
影响力是否能造成 pronouced drop, 即不能排除可能有其它因素共同作用.

Questions 15-16

313
Computer operating system software has become increasingly standardized. But when a large business with
multiple, linked computer systems uses identical operating system software on all of its computers, a computer
vandal who gains access to one computer automatically has access to the data on all the computers. Using a
program known as a "virus" the vandal can then destroy much of the data on all the computers. If such a business
introduced minor variations into its operating system software, unauthorized access to all the computers at the
same time could be virtually eliminated. Furthermore variations in operating system software can be created
without any lose of computer compatibility to the business. Therefore, it is advisable for businesses to implement
such variations.

15. Which one of the following, if true , supports the conclusion in the passage?

(A) Standardization of computer operating system software has increased computer compatibility among different
businesses.
(B) Correcting any damage resulting from an invasion by a computer virus program is more expensive than
preventing the damage
(C) It is not costly for a business to maintain incompatible computer operating systems.
(D) There are other kinds of destructive computer programs that do not depend on inter computer links.
(E) Not all businesses need to share date among their internal computer systems.

16. Which one of the following can be inferred from the passage?

(A) If a business does not introduce variety into its computer operating systems, It will lose data on its computers
because of damage from virus programs.
(B) If a computer virus program is introduced into a business computer, all of the data on that computer will be
destroyed.
(C) If a business introduces variety into its linked computer operating systems, it will have increased overall
protection for its systems, but will not have protected every computer from viral invasion.
(D) If a business does not have multiple, linked computer systems, its computers cannot be protected from
computer viruses.
(E) If minor variations are created in computer operating system software, it will be easier to access the data on the
computers that use that software.

15 题,答案(B)中 Correcting any damage 原文没有提到,是否 expensive 也没有说,怎么就成了答案呢?


16,答案(C)前半部分可以,后半部分 but will not have protected every computer from viral invasion 原文也没
有见到有讲,怎么就推出来了呢?奇怪 ing。

原文的论证过程如下:
同样版本电脑,一有病毒,就全都毁了。howerver,
If such a business introduced minor variations into its operating system software, unauthorized access to all the
computers at the same time could be virtually eliminated.
言下之意:不同版本电脑,有了病毒,不会毁全部电脑,但是还可能毁一部分电脑。

C 中 but will not have protected every computer from viral invasion. 说的是不能保护全部,和上面不正好一样
吗。

314
所以重要的是上下全文来理解,切断全文的论证过程,但看一个句子,很容易误解。

15 题是加强题,选项 B 从成本的角度说明了对电脑系统作变动在经济上的可行性。所以可选。加强题不一
定适用无关排除。
16 题。本文说的是如果每台电脑的操作系统都一样的话,那么一荣俱荣一损俱损。但如果只有一台电脑染
上病毒,其他电脑如果系统不一样不会有问题,但是这台电脑自然不能幸免了。

17. It is the mark of a superior conductor that he or she has the authority to insist, even with a top orchestra, that
rehearsal work must be intensified. This authority cannot simply be claimed, the conductor must earn it by winning
the orchestra's respect for the artistic interpretations he or she is currently pursuing.

In taking the position outlined, the author presupposes which one of the following?

(A) Superior conductors devise different interpretations of composition for each orchestra with which they perform
it.

(B) Superior conductors are perfectionists who are never satisfied with any performance even by a top orchestra.

(C) Top orchestras are always ready to put in additional work on rehearsals if the conductor considers additional
rehearsing necessary.

(D) Top orchestras can appreciate the merits of an interpretation even before they have brought it to full
realization.

(E) Even top orchestras are not always led by superior conductors.

为什么选 D 呢。第一句话中的 mark 是什么意思呢?

Mark 的意思我想是“标志”的意思,这道题的关键我想是最后一句话,意思是“这种权威必须通过获得乐队
对艺术解释的尊重才能获得”。因而 D 是必然的选项!不知道我的解释对不对,请 NN 们帮忙解释一下吧!

18. In the United States proven oil reserves—the amount of oil considered extractable
from known fields—are at the same level as they were ten years ago. Yet over this same
period no new oil fields of any consequence have been discovered, and the annual
consumption of domestically produced oil has increased.

Which one of the following, if true, best reconciles the discrepancy described above?

315
(A) Over the past decade the annual consumption of imported oil has increased more
rapidly than that of domestic oil in the United States.

(B) Conservation measures have lowered the rate of growth of domestic oil consumption
from what it was a decade ago.

(C) Oil exploration in the United States has slowed due to increased concern over the
environmental impact of such exploration.

(D) The price of domestically produced oil has fallen substantially over the past decade.
(E)

(E) Due to technological advances over the last decade, much oil previously considered
unextractable is now considered extractable.

油产量跟十年前一样,但每年油消耗在增长。问解决矛盾

看完提干,我就自己猜,肯定是因为进口国外的油

所以就选了 A,

E 好像再看也没什么道理啊技术进步,很多以前不能开采的,现在被认为能开采了。

可是美国就是知道能开采它也不去开采啊,它要保护自己的石油资源啊!

extractable = 提取 提炼 not exactly the same to exploit

fact 1, the available amount of extractable demestic oil is unchanged,

fact 2, the consumption of it increased.

E means the same amount of raw oil can be extracted much more refined oil due to
technology advance. so explain the discrepancy !

A totally irrelevent

19. Train service suffers when a railroad combines commuter and freight service. By dividing its attention between
its freight and commuter customers, a railroad serves neither particularly well. Therefore, if a railroad is going to
be a successful business, then it must concentrate exclusively on one of these two markets.

316
For the argument to be logically correct, it must make which one of the following assumptions?
(A) Commuter and freight service have little in common with each other.
(B) The first priority of a railroad is to be a successful business.
(C) Unless a railroad serves its customers well, it will not be a successful business.
(D) If a railroad concentrates on commuter service, it will be a successful business.
(E) Railroad commuters rarely want freight service as well.
为什么是 C?我选 A,其余四个都觉得可排除。

正相反, A 是无关选项. 你要把握 assumption 的原则:原文逻辑的必要条件. 原文的逻辑必然有一个 gap,


assumption 用来填补 gap. 我讲过 assumption 的类型, 你可以看看我以前的回复.

此题的逻辑关系中有一个很明显的 gap: 概念的跳跃. serve both-->serve neither well==>successful -->serve


only one. 这里 successful 和 serve customer well 被联系在一起, 但是并没有指出关系. 就是 gap. C 是标准答案.

A 也许可以做为你个推理或结论的答案. 如果你选了 A 说明你对 assumption 的题目还没有很了解. 不知道你


看的参考书是哪一本?

陈向东的书可能有帮助(本人多次推荐此书,世界知识出版社应该为 CD 的会员打 5 折). 由于题目中有明显的


gap, 这里只有 C 是标准答案. D 是结论题型中的典型错误答案. 原文的逻辑是 serve one 是成功的必要条件,
D 说它是充分条件.

MINDFREE,

概念的跳跃. serve both-->serve neither well==>successful -->serve only one. 这里 successful 和 serve customer
well 被联系在一起, 但是并没有指出关系. 就是 gap. C 是标准答案.

这个就是你上次教我的确认逻辑关系中的假设题。假设一定要是必要条件,不能是充分条件。你举过类似的
例子的。

怎么换了个长相,我就认不得了。

这个问题很好. 在此题中改变后 C 就不是答案. 因为可以只 serve freiht customers 而不 serve commuters. 所以


必要条件是无论 serve 哪一个, 都要 serve well.

很多假设题目的答案是很多假设其中的一个,别比如说:考砸了-->没复习好. assumption 就有很多: 不是考前


没睡好造成的;不是考试时晕到了;不是考试时睡着了; 复习好就能考好等等. 答案就是其中一个,比相当于一
个集合中的 factor.

如果把原文改动一下:
Train service suffers when a railroad combines commuter and freight service. By dividing its attention between its
freight and commuter customers, a railroad serves neither particularly well. Therefore, if a railroad is going to be a
successful business, then it must expand its resources to serve both well.

317
那么改动后的 C 就是正确答案, 因为必须要 serve commuter. 此答案也可以是 Unless ...serve freight customers
well, ...

因为改动前是在现有基础上要成功,就必须服务好其中一个,此时 C 中的 customers 可以是 freight custom


ers(充分条件一),也可以是 commuter customers(充分条件二);改动后则是要成功的话,必须加大力
度,把两个都服务好(充分条件)。unless 服务好 commuter (必要条件之一),或服务好 freight (必要条
件之二),才能成功。
不知这样解释行吗?

20. Most people in the United States view neither big nor small business as particularly
efficient or dynamic and regard both as providing consumers with fairly priced goods and
services. However, most people consistently perceive small business as a force for good
in society whereas big business is perceived as socially responsible only in times of
prosperity.
The statements above, if true would provide the strongest support for which one of the
following hypotheses?
(A) Most people in the United States give little thought to the value of business to society.

(B) If big business were more efficient, it would be perceived more favorably by the public
generally.
(C) If small business were regarded as being more dynamic, it too, would receive
strongly favorable ratings only in times of general prosperity.
(D) Even if people did not regard big business as providing consumers with value for their
money, they would still regard it as socially responsible in times of general prosperity.
(E) Many people in the United States regard the social responsibility of big business as
extending beyond providing consumers with fairly priced goods and services.
Answer: E

用排除法. 前四项都是无关选项. E 比较绕, 但是符合原文. 大多数人认为 BB 总是能提供 fairly priced goods,


但是认为 BB 只在经济繁荣时 socially responsible.

试着对第三题说两句:原文的意思是“大部分人认为无论大企业还是小企业都不是特别有效率或灵活的,
但都提供了价格合理的产品。但是大部分人一直认为小企业是一种良好的动力而大企业只是在繁荣时才有
社会责任感。”abc 的错误是明显的,只说 D AND E。D 的说法,文章是没有提供支持的,文中当然没有说
合理的产品价格和人们对小企业的好看法有什么关系,但绝对没说无关,所以 D 的结论得不出。E 的意思
是社会责任是超过提供合理定价的产品的。对呀,人们认为大企业一直是可以提供合理定价的产品,但只
在繁荣时才有责任感,可见责任感不仅仅是价钱合理的产品。

这道题目我的个人理解:
A little 错

318
B 无关比较
C 原文没提到的信息: more dymatic, ........及后半段
D value of money,原文没提到的信息
E 名词信息都是原文提到的。。只有一个动词信息:extend beyond 原文没提到。。可是该动词信息可以从原文
的 ARGUE 过程理解到:
小企业和企业都能提供 fairly priced goods and services(这句话提到的是两企业具体能提供的东西)
很多人认为小企业是一种 force for good in society 而大企业 is perceived as socially responsible only in times
of prosperity( 一 个 是 一 种 FORCE , 一 个 是 一 种 RESPONSIBLE 。 提 到 的 都 是 总 体 抽 象 的 DUTY 或 者
ROLE)不难看出这两样都是总体性的东西,当然 ROLE 和 RESPONSIBLE 这两种 ROLE 都包括了前面提
到的具体的 fairly priced goods and services。。所以 EXPEND BEYOND 是在对原文的理解上得出的。。

原文:美国很多人没将大小 business 看作有活力和有效率的。而是当作为消费者提供价格合理的商品和


服务。但是,多数人坚持认为小 business 是社会良好的一股力量,相比之下,大 BUSINESS 仅在社会兴
旺时负有社会责任。

其实说到底就是大小 Business 不仅仅提供商品和服务,还有其他作用。E 就是这意思:美国很多人认为


大 business 的社会责任不仅仅是提供价格合理的商品和服务。

21. The energy an animal must expend to move uphill is proportional to its body weight,
whereas the animal's energy output available to perform this task is proportional to its
surface area. This is the reason that small animals, like squirrel, can run up a tree trunk
almost as fast as they can move on level ground, whereas large animals tend to slow
down when they are moving uphill

Which one of the following is an assumption on which the explanation above depends?

(A) The amount of energy needed to move uphill is no greater for large animals that it is
for small animals.

(B) Small animals can move more rapidly than large animals can.

(C) The ratio of surface area to body weight is smaller in large animals than it is in small
animals.

(D) There is little variation in the ratio of energy output to body weight among animals.

(E) The amount of energy needed to run at a given speed is proportional to the surface
area of the running animal.

319
Answer is C. Could you please give me a math formula about the relationship? 晕!

required energy = a* body weight


available energy = b* surface area
ratio = (b*surface area)/(a*bodyweight) =available energy/required energy
Basically, the bigger the ratio, the easier the animal can move uphill. Small animals have smaller surface area and
body weight, but that doesn't necessarily mean the ratio is larger than big animals.

23. A medical journal used a questionnaire survey to determine whether a particular


change in its format would increase its readership. Sixty-two percent of those who
returned the questionnaire supported that change. On the basis of this outcome, the
decision was made to introduce the new format.
Which one of the following, if it were determined to be true, would provide the best
evidence that the journal's decision will have the desired effect?
(A) Of the readers who received questionnires, 90 percent returned them.
(B) Other journals have based format changes on survey results.
(C) The percentage of surveyed readers who like the format change was almost the
same as the percentage of the entire potential readership who would like format change.
(D) It was determined that the new format would be less costly than the old format.
(E) Ninety percent of the readers who were dissatisfied with the old format and only 50
percent of the readers who like the old format returned their questionnaires.
Answer: C 我知道 C 没问题,但是 a,e 能作为支持吗?
谢谢各位!

A 错在不能说明收到 survey 的人群有代表性, E 是削弱, 通过计算可以得到实际支持 format change 的比例小


于 50%.
22. The 1980's have been characterized as a period of selfish individualism that threatens the cohesion
of society. But this characterization is true of any time. Throughout history all human actions have been
motivated by selfishness. When the deeper implications are considered. even the simplest "unselfish"
acts prove to be instances of selfish concern for the human species.

Which one of the following is a flaw in the argument?

(A) The claim that selfishness has been present throughout history is not actually relevant to the
argument.

(B) No statistical evidence is provided to show that humans act selfishly more often than act unselfishly.

(C) The argument assumes that selfishness is unique to the present age.

(D) The argument mentions only humans and does not consider the behavior of other species.

320
(E) The argument relies on two different uses of the term selfish.

这个题目原文的文字游戏把我又给晃晕了?答案是 E,请解释一下原文和如何排除其他答案得到最后的答案
E.

one selfishness is for individual (first sentence), while the other is for human species (last sentence)

shanna: Owners of any work of art, simply by virtue of ownership, ethically have the right
to destroy that artwork if they find morally or aesthetically distasteful, or if caring for it
becomes inconvenient.

jorge: Ownership of unique artworks, unlike ownership of other kinds of objects, carries
the moral right to possess but not to destroy. A unique work of art with aesthetic or
historical value belongs to posterity and so must be preserved. whatever the personal
wishes of its legal owner.

24. Which one of the following principles, if accepted would contribute most to Shanna's
defense of her position against that of Jorge?

(A) Truly great works of art are never morally or aesthetically distasteful to any serious
student of the history of art.

(B) The right of future generations to have their artistic heritage preserved is of greater
importance than the rights of any presently living individual.

(C) It would be imprudent to allow the present stock of artworks to be destroyed without
some guarantee that the artists of the future will produce works as great as those
produced in the past.

(D) There are certain entities over which no one would be ethically justified in claiming
absolute rights to ownership.

(E) The autonomy of individuals to do what they wish with what is theirs must not be
compromised, in the absence of a threat to anyone's health or safety.

答案:E

E states that owners can do whatever they want to do with their possession or property,
as long as the action will not cause harm to others' health safety (no moral obligation). So

321
destroying the art works one owns is part of what E covers. So E is for Shannon's
opinion.

以上是原来一个人的解释,in the absence of a threat to anyone's health or safety.是不是


要排除 Jorge 反驳的可能性?

25. On the basis of their statements, Shanna and Jorge are committed to disagreeing
about the truth of which one of the following statements?
(A) Anyone who owns a portrait presenting his or her father in an unflattering light would
for that reason alone be ethically justified in destroying it.
(B) People who own aesthetically valuable works of art have no moral obligation to make
then available for public viewing.
(C) Valuable paintings by well-known artists are seldom intentionally damaged or
destroyed by their owners.
(D) If a piece of sculpture is not unique, its owner has no ethical obligation to preserve it if
doing so proves burdensome.
(E) It is legally permissible for a unique and historically valuable mural to be destroyed by
its owner if he or she tires of it.

答案:A

对 A 还是不是很理解。

LSAT-17-4

322
7. When Cortez arrived in Mexico in A.D.1519, he observed the inhabitants playing a ceremonial
game with a rubber ball. The pre-Columbian inhabitants of Mexico began to use rubber around
A.D1000.Thus we can be sure that the game must have originated sometime between
approximately A.D.1000 and Cortez arrival.
The conclusion reached above depends on which one of the following assumptions?
(A) The pre-Columbian inhabitants of Mexico played games on all ceremonial occasions.
(B) The making of rubber balls was one of the earliest uses of rubber by the inhabitants of
Mexico.
(C) The ceremonial game referred to was popular throughout Mexico.
(D) The game had been played since its inception with a rubber ball.
(E) The dating of the first use of rubber in Mexico was due to Cortez.
Answer: D 其他的我都知道不是,但是我觉得 D 很怪。。。D 好象重复了原文的意思,对吗?

原文讲这个 game 是用 rubber ball, 然后指出 rubber 的出现时间, 可是结论是 game 的出现时间.这里的概念跳
跃就是从 rubber ball 到 game. 它们之间的联系是 game 用 rubber ball.你可以很容易想到反驳的理由:如果
game 一开始不用 rubber ball, 或是 game 在 rubber ball 出现之前就存在, 结论就错. 所以 assumption 一定是要
将这个联系确定. 即 D

既然问的是 game 而不是 ball,那么 ball 是从 1000 年左右开始有的,作者以 ball 可能出现的时间作为 game
最早的时限,那么一定要确定 ball 之前 GAM E不存在所以D说游戏是从有球时开始的。

9. Some people are Montagues and some people are Capulets. No Montague can be crossed in love All
Capulets can be crossed in love .Therefore, Capulets are not Montague Anyone who is not a Montague is
intemperate.

Assume that all of the statements in the passage are true, If it is also true that no Montague is
intemperate, then which one of the following must be true?

(A) The only people who can be crossed in live are intemperate Capulets.

(B) Anyone who is not a Copulet is a Montagues

(C) All intemperate people can be crossed in love.

(D) All intemperate people are Capulets

(E) All Capulets are intemperate.

这 种 题 目 我 看 到 就 晕 , 能 否 再 讲 讲 ! ! !答 案是 e Capulets are not Montague Anyone who is not a


Montague is intemperate.这个句子是打印错误吗?

323
原题的信息: 1. ALL Montague are not crossed in love 2. ALL Capulets are crossed in love
3. Anyone who is not a Montague is intermerate 4. Montague 和 Capulets 集合不重叠

A 提到了 crossed in love 的信息,而且是 crossed in love 的必要条件..原文 1,2 信息根本无提到.错.


(B) 表示二选一..原文没提到.
(C) All intemperate people = NOT Montague can be crossed in love. 原文没提到,可能还有第三类人.
(D) All intemperate people = NOT Montague are Capulets 错
(E) All Capulets are intemperate. 由 3,4 可得

可以看到 BCD 的错误其实是一样的...都是 ASSUME 除了 Montague 和 Capulets 外没有第三类人.

10. The formation of hurricanes that threaten the Unite States mainland is triggered by
high atmospheric winds off the western coast Africa. When abundant rain falls in sub-
Saharan Africa, hurricanes afterward hit the United States mainland with particular
frequency. Therefore, the abundant rains must somehow promote the ability of the winds
to form hurricanes.

Which one of the following arguments contains a flaw that is most similar to one in the
argument above?

(A) People who exercise vigorously tend to sleep well. Therefore, people who exercise
vigorously tend to be healthy.

(B) Cars drive faster on long city blocks than on short city blocks. Long blocks are thus
more dangerous for pedestrians than short blocks.

(C) Many people who later become successful entrepreneurs played competitive sports
in college. Therefore, playing competitive sports must enhance a person’s entrepreneur
ability.

(D) The blossoms of the chicory plant close up in full sun. Therefore the chicory plant’s
blossoms must open up in the dark.(C)

(E) Events in Eastern Europe can affect the political mood in Central America. Therefore
liberalization in Eastern Europe will lead to liberalization in Central America.

此题我没太理清它的逻辑关系, 而选项 C 我觉得是:A=>B 推出 B=>A,我咋觉得不像呢?

line of reasoning,
since A =>B, and C =>B, so, C=> A.

324
A, high atmospheric winds off the western coast Africa, a person’s entrepreneur ability.
B, The formation of hurricanes, become successful entrepreneurs.
C, abundant rain falls, playing sports.
here, a common sense should get involved in that ability is not same to final success.
if you cannot reason it directly out of the correct answer, then try another tactic, to rule out the obvious
wrong. for A, B, D, E, they are very different from the original reasoning.

11. It is even more important that we criticize democracies that have committed human rights violations
than that we criticize dictatorships that have committed more violent human rights offenses. Human
rights violations are always inexcusable, but those committed by governments that represent the will of
the people are even more reprehensible than those committed by dictators. Further our criticism is more
likely to have an effect on the former than on the later.

Which one of the following is a proper inference from the passage?

(A) All government commit same inexcusable and reprehensible acts.

(B) Some human rights violations are more reprehensible than other, more violent human rights
violations.

(C)Criticism of human rights violations is certain to have no effect on a dictatorship.

(D)Human rights violations are more likely to occur in democracies than in dictatorship

(E)Those who do represent the will of the people are less likely to be moved by criticism than are those
who merely claim to represent the will of the people.

这个题目作对了,但是思路不是特别清楚.请讲讲思路.?
E 选项的意思有些烦人,可以解释一下吗?
顺便帮我解释一下 commit 的意思.这个词我总是理解的比较模糊,(1) committed human rights violations ;
(2) committed by governments
能分别讲一下吗?

我对 11 题原文的理解:
原文包含了以下信息:
1. 对那些触犯了人们权利的民主国家的批评比那些侵犯了人们权利的独裁国家的批评更重要(两种国家的
批评重要比较)
2。侵犯人们权利的行为都是需要谴责的。。但是代表了人们利益的政府的侵犯行为比独裁者的行为更值得谴
责(又是两种政府的行为的可谴责性比较)
3。而且,我们的批评对前者(民主国家)比对后者更有影响(还是批评对两种国家的影响比较)

325
根据 2 排除 A
C 中 CERTAIN NO 一定排除
D MORE LIKELY TO OCCUR 无原文的比较信息
E than are those who claim to represent the will of the people. 错 。 。 原 文 可 没 说 到 独 裁 者 CLAIM TO
REPRESENT。。。。

12. A recent study found that snoring, though not common in either group, is more common
among smokers than among nonsmokers. On the basis of this evidence, the author hypothesized
that smoking by itself can induce snoring.
Which one of the following, if true casts the most doubt on the author's hypothesis?
(A) Stress induces both snoring and smoking in certain individuals.
(B) Obesity induces many individuals to smoke
(C) Most snorers do not smoke
(D) Most smokers do not snore.
(E) Both smoking and snoring cause throat problems.
Answer: A why not D?

正如 siebel 所讲,这是一个关于相对值的题目,和绝对值无关.A 是典型答案,其它答案还有:不是 snoring 导致


smoking.

吸烟问题是一个典型的逻辑陷阱:AB 同时伴生,A 一定是 B 的原因吗?最有力的驳斥是 C 导致 A B

大多数的吸烟人不打呼噜不能说明吸烟不会造成人打呼噜。一个是效果,一个是原因。好比大多数吸烟人没
有得肺病不能证明吸烟不会导致肺病。

The press reports on political campaigns these days as if they were chess games. One candidate's
campaign advisor makes a move; the other candidate's advisor makes a countermove. The press then
reports on the campaign advisors and not on the candidates. The losers in the chess game are the voters.
They are deprived of the information they need to make informed decisions because the press is ignoring
substantive policy issues and reporting only on the process of the campaign. It is clear that the campaign
advisors should stay out of the limelight and let the press report on the most revealing positions on
substantive issues the candidates have taken.

13. Which one of the following is an assumption upon which the argument in the passage depends?

(A) Chess is the most appropriate analogy to reporting on political campaign.

(B) The candidates in the election are taking positions on substantive policy issues.

(C) How the press reports politics determines the substantive issues in the campaign.

(D) The voters are not paying enough attention to the election to be able to make informed decisions.

326
(E) There is no difference between reporting on the political process and reporting on substantive issues.

这个题目答案是 B!题目是作对了,但是思路好象有些混乱.原文的结论主要是什么呢?B 答案是如何弥补原文


的 gap 的?这个 gap 存在于原文什么地方呢?

我的理解:
A---> B ASSUMPTION 有几种可能..一种是补 GAP...还有一种是 ASSUME A 条件存在是可能的..
本题就是 ASSUME 前提是可能的...

最后一句话: let the press report on the most revealing positions on substantive issues the candidates have taken.
的前提当然是 candidates take substantive issues
就是 B

18. Serval cosmetics firms are committed to the active development, validation, and adoption of new product-
safety tests that use cultures of human cells. They argue that the new tests serve to reduce the need for tests on live
animals.
The statements above most strongly support which one of the following conclusions?
(A) The pressure on cosmetics firms to cease conducting experiments that use live animals was initiated by groups
of social activists.
(B) Consumers are no more likely to buy products whose safety was tested on cultures of human cells than they to
buy products whose safety was tested on animals.
(C) Financial consultants for the cosmetics firms believe that using human cell cultures rather than live animals to
test product safety will cost the firm less in actual product-development costs.
(D) Researchers in the cosmetics firms believe that fewer tests of products will be needed if cell cultures rather
than live animals are used.
(E) Managers of the cosmetics firms believe that it is better for their firms not to perform tests on live animals if
there is an acceptable alternative way of determining product safety.

答案:E

我选的 C,为什么原文支持 E,而没有支持 C,是因为 C 的 Financial consultant 是无关信息吗?

C 错在 cost the firm less。原文没有这方面的信息。E 实际是原文的换一种说法。alternative way


是指 new product-safety tests

Questions 19-20
Can any research be found to validate the contention that those who spend time plucking out their gray hairs have
more negative attitudes toward the elderly than those who shrug their shoulders about their gray hairs? Unless a
person’s psychopathology leads him or her to overgeneralize, there is no necessary connection. Certainly it is
reasonable to like the elderly yet dislike the idea of impaired eyesight and hearing. Furthermore, holding negative
attitudes toward older people merely because they are old is immoral, according to nearly universally accepted

327
ethical standards. But there is nothing immoral about disliking some concomitants of the aging process.
20. In order to advance her point of view, the author does all of the following EXCEPT:
(A) dismiss an assertion as unfounded
(B) appeal to reason
(C) appeal to a general principle
(D) discredit a common stereotype about the elderly(D)
(E) make a distinction about attitudes
1. Which sentence indicate (B) appeal to reason?
2. why can't we regard the following sentence as (D) discredit a common stereotype about the elderly ?
holding negative attitudes toward older people merely because they are old is immoral

1。通篇都在说 REASON。除了结论外

2。原文并没有老套,陈规(stereotype)。你的话的意思为:这种态度是不道德的。这并不是陈规看法。

21. A society in which there are many crimes, such as thefts and murders, should not be called
“lawless.” That is an abuse of the meaning of words. As a suffix, “-less” means “without,” so
“lawless” means “without laws.” However, a society that has no laws has no crimes, because no
laws can be broken. A lawless society would, therefore, be a crimeless society. So what some have
termed a lawless society should actually be called “crimeful.”

If the statements in the passage are true, which one of the following must also be true?
(A) A society that has laws has crimes.
(B) A society that has no crimes has no laws.
(C) A society that has many laws has many crimes.
(D) A society that has some crimes has some laws.(D)
(E) A society that has many crimes has many laws.
请问这题是怎样推理的, 如果选 D,那么 E 呢, some 与 many 有何区别?

if you can pick up D and E from other keys, then, you have already known the difference between
sufficent condition and necessary condition.
the nuance between D and E is that the latter is out of scope.
a society that has crimes must at least have some laws, but many crimes dont necessarily
guarantee many laws.
so, compared with E, D is better.

no law , no crime=> law is necessary condition for crime.


so, this is not complied by A, C ( sufficient condition) and B ( -b cannot to -a under necessary
condition)
some => at least there are some laws. (neccesary)

328
but, many => a lot of codes of law . ( possible but not must ) so it is worse than some.

put it another way, if a city has 1000 cases of robbery anually, we cannot say that there are 1000
codes of law in that city, right ? there is just one code of law, robbery.

23. Mr. Blatt: Expert consultants are sought after by management because they help executives
make better decisions. That is why they are worth the substantial fees they charge.
Ms. Fring: Nonsense. Expert consultants are hired in order to enable executives to avoid
responsibility. The more the experts cost, the more they can be blamed when things go wrong.
Which one of the following, if it occurred, would be the strongest evidence favoring Ms. Fring’s
position over Mr. Blatt’s position?
(A) A company that is trying to decide whether to move its manufacturing plant hires an expensive
expert to conduct a cost/benefit analysis.
(B) Two competing companies faced with very similar problems adopt different solutions.
(C) A successful firm of expert consultants seeks to increase its volume of business by reducing its
fees, but its volume of business drops.
(D) An expert consultant builds up a successful business by charging clients a substantial
percentage of the amount an independent assessor judges that the consultant saved the company.
(C)
(E) A company follows a consultant’s advice to open two new stores, but both stores are only
marginally profitable at first.
这个 C 跟 stimulus 有什么关系啊,看了好几遍都不明白

its volume: the consulting firm's business volume


its fees: the consulting firm's fees
Ms.Fring: the more expert consultants charge, the more willing the excecutives are to hire them,
because the higher the fees are, the more responsibility the consultants share, and thus the less
responsibility the executives take
answer C: when a successful consulting firm charges less, less executives are willing to hire the
firm, (thus the firm's volume of business drops), because the firm serves less well for the
executives to avoid their responsibilities.

24. Although all contemporary advertising tries to persuade, only a small portion of
contemporary advertising can be considered morally reprehensible. It nevertheless
follows that some attempts at persuasion can be regarded as morally reprehensible.
Which one of the following, in its logical features, most closely parallels the reasoning
used in passage?
(A) None of the chemicals used for cleaning the Sistine Chapel will affect the original
dyes. Hence, the colors used by Michelangelo will be fully restored.
(B) Not all operational tracking studies are conducted to illustrate exact corporate returns
on investment. Hence, some of these studies are not reliable.
(C) A good manager always makes important decisions on the basis of adequate data,

329
although of course some managers fail to do this. It follows that some managers are not
good managers.
(D) There is a direct correlation between the number of times you repeat something and
the degree to which you retain it. Therefore, repetition is always a critical factor in
remembering.
(E) Some short poems are thematically pluralistic, since some sonnets are characterized
by such pluralism, and all sonnets are short poems.

答案:E

原文:all。。。a small portion of(相当于 some)。。。,所以 some。。。

E:al。。。some。。。所以 some。。。

330
Chelseayang 的帖子

针对 这个比较 麻烦的问 题,我转 抄了一点 我认为简 单但比较 有用的东 西,希望 对大


家有所帮助。
Sufficient and necessary conditions
Conditional reasoning is a fundamental component of both the Logical Reasoning and Logic
Games section of the LSAT.
A sufficient condition can be defined as an event or circumstance whose occurrence indicates
that a necessary condition must also occur.
A necessary condition can be defined as an event or circumstance whose occurrence is
required in order for a sufficient condition to occur. In other words, if a sufficient condition
occurs, you automatically know that the necessary condition also occurs.
Consider the following statement:
To look sexy, you must smile.
We represent this statement as follows:
Sufficient Necessary
Look sexy Smile
One of the confusing elements in recognizing conditional statements is that so many different
terms can be used to introduce a sufficient or necessary condition. Consider the following
statements:
If you want to look sexy, you must smile.
Smiling is necessary to look sexy.
When someone looks sexy, she must be smiling.
Only someone who is smiling can look sexy.
Unless you smile, you cannot look sexy.
You will look sexy only if you smile.
All the above would be diagrammed the same way:
Sufficient Necessary
Look sexy Smile
The following list may be helpful to you.
Sufficient Condition Indicators Necessary Condition Indicators
If Then
When only
Whenever only if
Every must
All required
Any unless
People who except

331
In order to until
Without
You should be able to memorize the indicator words on each list, which could assist you in
recognizing the types of situations faster.

332

Você também pode gostar